You are on page 1of 453

ENVIRONMENTAL

1. Consider the following statements about biosphere.


1. Biosphere is the part of the earth where living organisms reside.
2. The energy required for life within the biosphere comes only from the sun.
Which of the above is/are correct?
(A) 1 only (B) 2 only
(C) Both 1 and 2 (D) None
2. Consider the following statements about ecotone.
1. It is larger than a biome.
2. It is a zone of tension because it has conditions intermediate to the adjacent ecosystems.
3. A well-developed ecotone may contain some unique organisms which might be absent in the adjacent
ecosystems.
4. Ecotone is an inter-junction between two or more diverse ecosystems.
Which among the above statements is/are incorrect?
(A) 1, 2 and 3 only (B) 2,3 and 4 only
(C) 1 only (D) 1, 2 and 4 only
3. Which of the following ecosystems can be considered as an example of ecotone?
1. Grassland
2. Mangroves
3. Estuaries
Select the correct answer using the codes given below.
(A) 1 and 3 only (B) 2 and 3 only
(C) 1,2 and 3 (D) None
4. Which of the following ecosystems is most likelyto be stable?
(A) A large ecosystem with no variety of species and small flora
(B) A small ecosystem with no variety of species and small fauna
(C) A large ecosystem with a rich variety of species and large number of flora and fauna
(D) A large ecosystem with very a small variety of species but large number of a particular flora]
5. Which of the following best defines the biosphere?
(A) It is the upper region of lithosphere.
(B) It is the narrow zone where land, water, and air come together to contain life forms.
(C) It is the lower atmospheric region near the hydrosphere.
(D) It is the abiotic component of all forms on earth.
6. The boundaries of different biomes on land are determined mainly by
(A) Climate (B) Political boundaries
(C) Drainage patterns (D) Altitude
7. Which of the following can be best characterized as a’biome’?
(A) Tundra region (B) Lake Baikal
(C) Western Himalayan forests (D) Ganges River
8. Among the following, the largest population islikely to be found in
(A) Community (B) Ecosystem
(C) Biome (D) Biosphere
9. If there are more number of ecotones
1. There will be more number of eco transition zones.
2. There will be greater species diversity.
Which of the above is/are correct?
(A) 1 only (B) 2 only
(C) Both 1 and 2 (D) None of these
10. Usually high biodiversity is witnessed in ecotones. This is because
1. Ecotone is a junction between two or more diverse ecosystems
2. An ecotone has the highest primary productivity amongst all ecosystems.
Which of the above is/are correct?
(A) 1 Only (B) 2 only
(C) Both 1 and 2 (D) None
11. With reference to a biotope, consider the following statements.
1. It is an ecological area that is usually larger than an ecosystem
2. It is a common practice to isolate biotopes from each other for niche biodiversity propagation.
Which of the above is/are correct?
(A) 1 only (B) 2 only
(C) Both 1 and 2 (D) None
Food Chain and Food Web
12. Consider the following statements with reference to food chains.
1. The grazing food chain is found only in terrestrial ecosystem.
2. The initial energy source for detritus food chain is dead organic matter.
3. The grazing and detritus food chains are interlinked.
Select the correct statements.
(A) All statements are correct (B) 1 and 2 only
(C) 2 and 3 only (D) 2 only
13. Arrange the following marine organisms in the order of increasing hierarchies in the food chain
(A) Dinoflagellates = Diatoms < Copepods < Ocean Sunfish
(B) Dinoflagellates < Copepods = Diatoms < Ocean Sunfish
(C) Diatoms <Dinoflagalletes< Copepods = Ocean Sunfish
(D) Diatoms <Dinoflagalletes< Ocean Sunfish < Copepods
Ecological Pyramid
14. With reference to ecological pyramids, consider the following statements.
1. The food producer represents the base of the pyramid and the top carnivore forms the tip in the
upright pyramid.
2. The pyramid of energy will always be upright.
Select the correct answer using the codes given below.
(A) 1 only (B) 2 only
(C) Both 1 and 2 (D) None
15. The pyramid of energy in an ecosystem is always upright because
1. The continuous loss of energy due to metabolic activity puts limits on how much energy is available
to higher trophic levels.
2. There is loss of energy at each trophic level in being transferred to the higher trophic level.
Which of the above is/are correct?
(A) 1 only (B) 2 only
(C) Both 1 and 2 (D) None
Pollutants and Trophical Level
16. Which of the following is/are the characteristics that the pollutant should possess for biomag-
nification to occur?
1. A pollutant should be long lived.
2. A pollutant should be biologically active.
3. A pollutant should be soluble in fats.
Select the correct answer using the codes given as follows.
(A) 1 only (B) 2 only
(C) 2 and 3 only (D) 1,2 and 3
17. Why chemical oxygen demand (COD) is a better mode to measure pollution load in water than
biochemical oxygen demand (BOD)?
1. COD can measure pollution load in flowing water which BOD cannot.
2. COD measures pollution load due to non-biodegradable sources which BOD does not.
Which of the above is/are correct?
(A)1 only (B)2 only
(C)Both 1 and 2 (D)None
18. Mosses is a bioindicator. What do you understand by this statement?
1.It reflects changes such as pollution in the ecosystems.
2.It cannot be cloned or genetically engineered.
3.It does not contain mRNA.
Select the correct answer using the codes below.
(A) 1 only (B) 2 and 3 only
(C) 3 only (D) 1 and 2 only
19. In which of the following trophic levels are you likely to find the highest concentration of an organic
toxin that has been found in ocean water?
(A)Phytoplankton
(B)Aquatic plants
(C) Small fishes at lower trophic levels
(D) Human being who consumes seafood
20. An indicator species is the one whose status provides information on the overall condition of the
ecosystem and of other species in that ecosystem. Consider the following species and the indications
they show in an ecosystem.
1. Mosses A. Help indicate acidic soil
2. Lichens B. Help indicate air pollution
3. Fungi C. Help indicate old-growth forests where an
abundance of coarse woody debris exists
Select the correct match using the codes given below.
(A) 1-A, 2-B, 3-C (B) 1-C, 2-B, 3-A
(C) 1-B, 2-C, 3-A (D) 1-A, 2-C, 3-B
21. BOD can be a measure of
1. Nutrient enrichment of water.
2. Level of pollution
Which of the above is/are correct?
(A) 1 only (B) 2 only
(C) Both 1 and 2 (D) None
Ecological Succession
22. Consider the following statements with reference to ecological succession.
1. Autogenic succession refers to succession brought about by living inhabitants of that
community itself.
2. Allogenic succession refers to succession brought about by outside forces.
Select the correct statements using the codes given below.
(A) 1 only (B) 2 only
(C) Both 1 and 2 (D) None
23. With reference to ecological succession, consider the following statements.
1. The first plants to colonize an area are known as the pioneer community.
2. The final stage of succession is called the climax community.
3. Allogenic succession is driven by the biotic components of the ecosystem.
Which of the statements given above is/arecorrect?
(A) 1 and2only (B) land3only
(C) 2only (D) 1, 2 and3
24. Lichen is a pioneer species. What do you understand by this statement?
(A) It has the ability to form symbiotic relation with a large number of species.
(B) It can live in inhospitable climatic conditions.
(C) It contributes substantially to nutrient recycling in the ecosystem.
(D) It is, generally, one of the first species to colonize an ecosystem.

25. Consider the following statements with reference to ecological succession.


1.It can occur in lifeless areas.
2. A climax community results when the webof biotic interactions becomes so intricate that no
other species can be admitted.
Which of the statements given above is/are correct?
(A) 1 only (B) 2 only
(C) Both 1 and 2 (D) None
26. Primary ecological succession is likely to occur in a region in which
1. Lava flows have occurred.
2. Rocks are left from a retreating glacier
3. Fires have occurred.
Select the correct answer using the codesgiven below.
(A) 1 and 2 only (B) 2 and 3 only
(C) land 3 only (D) 1,2 and 3
Ecological Terms and Concepts
27. Ecosystem is formed, most appropriately, by the interaction of
(A) Both biotic and abiotic factors of the environment.
(B) All living organisms present in the system.
(C) Diverse landforms found within a large zone.
(D) Communities found near the edge of a biome.
28. A biomarker is generally used as an indicator of
(A) Increase in the size of a biome.
(B) Biodiversity of an ecosystem.
(C) Diseases in an organism.
(d) Phenotype and genotype of a microbe.
29. Consider the following statements regardingproductivity of organisms.
1. Primary productivity is defined as the rate of formation of new organic matter by the major
‘consumers’ of the ecosystem.
2. Primary productivity depends on the plant species inhabiting a particular area, environmental
factors, and availability of nutrients and photosynthetic capacity of plants.
Which of the above is/are correct?
(A) 1 only (B) 2 only
(C) Both 1 and 2 (D) None
30. The terms ‘absorptive capacity’ and ‘carrying capacity’are used in the discourse of
(A) Evolutionary biology. (B) Geographical determinism.
(C) Employment elasticity. (D) Sustainable development.
31. Consider the following statements.
1. An ecological niche is the unique functional role or place of a species in an ecosystem.
2. No two species have exact identical niches.
Which of the above is/are correct?
(A) 1 only (B) 2 only
(C) Both 1 and 2 (D) None
32. Which of these is the most productive ecosystem in terms of primary productivity per unit area?
(A) Algal bed and coral reefs (B) Cultivated land
(C) Temperate forests (D) Savannah
33. pH determines the proliferation of any microbial species in a particular environment and the rate at
which it can reproduce. Consider the following statements with reference to this.
1. Most blue-green bacteria prefer an acidic environment.
2. Fungi generally prefer an alkaline environment.
3. The optimum pH of most bacteria and protozoa is near neutrality.
Select the correct answer using the codes given below.
(A) 1 and 2 only (B) 3 only
(C) 1 and 3 only (D) 1,2 and 3
34. With reference to the keystone species, consider the following statements.
1. It has a disproportionately large effect on its natural environment relative to its abundance.
2. Sea otter is considered an example of keystone species.
Which of the statements given above is/are correct?
(A) 1 only (B) 2 only
(C) Both 1 and 2 (D) None
35. With reference to the ecological niche, consider the following statements.
1. It is the role and position a species has in its environment.
2. Each species have a distinct niche.
Which of the statements given above is/are correct?
(A) 1 only (B) 2 only
(C) Both 1 and 2 (D) None
36. What do you understand by an indicator species?
(A) Its status provides information on the overall health of the ecosystem.
(B) It is a keystone species of an ecosystem without which the local food chain may collapse.
(C) It is an endemic species that is at the lowest level of the food chain.
(d) It is a representative species of all species that are threatened or endangered in the ecosystem.
37. What is a biotope?
(A) It is a well-defined geographical area, characterized by specific ecological conditions.
(B) It is an ecosystem that supports the genetic mutation of a contiguous ecosystem.
(C) It is a biome that harbours all species of the same phenotype.
(D) It is a community of species that reproduces entirely by asexual reproduction.
38. How is landscape connectivity between different protected areas crucial?
1. It helps to ensure natural seasonal migration pattern of species.
2. It increases the quality and diversity of available habitat that is crucial for species survival.
Which of the above is/are correct?
(A) 1 only (B) 2 only
(C) Both 1 and 2 (D) None
39. Ecozones are a method of dividing up the earth’s surface based on
(A) Common evolutionary history.
(B) Niche habitats of exotic species.
(C) Endemism and variety.
(D) Geographic landscapes.
40. Which of these is/are crucial in the transition of an economy to’green economy’?
1. Conservation of biodiversity
2. Reducing carbon emissions and pollution
3. Promoting consumption with sole focus on economic growth
Select the correct answer using the codes given below.
(A) 1 only (B) 1 and 2 only
(C) 2 and 3 only (D) 1 and 3 only
41. What do you understand by the term ‘social forestry’?
1. Reducing pressure on the traditional forest area by encouraging plantations on community land
2. Promoting commercial harvesting of timber and non-timber products from traditional forests by
community to meet local demands
Which of the above is/are correct?
(A) 1 only (B) 2 only
(C) Both 1 and 2 (D) None
42. An ecotype is a population that
1. Plays a specific functional ‘keystone’ role in an ecosystem
2. Has remained endemic to a particular ecological niche for a long period of time on
evolutionary scales.
3. Carries genes to successfully adapt to local environmental conditions.
Select the correct answer using the codes given below.
(A) 1 only (B) 1 and 2 only
(C) 3 only (D) 2 and 3 only
43. As an ecological phenomenon, a trophic cascade is definitely caused when
(A) Secondary succession immediately follows primary succession.
(B) Top predators are removed from an ecosystem.
(C) Human demand on resources exceeds what earth can regenerate each year.
(D) Energy flowing through the food chain is a constant.
44. If patches of green are seen amidst huge bodies of water, that signals presence of phytoplankton. A
large such patch would signify that
(A) The primary productivity of that water region is high.
(B) The region is oxygen deprived.
(C) The prevailing water is unusually hot.
(D) No waves or currents are found in its vicinity.
45. The philosophy of environmentalism advocates that
1. One should learn to live in harmony with the rhythms of the ecosystem.
2. Humans should not manipulate the natural environment to serve their immediate interests.
3. There is no way humans can preserve or restore the nature to its undisturbed form.
Select the correct answer using the codes given below.
(A) land2only (B) 2and3only
(C) 1 and3only (D) 1 only
46. What is land footprint in ecology?
(A) Area of land that is used to grow feed for animals within a country
(B) The proportion of built-up land as against the unused land
(C) The percentage of land to grow crops to the non-crop land
(D) Amount of land that is needed to produce a product by an organization or a nation
47. With reference to ecological value, consider the following statements.
1. It is a monetary assessment of minimum requirements for ecosystem service provision.
2. It includes assessment of ecosystem integrity, health, and resilience.
3. It accrues to both humans and non-humans.
Select the correct answer using the codes given below.
(A) 1 only (B) 1 and 2 only
(C) 1,2 and 3 (D) 2 and 3 only
48. What do you understand by ecological footprint?
(A) The way in which ecological agents reveal their preferences through ecological activity
(B) A degree of impairment to an ecosystem, which when surpassed is too severe to allow recovery
of that ecosystem
(C) An index of the area of a productive ecosystem required to produce the resources used and to
assimilate the wastes produced by a defined population
(d) Non-monetary assessment of ecosystem integrity, health, or resilience
49. Humus is the dark organic matter that forms in the soil when plant and animal matter decays. Consider
the following about it.
1. It contains nitrogen that is essential for plant growth and helps in nutrient retention.
2. It contributes to moisture retention in the soil by increasing microporosity.
Which of the above is/are correct?
(A) 1 only (B) 2 only
(C) Both 1 and 2 (D) None
50. A phenotype usually results from the interaction of
(A) Genotype and environment (B) Ecosystem and biome
(C) Species and genotype (D) Alleles and species
Forest Ecosystem
51. Which of the following biomes are correctly matched to their dominant flora and fauna?
Biome Flora and Fauna
1. Tundra Spruce and reptiles
2. Savannah Grasses, thorny shrubs and elephants
3. Desert Cactus and reptiles
4. Taiga Wolves, pines and firs
(A) only 1 (B) 2, 3 and 4 only
(C) 1, 3 and 4 only (D) 1 and 4 only
52. How do forests help in micro-climate control?
1. By affecting the relative humidity of air through transpiration
2. By absorbing greenhouse gases thus helping in temperature management
Which of the above is/are correct?
(A) 1 only (B) 2 only
(C) Both 1 and 2 (D) None
53. While travelling through Nilgiri Hills, you are least likely to encounter which of these tree pecies?
(A) Sal (B) Pine
(C) Eucalyptus (D) Cypress
54. Consider the following statements with reference to Eastern Ghats.
1. Starting from West Bengal, the ghats pass through Andhra Pradesh, Telangana, and Karnataka and
end m Tamil Nadu.
2. They play an important role in the monsoon break of both north-east and south-west monsoon.
Which of the statements given above is/are correct?
(A) 1 only (B) 2 only
(C) Both 1 and 2 (D) None
55. In what ways do naturally-occurring wildfires play an integral role in nature?
1. They help return nutrients to the soil.
2. They act as a disinfectant.
3. They allow sunlight to reach the forest floor, enabling a new generation of seedlings to grow.
Select the correct answer using the codes given below.
(A) 1 and 2 only (B) 2 and 3 only
(C) 1 and 3 only (D) 1,2 and 3
56. Which of the following is incorrect about temperate deciduous forests?
(A) It can be found in the coastal regions of Western Europe
(B) Oak, ash, and beech trees could be found here
(C) Pheasants and monals could be found in these forests
(D) None of the above
57. Mediterranean trees adapt themselves to dry summers with the help of their thick barks and wax
coated leaves. These help them to
(A) Reduce transpiration
(B) Increase rate of water flow from roots to shoots
(C) Cut down the need for photosynthesis
(D) Increase the area of the leaves to step up glucose intake
58. Depending upon the amount of net primary productivity, the various ecosystems can be arranged in
a decreasing sequence of yield. Which of these is the correct order?
(A) Tropical seasonal forest – Tropical rainforest – Temperate grassland – Woodland and shrubland
(B) Tropical seasonal forest – Tropical rainforest – Woodland and shrubland – Temperate grassland
(C) Tropical rainforest – Temperate grassland – Woodland and shrubland – Tropical seasonal forest
(D) Tropical rainforest – Tropical seasonal forest – Woodland and shrubland – Temperate grassland
59. Consider the following about the Veld.
1. Veld has a mild climate due to the influence of the Pacific Ocean
2. Veld receives rainfall mainly in the winter months due to the cold ocean currents washing its shores.
Which of the above is/are correct?
(A) 1 only (B) 2 only
(C) Both 1 and 2 (D) None
Eutrophication
60. ‘Brown tides’ in ocean occur due to
1. Algal blooms caused by phytoplankton
2. Increased human forage on ocean surface waters.
3. Upwelling of seagrass to the top layers
Select the correct answer using the codes given below.
(A) 1 only (B) 2 and 3 only
(C) 1 and 2 only (D) 1, 2 and 3
61. Eutrophication of a water body necessarily involves
1. Gradual heating of the water body
2. Inflow of nutrients in the water body
3. Calcification of bed deposits
Select the correct answer using the codes given below.
(A) 1 only (B) 2 and 3 only
(C) 2 only (D) 1, 2 and 3
62. Wetlands support human well-being in a number of ways. Which of these is/are such contribution(s)
of wetlands?
1. They cleanse polluted water in their vicinity
2. They help stabilize water supplied and protect shorelines
3. As an ecosystem, they support a rich biodiversity
Select the correct answer using the codes given below.
(A) 2 only (B) 1 and 3 only
(C) 1 only (D) 1, 2 and 3
63. You are working as the nodal officer of United Nations Environment Programme (UNEP) in India
reporting on the level of methane emissions form wetlands. You note that some local wetlands have
very high methane emissions. What can you do to reduce the emissions?
1. Increase the level of water saturation in wetlands
2. Make conditions more suitable in the wetlands for anaerobic digestion
Which of the above is/are correct?
(A) 1 only (B) 2 only
(C) Both 1 and 2 (D) None
64. A ‘floating treatment wetland’ (FTW) in the Neknampur Lake (Hyderabad) utilizes which among the
following techniques to help clean the lake?
(A) Phytoremediation (B) Bioventing
(C) Bioleaching (D) Composting
65. The wetlands (Conservation and Management) Rules, 2017
1. Stipulate setting up of State Wetlands Authority (SWA) in each state/UT headed by an eminent
ecologist from a state NGO.
2. Enable conversion of wetland for non-wetland used to seek a balance between growth and
sustainability
3. Make it mandatory for state authorities to prepare a digital list of all wetlands
Select the correct answer using the codes given below.
(A) 1 only (B) 2 and 3 only
(C) 3 only (D) 1, 2 and 3
66. Consider the following statements.
1. Wetlands in India occupy more area than the net sown area.
2. A greater area is under inland wetlands than coastal wetlands in India.
Which of the above is/are correct?
(A) 1 only (B) 2 only
(C) Both 1 and 2 (D) None
67. Which of the following will NOT be characterized as a wetland in India?
(A) Sunderbans delta (B)ReservoirsoftheDeccanPlateau
(C) Gulf of Kutch (D) Khasi and Jaintia region
68. Wetlands are a vital link between land and water. What is/are the importance of wetlands?
1. They are among the most productive ecosystems in the world.
2. They protect shores from wave action and reduce the impact of floods.
3. They absorb pollutants and improve water quality.
4. Many species of birds and mammals rely on wetlands for food, water, and shelter.
Select the correct answer using the codes given below.
(A) land2only (B) 2, 3 and4only
(C) 1,3 and 4 only (D) 1,2,3 and 4
69. With reference to peatlands, consider the following statements.
1. Peatlands are wetlands that contain a mixture of decomposed organic material.
2. Peatlands often lack oxygen as they are partially submerged in a layer of water.
3. If they are drained, their high carbon content makes them vulnerable to incineration.
Select the correct answer using the codes given below.
(A) 1 only (B) 2 and 3 only
(C) 1 and 3 only (D) 1,2 and 3
Estuary Ecosystem
70. Estuaries are among the most productive eco-systems in the world. What could be the possible
reason?
1. Some estuaries form an ecotone of fresh water and salty seawater.
2. Estuaries are not subject to disturbing forces such as tides and waves
Which of the above is/are correct?
(A) 1 only (B) 2 only
(C) Both 1 and 2 (D) None
Corals, Coral Reefs, and Coral Bleaching
71. Consider the following statements.
1. Cold water corals, in general, have a greater amount of zooxanthellae than warm water corals, and
they do not build reef-like structures.
2. Cold water corals differ from warm water corals because the former does not contain symbiotic
algae for photosynthesis and grows more slowly.
Which of the above is/are correct?
(A) 1 only (B) 2 only
(C) Both 1 and 2 (D) None
72. Consider the following statements regarding Aldabra Island.
1. It is the world’s second largest coral atoll.
2. It is one of UNESCO’s world heritage sites.
3. It is located near Maldives Island group.
Which of the statements given above is/are correct?
(A) 3 only (B) 1 and 2 only
(C) 2 only (D) 1,2 and 3
73. Consider the following about cold water corals.
1. They are rich in zooxanthellae, which allow them to survive in waters with low light insolation.
2. They can be found in the deep waters of continental shelves and offshore canyons.
3. These corals feed on zooplankton which is one of the sources of their energy.
4. They cannot be found in tropical regions.
Select the correct answer using the codes given below.
(A) 1 and 2 only (B) 2 and 3 only
(C) 3 and 4 only (D) 1 and 4 only
74. Consider the following statements.
Assertion (A): Area under coral reefs is lesser than the total area under forests on land.
Reason (R): Coral reefs are not found beyond temperate regions, whereas forests are.
In the context of the above, which of these is correct?
(A) A is correct, and R is an appropriate explanation of A.
(B) A is correct, but R is not an appropriate explanation of A.
(C) A is correct, but R is incorrect.
(D) Both A and R are incorrect.
75. Which of the following conditions generally leads to coral bleaching?
1. Low sedimentation of ocean beds
2. Unusually warm ocean water
3. Acidification of oceans
Select the correct answer using the codes given below.
(A) 1 and 2 only (B) 2 only
(C) 1 and 3 only (D) 2 and 3 only
76. Zooxanthellae loss in corals largely occurs due to
1. Exposure of corals to elevated concentrations of various chemical contaminants.
2. High temperature of ocean water and increased sedimentation of ocean bed.
Which of the above is/are correct?
(A) 1 only (B) 2 only
(C) Both 1 and 2 (D) None

77. Why is the Varadero reef––off the shore of the city of Cartagena, recently in the news, important for
marine biologists?
(A) It is the first successful relocation of a coral colony by humans.
(B) It is the first such extensive coral reef to be found at the mouth of a river.
(C) It thrives in a highly polluted environment.
(D) None of the above.
78. In which among the following ways do plastics raise the risk of disease outbreaks on coral reefs?
(A) Plastic debris stress corals through blocking out light and oxygen, thereby giving pathogens a
chance to take hold.
(B) Pathogens in the ocean thrive on plastic surfaces, thereby increasing their numbers and the
vulnerability of corals with elevated levels of plastic pollution around reefs.
(C) Coral organisms get infected by sharp plastic fragments that cut them, following which the
disease spreads across the coral colony.
(D) All of the above.
79. In the azure waters of this sea grows what scientists consider to be the planet’s most unique coral,
one that can survive global warming, at least for now. The sea in question is
(A) Red Sea (B) Java Sea
(C) Azov Sea (D) Alboran Sea
80. Consider the following about the CORAL project of NASA.
1. It will evaluate the state of existing rainforests and their evolutionary future discounting for
climate change.
2. It will survey keystone species in the harshest climates of earth which are the edge of a goldilocks
zone.
3. It will assess the conditions of the major reefs of the world through remote-sensing.
Select the correct answer using the codes given below.
(A) 1 and 2 only (B) 1 only
(C) 3 only (D) 2 and 3 only
81. What is/are the contribution(s) of coral reefs to an ecosystem?
1. They support fish production by providing them with food and shelter.
2. They control the concentration of carbon dioxide in the lower atmosphere and in the sea water.
3. They prevent the sea waves from going towards sea coasts.
4. They have the highest primary productivity amongst all ecosystems.
Select the correct answer using the codes given below.
(A) 1,3 and 4 only (B) 1,2 and 3 only
(C) 2 and 4 only (D) 1,2,3and 4
82. Consider the following statements.
1. Coral reefs break the power of the waves during storms, cyclones, and tsunamis.
2. Coral reefs form the spawning and nursery grounds for several fish species.
3. Coral reefs are being used in the treatment of cancer.
Select the correct answer using the codes given below.
(A) 1 only (B) 1 and 2 only
(C) 2 and 3 only (D) 1,2 and 3
83. The ocean/sea that has the most coral species is
(A) Pacific Ocean (B) Atlantic Ocean
(C) Indian Ocean (D) Mediterranean Sea
84. With reference to corals, consider the following statements.
1. Corals are marine vertebrate animals.
2. Each individual coral animal is a polyp.
3. Corals are not found at either of the poles of the earth.
4. All corals have a rock-like calcareous skeleton.
Select the correct answer using the codes given below.
(A) 1, 3 and 4 only (b) 2 only
(c) 2 and 4 only (d) 1,2,3 and 4
85. When ocean temperatures get too hot, corals often experience ‘bleaching’ events. What does this
mean?
(A) Plants and animals living in the vicinity of corals die.
(B) Corals move towards cooler waters.
(C) The symbiotic nature of the relationship between a certain plant and an animal breaks down.
(D) Corals submerge themselves completely under water, starving themselves of sunlight and air to
breathe.
Environmental Pollution and Measures
Pollutants
86. Consider the following about fluorinated gases.
1. They are emitted through semiconductor manufacturing processes.
2. They have the least global warming potential of all gases.
3. These gases are a cause of concern as they cannot be destroyed by any known natural
processes.
Select the correct answer using the codes given below.
(a) 1 only (b) 2 and 3 only
(c) land 3 only (d) 1,2, and 3
87. Which of the following pollutants is/are present in motor vehicle emission?
1. Carbon monoxide 2.Particulate matters
3. Hydrocarbons
Select the correct answer using the codes given below.
(a) 1 only (b) 2 and 3 only
(c) 3 only (d) 1,2, and 3
88. With reference to ‘Nitrogen trifluoride’ (NF3), consider the following statements.
1.It is primarily produced in the manufacturing of semiconductors and liquid crystal display (LCD)
panels.
2.It is considered a potent contributor to climate change and is therefore mandated to be included in
national inventories under the United Nations Framework Convention on Climate Change (UNFCCC).
3. It is used as a replacement for PFCs and SF6 in the electronics industry.
Which of the statements given above is/are correct?
(a) 2 only (b) 3 only
(c) land2only (d) 1,2,and3
89. Which of the following is/are the sources of oxides of Sulphur?
1.Petroleum extraction
2.Paper manufacturing
3.Municipal waste incinerating
Select the correct answer using the codes given below.
(a) land 2 only (b) 3 only
(c) 2 and 3 only (d) 1,2, and 3
90. Anthropogenic sources of mercury emission include
1. Coal-fired power plants
2. Mines engaged in gold production
3. Cement production
4. Manufacture of batteries
Select the correct answer using the codes given below.
(a) 1,2, and 3 only (b) 2 and 4 only
(c) 1 only (d) 1,2,3, and 4
91. Which of the following metals is/are used in fireworks to produce desired colours?
1. Copper for blue colour 2.Iron for green colour
3.Sodium for yellow colour 4.Potassium for violet colour
Select the correct answer using the codes given below.
(a) 2 and 4 only (b) 1,3, and 4 only
(c) land 4 only (d) 2 and 3 only
92. What are the sources that add nitrogen oxides into the atmosphere?
1. Bacteria living in soil
2. Reaction of ultraviolet radiation with ozone
3. Lightning stroke
Select the correct answer using the codes givenbelow.
(a) land 2 only (b) land 3 only
(c) 2 and 3 only (d) 1,2, and 3
93. Consider the following statements.
Assertion (A): Brown carbon is emerging as a solution to trap increasing atmosphericcarbon.
Reason (R): Brown carbon reflects all the ultraviolet and visible light falling on it.
In the context of the above, which of these iscorrect?
(a) A is correct, and R is an appropriate explanation of A.
(b) A is correct, but R is not an appropriate explanation of A.
(c) A is correct, but R is incorrect.
(d) Both A and R are incorrect.
94. The Government of India has been heavily regulating tanneries situated near river streams due to the
pollutants emitted by such industries, which are
1. Anthracene 2. Chromium
3. Arsenic 4. Formaldehyde
Select the correct answer using the codes given below.
(a) 1,2, and 4 only (b) 2 and 3 only f
(c) 3 and 4 only (d) 1,2, 3, and 4
95. Consider the following statements about nitrogen pollution.
1. Since nitrates cannot penetrate deep into the soil they pollute only surface water and not
groundwater.
2. Nitrogen dioxide is a greenhouse gas.
3. Sewage and organic solid wastes form the second largest source of nitrogen pollution in India.
4. Nitrous oxide (N20) is 300 times more potent than carbon dioxide but isn’t as prevalent in the
atmosphere.
Which of the above statements is/are correct?
(a) land 2 only (b) 3 and 4 only
(c) 1,2, and 3 only (d) 2,3, and 4 only
96. The Regulation of Polychlorinated Biphenyls Order, 2016 bans import of equipment containing
polychlorinated biphenyls (PCBs). Why is there concern about the use of PCBs?
1. PCBs are persistent organic pollutants (POPs).
2. PCBs are highly inflammable and volatile.
3. PCBs are carcinogens. Which of the above is/are correct?
(a) land 2 only (b) 2 and 3 only
(c) 1 only (d) 1 and 3 only
97. Burning coal generates fly ash which contains several toxic pollutants.These include
1. Lead 2. Zinc
3. Arsenic 4. Plutonium
5. Cadmium
Select the correct answer using the codes given below.
(a) 1, 2, 3, and 5 only (b) 2, 3, and 5 only
(c) 1,4, and 5 only (d) 1,2, and 3 only
98. What is/are the differences between a primary pollutant and a secondary pollutant?
1. A primary pollutant is an air pollutant emitted directly from a source unlike a secondary pollutant.
2. A primary pollutant has a much shorter life in the atmosphere than a secondary pollutant.
3. A primary pollutant is characterized as less potent or harmful than a secondary pollutant.
Select the correct answer using the codes given below.
(a) 1 only (b) 1 and 2 only
(c) 2 and 3 only (d) 1 and 3 only
99. Which of the following is/are harmful environmental effects of nitrogen oxides?
1. They cause irritant red haze in the traffic and congested places.
2. They can damage plant leaves and retardthe rate of photosynthesis. Which of the above is/are
correct?
(a) 1 only (b) 2 only
(c) Both 1 and 2 (d) None
100. Excess of nitrogen related compounds is harmful and may affect/cause
1. Blue baby syndrome 2. Textile fibres
3. Risk of acute respiratory diseases in children
Select the correct answer using the codes given below.
(a) 1 only (b) 2 and 3 only
(c) 1 and 3 only (d) 1,2, and 3
Air Pollution
101. Which of the following is/are the sources of radioactive pollution?
1. Accidents in nuclear power plants and nuclear waste
2. Nuclear weapon testing and explosion
3. Radiation therapy and direct exposures to radiation for diagnostic purposes
4. Cosmic rays from space and terrestrial radiations from radio-nuclides present in earth’s
crust such as radium-224
(a) 1,2, and 3 only (b) 2,3, and 4 only
(c) 1 and 3 only (d) 1,2,3, and 4
102. According to the Central Pollution Control Board, for the period between 2011 and2015
1. Nearly a third of Indian cities have breached annual pollution limits mandated by it.
2. The level of PM2.5 particles in most cities is within the prescribed limits.
Which of the statements given above is/are correct?
(a)1 only (b) 2 only
(c) Both 1 and 2 (d) None
103. Consider the following statements about fine particulate matter (PM2.5).
1. They are visible to the naked eye causing haze in the traffic.
2. Their levels are most likely to be elevated on days with little or no wind or air mixing.
3. They can bypass the nose and throat and can easily enter the circulatory system.
4. In particulate matter, the National Air Quality Index (AQI) monitors only PM10 particles.
Select the correct answer using the codes given below.
(a) 1,3, and 4 only (b) 2 and 3 only
(c) 2,3, and 4 only (d) 1,2, and 4 only
104. Consider the following statements regarding black carbon.
1. It is formed through the incomplete combustion of fossil fuels.
2. Its particles absorb heat but are extremely short-lived in the atmosphere in comparison to carbon
dioxide.
3. If deposited on snow, it could accelerate the heating of snow.
Select the correct answer using the codes given below.
(a) 1and2only (b) 2and3only
(c) 1 and3only (d) 1,2,and3
105. Open burning of coal can expose you to
1. Beryllium 2. Mercury
3. Sulfur dioxide 4. Nitrogen oxides
Select the correct answer using the codes given below.
(a) land 3 only (b) Z 3, and 4 only
(c) 1 and 4 only (d) 1, 2, 3, and 4
106. The largest human source of carbon dioxide emissions is from
(a) Industrial manufacturing (b) Combustion of fossil fuels
(c) Agricultural fields (d) Clearing of forests
107. photochemical smog consists of a mixture of air pollutants which may include
1.Sulphur oxides 2.Tropospheric ozone
3. Volatile organic compounds 4.Peroxyacyl nitrates
Select the correct answer using the codes given below.
(a) land 2 only (b) 2, 3, and 4 only
(c) 1, 2, and3only (d) land 4 only
108. Smog severity in a region is often aggravatedby
1. Stubble burning in neighboring agricultural areas.
2.Vehicular emissions.
3. Persistent cold weather conditions without sunlight.
Select the correct answer using the codes given below.
(a) 1 only (b) 2 and 3 only
(c) 2 only (d) 1,2, and 3
109. Photochemical smog generally forms in
(a)Cold and moist conditions (b)Warm, dry, and sunny climate
(c)Low lying mountain valleys (d)Regions where icefall is predominant
Fly Ash
110. Excessive generation of fly ash can lead to which of these harmful environmental effects?
1. It can result in leaching of toxic heavy metals in groundwater.
2. It contains crystalline silica which can cause silicosis.
Which of the above is/are correct?
(a) 1 only (b) 2 only
(c) Both 1 and 2 (d) None
111. Which is the first state to release a policy on fly ash utilization?
(a) Telangana (b) Karnataka
(c) Maharashtra (d) Punjab
112. Fly ash is becoming a popular construction material. What is/are the environmental benefits of fly ash
utilization in concrete?
1.Reduction in the amount of coal combustion products that must be disposed in landfills.
2.Net reduction in energy use and greenhouse gas emissions when fly ash is used to partly replace or
displace manufactured cement.
Which of the above is/are correct?
(a) 1 only (b) 2 only
(c) Both 1 and 2 (d) None
113. Fly ash is becoming a popular construction material. What is/are the environmental benefits of fly ash
utilization in concrete?
1.Reduction in the amount of coal combustion products that must be disposed in landfills.
2.Net reduction in energy use and greenhouse gas emissions when fly ash is used to partly replace or
displace manufactured cement.
Which of the above is/are correct?
(a) 1 only (b) 2 only
(c) Both 1 and 2 (d) None

Air Pollution in India


114. Consider the following statements.
1.Furnace oil refers to the heaviest commercial fuel that can be obtained from crude oil.
2.Furnace oil is used as a feed stock for fertilizer manufacturing.
Which of the above is/are correct?
(a) 1 only (b) 2 only
(c) Both 1 and 2 (d) None
115. Metal salts presently used in firework displays in India include
1.Sodium nitrate 2.Potassium chlorate
3. Strontium carbonate Select the correct answer using the codes given below.
(a) land 2 only (b) 2 and 3 only
(c) 1 and 3 only (d) 1,2, and 3
Water Pollution
116. Arsenic contamination of water is a high-profile problem in the Ganges Delta because
1.Arsenic is not found naturally within the earth’s crust and has been infused artificially due to
industrial pollution.
2.Use of deep tube wells for water supply is drawing arsenic closer to the surface.
Which of the above is/are correct?
(a) 1 only (b) 2 only
(c) Both 1 and 2 (d) None
117. Hot water discharged by industries is considered as a water pollutant because
1.Higher water temperature generally decreases the level of dissolved oxygen in water.
2.High temperature limits oxygen dispersion into deeper waters, contributing to anaerobic conditions
in deep water.
Which of the above is/are correct?
(a) 1 only (b) 2 only
(c) Both 1 and 2 (d) None
118. There are many documented cases of groundwater pollution which followed coal ash disposal. Which
of these elements in coal ash makes it so harmful?
1.Arsenic 2.Cadmium
3.Mercury 4.Sorbitol
Select the correct answer using the codes given below.
(a) 1,2, and 3 only (b) 2 and 4 only
(c) 1 and 3 only (d) 1,2, 3, and 4
119. All arsenic-affected river plains in India have their rivers originating from the
(a)Himalayan region (b)Western Ghats
(c)Eastern Ghats (d)Amarkantak region
120. Consider the following statements.
1.Arsenic contamination of groundwater is found in West Bengal.
2.High concentrations of fluoride in groundwater are common in some of the semi-arid areas of
Rajasthan.
3.The sulphate content in groundwater is high in regions such as Haridwarand Dehradun.
Select the correct answer using the codes given below.
(a) 1 and 2 only (b) 3 only
(c) 1 and 3 only (d) 1,2, and 3
121. Water can be considered polluted due to
1. High concentration of nutrients
2. Higher temperature
3. Presence of larvae eating fish
Select the correct answer using the codes given below.
(a) land 2 only (b) 2 and 3 only
(c) 1 and 3 only (d) None of the above
122. How does silting of water bodies affect aquatic life?
1. It results in increased rates of egg survival causing population boom in water bodies.
2. Coral polyps grow more favorably in silted and shallow water bodies.
3. Silt acts as a vehicle for certain pesticides into water bodies which adversely affect aquatic
life.
Select the correct answer using the codes given below.
(a) 1 only (b) 3 only
(c) 2 and 3 only (d) 1,2, and 3
Light Pollution
123. Light pollution can have which of the following effects on ecology and biodiversity?
1.It is known to disturb the reproductive cycles of some animals.
2.It adversely affects the migration of birds that navigate using the stars.
3. It disturbs circadian rhythms in humans affecting their sleep pattern. Select the correct answer
using the codes given below.
(a) 1 only (b) 2 and 3 only
(c) 1 and 3 only (d) 1,2, and 3
E-Waste
124. Due to improper/indiscriminate disposal of old and used computers or their parts, which of the
following is/are released into the environment as e-waste?
1.Beryllium 2.Cadmium
3.Chromium 4.Heptachlor
5.Mercury 6.Lead
7.Plutonium
Select the correct answer using the codes given below.
(a)1,3,4,6, and 7 only (b)1,2,3, 5, and 6 only
(c)2,4,5, and 7 only (d)1,2, 3,4, 5,6, and 7
125. Which of the following is often included in the category of harmful e-waste?
1.Lead 2.Cadmium
3.Beryllium 4.Brominated flame retardants
Select the correct answer using the codes given below.
(a) land2only (b) 1,2,and3only
(c) 3 and 4 only (d) 1,2,3, and 4
Solid Waste
126. The concept of ‘Extended Producer Responsibility (EPR)’ is often heard in the context of
(a)Renewable energy generation and transmission
(b)E-waste production
(c)Genetic modification of plant species
(d)Intellectual property rights
127. Consider the following statements.
1.The Solid Waste Management Rules, 2016 make it mandatory for fertilizer companies to co-market
compost.
2. Compost usage in farms enhances water holding capacity of the soil.
Which of the above is/are correct?
(a) 1 only (b) 2 only
(c) Both land 2 (d) None
Plastics Pollution
128. Which of the following is/are aims of thePlastic Waste Management Rules, 20167
1.Increase minimum thickness of plastic carry bags from 40 to 50 microns.
2.Expand the jurisdiction of applicability from the municipal areas to rural areas.
3.State Pollution Control Boards (SPCBs) have been mandated to formulate the guidelines for
thermoset plastic.
4.Manufacturing and use of non-recyclable multi-layered plastic to be phased out in two years.
Select the correct answer using the codes given below.
(a) 1,2, and 4 only (b) 2 and 3 only
(c) 1,3, and 4 only (d) 1,2,3, and 4
129. Consider the following statements.
1.To reduce the likelihood of dioxin and furan formation, refuse-derived fuel must be burnt at high
temperatures of more than 1,000’C.
2.About 80% of the total plastic waste generated in a year in India is not recycled.
Which of the statements given above is/are correct?
(a) 1 only (b) 2 only
(c) Both 1 and 2 (d) None
130. Consider the following statements.
1.About 29% of plastic produced gets recycled worldwide.
2.India has pledged to do away with single-use plastic by 2022.
Which of the statements given above is/are correct?
(a) 1 only (b) 2 only
(c) Both 1 and 2 (d) None
131. The #CleanSeas campaign has recently been launched by UN Environment to
(a)Accelerate innovation in eco-friendly plastic substitutes for all purposes related to the seas such as
fishing nets
(b)Ban the production of all forms of plastics
(c)Eliminate major sources of marine litter such as micro plastics in cosmetics
(d) None of the above
132. Of which celebration is India the host this year, whose theme is’Beat Plastic Pollution’?
(a)World Water Day (b)World Environment Day
(c)Earth Day (d)World Oceans Day
133. Consider the following statements.
1.The ‘Great Pacific Garbage Patch’ is a collection of marine debris that spans waters from the West
Coast of North America to Japan.
2.Ocean Gyres are formed by the earth’s wind patterns and the forces created by the rotation of the
planet.
Which of the statements given above is/are correct?
(a) 1 only (b) 2 only
(c) Both 1 and 2 (d) None
134. Consider the following about micro plastics.
1.They are small plastic pieces less than a nanometrology.
2.Bureau of Indian Standards (BIS) has classified them as unsafe for consumer products.
3.Micro beads are a type of micro plastic that are sometimes found in toothpastes.
4.If dumped in oceans, these pose a potential threat to aquatic life.
Select the correct answer using the codes given below.
(a) 1 and 4 only (b) 2,3, and 4 only
(c) 1 and 3 only (d) 2 and 4 only
135. Consider the following about multi-layered plastics (MLP), recently in news.
1.MLP is widely used in food packaging industries.
2.Some MLPs are non-recyclable and non-energy recoverable.
3.The Ministry of Environment recently notified that MLP is expected to be phased out from the
country.
Select the correct answer using the codes given below.
(a) 1 and 2 only (b) 1,2, and 3 (c) 2 and 3 only (d) land 3 only
Bio-Remediation
136. The highly degraded organic matter rich in phosphorus, nitrogen, and potassium in particular, resulting
from the activity of earthworms is known as
(a)Compost bedding (b)Humus
(c) Worm casting (d)Vermicomposting
137. Oil spills can be treated with bioremediation by using some species of
1.Archaea 2.Plants
3.Bacteria 4.Algae
Select the correct answer using the codes given as follows.
1 and 2 only (b) 4 only 1 and 4 only (d) 1,2,3, and 4
138. Vermicomposting toilet systems, which may come very handy in rural regions of India, work under
which of these limitations? 1. They must use larger quantities of water than traditional flush toilets
reducing their effectiveness in arid regions. The system cannot operate at high temperatures such as
those found in desert regions in Western India in summer seasons.
Which of the above statements is/are correct?
(a) 1 only (b) 2 only
(c) Both 1 and 2 (d) None
139. What do you understand by bioleaching?
(a)it is an in situ remediation technology that uses microorganisms to biodegrade organic constituents
in the groundwater system.
(b)It is the extraction of metals from their ores through the use of living organisms instead of using
harmful chemical substances.
(c)It is the decomposition of organic matter into compost by aerobic bacteria and fungi so that it can
be used as a fertilizer.
(d)It is the modification of the environment to stimulate existing bacteria capable of bi-oremediation.
140. Apart from certain types of insects, which of the following can be used as a bio-pesticide in the field?
1.Fungi 2.Bacteria
3.Viruses 4.Protozoa
Select the correct answer using the codes given below.
(a) 1,2, and 4 only (b) 2 and 3 only
(c) land 4 only (d) 1,2, 3, and 4
141. Consider the following statements.
1.Fungi can be used in the bioremediation of radioactively polluted sites.
2.Fungi can act as bio-fertilizers by actively competing for nutrients and space with pathogenic
microorganisms.
Which of the above statements is/are correct?
(a) 1 only (b) 2 only
(c) Both 1 and 2 (d) None
142. Rhizofiltration is the technique of
1. Enriching the nutrient base of soil by attaching symbiotic rhizobium with plant roots.
2. Absorbing and concentrating toxic metals from contaminated groundwater by plant roots.
Which of the above statements is/are correct?
(a) 1 only (b) 2 only
(c) Both 1 and 2 (d) None
143. Which of the following bacterium may be used for bioremediation operations?
1.Haemophilus 2.Mycobacterium
3.Rhodococcus 4.Ralstonia
Select the correct answer using the codes given below.
(a) land 2 only (b) 2, 3, and 4 only
(c) 1 and 3 only (d) 1,2,3, and 4
144. Bioremediation may NOT be most ideal for the removal of which of the following from a site?
(a)Cadmium (b)Lead
(c)Sodium chloride
(d)All of the above are toxic and may pose difficulty in degradation to microbes
145. Which of these are non-biodegradable materials?
1.Cotton cloth 2.Ball-point pen refills
3.Glass objects 4.Silver foil
Select the correct answer using the codes given below.
(a) 1,2, and 3 only (b) 2 and 3 only
(c) 2, 3, and 4 only (d) land 4 only
Environmental Pollution and Its Effects
146. Which of the following diseases is an occupational hazard?
(a) Silicosis (b) Tuberculosis
(c) Meningitis (d) Arthritis
147. Blood rain that had been observed for some time in Kerala is generally caused due to
(a) Presence of laterite soil in the region
(b) Excess of nitric and sulphur oxides in the atmosphere
(c) Specific type of green species of microalgae in the air
(d) Dispersal of iron oxides in the air
148. Consider the following pairs.
Pollutant Associated Disease
1. Nitrates Minamata
2. Mercury Black foot
3. Arsenic Blue baby syndrome
Which of the above pairs is/are correctly matched?
(a) 1 and 2 only (b) 1,2, and 3
(c) 3 only (d) None
149. Consider the following statements.
1. Methyl mercury can accumulate in edible fish tissue and pose a danger to humans.
2. Cadmium can enter the drinking water supply through corrosion of galvanized pipe and pollute it.
Which of the above is/are correct?
(a) 1 only (b) 2 only
(c) Both 1 and 2 (d) None
150. A substantial increase in atmospheric aerosols is likely to
1. Affect rainfall patterns
2. Cut down the destruction of stratospheric ozone significantly
3. Increase global warming by large proportions
Select the correct answer using the codes given below.
(a) 1 only (b) 2 and 3 only
(c) 3 only (d) land 2 only
151. A recent study has revealed that artificial lights are growing brighter and more extensive every year.
In this context, which of the following is/ are the associated effects/consequences?
1. It poses a threat to crop pollination.
2. More number of people cannot see the Milky Way.
3. It contributes to climate change.
Which of the statements given above is/are correct?
(a) 1 only (b) 1 and 3 only
(c) 3 only (d) 1,2, and 3
152. Though the casualties from the well-known pollutant PM2.5 are higher, the State of Global Air 2017
report highlights the alarming rate of increase in deatha 148% increase since 1990—related to which
one of the following air pollutants?
(a) Nitrogen dioxide (N02) (b) Lead
(c) Ozone (d) Sulphur dioxide (S02)
153. A study in The Lancet, a medical journal found that people who lived within 50m of high-traffic roads
had a higher chance of developing which one of the following diseases comparedto those who lived
more than 300m away from busy roads?
(a) Parkinson’s disease (b)Multiple sclerosis
(c) Dementia (d)Malnutrition
154. The Global Alliance to Eliminate Lead Paint is a voluntary partnership formed by UN Environment and
the World Health Organization to prevent exposure to lead, while promoting
Environmental Issues
155. Ecological balance may be disturbed due to the Introduction of new species
1. Introduction of new species 2.Natural hazards
3. Anthropogenic pollution
Select the correct answer using the codes given below.
(a) 1 only (b) 2 only
(c) 2 and 3 only (d) 1,2, and 3
Indian Himalayan Region
156. Which of the following organizations/institutions constituted the Himalayan State Regional Council?
(a) National Biodiversity Authority (b)Northern Zonal Council
(c) NITIAayog (d)None of the above
157. The oak trees of the Himalayan region are a concern to the region because
1. They are exotic species.
2. Their decay produces alkaline compounds that disturb the natural pH of forest soil.
3. Their introduction has reduced the population of keystone species in the region.
4. They soak a large amount of groundwater. Select the correct answer using the codes given
below.
(a) 4 only (b) 1 and 3 only
(c) 1,2, and 4 only (d) 2 and 3 only
Acid Rain
158. Acid rain is mainly a by-product of a variety of human activities that emit the oxides of
(a)Sulphur and nitrogen in the atmosphere
(b)Mercury and lead compounds in water bodies
(c)Radioactive compounds in effluent dis charge
(d)Carbon emitted by peatlands
Renewable and Non-Renewable Energy
Solar Energy
159. Which of the following is/are the features of ISRO’s ‘Solar Calculator’ app?
1.It suggests the optimum tilt angle for solar photovoltaic panel installation at any given location.
2. The latitude and longitude coordinates are required to be keyed in manually for the app to work.
3. It works only with smartphones that havean’auto brightness control’functionality Select the
correct answer using the codes giver below.
(a) lonly (b) 1 and 2
(c) 3 only (d) None of the above
International Solar Alliance
160. Consider the following statements.
1. International Solar Alliance (ISA) is a coalition of solar resource rich countries lying fully or partially
between the Tropic of Cancer and the Tropic of Capricorn to specifically address energy needs by
harnessing solar energy.
2. There are no targets or legal obligations imposed on member countries under the ISA Framework
Agreement.
3. The Private Financing Advisory Network (PFAN) is a multilateral public private partnership initiated
by the Climate Technology Initiative and the United Nations Framework Convention on Climate
Change (UNFCCC).
Which of the statements given above is/are correct?
(a) 1 only (b) 1 and 3 only
(c) 2 and 3 only (d) 1,2, and 3
161. In which of the following ways does the LSA seek to bring down the costs of producing solar power?
1. Promotion of research and development, particularly in areas of efficient storage systems
Members will be legally bound to increase solar energy generation capacity in thetr overall energy
basket to 25% by 2030 thereby boosting global demand Promoting standardization in the use of
equipment and processes for generating electricity
Select the correct answer using the codes given below.
(a) 1 only (b) 2 and 3 only
(c) 1 and 3 only (d) 1,2, and 3
Wind Energy
162. Wind energy forms a crucial component of theNational Renewable Energy Mission of India.Which of these
states in India has the largest capacity of installed wind energy?
(a) Gujarat (b) Maharashtra
(c) Tamil Nadu (d) Himachal Pradesh
Biofuels
163. Tata Motors has recently unveiled India’s first Bio-CNG (bio-methane) bus. Chemically, bio-methane
is identical to natural gas, however, natural gas is classified as a fossil fuel, whereas bio-methane is
called as a renewable source of energy. This is because
1.The nature of production process of bio-methane reduces its emissions of greenhouse gases into
the air.
2.Bio-methane is produced from fresh organic matter, unlike natural gas which is obtained from
decomposition of fossils.
Which of the above is/are correct?
(a) 1 only (b) 2 only
(c) Both 1 and 2 (d) None
164. The government is implementing an Ethanol Blended Petrol (EBP) Programme in India. Consider the
following with reference to it.
1.The government has introduced free market pricing mechanism for procurement of ethanol under
EBP Programme.
2.The government has allowed procurement of ethanol produced from cellulosic and lignocellulosic
feedstock.
Which of the above statements is/are correct

(a) 1 only (b) 2 only
(c) Both 1 and 2 (d) None
Other Energy Sources and Initiatives
165. Consider the following about some unconventional gas sources.
1.Tight gas is a natural gas with highest density that instantly leaks from reservoir rocks on first impact
at high pressure.
2.Coal mine methane (CMM) refers to the methane released from the coal and surrounding rock
strata due to mining activities.
Which of the above is/are correct?
(a) 1 only (b) 2 only
(c) Both 1 and 2 (d) None
166. Which of the following gases is the largest component of biogas?
(a) Methane (b) Hydrogen sulphide
(c) Nitric oxide (d) Carbon dioxide
167. Coal gasification is the process of producing syngas which consists of
(a) Methane, hydrogen, and water vapour only
(b)Hydrogen and carbon dioxide only
(c)Methane, hydrogen, carbon dioxide, carbon monoxide, and water vapour
(d)Carbon dioxide and carbon monoxide only
168. Compressed natural gas (CNG) is being promoted in place of diesel and petrol as a cleaner vehicle fuel
in India. Which of the following is/are the properties of CNG?
1. CNG is less likely to ignite on hot surfaces, since it has a high auto-ignition temperature.
2. CNG contains toxic lead.
3. While CNG is a cleaner fuel, it has a lower calorific value than diesel and petrol.
Select the correct answer using the codes given below.
(a) 1 only (b) 2 and 3 only
(c) 1 and 3 only (d) 1,2, and 3
169. Presently, biofuels in India are made from
1. Molasses
2. Municipal waste
3. Algae
4. Palm oil
Select the correct answer using the codes given below.
(a) 1 only (b) 2 and 3 only
(c) 1 and 3 only (d) 1,2, 3, and 4
170. FAME India Scheme is being implemented by
(a) Ministry of Environment, Forest and Climate Change
(b) Ministry of Heavy Industries and Public Enterprises
(c) Ministry of Science and Technology
(d) Ministry of Road Transport and Highways
171. Which of these is the lightest?
(a) CNG (b) Diesel
(c) Air (d) Liquefied petroleum gases (LPGs)
172. With reference to ‘flammable ice’, recently seen in news, consider the following statements.
1. It consists of methane trapped within water crystals.
2. It is another name for permafrost.
3. All of its reserves are trapped in plateaus at higher latitudes.
4. India is considering it as an alternative energy source.
Select the correct answer using the codes given below.
(a) 1,3, and 4 only (b) 2 and 3 only (c) 1 and 4 only (d) 2 only
173. Methanol is a promising fuel for waterways in India because
1. It does not have any smog-based emissions.
2. It is cheaper than most fossil fuels.
3. India manufactures all its requirement of methanol by domestic production.
Select the correct answer using the codes given below.
(a) land 2 only (b) 2 and 3 only
(c) 2 only (d) 3 only
174. Consider the following statements.
1. Hydrogen cyanide (HCN) is a colourless and non-flammable liquid.
2. HCN is used in manufacture of polymers and pharmaceuticals.
3. HCN is a strong base with a high pH value.
4. HCN has been detected in the interstellar medium.
Select the correct answer using the codes given below.
(a) 2 and 4 only (b) 1 and 3 only (c) 1,2, and 4 only (d) 2, 3, and 4 only
175. Consider the following statements.
1. Natural gas is often found with petroleumdeposits.
2. Russia is a major producer of natural gas.
3. In India, natural gas is found only in Western and Southern India.
Select the correct answer using the codes given below.
(a) land 2 only (b) 2 and 3 only
(c) 3 only (d) 1,2, and 3
Environmental Impact Assessment
176. Many development and industrial projects such as erection of dams, mining, and construction of
industries or roads require diversion of forest land. Consider the following in this regard.
1. Any project proponent, government or private, must apply for forest clearance from
Ministry of Environment, Forest, and Climate Change
(MoEFCC) through the concerned forest department of the state government.
2. As per the Forest (Conservation) Act, 1980, the project proponent recommends and
approves the compensation for the lost forest land.
Which of the above is/are correct?
(A) 1 only (B) 2 only
(C) Both 1 and 2 (D) None
177. The’ecosystem approach’ relies on
1. Integrated management of land, water, and living resources that promotes conservation
and sustainable use in an equitable way.
2. Adopting completely organic and natural methods, bypassing anthropogenic scientific
methodologies, focused on levels of biological organization harnessing mutual synergy.
Which of the above is/are correct?
(A) 1 only (B) 2 only
(C) Both 1 and 2 (D) None
EIA Cycle and Its Procedures
178. Environmental Impact Assessment (EIA) for a proposed plant generally includes which of these
factors?
1. Site and its surroundings
2. Hydrology along with quality of air and water
3. Occupational safety and health
4. Transportation of raw material and details of material handling
Select the correct answer using the codes given below.
(A) 1 and 2 only (B) 2 and 3 only
(C) 1, 3, 4 only (D) 1, 2, 3, and 4
179. All projects and activities under the EIA Notification, 2006 are broadly categorized in two categories
namely Category A and Category B based on the
1. Spatial extent of potential impacts
2. Potential impact on human health
3. Potential impact on natural and manmade resources
Select the correct answer using the codes given below.
(A) 1 only (B) 2 and 3 only
(C) 1 and 3 only (D) 1, 2, and 3
180. Discount rate tool is usually employed in environmental analysis to
(A) Evaluate the carbon sequestration derived from a particular initiative
(B) Understand the interconnection between ecological services
(C) Calculate inflation by using ecological GDP deflator
(D) Determine the present value of future benefits
Components of EIA
181. Which of the following items are relevant to properly conduct an EIA at a locality for an upcoming
project?
1. Landscape, including the details of drainage and natural water structures
2. Pattern of permanent wind flow in the locality
3. Availability of eateries in the locality
4. Position of sanitary fills in the locality
5. Calorie intake of the population
Select the correct answer using the codes given below.
(A) 1,4, and 5 only (B) 1,2, and 3 only
(C) 1,2, and 4 only (D) 2, 3,4, and 5 only
Acts and Policies
182. The harmful emissions that are identified for regulations in different Bharat Stages (BS) are
1. Carbon dioxide (CO2)
2. Methane
3. Particulate matter
4. Unburnt hydrocarbons (HC)
5. Nitrogen oxides (NOx)
Select the correct answer using the codes given below.
(A) 1 and 2 only (B) 3, 4 and 5 only
(C) 1, 2, and 5 only (D) 1, 2, 3, 4, and 5
183. Consider the following about Background Air Pollution Monitoring Network (BAPMoN).
1. India is not a part of this network.
2. It is a part of Global Atmospheric Watch (GAW) Programme.
3. It is being conducted by World Meteorological Organization (WMO).
Select the correct answer using the codes given below.
(A) 2 and 3 only (B) 3 only
(C) 1 only (D) 1 and 2 only
184. In India, National Air Quality Monitoring Programme (NAMP) is being implemented by
(A) The Energy and Resources Institute (TERI)
(B) Central Pollution Control Board (CPCB)
(C) Centre for Science and Environment (CSE)
(D) Environmental Information System (ENVIS)
185. Consider the following statements about National Clean Air Programme.
1. It involves the participation of central ministries, state governments, local bodies, and
other stakeholders.
2. The tentative target is 20%-30% reduction of PM2.5 and PM10 concentration by2022.
3. 2017 is taken as the base year for the comparison of concentration. Which of the above
statements is/are correct?
(A) 1 and 2 only (B) 2 and 3 only
(C) 1 and 3 only (D) 1, 2, and 3
186. Consider the following statements about National Mission for Clean Ganga (NMCG).
1. It is headed by the Union Minister of Water Resources, River Development and Ganga
Rejuvenation.
2. It is authorized to accord approval for all Ganga cleaning and rejuvenation projects.
Which of the statements given above is/are correct?
(A) 1 only (B) 2 only
(C) Both 1 and 2 (D) None
187. A seven-star rating system was recently launched by the Ministry of Housing and Urban Affairs.
This tool is specifically related to
(A) Scientific solid waste management
(B) Provisioning of safe piped water
(C) Making cities safe for pedestrians
(D) Eradication of open defecation
188. The Clean Air Campaign was jointly launched recently in Delhi by
(A) Ministry of Health and Family Welfare and the World Health Organization
(B) Ministry of Environment, Forest and Climate Change and the Delhi Government
(C) Ministry of Coal and the World Bank Group
(D) Ministry of Petroleum and Natural Gas and Clean AirAsia
189. The ‘Graded Response Action Plan’ was notified in year (2017) by the Environment Ministry The plan
is meant to tackle
(A) Air pollution in all urban regions of India
(B) Water pollution in river Ganga
(C) Soil pollution in eastern regions of India
(D) None of the above
190. The Ecosystem-based Management (EBM) Tools Network is focussing on tools for
1. Climate change vulnerability assessment and adaptation planning
2. Ecosystem-based coastal and marine spatial planning
Which of the above is/are correct?
(A) 1 only (B) 2 only
(C) Both 1 and 2 (D) None
191. Indian Coral Reefs Monitoring Network (ICRMN) is a programme of
(A) International Union for Conservation of Nature (IUCN)
(B) World Wildlife Fund (WWF)
(C) Ministry of Environment, Forest and Climate Change (MoEFCC)
(D) UNFCCC-India
192. Which of the following are reason(s) for adoption of bio-toilets in Indian Railways at a large scale?
1. They prevent erosion of railway tracks that was caused due to open defecation from the
coaches.
2. Bio-toilets do not require water and can operate in a variety of climatic situations.
Which of the above is/are correct?
(A) 1 only (B) 2 only
(C) Both 1 and 2 (D) None
193. Consider the following about the Comprehensive Environmental Pollution Index (CEPI).
1. It has been developed by the CPCB.
2. The index is a ‘rational number’ to characterize the environmental quality at a given location.
3. The index is used to earmark buffer zones around protected areas using different colour
schemes.
Select the correct answer using the codes given below.
(A) 1 only (B) 1 and 2 only
(C) 2 and 3 only (D) 1, 2, and 3
194. Consider the following about System of Air Quality and Weather Forecasting and Research (SAFAR).
1. It was introduced by Union Ministry of Earth Sciences (MoES) for greater metropolitan cities
of India.
2. The main objective of SAFAR project is to increase awareness among general public regarding
the air quality.
Which of the above is/are correct?
(A) 1 only (B) 2 only
(C) Both 1 and 2 (D) None
195. The central coordinating authority for oil-spill response in the maritime zones of India is the
(A) Indian National Centre for Ocean Information Services
(B) Indian Coast Guard
(C) Indian Navy
(D) National Disaster Response Force
196. Global Urban Ambient Air Pollution Database is released by
(A) International Meteorological Organization (IMO)
(B) United Nations Environment Programme (UNEP)
(C) UN-HABITAT
(D) World Health Organization (WHO)
197. With reference to the Central Pollution Control Board (CPCB), consider the following statements.
1. It is a statutory body under the Ministry of Environment, Forest, and Climate Change.
2. It was constituted in 1974 under the Water (Prevention and Control of Pollution) Act, 1974.
3. It is executing a nation-wide programme of ambient air quality monitoring known as National
Air Quality Monitoring Programme (NAMP).
Which of the statements given above is/are correct?
(A) 1 and 2 only (B) 1 and 3 only
(C) 2 only (D) 1, 2 and 3
Biodiversity
198. Which one of the following is the best description of the term ‘biodiversity’?
(A) A community of organisms together with the environment in which they live
(B) The interaction between biotic and abiotic elements within a given area
(C) The part of land, sea, and atmosphere in which organisms are able to live
(D) All kinds of living organisms within a given area
Levels of Biodiversity
199. Consider the following statements about naturalized species.
1. Most naturalized species are endemic to a certain environment.
2. Exotic species can become naturalized species if they do not compete for resources with the
native population.
Select the correct answer using the codes given below.
(A) 1 only (B) 2 only
(C) Both 1 and 2 (D) None
200. Among the following, highest levels of endemism in India is found in
(A) Mammals (B) Birds
(C) Fishes (D) Amphibians
201. Biodiversity of plants will generally tend to increase as one moves from
1. Lower to higher altitude
2. Higher to lower latitudes
Which of the above is/are correct?
(A) 1 only (B) 2 only
(C) Both 1 and 2 (D) None
202. Consider the following statements.
Assertion (A): The more the variety of species in an ecosystem, the more stable the ecosystem is likely
to be.
Reason (R): Genetic diversity is essential for a healthy breeding of and provision of a variety of services
between populations of species.
In the context of the above, which of these is correct?
(A) A is correct, and R is an appropriate explanation of A.
(B) A is correct, but R is not an appropriate explanation of A.
(C) A is correct, but R is incorrect.
(D) Both A and R are incorrect.
203. Endemic species are those that are
(A) Threatened to near extinction (B) Found exclusively in particular regions
(C) Most beneficial to the environment (D) Residing in ecologically rich regions

Marine Organism
Plankton
204. Consider the following statements.
1. Plankton are responsible for about half of earth’s oxygen.
2. Plankton are the only known organisms that always swim against the ocean current.
Which of the statements given above is/are true?
(A) 1 only (B) 2 only
(C) Both 1 and 2 (D) None
205. Highest concentration of plankton is found at
(A) Tropics (B) Sub-tropics
(C) Higher temperate latitudes (D) Polar regions

Phyto-plankton
206. What is the role of plankton in the marine ecosystem?
1. Establishing the foundation of food web of marine ecosystem
2. Absorbing atmospheric carbon dioxide into the ocean
Which of the above is/are correct?
(A) 1 only (B) 2 only
(C) Both 1 and 2 (D) None
207. Phytoplankton are generally found in upper layers of ocean water due to
1. Absence of nutrients in deep ocean water
2. Sunlight abundance in top layers
3. Calm top layers of water due to absence of mixing between top and bottom layers
Select the correct answer using the codes given below.
(A) 3 only (B) 1 and 2 only
(C) 2 only (D) 2 and 3 only
208. The reduction in population of phytoplankton may reduce
1. The amount of carbon dioxide drawn out of the atmosphere
2. Amount of carbon transferred to the deep ocean for long-term storage
3. Number of fishes in the sea
Select the correct answer using the codes given below.
(A) 1 only (B) 1 and 2 only
(C) 2 and 3 only (D) 1, 2, and 3
Seagrass
209. Seagrass is crucial for maintaining a balance in the marine ecosystem. Which of these conditions
promote the growth of seaweeds?
1. Shallow coastal waters with high temperature
2. Sandy ocean bottoms and clear water
3. Ocean water frequently disturbed by strong waves and currents
Select the correct answer using the codes given as follows.
(A) 1 and 2 only (B) 2 only
(C) 2 and 3 only (D) 1 and 3 only
210. Which one of the following statements about seagrasses is incorrect?
(A) These are flowering plants.
(B) They grow in near-shore waters, and are found around every continent.
(C) Seagrass habitats are commercially important due to their great productivity.
(D) They are the main diet for dugongs.
Seaweed
211. Seaweeds are potential natural fertilizers because
1. They are rich in soil nutrients and amino acids
2. They are not susceptiable to coastal pollution as they are found only in deep water.
Which of the above is/are correct?
(A) 1 only (B) 2 only
(C) Both 1 and 2 (D) None
212. Seaweeds can be used for which of the following purposes?
1. Production of agar-agar
2. Extraction of iodine
3. Generation of commercially important methane
Select the correct answer using the codes given below.
(A) 2 only (B) 1 and 3 only
(C) 1 only (D) 1, 2, and 3
Climate Change
213. Which of the following is/are important causes of long-term climate change?
1. Increase or decrease in number of sunspots
2. Milankovitch oscillations
Which of the above is/are correct?
(A) 1 only (B) 2 only
(C) Both 1 and 2 (D) None
214. Consider the following statements in the context of climate change.
1. The 20 warmest years on record have been in the past 22 years, with 2015-18 making up
the top four.
2. Together, China, the USA, and India account for nearly 40% of the global greenhouse gas
emissions today.
Which of the statements given above is/are correct?
(A) 1 only (B) 2 only
(C) Both 1 and 2 (D) None
215. Consider the following statements.
1. Climate justice compels us to understand the challenges faced by those people and
communities most vulnerable to the impacts of climate change.
2. Climate justice informs how we should act to combat climate change.
Select the correct answer using the codes given below.
(A) 1 only (B) 2 only
(C) Both1and 2 (D) None
216. Consider the following statements.
1. Climate justice focuses attention on people, rather than greenhouse gases.
2. Climate justice entails planning for evacuation of climate refugees due to rising seas.
3. Climate Justice is a moral argument.
Select the correct answer using the codes given below.
(A) 1 and 2 only (B) 2 and 3 only
(C) 1 and 3 only (D) 1, 2 and 3
217. Consider the following statements in the context of climate change.
1. 2014 is the hottest year on record, followed by 2015 and then 2016.
2. The increase in temperature in India over the long-term average is more than NASA’s
corresponding global figure.
Which of the statements given above is/are correct?
(A) 1 only (B) 2 only
(C) Both 1 and 2 (D) None
Global Warming
218. Some of the longer-term results of global warming can be
1. Coastal flooding
2. Increased incidence of tropical diseases
3. Disappearance of some ecological niches
4. Disruption of drinking water supplies
5. Increased frequency of tropical storms
Select the correct answer using the codes given below.
(A) 1, 3, and 5 only (B) 2, 4, and 5 only
(C) 1, 3, and 4 only (D) 1, 2, 3, 4, and 5
Greenhouse Effect
219. A greenhouse is made up of glass. What is the property of this glass?
(A) It is transparent to incoming short wave solar radiation and opaque to outgoing long wave
radiatio
(B) It is opaque to incoming short wave solar radiation and transparent to outgoing long wave
radiation.
(C) It is transparent to incoming long wave solar radiation and opaque to outgoing short wave
radiatio
(D) It is opaque to incoming long wave solar radiation and transparent to outgoing short wave
radiation.
220. During winter the vehicles with closed doors and windows remain warmer than the temperature
outside. This may be due to
(A) Greenhouse effect
(B) Heat conduction by metallic frame of vehicle
(C) Ionizing radiation effect
(D) High pollution outside the vehicle
Greenhouse Gases
221. Consider the following statements.
1. Tetrafluoromethane is a potent greenhouse gas.
2. Sulphur hexafluoride is the shortest living greenhouse gas in the troposphere and
stratosphere.
3. Recommendations for the use of perfluoroalkanes are included in the Kyoto Protocol.
Select the correct answer using the codes given below.
(A) 1 and 2 only (B) 2 and 3 only
(C) 1, 2 and 3 (D) 1 and 3 only
222. As per some reports, the Fashion industry, especially the synthetic fibres, may use up a quarter of
the world’s carbon budget by 2050. Which of the following is/are the major gases released in the
production of these synthetic fibres?
1. N2O
2. CO2
3. HFCs
Select the correct answer using the codes given below.
(A) 2 only (B) 1 and 2 only
(C) 1, 2 and 3 (D) 1 and 3 only
223. Which among the following greenhouse gases has the longest lifecycle in the atmosphere?
(A) Water vapour (B) Nitrous oxide
(C) Methane (D) Sulfur hexafluoride
224. Which one of the following is NOT a greenhouse gas generated from rice paddy fields?
(A) Nitrous oxide (B) Hydrofluorocarbons (HFCs)
(C) Methane (D) Carbon dioxide
Global Warming Potential
225. Global warming potential (GWP) is a measure of how much energy the emissions of one tonne of a
gas will absorb over a given period of time, relative to the emissions of one tonne of
(A) Methane (B) Nitrous oxide
(C) Ozone (D) Carbon dioxide
226. The total direct radiative greenhouse effect of methane is smaller than that of carbon dioxide due to
1. Lower concentration of methane in the atmosphere as compared to CO2
2. Lower direct radiative effect of a fixed mass of methane as compared to CO2
Which of the above is/are correct?
(A) 1 only (B) 2 only
(C) Both 1 and 2 (D) None
Ozone Depletion
227. The amount of incoming ultraviolet radiation reaching the terrestrial and marine ecosystems is
dependent on which of the following factors?
1. Cloud cover in troposphere
2. Depth of water column in water bodies
Select the correct answer using the codes given below.
(A) 1 only (B) 2 only
(C) Both 1 and 2 (D) None of these above
228. Ozone biosensors are able to record damage to marine life due to being exposed by
(A) Quartz crystals (B) Ultraviolet rays
(C) Metastatic cells (D) Floating nanoparticles
229. The amount of incoming ultraviolet radiation reaching the terrestrial and marine ecosystems is
dependent on which of the following factors?
1. Cloud cover in troposphere
2. Amount of aerosols scattered in the atmospheric layers
3. Depth of water column in water bodies
Select the correct answer using the codes given below.
(A) 1 only (B) 2 and 3 only
(C) 1 and 3 only (D) 1, 2 and 3
230. Which among the following statements is incorrect?
(A) The ozone molecule is harmful to air quality in the troposphere.
(B) The ozone is a secondary pollutant.
(C) Ozone is the main ingredient in smog.
(D) Emissions from industrial facilities and electric utilities are some of the major sources of
ground level ozone.
231. Which of the following chemicals is associated with the depletion of the ozone layer?
(A) Nitrogen dioxide (B) Methane
(C) Halons (D) Potassium hydroxide
232. How ultraviolet (UV) radiation is harmful to terrestrial life?
1. It accelerates ageing of skin in humans.
2. It can kill phytoplankton in large numbers.
3. Plant proteins are adversely affected by UV radiation.
4. It may lead to harmful mutation of plant cells.
Select the correct answer using the codes given below.
(A) 1 and 4 only (B) 2 and 3 only
(C) 1, 2, and 4 only (D) 1, 2, 3, and 4
233. Peak ground-level ozone concentrations are measured usually in which part of the day?
(A) Early morning (B) Afternoon
(C) Late evening (D) Midnight
234. Consider the following statements.
1. The ozone hole is caused due to reaction of chlorine monoxide and diatomic oxygen.
2. The ozone hole is produced when chlorofluorocarbons (CFCs) are hit by ultraviolet light.
Which of the above is/are correct?
(A) 1 only (B) 2 only
(C) Both 1 and 2 (D) None
Chapter 17
Impact of Climate Change – India
235. Consider the following statements.
Assertion (A): Low lying coastal areas may be permanently submerged under water due to global
warming.
Reason (R): Glaciers melt at higher temperature and heat causes water to expand in volume.
In the context of the above, which of these is correct?
(A) A is correct, and R is an appropriate explanation of A.
(B) A is correct, but R is not an appropriate explanation of A.
(C) A is correct, but R is incorrect.
(D) Both A and R are incorrect.
236. Melting permafrost would
1. Increase carbon content in the atmosphere
2. Raise sea levels
3. Reduce soil erosion at higher latitudes
Select the correct answer using the codes given below.
(A) 1 only 2 only (B) 2 and 3 only
(C) 2 only (D) 1 and 3 only
237. As global temperature increases
1. Increased frequency of extreme events like drought will reduce crop production
2. Pest attacks may increase on crops
3. Acidification of oceans will disturb the marine food chain
Select the correct answer using the codes given below.
(A) 1 and 2 only (B) 2 and 3 only
(C) 1 and 3 only (D) 1, 2, and 3
238. Oil extraction in the Arctic may lead to the release of
1. Black carbon
2. Methane
3. Carbon monoxide
Select the correct answer using the codes given below.
(A) 2 and 3 only (B) 2 only
(C) 1 and 2 only (D) 1 only
Agriculture and Food Security
239. Which of the following correctly assesses the impact of climate change on agriculture and food
security?
1. Crop yield may be reduced in most tropical and sub-tropical regions due to decreased
water availability.
2. Insect or pest incidence may increase leading to greater crop losses.
Which of the above is/are correct?
(A) 1 only (B) 2 only
(C) Both 1 and 2 (D) None
Mitigation Strategies
240. Which of the following will reduce global warming in the short-term?
(A) Increased rice cultivation
(B) Greater promotion of cattle breeding in developing countries
(C) Melting of permafrost in the Arctic region
(D) Major and sustained volcanic explosions
241. Global temperature potential is, most appropriately, a measure of
(A) The relative impact of different greenhouse gases on the earth’s warming
(B) The cyclical impact of increasing global temperatures and rising instances of forest fires on
earth’s warming
(C) The impact of countries not following through on commitments made by them to reduce
carbon emissions
(D) None of the above
242. In which one of the following declarations/ conferences did the’Polluter Pays Principle’first emerge
(though not explicitly)?
(A) United Nations Conference on the Human Environment, 1972
(B) ‘Our Common Future’ report, 1987
(C) Rio Earth Summit, 1992
(D) UN Conference on Earth and Development, 2002
243. Halting deforestation and increasing green cover is a cost-effective action that has a clear impact on
reduction of global GHG emissions. Why?
1. When forests are cleared or degraded, they can become a source of GHG emissions by
releasing the stored carbon in biomass or soil.
2. Farms such as rice fields soak atmospheric methane and reduce the overall stock of CO2 in
the atmosphere.
Which of the above is/are correct?
(A) 1 only (B) 2 only
(C) Both 1 and 2 (D) None
Carbon Sequestration
244. Artificial carbon sequestration can be achieved using
1. Sub-surface saline aquifers
2. Aging oil fields
3. Urban forestry
Select the correct answer using the codes given below.
(A) 2 and 3 only (B) 3 only
(C) 1, 2 and 3 (D) 1 and 2 only
245. Which of these ecosystems would sequester most carbon for a given unit of area?
(A) Saltmarsh (B) Mature tropical forests
(C) Grassland (D) Bare soil
246. The ‘CarbFix’ project in Iceland, recently in the news, aims to
(A) Accelerate the pace of incorporation of carbon into organic compounds by living
organisms
(B) Capture and convert carbon dioxide into stone forever
(C) Utilize carbon dioxide captured from air for the purpose of enhanced oil recovery
(D) Accelerate a naturally occurring process of carbon mineralisation in surface peridotite rocks
247. Ocean acts as a large carbon sink on earth due to
(A) Its large geographical coverage
(B) Rich population of phytoplankton and seagrass
(C) Difference in the partial pressure of carbon dioxide between seawater and air
(D) All of the above
248. Highest ‘carbon content’, among the following, will be found in
(A) Tropical rainforests (B) Atmosphere
(C) Ice in Antarctica (D) Oceans
249. Which among the following regions functions as the most effective ‘carbon sink’ and is also known as
the lungs of the earth?
(A) Savanna (B) Sahara region in Africa
(C) Tropical rainforests (D) Antarctica
250. Carbon Capture Utilization Storage (CCUS) is the process of
(A) Capturing waste carbon dioxide from large point sources and reusing it
(B) Redistributing captured carbon from storage in the atmosphere to mitigate de-calcification
in marine sources
(C) Generating carbon certificates from carbon-intensive activities and issuing them based on the
carbon footprint associated with each such activity
(D) Destroying atmospheric CO2 by way of electrostatic precipitators and storing the residue in
specialized storages
Geo-engineering
251. Which of the following is/are geo-engineering approaches to tackling climate change?
1. Dumping iron solution into the oceans to encourage plankton to bloom
2. Injection of sulphate aerosols into the atmosphere
3. Thinning down cirrus clouds of the upper troposphere
Select the correct answer using the codes given below.
(A) 1 only (B) 2 and 3 only
(C) 3 only (D) 1, 2 and 3
UNFCCC
252. Which of the following fund(s) is/are associated with the United Nations Framework Convention on Climate
Change (UNFCCC)?
1. Green Climate Fund (GCF)
2. Special Climate Change Fund (SCCF)
3. Least Developed Countries Fund (LDCF)
Select the correct answer using the codes given below.
(a) 1 only (b) 2 and 3 only
(c) 1 and 2 only (d) 1,2, and 3
253. Consider the following about the GCF.
1. GCF focuses on funding mitigation projects in developed countries only.
2. The G-20 countries are not the party members of the GCF.
3. It supports countries in fulfilling their Intended Nationally Determined Contributions (INDCs).
Select the correct answer using the codes given below.
(a)1 and 2 only (b) 2 and 3 only
(c) 3 only (d) 1 and 3 only
254. Which of the following has been accredited by the GCF as a national implementing entity for
undertaking climate change related projects in India?
(a) Reserve Bank of India (RBI)
(b) National Bank for Agriculture and Rural Development (NABARD)
(c) National Adaptation Fund for Climate Change
(d) Asian Development Bank (ADB)
255. The GCF has been established by the
(a) World Bank
(b) United Nations Convention on Biological Diversity
(c) UNFCCC
(d) Bonn Convention
256. With reference to the Conference of Parties (COP), consider the following statements.
1. It is the supreme decision-making body of the UNFCCC.
2. All states that are parties to the convention are represented at the COP.
3. COP reviews the implementation of the convention and any other legal instruments that the COP
adopts.
Select the correct answer using the codes given below.
(a) 1 only (b) 2 and 3 only
(c) 1 and 3 only (d) 1,2, and 3
257. Consider the following funds under UNFCCC and their parent management.
1. SCCF: Global Environment Facility
2. Adaptation Fund (AF): Green Environment Facility
3. LDCF: Kyoto Protocol
Select the correctly matched pairs using the codes given below.
(a) 1 only (b) 2 and 3 only
(c) 1 and 2 only (d) 1 and 3 only
258. The GCF is a fund within the framework of the
(a) Convention on Biological Diversity
(b) UNFCCC
(c) Montreal Convention
(d) IUCN
259. With reference to the UNFCCC consider the following statements.
1. It is the parent treaty of the 1997 Kyoto Protocol.
2. The framework sets binding limits on greenhouse gas emissions for individual countries.
3. It contains no enforcement mechanisms.
Select the correct answer using the codes given below.
(a) 1 only (b) 2 and 3 only
(c) land 3 only (d) 1,2, and 3
260. Parties to the UNFCCC are classified as Annex I, Annex II, and Annex B countries. Consider the following
about them.
1. Annex I countries are industrially developed countries.
2. Annex II countries are required to provide financial and technical support to developing countries
for climate change mitigation.
3. Annex B countries are the Least Developed Countries (LDCs) or developing island economies.
Select the correct answer using the codes given below.
(a) land 2 only (b) 2 and 3 only
(c) 1 and 3 only (d) 1,2, and 3
261. In climate change negotiations, the principle of Common but Differentiated Responsibilities (CBDR) is NOT
based on
(a) Historical emission responsibility of developed nations
(b) Need of sustainable development in developing countries
(c) Principle of equity in international affairs
(d) It is based on all of the above
262. With reference to climate change financing, the principle of CBDR evolved from the notion of
1. Nature as the common heritage of mankind
2. Equity in international law
3. Difference in economic and technical capacity of nations to tackle climatechange
4. Historical differences in the contributions of developed and developing states in creating
environmental problems
Select the correct answer using the codes given below.
(a) 1and4only (b) 1and2only
(c) 2 and 3 only (d) 1,2, 3, and 4
KYOTO Protocol
263. Consider the following about the Clean Development Mechanism (CDM).
1. CDM allows industrialized countries to buy Certified Emission Reduction (CER) and to invest in
emission reductions in developing economies.
2. ‘Additionally’ is ignored as an assessment factor for qualifying projects under CDM.
Which of the above is/are correct?
(a) 1 only (b) 2 only
(c) Both 1 and 2 (d) None
264. Which of the following amendments added new emission reduction targets for Second Commitment Period
(2012-20) of Kyoto Protocol?
(a) Doha Amendment (b) Cancun Amendment
(c) Singapore Amendment (d) New Delhi Amendment
265. The targets for the First Commitment Period of the Kyoto Protocol cover emissions of the six main
greenhouse gases. It did NOT include which of these gases?
(a) Methane (CH4) (b) Nitrous oxide (N20)
(c) Sulphur hexafluoride(SF0) (d) Carbon Monoxide (CO)
266. Consider the following statements.
1. A CER is generated from aCDM project activity.
2. CERs can be purchased only from the country that generated the emissions reduction potential.
Which of the above is/are correct?
(a) 1 only (b) 2 only
(c) Both 1 and 2 (d) None
267. Under Joint Implementation, countries with commitments are eligible to transfer and/or acquire emission
reduction units (ERUs) and use them to meet part of their emission reduction target. This has been
provided for under the
(a) Paris Climate Accord (b) Montreal Protocol
(c) Kyoto protocol
(d) Convention on Long-Range Transboundary Air Pollution (LRTAP)
REDD AND REDD+
268. Consider the following statements.
1. REDD+ is a mechanism jointly developed by the UNEP and the World Bank.
2. REDD+ incentivizes developing countries to keep their forest standing by offering result based
payments for actions to reduce or remove forest carbon emissions.
3. The World Resources Institute (WRI) established Global Forest Watch in 1997 as part of the Forest
Frontier Initiative.
Which of the statements given above is/are correct?
(a) 1 only (b) 2 and 3 only
(c) 2 only (d) 1,2, and 3
269. Consider the following statements.
1. Wood is a major renewable energy resource.
2. REDD+ (Reducing Emissions from Deforestation and Forest Degradation) is a climate change
mitigation programme developed by the US Agency for International Development and India’s
Environment Ministry.
Which of the statements given above is/are correct?
(a) 1 only (b) 2 only
(c) Both 1 and 2 (d) None
270. The United Nations Programme on Reducing Emissions from Deforestation and Forest Degradation (or UN-
REDD Programme) is a collaborative programme of the
1. Food and Agriculture Organization of the United Nations (FAO)
2. United Nations Educational, Scientific and Cultural Organization (UNESCO)
3. United Nations Framework Convention on Climate Change (UNFCCC)
4. United Nations Development Programme (UNDP)
Select the correct answer using the codes given below.
(a) land2only (b) 3and4only
(c) 1 and 4 only (d) 1,3, and 4 only
Global Environment Facility_
271. TrendsEarth, a tool developed by the Conservation International and funded by the Global Environment
Facility (GEF), concerns which among the following issues?
(a) Land degradation (b) Species extinction
(c) Plastic pollution (d) Methane emissions
272. The Global Wildlife Programme aims to promote wildlife conservation and sustainable development by
(a) Protecting keystone species
(b) Accelerating national afforestation projects in Asia, Africa, and South America
(c) Combating illicit trafficking in wildlife
(d) Studying which wildlife species need to thrive
273. GEF serves as a financial mechanism for the following conventions/protocols
1. UN Convention to Combat Desertification (UNCCD)
2. Stockholm Convention on Persistent Organic Pollutants (POPs)
3. Minamata Convention on Mercury Select the correct answer using the codes given below.
(a) 1 and 2 only (b) 2 and 3 only
(c) 1,2, and 3 (d) 1 only
274. The Sustainable Land and Ecosystem Management (SLEM) Programme is a joint initiative of the
Government of India and the GEF. What is/are the objectives of SLEM?
1. To promote sustainable land management and use of biodiversity
2. To maintain the capacity of ecosystems to deliver goods and services while taking into account
climate change
Which of the above is/are correct?
(a) 1 only (b) 2 only
(c) Both 1 and 2 (d) None
275. The Union Government signed the Global Environment Facility (GEF) Grant Agreement of the World
Bank for Ecosystem Service Improvement Project. It has which of the following objectives?
1. Improve the livelihoods of forest dependent communities
2. Strengthen the institutional capacity of the community organizations and departments of forestry
to enhance forest ecosystem services
Which of the above is/are correct?
(a) 1 only (b) 2 only
(c) Both 1 and 2 (d) None
276. Consider the following statements.
1. The Special Climate Change Fund (SCCF) was established under the UNFCCC to finance projects
relating to adaptation, technology transfer, and capacity building.
2. Developed countries are contributing to the Fast-start Finance (FSF) as a step towards mobilizing
US$1 trillion per year from public and private sources by 2020.
Which of the above is/are correct?
(a) 1 only (b) 2 only
(c) Both 1 and 2 (d) None
Intergovernmental Panel on Climate Change
277. With reference to the Intergovernmental Panel on Climate Change (IPCC), consider the following
statements.
1. It was created in 1988 by the World Meteorological Organization (WMO) and the United Nations
Environment Programme (UNEP).
2. The IPCC does not conduct its own research.
Which of the statements given above is/are correct?
(a) 1 only (b) 2 only
(c) Both 1 and 2 (d) None
278. Consider the following statements.
1. As per the recently released Special Report on Global Warming of 1.50C, India could annually
experience conditions like the 2015 heat wave if the average global temperature rises by more than
10C from the present.
2. The Paris Climate Pact calls for efforts to be pursued to limit the temperature increase to 1.50C
above pre-industrial levels.
Which of the statements given above is/are correct?
(a) 1 only (b) 2 only
(c) Both 1 and 2 (d) None
279. In order to limit global warming to 1.5°C, the Special Report on Global Warming of 1.5°C sets which
of the following targets?
1. By 2030 global emissions must decline by 45% from 2010 levels
2. By 2040 global net zero emissions must be achieved
3. By 2050 renewables must provide up to 85% of global electricity
Select the correct answer using the codes given below.
(a) 1 only (b) 2 and 3 only
(c) 1 and 3 only (d) 1,2, and 3
280. With reference to key takeaways from the Special Report on Global Warming of 1.5°C released recently,
consider the following statements.
1. Limiting warming to 1.5°C brings a lot of benefits compared with limiting it to2°C.
2. Rapid and far-reaching changes must take place in energy generation, land use, cities and industry
to limit warming to 1.5°C
3. As opposed to previous approaches of IPCC’s study of warming, this report clearly links lifestyle
choices with warming.
Which of the statements given above is/arecorrect?
(a) land 2 only (b) 2 and 3 only
(c) land 3 only (d) 1,2, and 3
281. The idea of an apparent pause In temperature rise between 1998 and 2014 had gained support in recent
years with even the IPCC In 2013 largely endorsing the findings. However, a number of studies in
recent times have found there has been no such pause. Which one of the following is the primary
reason cited by researchers for their contrarian conclusion?
(a) Better models have been developed in recent times to fill the gap in data of ocean temperatures
that was hitherto insufficiently measured from the Eastern Hemisphere.
(b) Global warming has taken the form of Increased glacial and ice melt in the past two decades,
acting as a veil for the rise in temperature.
(c) Ocean buoys used to measure sea temperatures, tend to report slightly cooler temperatures than
the older ship-based systems.
(d)Temperature measurements from the older ship-based systems continued to be utilized to develop
the ma\n global climate models, even as technologically advanced ocean buoys were deployed for
improved accuracy.
282. The IPCC is a scientific and intergovernmental body under the
(a) Auspices of the United Nations
(b) Conservation International NGO
(c) World Wildlife Fund (WWF)
(d) Climate division of IUCN
Miscellaneous
283. Consider the following statements regarding ‘Mobilize Your City Initiative!
1. It was launched as a part of COP 21 UNFCCC Paris Climate Conference.
2. It seeks to provide support to 100 cities worldwide in 3 years to engage in sustainable urban mobility
planning to reduce greenhouse gas emissions.
3. India is one of the very first countries to benefit from this initiative.
Which of the statements given above is/are correct?
(a) 2 only (b) 1 and 2 only
(c) 1only (d) 1,2, and 3
284. The C40 Cities group is an umbrella organization that coordinates the climate change activities of 96 major
urban centres around the world. Indian cities that are part of this network include
1. Mumbai
2. Bengaluru
3. Kolkata
4. Chennai
5. Delhi
6. Jaipur
Select the correct answer using the codes given below.
(a) 1,3,4, and 5 only (b) 2,4, and 6 only
(c) 1,2, 3,4, 5, and 6 (d) None of the above
285. As per IUCN, what are the defining characteristics of Indigenous and Community Conserved Areas (ICCAs)?
1. A community is closely connected to this well-defined ecosystem.
2. The area is untouched by human civilization other than the local indigenous communities.
3. The community is the major player in decision-making (governance) and implementation regarding
the management of the site.
Select the correct answer using the codes given below.
(a) 1 and 3 only (b) 2 only
(c) 2 and 3 only (d) lonely
286. International Fund for Animal Welfare (IFAW)
1. Is a non-profit organization
2. Provides hands-on assistance to animals
3. Campaigns to end commercial whaling
4. Was founded by the World Wildlife Fund (WWF)
Select the correct answer using the codes given below.
(a) 1 and 4only (b) 2 and 3only
(c) 1,2,and3only (d) 2and4only
287. The Clean Energy Ministerial (CEM)is
1. A high-level global forum to share lessons learnt and best practices for the transition to a global
clean energy economy
2. An initiative of the World Trade Organization (WTO)
3. Financing Green Carbon Credits (GCC) allotted to the Least Developed Countries (LDCs)
4. The main implementing agency for the programs of the United Nations Environment Programme
(UNEP)
Select the correct answer using the codes given below.
(a) 1,2, and 3only (b) 2and4only
(c) 1 only (d) 2,3, and 4only
288. The main foci of the World Climate Research Programme (WCRP) are
1. Observing changes in the components of the Earth system
2. Developing and improving numerical models that are capable of simulating and assessing the
climate system
3. Investigating the sensitivity of the climate system to natural and human-induced forces
Select the correct answer using the codes given below.
(a) 2and3only (b) 1 only (c) 1 and 3 only (d) 1,2, and 3
289. United Nations High-level Political Forum on Sustainable Development (HLPF) was established as a result
of the mandate in
(a) Rio 20 Conference, 2012
(b) UN Conference on the Human Environment (Stockholm 1972)
(c) Convention on Biological Diversity (1992)
(d) Paris Conference, 2014
290. Which of the following issues are covered by the United Nations Convention on the Law of Seas
(UNCLOS)?
1. Laws and regulations of the coastal state relating to innocent passage
2. Sea lanes and traffic separation schemes in the territorial sea
3. Immunities of warships and other government ships operated for non-commercial purposes
4. Submarines and other underwater vehicles Select the correct answer using the codes given below.
(a) 1 and 4 only (b) 2 and 3 only
(c) 1 and 3 only (d) 1,2,3, and 4
291. The first World Parliamentary Forum on Sustainable Development was hosted by
(a) UNEP
(b) World Economic Forum (WEP)
(c) Indonesian House of Representatives
(d) Senate of the United States of America
292. The objective of the National Carbonaceous Aerosols Programme (NCAP) is to
1. Prepare inventory of the carbonaceous aerosols including black carbon
2. Study the impact of aerosols on regional and global climate
Which of the above is/are correct?
(a) 1 only (b) 2 only
(c) Both 1 and 2 (d) None
293. The New Urban Agenda is an important outcome document agreed upon at the
(a) Paris Climate Conference, 2016 (b) Addis Ababa Climate Conference, 2014
(c) Habitat III Cities Conference, 2016
(d) United Nations Climate Change Conference, 2015
294. The blue label certification granted by the Marine Stewardship Council essentially depicts that the seafood
that is labelled blue is
(a) Seafood extracted in a sustainable and eco-friendly way
(b) Legally extracted from international waters
(c) The most exotic seafood available in the international market
(d) Safe for human consumption
295. The Restoration Opportunities Assessment Methodology (ROAM), produced by IUCN and the World
Resources Institute (WRI)
1. Provides a flexible and affordable frame work for countries to rapidly identify analyses areas th.it
.ire primed for forest lands ape restoration (FIR)
2. Gives a method for I nvlronment.il Impact Assessment (IIA) for developing countries that are parties
to the Convention or) Rio logical Diversity (CRD) only
Which of the above is/are correct?
(a) 1 only (b) 2 only
(c) Both 1 and 2 (d) None
296. Consider the following statements.
1. Circumarctic Environmental Observatories Network (CEON) is a network of terrestrial and freshwater
observation platforms, science experts, and network partners.
2. CEON promotes the collection and dissemination of environmental data from the Arctic.
3. CEON is located hundred feet underneath the thick layer of Arctic Sea.
Select the correct answer using the codes given below.
(a) land 2 only (b) 2 and 3 only
(c) 1 and 3 only (d) 1,2, and 3
297. The Climate and Development Knowledge Network (CDKN) is a/an
(a) WingoftheUNEP
(b) Alliance of private organizations and think tanks that links developing countries with experts on
climate change
(c) Inter-governmental organization established at the Kigali Meet of the Conference of Parties (COP)
to the UNFCCC
(d) Non-governmental organization established bythelUCN
298. The initiative ‘Future Earth’ was launched in
(a) Cancun Declaration, 2016—to the Convention on Biological Diversity
(b) First Earth Summit
(c) Rio 20 Summit, 2012
(d) Bali Climate Change Conference, 2007
299. The ‘Transformative Carbon Asset Facility (TCAF)’that would help developing countries pay for
emission reductions, was launched by
(a) World Hank (b) UNFCCC
(c) WWI (d) UNEP
300. Consider the following statements.
1. Bio Carbon Fund Initiative for Sustainable Forest Landscapes, (ISFL) promotes reducing GHG
emissions from the land sector by helping tackle forest degradation.
2. The Carbon initiative for Development (Ci-Dev), managed by WWF, helps the world’s poorest
countries access carbon finance, mainly in the area of energy access.
Which of the above is/are correct?
(a) 1 only (b) 2 only
(c) Both 1 and 2 (d) None
301. ‘Green Good Deeds’ campaign, recently in news, has been launched by
(a) WWF-lndia
(b) Union Ministry of Environment, Forests and Climate Change
(c) Greenpeace India
(d) Conservation International - India Chapter
302. Operation IceBridge is a 2009-18 NASA mission that aims to
1. Monitor changes in polar ice from a satellite
2. Create physical underground networks for clutter-free movement on frozen surfaces
Which of the above is/are correct?
(a) 1 only (b) 2 only
(c) Both 1 and 2 (d) None
303. Consider the following statements.
1. Green Initiatives for Future Transport (GIFT) aims to develop and demonstrate hydro gen powered
IC engine and fuel cell based vehicles.
2. Green Initiative for Power Generation (GIP) envisages developing fusion reactors for supplying on
grid power to the metropolitan areas.
Which of the above is/are correct?
(a) 1 only (b) 2 only
(c) Both 1 and 2 (d) None
304. The scope of World Climate Services Programme (WCSP) includes
1. Climate data and analysis
2. Climate adaptation and risk management
3. Climate mitigation finance
4. Climate monitoring and prediction Select the correct answer using the codes given below.
(a) land2only (b) land4only
(c) 1,2, and 4 only (d) 1,3, and 4 only
305. The Global Carbon Project (GCP) seeks to
1. Quantify global carbon emissions and their causes
2. Sequester global carbon generation using geo-engineering
Which of the above is/are correct?
(a) 1 only (b) 2 only
(c) Both 1 and 2 (d) None
National Wildlife Action Plan_
306. The National Wildlife Action Plan (NWAP) for 2017-31 adopts alandscape approachto conserve wildlife.
What does this approach mean?
1. Conserving all the wildlife rather than the areas where specific wildlife occurs
2. Focusing on conservation of wildlife by establishing smaller national parks and sanctuaries
3. Doing away with the system of reserved forests
Select the correct answer using the codes given below.
(a) 1 only (b) 2 and 3 only
(c) 1 and 2 only (d) 1,2, and 3
National Afforestation and Eco-Development
307. The main roles and functions of the National Afforestation and Eco-Development Board (NAEB)
include
1. Financing and overseeing the compensatory afforestation work
2. Rehabilitating and resettling communities displaced by deforestation for developmental works
3. Sponsoring research and extension of research findings to disseminate new technologies for the
regeneration and development of degraded forests
Select the correct answer using the codesgiven below.
(a) land 2 only (b) 3 only
(c) land 3 only (d) 2 and 3 only
CAMPA
308. Compensatory Afforestation Funds have been established only under the
(a) Public Account of each state
(b) Public Account of India and of each state
(c) Consolidated Fund of each state
(d) Consolidated Fund of India and of each state
309. Compensatory Afforestation Fund Management and Planning Authority (CAMPA) was established
(a) On the directions of the Supreme Court in 2009
(b) By a UNEP Project in India in 2012
(c) With India’s accession to the Convention on Biological Diversity (CBD)
(d) After the ratification of the Cartagena and Nagoya protocols
National Clean Energy Fund_
310. Consider the following about the ‘National Clean Energy Fund’ (NCEF).
1. It was constituted in the Public Account of India.
2. It invests in entrepreneurial ventures and innovative projects in the field of clean energy technology.
3. The fund does not receive any government assistance and is funded entirely from clean energy
cess.
Select the correct answer using the codes given below.
(a) 2 only (b) 1 and 3 only
(c) 1 and 2 only (d) 1,2, and 3
311. With reference to the NCEF, consider the following statements.
1. The fund is designed as a non-lapsable fund under the Public Account of India.
2. The Cabinet Committee on Economic Affairs approves all the projects under the fund.
3. Any project or scheme relating to innovative methods to adopt clean energy technology and R&D
are eligible for funding under the NCEF.
Select the correct answer using the codes given below.
(a) 1 only (b) 2 only
(c) 1 and 3 only (d) 2 and 3 only
National Green Tribunal_
312. Members of the National Green Tribunal (NGT) are chosen by a selection committee headed by
(a) Minister of Law and Justice
(b) Sitting judge of the Supreme Court of India
(c) Minister of Environment, Forests and Climate Change
(d) Cabinet Secretary
313. Matters related to which of the following acts can be adjudicated by the NGT?
1. Wildlife (Protection) Act, 1972
2. Environment (Protection) Act, 1986
3. Forest (Conservation) Act, 1980
Select the correct answer using the codes given below.
(a) 1 only (b) 2 and 3 only
(c) 1 and 3 only (d) 1,2, and 3
Miscellaneous
314. With reference to the ECOMARK Scheme, consider the following statements.
1. It is issued by the Bureau of Indian Standards (BIS) to products conforming to a set of standards
aimed at the least impact on the ecosystem.
2. An earthen pot has been chosen as the logo for the label.
3. The Central Pollution Control Board has become the member of Global Eco-label-ling Network
(GEN).
Which of the statements given above is/are correct?
(a) 1 only (b) 2 and 3 only
(c) 1 and 2 only (d) 1,2, and 3
315. With reference to the ‘National Nature Camps Programme’, consider the following statements.
1. It is an initiative of Ministry of Environment, Forests and Climate Change.
2. It is aimed at creating greater awareness, understanding, and empathy of children with and for the
environment.
Which of the statements given above is/are correct?
(a) 1 only (b) 2 only
(c) Both 1 and 2 (d) None
316. Consider the following about The Energy and Resources Institute (TERI).
1. It is the research arm of the Ministry of Environment, Forests and Climate Change.
2. Oil zapper technology for bio-remediation of oily sludge and oil spills was developed byTERI.
Which of the above is/are correct?
(a) 1 only (b) 2 only
(c) Both 1 and 2 (d) None
317. Wildlife Trust of India is
1. Credited with recovering population of critically endangered species, translocation of species,
reduction of human-animal conflict, rescue, and rehabilitation of animals like elephants, tigers,
leopards, one-horned rhinos, and bears
2. A registered charity in India
Select the correct answer using the codes given below.
(a) 1 only (b) 2 only
(c) 1 and 2 only (d) None
318. Consider the following statements about PARIVESH.
1. It is an environmental single-window hub for environment, forest, and wildlife clearances.
2. The system has been designed, developed, and hosted by the Ministry of Environment, Forests and
Climate Change.
3. Coastal Regulation Zone (CRZ) clearances are absent.
Which of the above statements is/are correct?
(a) land2only (b) 2and3 only
(c) land3only (d) 1,2,and3
319. Consider the following statements.
1. National Mission for Clean Ganga (NMCG) is the implementation wing of the National Ganga Council
2. NMCG is a statutory authority.
3. NMCG is authorized to approve projects for cleaning Ganga.
Select the correct answer using the codes given below.
(a) 1 and 2 only (b) 1 and 3 only
(c) 2 only (d) 2 and 3 only
320. National Water Development Agency (NWDA) carries out which of these functions?
1. Conducts detailed surveys and investigations of possible reservoir sites and interconnecting links
for peninsular rivers
2. Monitors, regulates, and controls water flows between inter-state river basin embankments
3. Assesses groundwater potential in the country and makes adequate arrangements for exploration
of groundwater
Select the correct answer using the codes given below.
(a) 1 only (b) 2 only
(c) 1 and 3 only (d) 1,2, and 3
321. Consider the following about TERI.
1. It is a non-profit policy research organization.
2. It is an attached agency of the International Centre for Climate Governance (ICCG).
Which of the above is/are correct?
(a) 1 only (b) 2 only
(c) Both 1 and 2 (d) None
322. Consider the following about the Centre for Science and Environment (CSE) in India.
1. It is a not-for-profit public interest research and advocacy organization.
2. It provides grants to Central Universities for research in ecology and zoology.
3. It publishes ‘Down to Earth’ magazine on a regular basis.
Select the correct answer using the codes given below.
(a) 1 and 3 only (b) 2 and 3 only (c) 3 only (d) 1 and 2 only
323. Which of these environmental/survey related organizations was established after India’s Independence?
(a) Forest Survey of India (FSI) (b) Zoological Survey of India (ZSI)
(c) Botanical Survey of India (BSI) (d) Geological Survey of India (GSI)
324. What are the primary functions of BSI?
1. Publication of national, state, and district floras
2. Survey and documentation of traditional knowledge and ethno-botany, associated with plants
3. In-situ conservation of critically threatened species in botanical gardens
4. Identification of threatened and red list species and species rich areas needing conservation
Select the correct answer using the codes given below.
(a) 1,2,and3only (b) 1,2,and4only
(c) 2 and 4 only (d) 1 and 3 only
325. The Long Term Ecological Observatories (LTEO) programme of the Ministry of Environment, Forests
and Climate Change
1. Aims to understand the biophysical and anthropogenic drivers of ecosystem change in the selected
biomes
2. Focuses on establishing international observatories linked through a common ecoentre to assess
the impact of climate change on global biodiversity
Which of the above is/are correct?
(a) 1 only (b) 2 only
(c) Both 1 and 2 (d) None Consider the following statements.
1. To undertake, aid, promote, and coordinate forestry education, research and their applications.
2. To develop and maintain a national library and information centre for forestry and allied sciences.
3 To act as a clearing-house for research and general information related to forests and wildlife.
The above are the objectives of the
(a) ENVIS Portal of the MoEFCC
(b) Indian Council of Forestry Research and Education (ICFRE)
(c) Wildlife Institute of India (Wll)
(d) FSI
326. Facility for Low Carbon Technology Deployment (FLCTD) aims to promote innovation of low-carbon
technologies and its deployment in which of these sectors of the Indian economy?
(a) Agriculture
(b) Hospitality and Tourism
(c) Industrial and other related sectors
(d) Import-export
327. Conservation and management of wetlands in India is primarily vested with the
(a) Central Government
(b) State/UT government
(c) Local government
(d) Ramsar Convention-India Chapter
328. ECOLEX isa/an
(a) Information service on environmental law
(b) Global ecological patents system
(c) Newly developed phytosanitary standard of WTO
(d) International NGO that protects the rights of communities who are ecologically vulnerable
329. Consider the following statements.
1. The Oil and Gas Climate Initiative (OGCI) is an inter-governmental initiative as a part of the post-
Paris agenda.
2. OGCI is under the auspices of the United Nations Environment Programme (UNEP).
3. OGCI serves as a platform to collaboratively advance technological solutions for meaningful action
on climate change.
Select the correct answer using the codes given below.
(a) 2 only (b) 3 only
(c) 1 and 3 only (d) 1,2, and 3
General Studies & Engineering Aptitude Answer Keys
330. Consider the following statements regarding the generation of biomass energy:
1. It is generated from organic material, plant or animal waste.
2. It is easily available.
3. Biomass boilers are less expensive than gas or oil boilers.
Which of the above statements are correct?
(A) 1 and 2 only (B) 1 and 3 only
(C) 2 and 3 only (D) 1, 2 and 3
331. Consider the following statements regarding solar energy:
1. To encourage the adoption of solar energy production, many State Governments and the
Centre have announced plans by way of buy back as well as subsidies for installation.
2. Land acquisition of several hectares is a bottleneck in implementing this programme.
3. Considerable R & D effort is needed to bring down the cost of P-V cells.
Which of the above statements are correct?
(A) 1 land 2 only (B) 1 land 3 only
(C) 1,2 and 3 (D) 2 and 3 only
332. Which one of the following statements is correct?
(A) The end product of fossil fuels is in the form of electrical energy.
(B) Watershed protection increases the rate of surface runoff of water.
(C) If timber is overharvested, the ecological functions of the forest are improved.
(D) Rivers change their course during floods and lots of fertile soils are lost to the sea.
333. Statement (I): Increased cloud cover will lead to warmer winters due to clouds reflecting more
intense solar energy.
Statement (II): Overcast cloud conditions result in decrease in the day-night temperature
difference.
(A) Both Statement (I) and Statement (II) are individually true and Statement (II) is the
correct explanation of Statement (I)
(B) Both Statement (I) and Statement (II) are individually true but Statement (II) is not the
correct explanation of Statement (I)
(C) Statement (I) is true but Statement (II) is false
(D) Statement (I) is false but Statement (II) is true
334. Consider the following statements regarding Insolation:
1. It is the solar radiation that reaches the Earth’s surface.
2. It is measured by the amount of solar energy received per square centimeter per minute.
3. It is the amount of solar energy absorbed by the stratosphere.
Which of the above statements are correct?
(A) 1, 2 and 3 (B) 1 and 2 only
(C) 1 and 3 only (D) 2 and 3 only
335. Statement (I): Green energy refers to one which does not harm the ecosystem of planet Earth.
Statement (II): All renewable energy is green energy.
(A) Both Statement (I) and Statement (II) are individually true and Statement (II) is the correct
explanation of statement (I)
(B) Both Statement (I) and Statement (II) are individually true but Statement (II) is not correct
explanation of Statement (I)
(C) Statement (I) is true but Statement (II) is false
(D) Statement (I) is false but Statement (II) is true
336. What are the limitations of solar energy?
1. Collecting solar energy over large areas and converting it to other forms that can be
convenient!, transported, stored and used in existing equipment is not economical.
2. Low density of solar energy as compared to coal, oil and gas.
3. Its major applications are photothermal conversion, solar water heating, green housing
technology’ and photo voltaic conversion.
Select the correct answer using the codes given below:
(A) 1, 2 and 3 (B) 1 and 2 only
(C) 1 and 3 only (D) 2 and 3 only
337. What are the advantages of Biomass as a source of energy?
1. Its storage and transportation is possible
2. It is ecologically safe and is inoffensive
3. Can be developed with present man and material abilities
4. Low capital input required
Select the correct answer using the codes given below:
(A) 1, 2, 3 and 4 (B) 1, 2 and 3 only
(C) 1, 3 and 4 only (D) 2, 3 and 4 only
338. Hydrogen fuel cell vehicles produce which of the following as exhaust?
(A) NH3 (B) CH4
(C) H2O (D) H2O2
339. With reference to technologies for solar power production, consider the following statements:
1. Photovoltaics is a technology that generates electricity by direct conversion of light
into electricity.
2 Photovoltaics generates alternating current (AC), while Solar Thermal generates direct
current (DC).
3. India has manufacturing base for Solar Thermal technology, but not for Photovoltaics.
Which of the statements given above is/are correct?
(A) 1 only (B) 2 and 3 only
(C) 1, 2 and 3 (D) None of these
340. Consider the following statements regarding ‘biogas’:
1. This is a mixture of different gases which is formed after breakdown of an organic
compound.
2. This is produced by decomposition by fungi and algae in an open system where
sufficient oxygen supply is available.
3. This gas can be also compressed same like CNG and used for energy needs.
Select the correct answer using the codes given below:
(A) 1 and 2 only (B) 2 and 3 only
(C) 1 and 3 only (D) All of the above
341. Both power and manure are provided by
(A) nuclear plants (B) biomass plants
(C) tidal plants (D) hydroelectric plants
342. Consider the following statements regarding solar panel:
1. It has high maintenance cost.
2. It has no moving parts.
3. It provides pollution free, noiseless energy.
Which of the above statements are correct?
(A) 1 and 2 only (B) 2 and 3 only
(C) 1 and 3 only (D) 1, 2 and 3
343. The value of Solar Constant is
(A) 140 Wm-2 (B) 1.4 Wm-2
(C) 1.4 kWm-2 (D) 1.4 MWm-2
344. The main constituent of CNG is
(A) Methane (B) Butane
(C) Ethane (D) Propane+Butane
345. Consider the following statements regarding REN 21:
1. REN 21 is the global multi-stakeholder network to assist governments and other
institutions regarding renewable energy policy.
2. It is an international non-profit association and headquartered in Nairobi, Kenya.
Which of the above statements is/are correct?
(A) 1 only (B) 2 only
(C) Both 1 and 2 (D) Neither 1 nor 2
346. Consider the following statements regarding renewable energy resources:
1. These are the sources which can be renewed by nature again and again.
2. These are the sources whose supply isn’t affected by rate of their consumption.
3. These sources are exhaustible
Which of the statements given above are correct?
(A) 1 and 2 only (B) 2 and 3 only
(C) 1 and 3 only (D) All of the above
347. Consider the following statements regarding hydro power:
1. An impoundment facility of hydro power plant generally uses a dam to store water in a
reservoir.
2. Run-of-river facility of hydro power plant captures the kinetic energy in rivers or
streams, without a large reservoir and sometimes without the use of dams.
3. Small hydro power (SHP) plant is defined as any hydro power project which has an
installed capacity of less than 50 MW.
Which of the above statements are correct?
(A) 1 and 2 only (B) 2 and 3 only
(C) 1 and 3 only (D) 1, 2 and 3

348. Consider the following statements regarding ITER:


1. ITER is one of the most ambitious energy projects based on the nuclear fission and
located in France.
2. ITER is a joint effort of many parties including China and India.
Which of the above statements is/are correct?
(A) 1 only (B) 2 only
(C) Both 1 and 2 (D) Neither 1 nor 2
349. Consider the following statements regarding Tidal power:
1. Tides are created by movement of wind on the surface of sea.
2. Tidal power has traditionally suffered from relatively high cost and limited availability
of sites.
Which of the above statement(s) is/are correct?
(A) 1 only (B) 2 only
(C) Both 1and 2 (D) Neither 1 nor 2
350. Consider the following statements regarding geothermal energy:
1. The slow decay of radioactive particles in the earth’s core produces geothermal energy.
2. Geothermal energy is relatively expensive with respect to other sources of renewable
energy.
3. The carbon footprint of a geothermal power plant is minimal.
Which of the above statements is/are correct?
(A) 1 only (B) 2 and 3 only
(C) 1 and 3 only (D) 1, 2 and 3
351. Consider the following statements:
1. Shale gas is colorless, odourless and lighter than air.
2. Shale gas is unconventional source of methane, just like coal-bed gas and tight gas
3. Shale gas is used in Europe but not in USA.
Which of the above statements is/are correct?
(A) 1 only (B) 1 and 2 only
(C) 1 and 3 only (D) 1, 2 and 3
352. Consider the following statements regarding National Solar Mission:
1. National Solar Mission is one of the eight key national missions which comprise
National Action Plan on Climate Change.
2. Government has revised the target of grid connected solar power projects from earlier
20,000 MW to 100,000 MW by the year 2022.
Which of the above statements is/are correct?
(A) 1 only (B) 2 only
(C) Both 1and 2 (D) Neither 1 nor 2
353. Consider the following statements regarding International Solar Alliance (ISA):
1. It is the first treaty-based international organisation which is headquartered in India.
2. It was launched at the Conference of Parties (CoP21) in Paris, France.
3. Its most of the members are the countries lying on or within the tropics.
Which of the above statements are correct?
(A) 1 and 2 only (B) 2 and 3 only
(C) 1 and 3 only (D) 1, 2 and 3
354. Consider the following statements:
1. Thorium cannot be used directly to produce nuclear energy.
2. Thorium does not contain any fissile isotope.
Which of the above statements is/are correct?
(A) 1 only (B) 2 only
(C) Both 1and 2 (D) Neither 1 nor 2
355. Which one of the following is the major characteristic of deciduous trees?
(A) They do not lose their leaves.
(B) They shed their leaves annually
(C) They synthesize their own food.
(D) They depend on other factors for their food.
356. An association of two organisms of different species for mutual benefit, and where the
individuals may not be able to survive separately, is called
(A) Commensalism (B) Parasitic
(C) Non-symbiotic (D) Symbiotic
357. Which one of the following is a terrestrial type of ecosystem?
(A) Limnetic (B) Estuary
(C) Prairie (D) Reefs
358. Which of the following are reasons for decliningcoral reefs around the world?
1. Rise in sea temperature
2. Ocean acidification
3. Overfishing
4. Pollution from land
Select the correct answer using the codes given below:
(A) 1, 2 and 3 only (B) 2, 3 and 4 only
(C) 1, 2 and 4 only (D) All of the above
359. Consider the following statements regarding Wetlands International:
1. It is one of the party in implementation ofRamsar Convention.
2. It is a inter-governmental body which strives to protect wetlands and preserve their
richness.
3. Wetlands International’s work ranges from research and community-based field projects
to advocacy and engagement with governments, corporate and international conventions.
Select the correct answer using the codes given below:
(A) 1 and 3 only (B) 1 and 2 only
(C) 2 and 3 only (D) All of the above
360. With reference to Wetlands, consider the following statements:
1. The Convention on Wetlands, signed in Ramsar, Iran in 1971, is an intergovernmental
treaty which provides the framework for national action and international cooperation
for the conservation and wise use of wetlands and their resources.
2. 2nd February of each year is observed as World Wetlands Day (WWD).
3. Indo-Gangetic wetlands are the largest wetland system in India, extending from the
river Indus in the west to Brahmaputra in the east.
Which of the above statements are correct?
(A) 1 and 2 only (B) 2 and 3 only
(C) 1and 3 only (D) All of the above
361. Consider the following statements regardingMangroves:
1. Mangroves have capacity to tolerate excessive salt content and regular water logging
condition.
2. Mangrove forests stabilize the coastline, reduce erosion from storms, waves, tides, etc.
3. Sundari tree is an example of Mangroves.
Which of the above statements are correct?
(A) 1and 2 only (B) 2 and 3 only
(C) 1 and 3 only (D) 1, 2 and 3
362. With reference to ecosystems, consider the following statements:
1. Organisms at higher trophic level receive energy from more than one trophic level.
2. Energy tends to decrease at higher trophic levels.
Which of the statements given above is/are correct?
(A) 1 only (B) 2 only
(C) Both 1 and 2 (D) Neither 12 nor 2
363. With reference to Earth Overshoot Day, considerthe following statements:
1. It is an illustrative date observed every year on which the consumption of Earth’s
resources for the year exceeds Earth’s capacity to regenerate those resources.
2. It is also known as Ecological Debt Day.
3. In the year 2017, it was observed on August 2.
Which of the above statements is/are correct?
(A) 1 only (B) 1 and 2 only
(C) 2 and 3 only (D) 1, 2 and 3
364. Consider the following statements regarding ecological niche:
1. Ecological niche is a description of all the q g biological, physical and chemical factors
that a species needs to survive, stay healthy and reproduce.
2. Two species may or may not have exact identical niche.
3. It plays an important role in conservation of organisms.
Which of the above statements are correct?
(A) 1 and 2 only (B) 2 and 3 only
(C) 1 and 3 only (D) 1, 2, and 3
365. Consider the following statements regarding food web:
1. A food web illustrates all possible transf6r of energy and nutrients among organisms in
an ecosystem.
2. The presence of complex food webs increases the stability of the ecosystem
Which of the above statements is/are correct?
(A) 1 only (B) 2 only
(C) Both 1 and 2 (D) Neither 1 nor 2
366. Pyramid of energy in a pond ecosystem is always
(A) inverted (B) upright
(C) irregular (D) linear
367. With reference to Bombay Natural History Society (BNHS), consider the following statements;
1. It is an autonomous organization under Ministry of Environment, Forests and Climate
Change.
2. It strives to conserve nature through action based research, education and public
awareness.
Which of the above statements is/are correct?
(A) 1 only (B) 2 only
(C) Both 1 and 2 (D) Neither 1 nor 2
368. The transition area between two biomes is known as
(A) Ecotone (B) Ecological niche
(C) Biosphere (D) Succession

369. Consider the following statements:


1. Autotrophs are those organisms which obtain their food by eating plants or other
animals or their products.
2. Heterotrophs are those plants which synthesize their food themselves.
Which of the statements given above is/are correct?
(A) 1 only (B) 2 only
(C) Both 1 and 2 (D) Neither 1nor 2
370. Consider the following statements regarding ecocline:
1. Ecocline describes an ecotone in which a series of bio communities display a
continuous gradient.
2. Ecocline refers variation in physio-chemical environment of ecotone.
Which of the above statements is/are correct?
(A) 1 only (B) 2 only
(C) Both 1 and 2 (D) Neither 1 nor 2
371. Consider the following statements:
1. Pyramid of Energy is always upright irrespective of type of ecosystem.
2. Pyramid of Number for both the grassland and forest ecosystems are similar.
Select the correct answer using the codes given below:
(A) 1 only (B) 2 only
(C) Both 1 and 2 (D) Neither 1 nor 2
372. If a wetland of international importance Is brought under the ‘Montreux Record’, what does it
imply?
(A) Changes in ecological character have occurred, are occurring or are likely Io occur in
the wetland as a result of human interference.
(B) The country in which the wetland is located should enact a law to prohibit any human
activity within five kilo meters from the edge of the wetland
(C) The survival of the wetland depends on the cultural practices and traditions of certain
communities living in its vicinity and therefore the cultural diversity therein should not
be destroyed
(D) It is given the status of ‘World Heritage Site’
373. Consider the following statements.
1. Bioremediation is the phenomena in which living organism are used for solving any
degradation m environment.
2 The leguminous crops like oilseeds, pulses etc., are used to replenish N2 content of soil,
which in o/limr.md duo to mono cropping.
3. The rhizobium bacteria in roots of leguminous plants re-fixes N2 in soil.
Which of the above statements are corrects?
(A) 1 and 2 only (B) 2 and 3 only
(C) 1 and 3 only (D) All of the above
374. Consider the following statements regardEarth Hour:
1. It is an initiative by World Wide Fund Nature (WWF).
2. It is a movement to raise the awareness about the climate change and need to save the
Earth.
Which of the above statements is/are correct?
(A) 1 only (B) 2 only
(C) Both 1 and 2 (D) Neither 1 nor 2
375. Consider the following statements regarding biomagnification:
1. It is the phenomenon of increasing concentration of toxic chemicals in the tissues of
organism through food chain.
2. The concentration of toxic compound increases with decreasing the tropic level.
3. This process is also known as bioamplification.
Which of the above statements is/are correct?
(A) 1 only (B) 2 and 3 only
(C) 1 and 3 only (D) 1, 2 and 3 only
376. Eutrophication is a process in which a water body becomes rich in nutrients as well as deprive o f
oxygen.
Which of the following nutrients primarily cause it?
1. Nitrates
2. Sulphates
3. Phosphates
Select the correct answer using the codes given below:
(A) 1 and 2 only (B) 2 and 3 only
(C) 1 and 3 only (D) 1, 2 and 3
377. Consider the following statements
1. Energy flow through the trophic levels from producers to subsequent trophic level is bi-
directional.
2. Energy level increases from the first trophic level upwards due to addition of energy in
the form of heat at each trophic level.
Which of the above statements is/are correct?
(A) 1 only (B) 2 only
(C) Both 1 and 2 (D) Neither 1 nor 2
378. Which one of the following is the correct sequence of ecosystems in the order of decreasing
productivity?
(A) Oceans, lakes, grasslands, mangroves (B) Mangroves, oceans, grasslands, lakes
(C) Mangroves, grasslands, lakes, oceans (D) Oceans, mangroves, lakes, grasslands
379. What is correct order of size of ecological niche of different consumers?
(A) Herbivores < Carnivores < Omnivores (B) Herbivores < Omnivores < Carnivores
(C) Omnivores < Carnivores < Herbivores (D) Omnivores < Herbivores < Carnivores
380. Which of the following statements is/are correct?
(A) International Day for Biodiversity is sanctioned by United Nations and observed on
May 22.
(B) World Wetland Day occurs on day of signing of convention on wetlands, called
Ramsar Convention, on February 2, every year.
(C) International Day for Preservation of Ozone layer is observed on September 16, on the
date of signing of Montreal Protocol on Substances that deplete ozone layer.
(D) All of the above
381. Consider the following statements:
1. Aphotic zone is the upper layer of the aquatic ecosystems, up to which light penetrates
and within which photosynthetic activity is confined.
2. With the increase in the temperature of a water body, the rate at which oxygen is
depleted from water also increases.
Which of the above statements is/are correct?
(A) 1 only (B) 2 only
(C) Both 1 and 2 (D) Neither 1 nor 2
382. Besides species diversity and ecosystem diversity, which of the following is included in the term
‘biodiversity’?
(A) Genetic diversity (B) Climatic diversity
(C) Cultural diversity (D) Lingual diversity
383. Consider the following statements regarding structure and design of biosphere reserves:
1. Biosphere reserve is divided into two interrelated zones namely core and buffer.
2. Core area is kept absolutely undisturbed and also contains exceptional scientific interest
as it may contain endemic species.
3. Buffer area is a outermost zone usually not delimited and all kinds of activities which
are not harmful to environment are allowed.
Select the correct statement(s) using the codes given below:
(A) 1 and 2 only (B) 1 only
(C) 2 and 3 only (D) 2 only
384. Consider the following statements regarding TRAFFIC - a wildlife trade monitoring network: 1.
It has objective to ensure that trade in wild plants and animals is not a threat to the
conservation of nature.
2. It is a joint conservation programme of World Wildlife Fund (WWF) and International
Union for Conservation of Nature (IUCN).
Which of the above statements is/are correct?
(A) 1 only (B) 2 only
(C) Both 1 and 2 (D) Neither 1nor 2
385. Consider the following statements:
1. Buffer zone of biosphere reserve is an area of active cooperation between the
management and the local people.
2. Core zone of biosphere consists of an undisturbed and legally protected ecosystem.
Which of the statements given above is/are correct?
(A) 1 only (B) 2 only
(C) Both 1 and 2 (D) Neither 1 nor 2
386. Consider the following statements:
1. Conserving the animals and plants in their natural habitat is known as ex-situ
conservation.
2. Seed banks, botanical gardens, zoological parks are examples of ex-situ conservation.
Which of the above statements is/are correct?
(A) 1 only (B) 2 only
(C) Both 1 and 2 (D) Neither 1 nor 2
387. With reference to an organization known as ‘Birdlife International’ which of the following
statements is/are correct?
1. It is a Global Partnership of Conservation Organizations.
2. The concept of ‘biodiversity hotspots’ originated from this organization.
3. It identifies the sites known/referred to as ‘Important Bird and Biodiversity Areas’.
Select the correct answer using the code given below:
(A) 1 only (B) 2 and 3 only
(C) 1 and 3 only (D) 1, 2 and 3
388. Consider the following statements:
1. Biodiversity hotspots are located only in tropical regions.
2. India has four biodiversity hotspots i.e. Eastern Himalayas, Western Himalayas,
Western Ghats and Andaman and Nicobar Islands.
Which of the statement(s) given above is/are correct?
(A) 1 only (B) 2 only
(C) Both 1 and 2 (D) Neither 1 nor 2
389. MIDORI Prize is related to which of the following fields?
(A) Genetics resources (B) Pollution control measures
(C) Biodiversity (D) Biosafety
390. With reference to National Green Tribunal, consider the following statements
1. The Tribunal has been established for the effective and expeditious disposal of cases
relating to environmental protection and conservation of forests.
2. No civil court shall have jurisdiction to entertain any appeal in respect of any matter
which the Tribunal is empowered to determine under its appellate jurisdiction.
3. At present only Delhi and Bhopal are the places of sitting for the Tribunal.
Which of the statements given above is/are correct?
(A) 1 and 2 only (B) 2 and 3 only
(C) 1 and 3 only (D) All of the above
391. The ‘Red Data Book’ published by the International Union for Conservation of Nature ( I U C N )
contains lists of
1. Endemic plant and animal species present in the biodiversity hotspots.
2. Threatened plant and animal species.
3. Protected sites for conservation of nature and natural resources in various countries.
Select the correct answer using the codes given below:
(A) 1 and 3 only (B) 2 only
(C) 2 and 3 only (D) 3 only
392. With reference to the Convention on Biological Diversity (CBD), consider the following
statements:
1. The governing body of the CBD is the Conference of the Parties (COP).
2. The secretariat of the CBD is based in Montreal, Canada.
Which of the statements given above is/are correct?
(A) 1 only (B) 2 only
(C) Both 1 and 2 (D) Neither 1 nor 2
393. Which of the following can be done as per Cartagena Protocol of Biosafety:
1. Banning of imports of genetically modified (GM) crop seeds.
2. Labeling of GM related products.
3. Scrapping of Intellectual property right of GM crop and opening up for more
transparent research till safety is proved.
Select the correct answer using the codes given below:
(A) 1 and 2 only (B) 2 only
(C) 1 and 3 only (D) All of the above
394. Consider the following tertiary treatment methods for treatment of waste water.
1. Ion-exchange method 2.Reverse osmosis
3. Chemical oxidation method 4. Activated sludge process
Which of the above methods are correct?
(A) 1, 2 and 4 (B) 1, 3 and 4
(C) 2, 3 and 4 (D) 1, 2 and 3
395. Which industrial waste is commonly used in construction industry?
(A) Fly ash (B) Slag
(C) Sludge (D) Red oxide
396. Consider the following statements concerning environmental pollution:
1. Nuclear explosions cause radioactive radiation.
2. Earthquakes do not cause Tsunamis.
3. Acid rain is not a major environmental issue.
4. Air pollution has some impact on meteorology.
Which of the above statements are correct?
(A) 1 and 2 (B) 2 and 3
(C) 3 and 4 (D) 1 and 4
397. Consider the following statements regarding SMOG:
1. SMOG ‘was coined during the 1950’s to describe a mixture of smoke and fog
experienced in London.
2. The principal pollutants in London SMOG are particulates and sulphur compounds.
3. The London SMOG occurs generally early in the morning on cold wet winter days.
(A) 1, 2 and 3 (B) 1and 2 only
(C) 1 and 3 only (D) 2 and 3 only
398. The Rotterdam Convention deals with
(A) Reducing nuclear weapon stockpiles (B) Limiting the use of toxic chemicals
(C) Protecting the oceans (D) Banning of human clone experiment
399. Sanitary/municipal fills and waste heaps are unavoidably hazardous due to
1. Leachates
2. Emanating gases
3. Rodents and wandering animals
4. Automobile workshops that seems to have an affinity for such neighbourhoods.
(A) 1 and 4 only (B) 1 and 2 only
(C) 2 and 3 only (D) 3 and 4 only
400. Increased biological oxygen demand is an indication of
1. Low microbial contamination.
2. Absence of microbial pollution.
3. High level of microbial contamination
Which of the above statements is/are correct?
(A) 1 only (B) 2 only
(C) 3 only (D) 1, 2 and 3
401. Statement (I): Volcanic eruptions is often accompanied by earthquakes.
Statement (II): Volcanoes erupt dust particles in the atmosphere.
(A) Both Statement (I) and Statement (II) are individually true and Statement (II) is the
correct explanation of Statement (I)
(B) Both Statement (I) and Statement (II) are individually true but Statement (II) is not the
correct explanation of statement (I)
(C) Statement (I) is true but Statement (II) is false
(D) Statement (I) is false but Statement (II) is true
402. Statement (I): Normally carbon dioxide is not considered an air pollutant.
Statement (II): Carbon dioxide is a constituent of atmospheric air.
(A) Both Statement (I) and Statement (II) are individually true and Statement (II) is the
correct explanation of Statement (I)
(B) Both Statement (I) and Statement (II) are individually true but Statement (II) is not the
correct explanation of Statement (I)
(C) Statement (I) is true but Statement (II) is false
(D) Statement (I) is false but Statement (II) is true
403. Venturi scrubber, a device used to remove particulate matter from the atmosphere, works on t h e
principle of
(A) Settling by gravitational force (B) Removal by centrifugal force
(C) Removal by electrically charged particles (D) Removal by atomized water vapour
404. Acid rain results when gaseous emissions of Sulfuric oxides (SOX) and nitrogen oxides (NOX)
interact with water vapour and
(A) Moonlight, and are chemically converted to strong acidic compounds such as sulphuric
acid (H2SO4) and nitric acid (HNO3)
(B) Sunlight, and are chemically converted to strong acidic compounds such as sulfuric acid
(H2SO4) and nitric acid (HNO3)
(C) Moonlight, and are chemically converted to weak acidic compounds such as sulfuric
acid (H2SO4) and nitric acid (HNO3)
(D) Sunlight, and are chemically converted to week acidic compounds such as sulfuric acid
(H2SO4) and nitric acid (HNO3)
405. The ‘Minamata Tragedy’ was caused by the eating of fish growing in the Minamata Bay
contaminated with
(A) Peroxyalylnitrate (B) Methyl isocyanate
(C) Potassium cyanide (D) Methylmercury
406. BOD of a waste water sample is estimated to be 180mg//. Assuming 4 mg// BOD can be
consumed in the BOD bottle, the volume of undiluted sample to be added to a 300 ml bottle is
nearly
(A) 6.7 ml (B) 5.6 ml
(C) 4.4 ml (D) 3.3 ml
407. Consider the following statements regarding Biomagnification:
1. Biomagnification is increase in concentration of metals in successive tropics levels
which will lead to drastic negative effects.
2. Biomagnification happens with metals with density less than 5 gm/cm3 which are
medium density metals.
3. DDT which biomagnifies and disturb calcium metabolism in birds and results in
premature breaking of eggs.
Select the correct answer using the codes given below.
(A) 1 and 2 only (B) 2 and 3 only
(C) 1 and 3 only (D) All of the above
408. Excessive release of the pollutant carbon monoxide (CO) into the air may produce a condition i n
which oxygen supply in the human body decreases. What causes this condition?
(A) When inhaled into the human body, CO is converted into carbon dioxide.
(B) The inhaled CO has much higher affinity for haemoglobin as compared to oxygen.
(C) The inhaled CO destroys the chemical structure of haemoglobin.
(D) The inhaled CO adversely affects the respiratory centre in the brain.
409. Consider the followingstatements regarding fly ash:
1. Fly ash is one such residue produces whenever a combustion of solid material takes
place.
2. Coal based thermalpower plants are majoremitter of fly ash.
3. Fly ash is collected by passing hot steam before the flue gases goes into chimney.
Select the correct answer using the codes given below.
(A) 1 and 2 only (B) 2 and 3 only
(C) 1 and 3 only (D) All of the above
410. Consider the following statements regarding effects because of particular air pollutant:
1. Carbon monoxide will result into difficulty in breathing problem, severe headache,
unconsciousness and even death.
2. Ozone will result into nervous disorder.
3. Mercury will result in Minamata disease.
Select the correct answer using the codes given below:
(A) 1 and 2 only (B) 2 and 3 only
(C) 1 and 3 only (D) All of the above
411. Consider the following statements regarding chemical oxygen demand:
1. It is amount of oxygen needed for chemical reactions of inorganic matter in water.
2 It is slightly better mode of pollution measure in water rather than biological oxygen
demand.
Select the correct answer using the codes given below:
(A) 1 only (B) 2 only
(C) Both 1 and 2 (D) Neither 1 nor 2
412. Consider the following statements regarding “oil zapper”:
1. This is a bioremediation technique whichis developed by TERI.
2. It is a mixture of bacteria which degrades pollutants in oil contaminated sites leaving
behind no harmful substances.
3. The production of oil zapper is very costly which makes this process economically
enviable.
Select the correct answer using the codes given below.
(A) 1 and 2 only (B) 2 and 3 only
(C) 1 and 3 only (D) All of the abvoe
413. Consider the following statements regardingTetraethyl lead (TEL):
1. It works as a antiknock agent and increases the efficiency of a vehicle.
2. This also complements catalytic convertor in vehicles in collecting particulate
pollutants.
3. Because of toxic nature, it is now banned in most of the countries and replaced with
other materials.
Select the correct answer using the codes given below.
(A) 1 and 2 only (B) 1and 3 only
(C) 2 and 3 only (D) All of the above
414. The National Air Quality Monitoring Programme (NAMP) run by Central Pollution Control
Board, monitors which of the following pollutants at all locations?
1. carbon dioxide
2. sulphur dioxide
3. oxides of nitrogen
4. suspended particulate matter
Select the correct answer using the codes given below.
(A) 1, 2 and 3 only (B) 1, 3 and 4 only
(C) 2, 3 and 4 only (D) 1, 2 and 4 only
415. Formation of algal bloom in waterbody is due to the
(A) enrichment of plant diversity
(B) enrichment of animal diversity
(C) enrichment of hazardous pollutants
(D) enrichment of nutrients
416. Consider the following statements regarding Basel Convention:
1. It is a treaty on the control of trans-boundary movements of hazardous wastes and their
disposal.
2. It does not address the movement of radioactive wastes.
Which of the above statements is/are correct?
(A) 1 only (B) 2 only
(C) Both 1and 2 (D) Neither 1 nor 2
417. Which of the following is the most dangerous and long-lasting?
(A) Biomedical waste (B) Ash from volcano
(C) Mining waste (D) Nuclear waste
418. Consider the following statements regarding catalytic convertors:
1 Catalytic convertors used in vehicles. They have platinum-palladium and Rhodium as
catalyst.
2. Unleaded petrol should be used in vehicles because it will inactivate the catalyst.
Select the correct answer using the codes given below:
(A) 1 only (B) 2 only
(C) Both 1 and 2 (D) Neither 1 nor 2
419. Pollutants can be classified as primary and secondary. Which of these is/are primary pollutant?
1. Volatile Organic Compounds
2. Ground Level Ozone
3. Acid Rain
Select the correct answer using the codes given below.
(A) 1 only (B) 1 and 2 only
(C) 1 and 3 only (D) 1, 2 and 3
420. With reference to bio-toilets used by the Indian Railways, consider the following statements:
1. The decomposition of human waste in the bio-toilets is initiated by a fungal inoculum.
2. Ammonia and water vapour are the only end products in this decomposition which are
released into the atmosphere.
Which of the statements given above is/are correct?
(A) 1 only (B) 2 only
(C) Both 1 and 2 (D) Neither 1 nor 2

421. Consider the following statements regarding persistent organic compounds.


1. These are organic compounds which are resistant to degradation.
2. They are main causes for bioaccumulation.
3. Stockholm Convention 2001 is signed to eliminate and restrict the production and use
of these.
Select the correct answer using the codes given below:
(A) 1 and 2 only (B) 2 and 3 only
(C) 1 and 3 only (D) All of the above
422. Consider the following statements regarding National Air Quality Index (NAQI):
1. It was launched by Union Government in October, 2014.
2. NAQI uses color coded table to show air quality and its effect on health.
3. Green color in NAQI table indicates moderately polluted air which may cause breathing
discomfort to Asthma patient, elderly and children.
Which of the above statements are correct?
(A) 1 and 2 only (B) 2 and 3 only
(C) 1and 3 only (D) All of the above
423. Consider the following statements regarding Biological Oxygen Demand (BOD):
1. It is expressed in milligrams of oxygen per litre of water.
2. Microbes in sewage treatment plants increases the BOD of the effluent.
Which of the above statements is/are correct?
(A) 1 only (B) 2 only
(C) Both 1 and 2 (D) Neither 1 nor 2
424. Consider the following statements regarding photochemical smog:
1. Photochemical smog occurs in a cool and humid climate
2. Presence of high levels of nitric oxide is essential for formation of photochemical smog
3. Photochemical smog is chemically an oxidizing mixture.
Which of the following statements are correct?
(A) 1 and 2 only (B) 2 and 3 only
(C) 1 and 3 only (D) 1, 2 and 3
425. In context of environment, the term ‘dirty dozen’ refers to which of the followings?
(A) 12 Radioactive elements (B) 12 most harmful greenhouse gases
(C) 12 persistent organic pollutants (D) 12 ozone depleting substances
426. Air freshener which contains phthalates have following adverse effects on human:
1. It suppresses androgen and causes asthma problem.
2. It retards development of children.
3. It is carcinogenic and also affects Central Nervous System (CNS).
Which of the above statements are correct?
(A) 1 and 2 only (B) 2 and 3 only
(C) 1 and 3 only (D) All of the above
427. Consider the following statements:
1. Smog formed in the area of intense solar radiation is called Grey air.
2. Smog formed in the area of lesser solar radiation is called Brown air.
Which of the above statements is/are correct?
(A) 1 only (B) 2 only
(C) Both 1 and 2 (D) Neither 1 nor 2
428. Consider the following statements:
1. Black carbon is a form of particulate pollutant produced from incomplete combustion.
2. Black carbon is emitted by high temperature combustion process whereas Brown
carbon by low temperature combustion.
Which of the above statements is/are correct?
(A) 1 only (B) 2 only
(C) Both 1 and 2 (D) Neither 1 nor 2
429. Consider the following statements regarding Minamata Convention, 2013:
1. The convention has prohibited a myriad of products containing mercury, and their
production and trade will be altogether prohibited by 2020.
2. This also prohibited vaccines containing mercury and use of mercury in dental fillings.
3. The biggest mercury release comes from coal-fired power stations and usage of
mercury’ to separate gold from ore-bearing rock.
Select the correct answer using the codes given below.
(A) 1 and 2 only (B) 2 and 3 only
(C) 1 and 3 only (D) 1, 2 and 3
430. The Rotterdam Convention is related to which of the following?
(A) Control of persistent organic pollutants (POPs)
(B) Control of trans-boundary movements of hazardous wastes and their disposal
(C) Control of pesticides and industrial chemicals
(D) Protection of ozone layer from depletion
431. Statement (I): Greenhouse gases increase the earth’s surface temperature.
Statement (II): Greenhouse gases help to retain the incoming heat energy from the sun.
(A) Both Statement (I) and Statement (II) are individually true; and Statement (II) is
the correct explanation of Statement (I)
(B) Both Statement (I) and Statement (II) are individually true; but Statement (II) is
NOT the correct explanation of Statement (I)
(C) Statement (I) is true; but Statement (II) is false
(D) Statement (I) is false; but Statement (II) is true
432. The UN Conference on Environment and Development, known as Rio Summit, led to the fi n a l
development of which international treaty?
(A) Kyoto Protocol (B) World Environment Treaty
(C) Environment Treaty of the G8 Nations (D) Universal Environment Treaty
433. Consider the following statements regarding ozone:
1. It is continuously being produced and destroyed.
2. It helps to sustain life on earth.
3. It is formed in the atmosphere though photochemical reaction.
Which of above statements are correct?
(A) 1 and 2 only (B) 1 and 3only
(C) 2 and 3 only (D) 1, 2 and 3
434. Statement (I): Global warming is the cause for climate change.
Statement (II): Ozone depletion will cause global warming.
(A) Both Statement (I) and Statement (II) are individually true and Statement (II)
is the correct explanation of Statement (I)
(B) Both Statement (I) and Statement (II) are individually true but Statement (II) is
not the correct explanation of Statement (I)
(C) Statement (I) true but Statement (II) is false
(D) Statement (I) false but Statement (II) is true
435. The meaning of ‘Carbon Footprint’ is described by the amount of
(A) Carbon dioxide released into the atmosphere as a result of the activities of a
particular individual, organization or community.
(B) Greenhouse gases emitted by industries contributing to global warming.
(C) Carbon emission released by the burning of jet fuel.
(D) increase in the carbon content of the atmosphere due to the felling of trees.
436. Consider the following statements regarding depletion of the ozone layer:
1. Excessive release of chlorine and bromine in the environment from man-made
compounds, such as chlorofluorocarbons.
2. Occurrence of certain natural phenomena such as sunspots, and stratospheric
winds.
3. Degradation of materials by ultra-violet radiation.
4. Major volcanic eruptions.
Which of the above can be categorized as causing ozone depletion?
(A) 1, 2 and 3 only (B) 1, 3 and 4 only
(C) 1, 2 and 4 only (D) 2, 3 and 4 only
437. Statement (I): A greenhouse gas is any gas in the atmosphere which absorbs and re- emits heat
and thereby keeps the plant’s atmosphere warmer than it otherwise would be .
Statement (II): In the Earth’s atmosphere, water vapour is one of the main greenhouse
gasses.
(A) Both Statement (I) and Statement (II) are individually true and Statement (II) is
the correct explanation of Statement (I)
(B) Both Statement (I) and Statement (II) are individually true but Statement (II) is
not the correct explanation of Statement (I)
(C) Statement (I) is true but Statement (II) is false
(D) Statement (I) is false but Statement (II) is true
438. Consider the following statements with regard to atmospheric humidity.
1. Absolute humidity is the amount of water vapour per unit volume.
2. Hygrometer is used to measure relative humidity.
3. Dew point is the temperature at which the relative humidity is 75 %.
Which is the above statements are correct?
(A) 1 and 2 only (B) 1 and 3 only
(C) 2 and 3 only (D) 1, 2 and 3
439. Ozone layer present in the atmosphere protects life on earth by not permitting harmful
radiations present in the sunlight to penetrate through it. Ozone layer is formed by the
reaction of
(A) Chlorofluorocarbons (CFCs) on oxygen (O2)
(B) Chlorine (CI) on oxygen (O2)
(C) Solar Ultraviolet rays on oxygen (O2)
(D) Chlorine nitrate (CINO3) on oxygen (O2)
440. Which of the following are sources of methane?
1. bacteria in wetlands
2. digestion of live-stock
3. Production of natural gas
Select the correct answer using the code given below:
(A) 1 and 2 only (B) 2 and 3 only
(C) 1 and 3 only (D) All of the above
441. Consider the following regarding various protocols of environmental protection:
1. Hydroflorocarbons should be phased out by all countries as per Montreal
Protocol.
2. Kyoto Protocol is a legally binding agreement on all developed countries but not
on developing countries.
Select the correct answer using the code given below:
(A) 1 only (B) 2 only
(C) Both 1 and 2 (D) Neither 1 nor 2
442. Consider the following statements:
1. Global Warming Potential (GWP) of a gas is the measure of the total energy that
a gas absorbs over a particular period of time, compared to methane.
2. Among Greenhouse Gases, Hydrofluorocarbons (HFCs) have maximum global
warming potential.
Which of the above statements is/are correct?
(A) 1 only (B) 2 only
(C) Both 1 and 2 (D) Neither 1 nor 2
443. Selling of carbon by the developing industries is known as
(A) carbon sequestration (B) carbon dating
(C) carbon credit (D) None of the above
444. Unit of ozone measurement is
(A) ppm (B) Domg/1
(C) Dobson (D) Decibel
445. Consider the following statements:
1. Photosynthesis process adds carbon dioxide in carbon cycle.
2. The increasing amount of carbon dioxide in the air is raising the temperature of
the atmosphere as it absorbs the outgoing radiation.
Which of the above statements is/are correct?
(A) 1 only (B) 2 only
(C) Both 1 and 2 (D) Neither 1 nor 2
446. Which one of the following gases is not included in the first commitment period of the Kyoto
Protocol?
(A) Carbon tetrachloride (CCI4) (B) Hydrofluorocarbons (HFCs)
(C) Perfluorocarbons (PFCs) (D) Sulphur hexafluoride (SF6)
447. Consider the following statements:
1. Carbon sequestration describes long-term storage of carbon dioxide to mitigate or defer
global warming.
2. Carbon offsets are credits for reductions in greenhouse gas emissions made at
locations which create renewable energy.
3. Green carbon refers to coastal, aquatic and marine carbon sinks held by vegetation,
marine organisms and sediments.
Which of the above statements is/are correct?
(A) 1 only (B) 1 and 2 only
(C) 2 and 3 only (D) 1, 2 and 3
448. Kigali Amendment is related to which of the following protocols?
(A) Kyoto Protocol (B) Montreal Protocol
(C) Nagoya Protocol (D) Geneva Protocol
449. Which of the following items will be relevant to properly conduct the needful Environmental impact
Assessment at a locality for any project coming up there?
1. The lay of the land, particularly large depressions which may hold water of any
source
2. Prevailing or predominant wind directions throughout the year in the locality
3. Food habits of the local population
4. Whether sanitary fills for disposal of Municipal wastes are in the neighbourhood
5. Nearness to National Highways
6. Availability of higher educational institutions in the vicinity
Select the correct answer using the codes given below:
(A) 2, 3 and 6 (B) 1, 3 and 5
(C) 1, 2 and 4 (D) 2, 4 and 6
450. Environmental Impact Assessment (EIA) is aimed to help
(A) Estimate future needs of the society
(B) Smooth implementation of a project
(C) Cope with rapid increase in population
(D) Resource conservation
451. Statement (I): Training should be conducted among the line and low management for ensuring the
importance of environmental protection plan.
Statement (II): Environmental science is a developing subject and the people i m p l e m e n ti n g
environment strategies should remain up to date with the environmental control processes.
(A) Both Statement (I) and Statement (II) are individually true; and Statement (II) is
the correct explanation of Statement (I)
(B) Both Statement (I) and Statement (II) are individually true; but Statement (II) is
NOT the correct explanation of Statement (I)
(C) Statement (I) is true; but Statement (II) is false
(D) Statement (I) is false; but Statement (II) is true
452. Statement (I) : An EIA is a study of the probable changes in socio-economic and biophysical
characteristics of the environment that may result from a proposed action.
Statement (II): The purposes of an EIA is to help design projects, which do not disturb the
quality of the environment by examining alternatives.
453. Consider the following statements regarding Environmental Impact Assessment:
1. This is mandatory under Biological Diversity Act, 2002.
2. This acts as a tool which helps in decision making for government to access the
impact of the project.
3. All industries are required to conduct a environmental impact assessment before
taking approval from government.
Select the correct answer using the code given below:
(A) 1 only (B) 2 only
(C) 1 and 2 only (D) 2 and 3 only
454. Which of the following is not an objective of EIA?
(A) Recycling and reduction of waste
(B) Risk analysis and disaster management
(C) Assessment of international funding
(D) Sustained development with minimal environmental degradation.
455. Consider the following statements:
1. Screening is done to see whether a project requires environmental clearance as
per the statutory notifications.
2. Scoping is a process of detailing the terms of reference of EIA.
Select the correct answer using the code given below:
(A) 1 only (B) 2 only
(C) Both 1 and 2 (D) Neither 1 nor 2
456. In EIA, the baseline data describes the
(A) Demographic and socioeconomic data.
(B) Assessment of risk on the basis of proposal.
(C) Environmental consequences by mapping.
(D) Existing environmental status of the identified study area.
457. Which of the following is/are objectives of the Environmental Impact Assessment (EIA):
1. Sustainable development with minimum environmental degradation.
2. Risk analysis and disaster management.
3. Recycling and reduction of waste.
Select the correct answer using the codes given below:
(A) 1 and 2 (B) 2 only
(C) 1 and 3 only (D) All of the above
458. Consider the following statements regarding Strategic Environment Assessment:
1. It takes place at earlier stages of decision making cycle.
2. It considers limited number of feasible alternatives.
Which of the above statements is/are correct?
(A) 1 only (B) 2 only
(C) Both 1 and 2 (D) Neither 1 nor 2
459. Consider the following statements about monitoring:
1. It ensures the fulfillment of all the commitments made in the approved
Environmental Impact Assessment.
2. It keeps track of changes that may happen in environment and communities because of
the project.
Which of the above statements is/are correct?
(A) 1 only (B) 2 only
(C) Both 1 and 2 (D) Neither 1 nor 2
460. Consider the following statements:
1. Strategic Environmental Assessment is undertaken much earlier in the
decision-making process than Environmental Impact Assessment.
2. Matrix methods are used primarily for organizing information of Strategic
Environmental Assessment only.
Which of the above statements is/are correct?
(A) 1 only (B) 2 only
(C) Both 1 and 2 (D) Neither 1 nor 2
461. Arrange the following steps of Environmental Impact Assessment in the chronological order:
1. Management and monitoring
2. Screening
3. Audit
4. Prediction and mitigation
5. Scoping
Select the correct answer using the codes given below:
(A) 2-5-4-3-1 (B) 5-2-3-1-4
(C) 2-5-4-1-3 (D) 5-2-3-4-1
462. With reference to environmental impact assessment (EIA), consider the following statements:
1. Baseline data provide a means of detecting actual change by monitoring once a
project has been initiated.
2. The final EIA report is referred to as an Environmental Management Plan.
Which of the above statements is/are correct?
(A) 1 only (B) 2 only
(C) Both 1 and 2 (D) Neither 1 nor 2
463. The LEED and GRIHA:
1. Are green building rating systems.
2. Issue energy compliance certificate
3. Refer to Global standards.
4. Are India standards under finalization.
Select the correct answer using the codes given below:
(A) 1 and 2 (B) 2 and 3
(C) 3 and 4 (D) 1 and 4
464. What is the full form of GPP?
(A) Green Public Policy (B) Green Private Procurement
(C) Green Public Procurement (D) Green Private Policy
465. Statement (I): Environmental considerations are not necessarily the same as, or congruent
with, Ecological considerations.
Statement (II): Environmental considerations address more towards maintaining, whereas,
Ecological considerations address more towards sustaining.
(A) Both Statement (I) and Statement (II) are individually true and Statement (II) is
the correct explanation of Statement (I)
(B) Both Statement (I) and Statement (II) are individually true but Statement (II) is
not the correct explanation of Statement (I)
(C) Statement (I) is true but Statement (II) is false
(D) Statement (I) is false but Statement (II) is true
466. Consider the following statements about LEED and GRIHA energy compliance certifications:
1. LEED has backing of Confederation of Indian Industry (CII) and been used
in India since 2001.
2. LEED rating is based on per capita energy consumption while GRIHA works on
absolute number.
3. LEED is benchmarked with Global Standards and GRIHA is home-grown.
Which of the above statements is/are correct?
(A) 1 only (B) 2 only
(C) 1 and 2 only (D) 1, 2 and 3
467. Which of the following correctly defines the term sustainable development?
(A) the development which does not cause pollution.
(B) the development which meets need of present generation without compromising
ability of future generations to meet their own needs.
(C) the development which tends to maximum utilization of natural resources.
(D) the development which creates more revenue than investment.
468. Consider the following statements regarding Green Buildings:
1. Green buildings preserve precious natural resources and improve our quality of
life.
2. Good indoor environmental air quality is one of the features of Green buildings.
Which of the above statements is/are correct?
(A) 1 only (B) 2 only
(C) Both 1 and 2 (D) Neither 1 nor 2

469. Consider the following statements regarding Green Public Procurement (GPP):
1. It is an effective means for public administrations to manage the balance
between cost effectiveness and sustainable development.
2. GPP is a voluntary instrument which has a key role to play in the European
Union’s efforts to become a more resource-efficient and circular economy.
Which of the above statements is/are correct?
(A) 1 only (B) 2 only
(C) Both 1 and 2 (D) Neither 1 nor 2
470. Consider the following statements regarding objectives of National Mission on Sustainable
Habitat:
1. To reduce energy demand by promoting alternative technologies and energy
conservation practices in both residential and commercial areas.
2. To encourage community involvement and dialogue for sustainable development.
Which of the above statements is/are correct?
(A) 1 only (B) 2 only
(C) Both 1 and 2 (D) Neither 1 nor 2
EES

471. As per Noise pollution (Regulation and Control) Rules 2000 of India, the day time noise limit for a
residential zone expressed in dB (A) Leq is_____________
(a) 54 (b) 55 (c) 57
(d) 60
472. The permitted noise level in a silent zone during day time and night time are respectively:
(a) 65dB and 55dB (b) 55dB and 50dB
(c) 70dB and 60dB (d) 50dB and 40dB
473. The 20dB noise level is how many times more powerful than the 1dB noise level?
(a) 10 times (b) 20 times (c) 100 times
(d) 200 times
474. Biodiversity in India is governed through the following act
(a) The Biological Diversity Act, 1992
(b) The Biological Diversity Act, 2000
(c) The Biological Diversity Act, 2002
(d) The Biological Diversity Act, 2004
475. The Paris accords legally bound its signatories collectively to limit greenhouse gas emissions to keep
the temperature rise “well below| ……… degrees Celsius this century.
(a) 1.5 (b) 2 (c) 2.5
(d) 3
476. Paris Accord was agreed to in ________?
(a) 2010 (b) 2015 (c) 2017
(d) 2020
477. Which of the following expresses the meaning of Environment Management System?
(a) It is statutory need of the organisation
(b) It is obligation of the institute for environment
(c) It is a system and database
(d) None of these
478. Which of the following is related to Environment Management System?
(a) ISO 9000 (b) ISO 9001
(c) ISO 14000 (d) ISO 14001
479. Which of the following is NOT the stage of Environment Management System?
(a) Planning (b) Implementation
(c) Evaluation (d) Scheduling
480. Which of the following is not one of the basic elements of Environment Management System?
(a) Analysing its environmental impacts
(b) Reviewing the organization’s environmental goals
(c) Setting environmental objectives
(d) Penalising the organisations for their non commitments
481. The Biodiversity Act creates the following authority
(a) National Biodiversity Authority
(b) National Green Tribunal
(c) Indian Biodiversity Commission
(d) National Biodiversity Commission
482. What is CoP?
(a) Convergence of the Parties (b) Convergence of the People
(c) Conference of the People (d) Conference of the Parties
483. The body responsible for international environmental agreement is
(a) UNEP (b) UNSC (c) UNFCCC (d) UN
484. UNEP was established by
(a) Maurice Strong (b) Janathon Curie
(c) Marie Hard (d) Paul Strong

485. Lichen is the best indicator of


(a) Water pollution (b) Air pollution
(c) Soil pollution (d) All the above
486. Pollution which has far-reaching consequences and has many physical as well as psychological effects
on human beings is termed as: -
(a) Air pollution (b) Water pollution
(c) Marine pollution (d) Noise pollution
487. The threshold of sound pressure that a normal healthy person can respond to is
(a) 0.0002Pa (b) 0.00002Pa (c) 0.002Pa (d) 0.02Pa
488. Sustainable Development Goals were set up by
(a) UNEP (b) UNSC (c) UNGA (d) None of these
489. Radio carbon dating technique is used to estimate the age of
(a) rocks (b) soil (c) fossils (d) buildings
490. Which electromagnetic waves are mainly used as a treatment for cancer?
(a) Alpha-rays (b) β-rays
(c) X-rays (d) Gamma rays
491. Symbiotic association between plant roots and Fungi?
(a) Micorhiza (b) Lichen
(c) Rhizophore (d) Phycobiont
492. Bioventing is
(a) Incorporation of water through soil to increase the moisture content is the waste
(b) Incorporation of additional microbes to increase the rate of degradation
(c) Incorporation of air through soil in order to increase oxygen rate in waste
(d) Addition of earthworm to the waste for increasing rate of degradation
493. What is the validity of the statement – Anaerobic digestion is widely used as a source of renewable
energy.
(a) Valid (b) Invalid
(c) Depends upon the waste type
(d) None of the above
494. _________(as opposed to waste management) uses LCA (life cycle analysis) attempts to offer
alternative to waste management.
(a) Pyrolysis (b) Resource recovery
(c) Incineration (d) Energy recovery
495. Which of the following diseases are caused by the consumption of water contaminated by mercury
and nitrates respectively?
(a) Osteoporosis and blue baby syndrome (b) Osteoporosis and Minamata disease
(c) Minamata disease and blue baby syndrome (d) Minamata disease and osteoporosis
496. Which of the following is NOT true related to Kyoto Protocol
(a) It is international treaty that seeks to reduce GHG emissions
(b) It is obligatory on both developing and developed nations
(c) It came into force in 2005
(d) Its first implementation period was 2008 to 2012
497. The United Nations framework convention on climate change (UNFCCC) is an international treaty
drawn at:
(a) United Nations conference on the Human Environment, Stockholm (1992)
(b) UN conference on Environment and development, Rio de Janeiro (1992)
(c) World summit on sustainable sustainable development, Johannesburg (2002)
(d) UN conference of parties 21, Paris, (2015)
498. What is noise?
(a) Desirable sound
(b) Desirable and unwanted sound
(c) Undesirable and unwanted sound
(d) Undesirable and wanted sound
499. In anaerobic conditions, mercury is present in; metallic form and
(a) Humic acid (b) Nitric acid
(c) Methane form (d) Sulphide form
500. Which of the following doesn’t influence energy consumption?
(a) Fuel prices (b) Public education
(c) Quality of transportation system (d) All of the above
501. The stability of an ecosystem depends upon
(a) Primary productivity
(b) Interchange between producers and consumers
(c) Number of producers
(d) Number of consumers
502. Which of the following is not an objective of EIA?
(a) Recycling of waste (b) Risk analysis and disaster management
(c) Assessment of national funding (d) Sustained development
503. Which among the following is/are the renewable sources of energy?
(a) Solar energy, Hydro-power and nuclear energy
(b) Nuclear energy, Hydro-power and Biomass energy
(c) Geothermal energy, Hydro-power and Biomass energy
(d) Geothermal energy, nuclear energy and solar energy
504. Consider the following regarding solar panels:
1. High maintenance cost
2. No moving parts
3. Pollution free, noiseless energy
Which of the above statements are correct?
(a) 1 & 2 only (b) 2 & 3 only (c) 1 & 3 only (d) 1, 2 & 3
505. In which phase of LCA you draw the conclusions that allow you to make better business decisions
(a) Life cycle Inventory (b) Impact Assessment
(c) Interpretation (d) Goal and scope
506. National Environment Policy, 2006 is framed by which ministry
(a) Ministry of Finance
(b) Ministry of Home Affairs
(c) Ministry of Environment, Forest and Climate Change
(d) Ministry of Foreign Affairs

507. Biomass, a renewable energy source can be obtained from:


(a) Groundnut shells
(b) Sugarcane bagasse (c) Rice husk (d) All of these
508. In EIA, the baseline data describes the
(a) Socioeconomic and Biological data
(b) Assessment of risk on the basis of proposal
(c) Existing environmental status of the identified study area
(d) Environment consequences
509. Which event gave an impetus for enacting many environmental laws in India?
(a) Stockholm conference (b) Bhopal Tragedy
(c) Arrival of the British (d) Independence
510. Which of the following statements is/are true with regard to the Sustainable Development Goals?
1. They were adopted at the UN Summit in 2016
2. They are the renamed version of the Millennium Development Goals
3. They are legally binding on the countries.
4. They are to be reached by 2030
Select the correct answer using the code given below
(a) 1, 2 and 3 only (b) 1, 3 and 4 only (c) 4 only (d) 1,
2, 3 and 4
511. Among the sources of energy listed below which one is non-conventional in present day rural India?
(a) Fuel wood (b) Cow-dung cake (c) Biogas ( d )
Hydel
512. Which of the following is a disadvantage of most of the renewable energy sources?
(a) Highly polluting (b) High waste disposal cost
(c) Unreliable supply (d) High running cost

513. Which of the following is/are advantage(s) of most of the renewable energy sources?
1. Less polluting
2. Low waste disposal cost
3. Unreliable supply
4. Low running cost
Codes:
(a) 1 & 2 only (b) 2 & 4 only
(c) 1, 2 & 4 only (d) 1, 2, 3 & 4
514. Choose the correct statement for Bioaugmentation and Bio-stimulation
(a) Bioaugmentation is the use of nutrients, electron acceptors etc to stimulate naturally occurring
microbial population whereas bio-stimulation is the process of adding microorganism to enhance bio-
stimulation process
(b) Bio-stimulation, in incorporation of oxygen through soil where as Bio-stimulation is in corporation
of microbes to enhance Bioaugmentation process
(c) Bioaugmentation is use of plants to remove metals in water while Bio-stimulation is addition of
base/acid in order to maintain a pH of 6.5-7.5
(d) Bioaugmentation is process of adding microbes to enhance bio degradation process and Bio-
stimulation is the process of adding nutrients electron accepter etc to stimulate naturally occurring
microbial population.
515. _______is the reduction of the environmental impact from the production and consumption of these
goods, from final raw material extraction to use and disposal.
(a) Waste hierarchy (b) Resource efficiency
(c) Polluter pays principle (d) Zero waste
516. Consider the following statements regarding the pollutants
1. Lead can lead to the Damage to brain and central nervous system, kidneys and brains, impaired
intelligence and interference with development of RBCs
2. Thermal power plants and industries are the major source of SO2 release into the atmosphere
3. SO2 can lead to Eye and throat irritation cough, allergies, impairs enzyme function in respiratory
system
4. Mercury can lead to Nervous disorders, insomnia, memory loss, gingivitis and Minamata disease
Which of the above statements are correct?
(a) 1, 2 and 3 (b) 2 and 3 only
(c) 1, 2 and 4 (d) 1, 2, 3 and 4
517. Which of the following is not the objective of Ministry of Environment, Forest and climate change?
(a) Ensuring the welfare of animals
(b) Afforestation and regeneration of degraded areas
(c) Prevention and control of pollution
(d) Prevention of volcanic eruptions
518. The concept of carbon credit originated from which one of the following?
(a) Earth Summit Rio de Janerio (b) Kyoto Protocol
(c) Montreal Protocol (d) G-8 Summit, Heiligendamm
519. Treeless terrestrial biome of cold climate is
(a) Taiga (b) Tundra
(c) Savanna (d) Plankton
520. The transition zone between two communities is known as
(a) Ecotone (b) Territory
(c) Ecosystem (d) Biome
521. The presence of diversity at the junction of territories of two different habitats is known as
(a) Edge effect (b) Junction effect
(c) Boarder effect (d) Side effect
522. Kyoto Protocol, held in 1997 at Japan’s historical city Kyoto, which of the following facts about Kyoto
Protocol are correct:-
(1) It sets legally binding targets related to climate change.
(2) During second commitment Period (2013-20), Parties committed to reduce GHG emissions by at
least 19% below 1990 levels.
(3) International Emissions Trading (Carbon trading) was proposed at Kyoto Meet only
(4) USA withdrew from Kyoto Protocol in 2012
(a) 1 & 2 (b) 2, 3 & 4 (c) 4 only (d) 1, 2 & 3
523. Pre-cursors to photochemical oxidants are:
(a) Nox, VOCs and sunlight (b) SO2, CO2 and sunlight
(c) H2S, CO and sunlight (d) SO2, NH3 and sunlight
524. Vienna convention took place in which year
(a) 1985 (b) 1972 (c) 1975 (d) 1995
525. The Government of India has set a target of 175 GW renewable power installed capacity by the end of
2022. Which of the following is the correct sequence in the descending order of contribution expected
from various sectors?
(a) Biomass powers, solar power, wind power
(b) Solar power, wind power, biomass power
(c) Wind power, solar power, biomass power
(d) Solar power, biomass power, wind power
526. Extinction of a species in a food chain is compensated by
(a) Food chain (b) Ecological pyramid
(c) Food web (d) None of these
527. Which of the following statements is incorrect?
(a) The Great Barrier Reef along the east coast of Australia can be categorized as a Biome
(b) In a comparative study of grassland ecosystem and pond ecosystem it may be observed that both
biotic and abiotic components are similar
(c) The term ecosystem was coined by A.G. TANSLEY
(d) None of the above
528. Which one of the following is one of the characteristics of a biological community?
(a) Stratification (b) Natality (c) Mortality (d) Sex ratio
529. Both power the manure are provided by
(a) Nuclear plants (b) Biomass plants
(c) Tidal plants (d) Hydroelectric plants
530. Sustainable Development Goals are to be achieved by
(a) 2025 (b) 2030 (c) 2035 (d) 2050
531. Consider the following statements regarding ‘biogas’:
1. This is a mixture of different gases which is formed after breakdown of organic compounds.
2. This gas can be compressed and used for energy needs.
Select the correct answer:
(a) 1 only (b) 2 only
(c) 1 & 2 (d) None of these
532. Consider the following statements regarding objectives of National Mission on Sustainable Habitat:
1. Better urban planning and modal shift to public transport
2. Conservation through appropriate changes in legal and regulatory framework
Which of the above statements is/are correct?
(a) 1 only (b) 2 only
(c) Both 1 & 2
(d) neither 1 nor 2
533. Which of the following is/are objectives of the Environment Impact Assessment (EIA):
1. Conservation and sustainable use of natural resources
2. To assess the nature, intensity and duration of impact of proposed development projects
3. To promote social and cultural considerations in project design
Select the correct answer using the codes given below:
(a) 1 & 2 (b) 2 only (c) 1 & 3 only (d) All of above
534. Assertion: Geothermal energy is heat energy generated inside the earth
Reasoning: Geothermal energy of earth’s crust originates from the original formation of the planet
and from radioactive decay of material.
(a) Both A and R are true and R is the correct explanation of A.
(b) Both A and R are true but R is NOT the correct explanation of A.
535. Which of the following is main objective of creation of National Green Tribunal?
(a) To protect environment
(b) To ensure implementation of environmental laws
(c) To provide speedy justice to reduce burden of litigation in higher courts
(d) all of these
536. Who can be chairperson of National Green Tribunal?
(a) Sitting Judge of Supreme Court
(b) Sitting Chief Justice of Supreme court
(c) Retired Chief Justice of Supreme Court
(d) Retired Judge of Supreme Court
537. Which of the following statements is not true with regard to the Environment (Protection) Act?
(a) It is an enabling law
(b) It sets the standards for emission or discharge of environmental pollutants.
(c) It provides powers to the executive to frame various rules and regulations
(d) It authorizes the central government to protect and improve environmental quality.
538. Consider the following statements:
1. National Green Tribunal is a statutory body to undertake expeditious disposal of the cases pertaining
to environmental issues
2. All the members of the National Green Tribunal are chosen by the President in consultation with
the council of ministers.
Select the correct answer using the codes given below:
(a) 1 only (b) 2 only
(c) both 1 & 2 (d) neither 1 nor 2
539. With reference to two non-conventional energy sources called ‘coalbed methane’ and ‘shale gas’,
consider the following statements:
1. Coalbed methane is the pure methane gas extracted from coal seams, while shale gas is a mixture
of propane and butane only that can be extracted from fine-grained sedimentary rocks.
2. In India, abundant coalbed methane sources exist, but so far no shale gas sources have been found.
Which of the statements given above is/are correct?
(a) 1only (b) 2 only
(c) both 1 & 2 (d) neither 1 nor 2
540. Algal bloom is an example of
(a) Ammensalism (b) Antagonism
(c) Synergism (d) Competition
541. What is a Geo-net
(a) A synthetic material used for drainage of liquids
(b) A synthetic material used for drainage of gases
(c) A ceramic material used for drainage of liquids
(d) A fibrous material used for drainage of liquids
542. Which of following subjects fall under the Ministry of Environment, Forest and Climate Change?
(a) Bio-diversity Conservation including that of lakes and wetlands
(b) Environmental Health
(c) Green Skill Development Programme
(d) All of the above
543. Statement-1: Kyoto Protocol and Vienna Convention are both related to ozone depletion substances
(ODS)
Statement-2: Ozone depletion will affect human body by enhancing the chances of skin cancer.
Which of the above statement (s) is/are correct?
(a) Only S1 (b) Only S2
(c) Both S1 & S2 (d) None
544. Corals are useful to mankind in many ways, which of the following statement is incorrect?
(a) They are home to more than 25% of all known Marine species
(b) If corals are colourful, it means that there is a pollution problem
(c) Fisheries, tourism, medicines etc are benefits of corals to mankind
(d) They protect nearby shoreline area from erosion
545. The biomass available for consumption by the herbivores is called
(a) Net primary productivity (b) Secondary productivity
(c) Standing crop (d) Gross primary productivity
546. The greatest biomass of autotrophs in the world’s oceans is that of
(a) Benthic brown algae, coastal red algae and daphnids
(b) Benthic diatoms and marine viruses
(c) Sea grasses and slime moulds
(d) Free floating micro algae, cyanobacteria and nanoplanktons.
547. Energy storage at consumer level is called
(a) Gross primary productivity (b) Secondary productivity
(c) Net primary productivity (d) Net productivity
548. The ecological niche of population is a
(a) Geographical area that it covers (b) Place where it lives
(c) Set of conditions and resource it uses (d) None of the above
549. Which of the following is NOT correct related to Convention on Biological Diversity (CBD)?
(a) It is multilateral treaty
(b) It entered into force in 1993
(c) It deals with sharing of benefits from genetic resources
(d) It has mentioned some sustainable developmental goals.
550. Ocean thermal energy conversion produces electricity using difference in
(a) Temperature (b) Sunlight
(c) Salinity (d) None of these
551. Consider the following statements about monitoring:
1. It ensures the fulfilment of all the commitments made in the approved Environmental Impact
Assessment.
2. It keeps track of changes that may happen in environment and communities because of the project.
Which of the above statements is/are correct?
(a) 1 only (b) 2 only
(c) both 1 & 2 (d) neither 1 nor 2
552. Consider the following statements regarding International Solar Alliance (ISA):
1. It was launched at the Conference of Parties (CoP 21) in Paris, France.
2. It most of the members are the countries lying on or within the tropics.
Which of the above statements are correct?
(a) 1 only (b) 2 only (c) 1 & 2 (d) None of these
553. A systematic process for evaluating the environmental implications of a proposed policy, plan or
programme and provides means for looking at cumulative effects and appropriately address them at
the earliest stage of decision making alongside economic and social considerations is known as
(a) Strategic Environmental Assessment (SEA) (b) EIA
(c) Policy assessment (d) None of the above
554. Biomass gasification is considered to be one of the sustainable solutions to the power crisis in India.
In this context, which of the following statements is/are correct?
1. Coconut shells, groundnut shells and rice husk can be used in biomass gasification.
2. The combustible gases generated from biomass gasification consist of hydrogen and CO2 only
3. The combustible gases generated from biomass gasification can be used for direct heat generation
but not in internal combustion engines.
Select the correct answer using the codes given below:
(a) 1only (b) 2 & 3 only
(c) 1 & 3 only (d) 1, 2 and 3
555. How is the National Green Tribunal (NGT) different from the Central Pollution Control Board (CPCB)?
1. The NGT has been established by act whereas the CPCB has been created by an executive order of
the government.
2. The NGT provides environmental justice and helps reduce the burden of litigation in the Higher
Courts whereas the CPCB promotes cleanliness of streams and wells, and aims to improve the quality
of air in the country.
Which of the statements given above is/are correct?
(a) 1 only (b) 2 only (c) both 1 & 2 (d) neither 1 nor 2
556. Which of the following statements is correct regarding Environmental Impact Assessment (EIA), 1994?
(a) Responsibility of implementation of EIA notification lies with Ministry of Environment, Forests and
climate change
(b) EIA notification is applicable to all states and Union Territories
(c) Schedule 1 of the guidelines lists the projects requiring environmental clearances
(d) All of the above
557. Consider the following statements regarding Strategic Environment Assessment:
1. It takes place at earlier stages of decision-making cycle.
2. It considers limited number of feasible alternatives.
Which of the above statements is//are correct?
(a) 1 only (b) 2 only (c) both 1 & 2 (d) neither 1 nor 2
558. Which of the following is not the advantage of EIA?
(a) Reduced environmental damage (b) Improved project design
(c) Improved income to Environmental agencies (d) Environmentally sensitive decisions.
559. Deforestation generally reduces which one of the following natural phenomena
(a) Draught (b) Rainfall
(c) Soil erosion (d) Global warming
SOLUTIONS

1. The answer is (a).
Justification: Statement 1:Biosphere is that part of the earth where life can exist. A narrow layer
around the surface of the earth comprising atmosphere (air), hydrosphere (water), and lithosphere
(land), it is a highly integrated and interactive zone.
Statement 2: The energy required for the life within the biosphere usually comes from the sun. But
there are certain organisms such as chemosynthetic bacteria, which do not need sunlight for their
survival. Instead these organisms use inorganic molecules as a source of energy and convert them
into organic substances. They are found in regions where sunlight is not able to reach or penetrate
such as the deep sea and are generally considered inhospitable.
2. The answer is (c).
Justification: Ecotone is a zone of transition or boundary between two or more diverse ecosystems.
Examples of ecotones are mangrove forests, estuary grassland, river bank, marshes, etc.
Characteristics of ecotone
• It may be very narrow or quite wide, but not larger than a biome which is a much larger entity.
• It is a zone of tension because the conditions present here are intermediate to the adjacent biomes
or ecosystems.
• In this zone, there is progressive increase in the species composition of incoming community and a
simultaneous decrease in the species of the outgoing community.
• In a well-developed ecotone, there are certain organisms which are entirely different from those
found in the adjoining communities.
• Ecotones may display edge effect where the population density of some species is more in an ecotone
than its adjoining communities.

3. The answer is (c).


Justification: Ecotone is a zone of transition or boundary between two or more diverse ecosystems.
Examples of ecotones are mangrove forests, estuary, grassland, river bank, marshes, etc.

4. The answer is (c).


Justification: The stability of an ecosystem is higher if the chance of extinction is less. The stability of
an ecosystem is generally measured by measuring the variability of aggregate community properties
such as total biomass and species diversity.
Higher biomass (i.e. large number of flora or fauna) usually leads to higher stability.
Moreover, it has been observed that diverse ecosystems have better inter-linkages between species
and allow more ground (ecosystem services) to survive So, option (c) is the most appropriate
choice.
5. The answer is (b).
Justification: Biosphere is that part of the earth where life can exist. A narrow layer around the surface
of the earth comprising atmosphere (air), hydrosphere (water), and lithosphere (land), it is a highly
integrated and interactive zone.
6. The answer is (a).
Justification: A biome is a community of plants and animals that covers a large geographical area.
The plants and animals existing in a biome interact with each other within specific conditions such as
rainfall, temperature, humidity, and soil. Hence the boundaries of biomes are determined mainly by
climate. A biome may or may not have a collection of endemic species. It is not a meeting boundary
and is distinct from small ecosystems or communities.
7. The answer is (a).
Justification: Some of the major biomes of the world are forests, grasslands, deserts, and tundra
biomes.
8. The answer is (d).
Justification: Biosphere forms the highest-level ecosystem; it comprises all life forms present on earth.
Hence largest population can be found in biosphere.

9. The answer is (c).


Justification: A more diverse ecosystem is more stable. If there is less evenness among species,
then there will be greater diversity. If there are a greater number of ecotons, there will be a lot
of eco transition zones (e.g. ponds, grasslands, etc.) and there will be greater species diversity and
interdependence. Finally, if a smaller number of species dominate, it is good for the ecosystem
diversity even more.
10. The answer is (a).
Justification: Statement 1: Ecotone is a zone of transition or boundary between two or more diverse
ecosystems.
Statement 2: It is not necessarily true. For example, an ecotone with desert and lake junction will
not have a higher productivity than an estuary ecosystem.

11. The answer is (d).


Justification: Statement 1: Biotope is an ecological area that supports a particular range of biological
communities. Biotope is almost synonymous with the term habitat and it is generally not considered
to be a large-scale phenomenon. Examples of biotope are a neighbouring park, a garden, or a fish
tank.
Statement 2: Biotopes are not isolated. Biotopes need to be connected to each other and other
surrounding life for without these connections to life forms such as animals and plants, biotopes
would not effectively work as a place in which diverse organisms live.
12. The answer is (c)
Justification: A sequence of organisms that feeds on one another forms a food chain. The two main
types of food chains found in nature are’grazing food chain’ and ‘detritus food chain’.
The grazing food chain starts with producers or autotrophs as base, which is consumed by heterotrophs.
It is found in both terrestrial and aquatic ecosystems.
The detritus food chain starts from dead organic matters of decaying plants and animals and then to
detritus feeding organism called detritivores or decomposer and to other predators.
The waste and dead organic matter from the grazing food chain is the initial energy source for detritus
food chain and these two food chains are interlinked with each other.
13. The answer is (a).
Justification: In a food chain, dinoflagellates and diatoms are producers. Copepods are primary
consumers, ocean sunfish are secondary consumers and sharks are tertiary consumers.
14. The answer is (c).
Justification: Statement 1: Ecological pyramid is a ‘graphical representation of relationship’ between
the organisms from different trophic levels of food chains. It is represented based on energy, number,
and biomass productivity. The base of the upright pyramid is represented by the ‘lowest trophic level,
i.e., producers’. The next level is occupied by the primary consumer and so on. The tip is occupied by
top carnivore of a food chain.
Statement 2: Pyramid of energy in an ecosystem is always upright as the energy flow in a food chain
is always unidirectional and with increasing trophic level, some energy is lost as heat and respiration.
15. The answer is (c).
Justification and Learning: Pyramid of energy is a graphical representation of the amount of energy
trapped per unit time and area in different trophic levels of a food chain. Pyramid of energy in an
ecosystem is always upright as the energy flow in a food chain is always unidirectional and with
increasing trophic level, some energy is lost as heat and respiration.
It follows 10% rule, that is, during each transfer about 90% of the energy available at lower trophic
level is used up to overcome its entropy and to perform metabolic activities and only about 10% of
the energy is available to the next trophic level.
16. The answer is (d).
Justification: Biological magnification or in general biomagnification means the ‘increase of
contaminated substances or toxic chemicals’that takes place in the food chains as the pollutants pass
from lower trophic to higher trophic.
The pollutants are heavy metals such as mercury, arsenic, pesticides such as DDT, and polychlorinated
biphenyls (PCBs). These compounds are taken up by organisms because of the food they consume
from an intoxicated environment.
Pollutants need to satisfy characteristics such as having a long life and being biologically active and
soluble in fat to make bio-magnification possible.
17. The answer is (b).
Concept: The level of dissolved oxygen is an important indicator of water pollution.
Justification: Water pollution by biodegradable organic wastes is measured in terms of BOD. Whereas
water pollution by both biodegradable and non-biodegradable organic wastes is measured in terms
of COD. Hence COD is a better measure of water pollution than BOD.
18. The answer is (a).
Justification: Mosses can be used as a low-cost bio-indicator to monitor urban pollution and to measure
the impact of atmospheric change. As a bio indicator, mosses responds to any change in environment
in terms of pollution or drought-stress, by changing its shape, density, or even disappearing from the
environment.

19. The answer is (d)


Learning: Generally, organic toxins are water insoluble and non-biodegradable. They are transferred
from lower trophic level to higher trophic level through food chain and lead to biomagnifications.
Over time, the concentration of these toxins is highest in higher animals and reaches a level which
cause serious metabolic and physiological disorders.

20. The answer is (a).


Learning: They reflect the quality and changes in environmental conditions as well as aspects of
community composition. Following are some examples of indicator species.
Stoneflies: Indicator of high oxygen water
Mosses: Some mosses species indicate acidic soil and some monitor urban pollution
Lichens: Some species of lichens indicate low air pollution
Fungi: Some species of fungi can indicate old-growth forests where an abundance of coarse woody
debris exists
Mollusca: Can indicate water pollution status
21. The answer is (c).
Justification: Water pollution by biodegradable organic wastes is measured in terms of BOD. The
quality of water can be measured by BOD as
It provides an index to assess the discharged wastewater (nutrient discharge)
A higher BOD value means greater amount of organic matter available for oxygen consuming bacteria
Thus, both 1 and 2 are correct.
22. The answer is (c).
Justification: There are two types of succession on the basis of factors causing succession.
‘Autogenic succession’is a succession brought about by the living inhabitants of that community itself.
‘Allogenic succession’ is a succession brought about by outside forces. The forces can be
• Flooding
• Volcanic eruptions and earthquakes
• Meteor or comet strikes
• Drought
23. The answer is (a).
Justification: The gradual process by which ecosystems change and develop over time is known as
ecological succession.
Statement 1:Pioneer community are the plants which colonise the area for the first time.
Statement 2: The successive stages are known as seral community and the final stage is known as the
climax community.
Statement 3: Allogenic succession is a succession brought about by outside forces. The forces can be
• Flooding
• Volcanic eruptions and earthquakes
• Meteor or comet strikes
• Drought
24. The answer is (d).
Justification: In any ecosystem, there are a series of steps which lead to its enrichment, i.e., ecological
succession takes place in a series of stages. Most ecosystems will start from purely inorganic material
such as rocks. Only a few organisms such as lichens are capable of surviving in such an inhospitable
area. The plants which colonise the area for the first time are known as pioneer community or pioneer
species.
Lichens is therefore a pioneer species since growth in its population will give rise to the entire
ecosystem biodiversity in a series of steps. As a symbiotic relationship between an alga and a fungus,
a lichen can survive considerable drying out.
25. The answer is (c).
Justification: There are two types of ecological succession.
Primary succession: This type of succession essentially occurs in a lifeless area. These are the regions
which are incapable of sustaining life. These regions are formed by volcanic eruptions, sand dunes,
etc. Hence statement 1 is correct.
Secondary succession: This type of succession occurs in areas where a community that previously
existed has been removed by disturbances but all life and nutrients have not yet been eliminated
from the environment.
Statement 2: Climax community forms the part of climax stage of succession. When the web of biotic
interactions becomes so intricate that no other species can be admitted, then the existing ecological
community is known as the climax community.
26. The answer is (a).
Justification: Primary succession essentially occurs in a lifeless area. These are the regions which are
incapable of sustaining life. These regions are formed by volcanic eruptions, sand dunes, etc. Hence
statements 1 and 2 are correct.
Secondary succession occurs in regions which are affected by disturbances such as a fire or a flood.
27. The answer is (a).
Justification: The system formed by the interaction of all living organisms (biotic) with each other and
with the physical and chemical factors (abiotic) of the environment in which they live is known as
ecosystem.
28. The answer is (c).
Learning : A biomarker is a biological molecule found in blood, other body fluids, or tissues that is a
sign of a normal or abnormal process, or of a condition or disease. It can also be a substance whose
detection indicates a particular disease state.
29. The answer is (b)
Justification: Statement 1 : The rate of formation of new organic matter by consumers is known as
secondary productivity. Hence statement 1 is wrong. Whereas the rate of formation of new organic
matter by plants (producers) is known as primary productivity.
Statement 2: The factors that affect primary productivity are
Plant specifies inhabiting a particular area
Availability of nutrients
Photosynthetic capacity of plants
30. The answer is (d)
Learning: The environment is able to perform ecological functions without any interruption as long
as these functions are within the carrying capacity of that environment. This means that resource
extraction is not above the rate of regeneration of the resource and the wastes generated are within
the assimilating capacity of the environment.
31. The answer is (c).
Justification: An ecological niche is the unique functional role or place of a species in an ecosystem.
The ecological niche is the description of all the biological, physical, and chemical factors that a species
needs to survive, stay healthy, and reproduce in an environment. The ecological niche is unique for
each species and no two species can have identical and exact niches.
32. The answer is (a).
Learning: The following table shows the most productive ecosystem in terms of primary productivity
per unit area.
Ecosystem Type Primary Productivity (Per Unit Area)
Algal bed and reef 2500
Cultivated land 650
Temperate forest 1200
Savannah 900
33. The answer is (b).
Justification: Different microorganisms require different environments such as varied temperature,
light, levels of oxygen, and pH level. Some microbes grow faster in environment with extremely low
pH values. The optimum pH for most bacteria and protozoa is near neutrality. Hencestatement 3 is
correct.
Fungi generally prefer an acidic environment whereas blue-green bacteria generally prefer an alkaline
environment. Hence statements 1 and 2 are wrong.
34. The answer is (c).
Justification: A keystone is that stone in an arch’s crown that secures the other stones in place. In
many ecological communities, keystone species also act like a keystone by maintaining the structure
and integrity of the community.
If the population of the keystone species in an ecosystem changes or diminishes, then the ecosystem
would be dramatically different or it may even cease to exist altogether. No other species will be able
to replace the ecological niche of a keystone species.They are not always the largest or most abundant
species in an ecosystem. Sea otter is considered as an example of keystone species.
35. The answer is (c).
Justification: An ecological niche is the unique functional role or place of a species in an ecosystem.
The ecological niche is the description of all the biological, physical and chemical factors that a species
needs to survive, stay healthy and reproduce in an environment. The ecological niche is unique for
each species and no two species can have identical and exact niches.
36. The answer is (a).
Learning: Indicator species provide information on the overall conditions of an ecosystem and reflect
the quality and changes in environmental conditions as well as aspects of community composition.
River otters (Lontra canadensis) can be an indicator species of clean rivers or presence of too much
algae can be the indication of toxicity in a marine ecosystem.
37. The answer is (a).
Justification and Learning: Biotope is a well-defined geographical area, characterized by specific
ecological conditions. The whole area has uniformity in environmental conditions and distribution of
animal and plant life.
38. The answer is (c).
Justification: One of the most commonly cited threats to species extinction and an ensuing loss of
biological diversity is habitat fragmentation, making it perhaps the most important contemporary
comer vation issue. Loss of connectivity between contiguous protected areas can reducethe size
and quality of available habitat, impede and disrupt movement of organisms to new habitats, and
also affect their seasonal migration patterns. These changes can decrease carrying capacity of the
ecosystem, lead to loss of genetic variation and ultimately species extinction.
39. The answer is (a).
Learning: A large area that contains a number of habitats, which are linked by the evolutionary history
of the animals and plants within them is known as ecozone. An example of ecozone is Australasia,
because the marsupials in this area evolved in isolation to mammals in the rest of the world.
40. The answer is (b).
Justification: A green economy is an economy whose growth in income and employment is driven by
public and private investments that promote sustainable development and help in overcoming the
deeply rooted causes of unsustainable economic development.
Three priorities in transition of economy to green economy are
• decarbonizing the economy
• committing the environmental community to justice and equity; and
• conserving the biosphere.

41. The answer is (a).


Justification: The term ‘social forestry’ was first used by the National Commission on Agriculture,
Government of India in 1976. Social forestry means managing and protecting forests and afforesting
on barren lands to help in environmental, social, and rural development. The main aim of social
forestry is to raise plantations by the common man so as to meet the growing demand for 5Fs: food,
fuel, fodder, fibre, and fertilizer and reduce pressure on the traditional forest area.
Statement 2: It does not aim at displacement or exploitation of traditional forests, but at supplementing
them with community grown forests. So, statement 2 is wrong.
42. The answer is (c).
Justification: Species that suit the description in Statement 1 are known as keystone species. Those
suiting Statement 2 description are more similar to those living in ecological niches.
Statement 3: An ecotype is a population that has successfully adapted to local environmental
conditions. Theindividuals in these ecotypes carry genes that are partly responsible for their success
in that environment and the adaptation is based on the interactions of their own special sets of genes
with their own environment.
43. The answer is (b).
Justification and Learning: Trophic cascade is an ecological phenomenon triggered by the addition or
removal of top predators. With the change in population of top predator, there are reciprocal changes
in the relative populations of predator and prey through a food chain. These changes often result in
dramatic changes in the ecosystem structure and the nutrient cycle.

44. The answer is (a).


Justification and Learning: Phytoplankton, also known as microalgae, are similar to terrestrial plants
in that they contain chlorophyll and require sunlight in order to live and grow. Phytoplankton are
producers and form the base of several aquatic food webs. They are buoyant and float in the upper
part of the ocean, where sunlight penetrates the water.
45. The answer is (a).
Justification: Statements 1 and 2: Environmentalists maintain that human beings should learn to live
in harmony with the rhythms of the ecosystem and not manipulate the natural environment to serve
their immediate interests. Environmentalism is an attempt to balance the relations between humans
and the natural ecosystems on which they depend.
Statement 3: It does not say that we cannot restore the environment.
46. The answer is (d).
Learning: Land footprint is a consumption-based indicator, which looks at the resources needed to
create a final product by an organization or a country.
47. The answer is (d).
Justification: The ‘non-monetary’ assessment of ecosystem integrity, health, or resilience is known
as ‘ecological value’. All of these indicators are crucial to determine critical thresholds and minimum
requirements for ecosystem service provision. It is non-monetary because the value of ecology is
evaluated in terms of its intrinsic value as a part of the earth, and not because it provides secondary
benefits that can be capitalized by humans. For example, the value of pollination by bees can hardly
be measured monetarily, and is beneficial to both humans and non-humans.
48. The answer is (c).
Justification: According to Millennium Eco Assessment, ecological footprint is an Index of thearea ofproductive
ecosystems requited to produce the resources used and to assimilate the wastes produced by a
defined population. Ecological footprint analysis is used in support of sustainability assessment and
to measure and manage the use of resources throughout the economy. It is also used to explore the
sustainability of individual lifestyles, industry sectors, neighbourhoods, cities, and nations.
49. The answer is (c).
Learning: Humus has a uniformly dark, spongy, and jelly-like appearance. It is organic due to accumulation
of organic carbon. The process of humification occurs naturally in soil but it can also be induced
artificially in the production of compost. Humus is a source of nutrients to microbes. It provides a
readily available supply of nutrients, and also acts as alonger term storage reservoir of nutrients. It
contributes to moisture retention in the soil by increasing micro porosity.
50. The answer is (a).
Concept and Learning: The phenotype of an organism is a result of the expression of an organism’s genetic code,
its genotype, as well as the influence of environmental factors on the organism and the interactions
between the environment and the genotype. Phenotypic variation is a fundamental prerequisite for
evolution by natural selection. The interaction between genotype and phenotype has often been
conceptualized by the following relationship.
Genotype (G) Environment (E) → Phenotype (P)
51. The answer is (b).
Justification: Tundra: It is a treeless region. Vegetation of this region mainly consists of lichen, mosses, and
sedges. The fauna of this region consists of reindeer, polar bear, arctic fox; reptiles and amphibians
are almost absent.
Savannah: The vegetation mainly consists of grasses with scattered trees and fire resisting thorny shrubs. The
fauna of this region includes elephants, lion, cheetah, hyena, antelopes, zebras, buffaloes, and
rhinoceros.
Desert: The vegetation mainly consists of drought resistant plants such as cactus, euphorbias, and sagebrush and
fauna includes reptiles, mammals, and birds.
Taiga: The vegetation is mainly characterized by coniferous evergreen trees mostly spruce, pine and firs. The
fauna includes small seed eating birds, hawks, and fur bearing carnivores such as puma, Siberian tiger,
wolverine, and wolves.
52. The answer is (c).
Justification: Statement 1: They increase the relative humidity of the air and retard evaporation by transpiration.
Forests also offer protection to animals and crops against hot, dry winds or strong, cold winds, and
rays of the sun. Forests also prevent desiccation and vegetative retrogression.
Statement 2: During photosynthesis, they absorb CO2, an important greenhouse gas, leading to reduced
temperature absorption by the atmosphere.
53. The answer is (a).
Justification: Sal tree is a deciduous tree and Nilgiris are evergreen forests.
Pine generally occurs at an altitude of 1,600-6,600 feet in Nilgiri forests.
Eucalyptus plantations have been raised in Nilgiri and Kodaikanal hills in the past to meet firewood needs.
Cypress can be found planted along the highways inNilgiris.
54. The answer is (c).
Justification: Eastern Ghats play an important rolein the monsoon break of both south-west and north east
monsoons. The Eastern Ghats are a discontinuous range of mountains running parallel to the Bay of
Bengal. They pass through West Bengal, Odisha, Andhra Pradesh, Telangana, and Tamil Nadu. They
also pass through some parts of Karnataka.
55. The answer is (d).
Justification: Forest fires
• Return the nutrients to the soil by burning dead or decaying matter present in the forest.
• Act as ‘disinfectant’and remove disease ridden plants and harmful insects from a forest ecosystem.
• Burn thick canopies and bushy undergrowth, which ‘allows sunlight to reach the forest floor’ and
enables a new generation of seedlings to grow in the forest
56. The answer is (d).
Learning: At higher latitudes, temperate deciduous forests are found in the north-eastern part of USA, China,
New Zealand, Chile, and in the coastal regions of Western Europe. In these forests, trees shed their
leaves in the dry season. The vegetation consists of oak, ash, beech, etc. Deer, foxes, wolves are the
animals commonly found. Birds such as pheasants and monals are also found here.
57. The answer is (a).
Learning: Transpiration is the loss of water from the leaves through stomata.This creates a suction pull in plants,
which creates more demand for water from the roots. If transpiration is high due to large area of
large pores of the leaves, water demand for the plant is likely to be high, which would not work
for a climate like Mediterranean. So, waxy leaves and thick barks reduce transpiration and thus the
trees’water need.
58. The answer is (d).
Justification: The order can be logically arrived at by elimination. Tropical rainforests receive overall more rainfall
than seasonal forests and grasslands. So, (a) and (b) are eliminated. Also, a forest will certainly be
more productive than grassland. So, (c) is also eliminated. Thus, the correct answer will be (d).
59. The answer is (d).
Justification: Statement 1: Veld has mild climate due to the influence of Indian Ocean, where the
winters are dry and cold. Temperature vary between 5°C and 10°C and July is the coldest month.
Summers are short and warm.
Statement 2: The veld receives rainfall in the summer months (from November to February) because
of the warm ocean currents that wash its shores. If the rainfall is scanty in the winter months, i.e.,
June to August, drought may occur.
Learning: The veld is rich in mineral reserves. Iron and steel industry has developed in this region
due to the presence of coal and iron. Gold and diamond mining create major employment for people
of this region. Johannesburg is called the gold capital of the word and Kimberley is famous for its
diamond mines.
60. The answer is (a).
Statement 1: Brown tides are part of growing worldwide incidences of harmful algal blooms (HAB)
which are caused by a proliferation of single-celled marine plants called phytoplankton.
Statement 2; Brown tide is a natural phenomenon with no or little-known linkage to human activity.
Moreover, brown tide is fortunately not toxic to fish, so it does not cause fish kills like red tide does.
Statement 3: It is not caused by seagrass. Instead, brown tides for long periods of time and in dense
enough concentrations can harm seagrasses by blocking the sunlight they need to survive.
61. The answer is (c).
Eutrophication is a process by which a water body acquires a high concentration of nutrients
mostly phosphates and nitrates. This helps promote excessive growth of algae. As the algae die and
decompose, the decomposing organisms deplete available oxygen in the water, causing the death of
other marine organisms such as fish.
Statement 1: Heated industrial effluents increase the temperature of the water body and typically
decrease the level of dissolved oxygen in water. This can be harmful to aquatic animals such as fish,
amphibians, and other aquatic organisms. But this might not generally happen with eutrophication.
Statements 2 and 3: Eutrophication is a natural phenomenon with slow-aging of a water body, but
human activity (industrial effluents) aids in speeding up of the process.
62. The answer is (d).
` Statement 2: Wetlands act as natural buffers against natural disasters and risks such as floods,
tropical cyclones, and droughts. Wetlands also act as sponges that help in storing water during peak
rainfall and releasing water during lean season.
Statement 1: Acting as ‘kidneys of landscape’, wetlands generally receive flows of water and waste
from upstream sources. They help in stabilizing water supply, cleansing of water, and recharging of
groundwater aquifers.
Statement 3: The extensive food chain and biological diversity in wetlands make them ‘biological
supermarkets’ as they are rich in nutrients and help support a wide variety of communities.
63. The answer is (d).
Wetlands have distinctive communities of plant and animal species and are characterized by water-
logged soils. The organisms of wetlands have evolved and adapted to the constant presence of water.
Statement 1: Since wetlands are one of the most significant natural sources of atmospheric methane,
ideally decreasing the level of water saturation will lead to decrease in release of methane. Thus,
statement 1 is wrong.
Statement 2: Most of the methane production occurs in oxygen poor/anaerobic environmental
condition. The microbes that live in warm and moist environments consume oxygen more quickly
than they can diffuse in from the atmosphere, and since wetlands are anaerobic or oxygen poor, they
have the ideal environment for fermentation and methane production. So, statement 2 is wrong as it
increases methane production.
64. The answer is (a).
Several plants on the floating treatment wetland (FTW) help clean the lake by absorbing excess
nutrients such as nitrates and oxygen that are present in the water, thereby reducing the content of
these chemicals. ‘Phytoremediation’ (phyto means plant) is the term used for technologies that use
plants for remediating soils, sludges, sediments, and water contaminated with organic and inorganic
contaminants. ‘Hydroponics’ is a technique of growing plants in the absence of soil with nutrient
solution. The FTW is based on the concept of soilless hydroponics technique. It is a joint effort of an
NGO Dhruvansh with the Hyderabad Metropolitan Development Authority, the district administration,
and other organizations.
65. The answer is (c).
The rules stipulate the setting up of a National Wetlands Committee, which should be chaired by the
Ministry of Environment, Forest and Climate Change (MoEFCC) Secretary. The aim of the body is to
monitor the implementation of these rules and oversee the work carried out by the states.
Statement 1: The rules also stipulate setting up of a State Wetlands Authority (SWA) in each state/UT
headed by the particular state/UT’s environment minister. State government will also nominate one
expert from each field of wetland ecology, hydrology, fisheries, and socioeconomics and landscape
planning to SWA. SWA will develop a comprehensive list of activities that have to be regulated and
permitted within the notified wetlands and their zones.
Statement 2: The rules include prohibition of activities such as conversion of wetland for non-wetland
uses including encroachment of wetland in any way, setting up and expansion of industries, and
activities of waste dumping and discharge of untreated wastes and effluents from industries, cities,
and other human settlements and so on.
66. The answer is (b).
Justification: Statement 1: Of the total geographical area of India (329 million hectares), 195 million
hectares is gross cropped area and 141 million hectares is net sown area. On the other hand, the area
of wetlands in India is around 15-16 million hectares, accounting for 4.7% of the total geographical
area of the country.
Statement 2: Of the total wetland area in India, inland wetlands account for 69% and coastal wetlands
account for 27%, other wetlands account for (smaller than 2.25 ha) 4% (SAC, 2011). In terms of average
area, natural coastal wetlands have the largest area.
67. The answer is (d).
Option (d) talks about north-east India’s hill tracts which are not wetlands. Learning: A wetland is a
body of land that is saturated with water. The country’s wetlands have generally been grouped into
the following categories.
• Reservoirs and lagoons of the Deccan Plateau in the south and other wetlands of the southern west
coast
• Fresh water lakes and reservoirs extending from Gujarat eastwards through Rajasthan (Keoladeo
National Park) till Madhya Pradesh
• Saline stretches of Rajasthan, Gujarat, and the GulfofKutch
• Lagoons and delta wetlands of India’s east coast (Chilika Lake)
• Rivers and lakes of the montane region of Kashmir and Ladakh
• Fresh water marshes of the Ganga Plain
• Brahmaputra floodplains
• Marshes and swamps in the Himalayan foothills and the hills of north east and the mangrove forest
and other wetlands forming island arcs of the Andaman and Nicobar Islands
68. The answer is (d).
Wetlands play an integral role in the watershed ecology. Due to the combination of shallow water,
high nutrients level, and primary productivity, wetlands are ideal for the development of organisms
that form the base of the food web and feed many species of fish, amphibians, shellfish, and insects.
Wetlands are amongst the most productive ecosystems of the world, comparable to rainforests and
coral reefs. An extensive variety of species are part of the wetland ecosystem and provide habitat for
animals and plants with wide diversity of life. Wetlands provide a wide range of environmental, social,
and economic services. They function as natural sponges in trapping and releasing of surface water,
rain, snow-melt, groundwater, and flood waters. Wetlands also provide important industrial benefits
such as they form nurseries for fish that are critical to Australia’s commercial and recreational fishing
industries.
69. The answer is (d).
The peatlands are complex in biodiversity and are home to a variety of species. Their high carbon
content makes them vulnerable to incineration if they are drained.
They are a globally important carbon store hence any kind of unregulated exploitation of peatlands
can be detrimental to the environment and the climate, as it could release carbon emissions that have
been locked in peatlands for millennia.
Learning and Context: The Brazzaville Declaration was signed for promoting better management and
conservation of the world’s largest tropical peatlands found in Cuvette Centrale region of Congo Basin
from unregulated land use and also to prevent its drainage and further degradation. It was signed
jointly by the Democratic Republic of Congo (DRC), the Republic of Congo, and Indonesia on the
sidelines of the Third Partners Meeting of Global Peatlands Initiative held in Brazzaville, Republic of
Congo. The Cuvette Centrale region in the Congo Basin is the world’s largest natural tropical peatlands
and is about the size of England. It stores three years equivalent of global greenhouse gas emissions.
70. The answer is (a).
Statement 1: Estuaries are the bodies of water with surrounding wetlands where rivers meet the sea.
Estuaries are home to unique animal and plant communities that have adapted to brackish water
with a mixture of fresh water drained from the land and salty water from the sea. An ecotone is
defined as a transition area between two biomes, where two communities meet together
And integrate. It has the characteristics of bordering biological community and contains species
often not found in the overlapping communities. For example, the mangroves represent an ecotone
between marine ecosystem and terrestrial ecosystem and terrestrial ecosystem.
71. The answer is (b).
Statement 1: Cold water corals inhabit the colder deep waters of continental shelves and offshore
canyons, ranging from depths of 50 to over 1000 m. These corals lack zooxanthellae. They may even
build reef-like structures or may occur solitarily.
Statement 2: Unlike warm water corals, cold water corals do not contain symbiotic algae for
photosynthesis, instead they obtain all their energy from organic matter and zooplankton and grow
slowly. Cold-water corals can be found from tropical to Polar Regions and from shallow to deep seas.
Their distribution is largely defined by water temperatures, which must generally be between 4° and
12°C At high latitudes, these conditions are generally found in relatively shallow waters.
72. The answer is (b).
Aldabra is the ‘world’s second-largest coral atoll’. It is situated in the Aldabra Group of islands—
Seychelles in the Indian Ocean. Hence, statement 3 is incorrect. It became a UNESCO world heritage
site on November 19,1982.
73. The answer is (b).
Statement 1: These corals lack zooxanthellae. They may even build reef-like structures or may occur
solitarily.
Statement 2: Cold water corals inhabit the colder deep waters of continental shelves and offshore
canyons, ranging from depths of 50 to over 1000 m.
Statement 3: Unlike warm water corals, cold water corals do not contain symbiotic algae for
photosynthesis, instead they obtain all their energy from organic matter and zooplankton and grow
slowly.
Statement 4: Cold water corals can be found from tropical to Polar Regions and from shallow to deep
seas. Their distribution is largely defined by water temperatures, which must generally be between
4* and 12°C At high latitudes, these conditions are generally found in relatively shallow waters.
74. The answer is (c).
The forest cover is about 31% of the world’s land surface; world’s coral reefs amount to less than one
quarter of 1% of the entire marine environment. Hence, the forest cover has a greater area than coral
reefs. So, A is correct. Coral reefs can be found across the world. Cold water coral reefs have been
found off the coast of Norway and deep underwater in the Mediterranean Sea. So, R is incorrect.
75. The answer is (d).
Coral bleaching is a phenomenon which causes corals to expel zooxanthella (tiny pho to synthetic
algae), draining them of their colour and turning them white. The causes of coral bleaching are
• High sedimentation
• Exposure to direct sunlight for a long duration
• Exposure to unusually warm water with high temperature
• Acidification of water
• Chemical pollution and increased concentration of elemental nutrients
76. The answer is (c).
The phenomenon which leads to loss of zooxanthellae is known as coral bleaching. The causes of
coral bleaching are given above.
77. The answer is (c).
The Varadero Reef is located in Cartagena Bay and Cartagena is a port city along the Caribbean coast
in Colombia.The bay is part of a major waterway and is contaminated by industrial and sewage waste.
According to scientists, the reef should not be alive, and yet it has flourished in the highly polluted
environment and is home to a large number of corals, fish, urchin species and other marine organisms.
Studying this reef will provide clues to recover the world’s bleached and dying corals.
78. The answer is (d).
With increasing plastic pollution in the ocean, billions of pieces of plastic have snagged the coral reefs.
Research has revealed that pollution has led to soaring disease rates and damage is being done to a
vital habitat that already faces an existential threat from global warming.
• The study of 125,000 corals across the Asia-Pacific region, home to half the world’s reefs, had found
that 89% of those fouled by plastic were suffering from disease, while on the other hand, only 4% of
the corals were diseased in plastic-free coral reefs.
• The reason for high rates of disease was that the sharp plastic fragments cut the coral organisms,
while plastic fabrics smother them and block light and oxygen.
• Once a coral is infected, the disease usually spreads across the colony as plastics act as vessels for
microorganisms with pits and pores. If a piece of plastic happens to entangle on a coral, the coral has
only a slim chance of survival.
79. The answer is (a).
Global warming has led to coral bleaching but not in the Gulf of Aqaba, part of the northern Red
Sea. The reason due to which the corals fare well in the heat is attributed to their slow journey
from the Indian Ocean through the Bab al-Mandab Strait, between Djibouti and Yemen, where water
temperatures are much higher. Over the past 6,000 years, they underwent a form of selection through
a very, very hot body of water, and only those that could pass through that hot water body reached
the northern Red Sea and the Gulf of Aqaba.
80. The answer is (c).
Coral Reef Airborne Laboratory (CORAL) is NASA’s air-borne three-year field experiment. The aim
of this experiment is to survey the conditions of the major reefs of the world through remote-
sensing. CORAL will involve the analysis of this data using underwater photographs and reef primary
productivity. The condition of the entire reef systems in Hawaii, Palau, the Mariana Islands, and
Australia will be surveyed under this project.
81. The answer is (b).
Learning: Following points highlight the importance of coral reefs for an ecosystem.
• They protect shorelines by absorbing the wave energy. The survival of many islands is dependent on
them.
• They control the C02 in ocean water.
• They are the best breeding ground for fishes.
• They promote eco-tourism.
Statement 2: The excess C02 is used for calcification of reefs and in this way the additional C02 is
taken care of. So, coral reefs are helpful in controlling the Green House Effect as they absorb carbon ^
dioxide.
Statement 3: They usually form the first barrier against a tsunami or cyclone or sea waves by blocking
the force of the wave.
Statement 4: Corals have the highest primary productivity in the ocean ecosystem and not outside.
Amongst all, estuaries have the highest primary productivity.
82. The answer is (d).
Statement 1: Corals help in preventing coastal erosion, coastal flooding, and loss of property along
the shores especially during natural disasters.
Statement 2: Coral ecosystem forms the nursery and breeding ground for about a quarter of the
ocean’s fish, and thus provides revenue for local communities.
Statement 3: Research on corals has led to discovery of a new class of proteins capable of blocking the
HIV virus from penetrating T-cells. The proteins have been found in a coral from Australia’s northern
coast, which is being used in the treatment of cancer.
83. The answer is (a).
Learning: Generally, there are about twice as many coral species in the Pacific Ocean reefs as in the
Atlantic Ocean reefs. The important coral reefs in the Pacific Ocean are the Australian Barrier Reef and
the Coral Triangle.
84. The answer is (b).
Statement 1: Corals are invertebrate organisms that belong to Cnidaria, a large group of colorful and
fascinating animals.
Statement 2: Each individual coral animal is called a polyp. These coral polyps live in groups of
hundreds to thousands of genetically identical polyps that form a ‘colony’.
Statements 3 and 4: There are two types of corals— hard coral and soft coral. Hard corals are the ‘reef
buildingcorals and have a rock-like calcareous skeleton. Whereas the soft corals, which include seas
fans, sea feathers, and sea whips, do not have the rock-like calcareous skeleton. Instead these soft
corals grow wood-like cores for support and fleshy rinds for protection.
Statement 4: It is true that corals reefs are usually found in warm waters absent at the poles, but soft
corals are found in oceans from the equator to the north and south poles, generally in caves or ledges.
85. The answer is (c).
Corals have a symbiotic relation with algae, that is, they provide services for each other. Algae
provides food and energy to corals through photosynthesis and corals provide protection and support
for algae. In the event of coral bleaching, this symbiotic relationship breaks down, that is the algae is
lost from the coral, which means that the coral is effectively starving and it starts to look white.
Additional Information: When a coral bleaches, it is not dead but if the bleaching event persists for a
long period of time then it may face mortality.
86. The answer is (a).
Certain powerful synthetic gases such as HFC’s, SF6, and fluorinated gases are emitted through
industrial processes such as semiconductor and aluminium manufacturing. Fluorinated gases are
often used as substitutes for other ozone-depleting gases such as halons and HCFC’s.
Statement 2: Fluorinated gases have high global warming potentials (GWPs).
Statement 3: Fluorinated gases can be removed or destroyed naturally in upper atmosphere by
exposure to sunlight.
87. The answer is (d).
Apart from the pollutants mentioned in the question, other pollutants such as sulphur dioxide, volatile
organic compounds, nitrogen oxides, and carbon monoxide are also released from motor vehicles.
88. The answer is (d).
NF3 is used in semiconductor manufacturing and electronics industry as replacement for SF6 and PFC.
It is included under United Nations Framework Convention on Climate Change (UNFCC) as one of the
potential contributors of climate change.
89. The answer is (d).
Sulphur dioxide is used widely in petroleum and paper industries. It has a pungent odour and is
colourless. Along with paper manufacturing other sources of sulphur dioxide emissions include
volcanic eruptions and burning of coal and natural gas.
90. The answer is (d).
Statements 1 and 2: Coal powered plants and mines such as those engaged in gold production emit
mercury and other harmful gases and toxins.
Statement 3: The cement sector produces other harmful gases such as nitrogen oxides and sulphur
dioxide including mercury.
Statement 4: Manufacturing of batteries also releases mercury.
91. The answer is (b).
Statement 2: Iron does not determine the colour; it only aids in producing sparks.
Along with those compounds mentioned in the options, except iron others such as caesium, chlorine
also determine the colour of fireworks.
92. The answer is (b).
Statement 1: Bacteria living in soil are associated with nitrogen cycle that adds nitrogen into the
atmosphere.
Statement 2: This statement is wrong because reaction with ultraviolet radiation destroys nitrogen
oxides.
Statement 3: Lightning strokes act as natural source for nitrogen oxides.
93. The answer is (d).
Both statements are wrong, because one of the reasons for global warming and climate change is
brown carbon. Brown carbon absorbs radiation in the range of ultraviolet wavelengths.
94. The answer is (d).
Tanning as a process makes use of organic and chemical compounds while treating animals for
production of leather. These pollutants affect the environment. These chemicals are considered
carcinogens that lead to several health problems. The most significant example of the ill effects of
tanneries is Kanpur, UP—the city considered to be the largest leather exporter in India, its citizens
face various health problems due to the presence of many tanneries in the city.
95. The answer is (b).
Nitrates also pollute groundwater andnot just surface water. Not all oxides of nitrogen are greenhouse
gases. Only nitrogen in the form of nitrous oxide is a greenhouse gas. Nitrous oxide is also a more
potent greenhouse gas than carbon.
96. The answer is (d).
Polychlorinated biphenyls (PCBs) are synthetic organic chemicals that have been banned by the Union
Environment Ministry in line with the Stockholm Convention with complete prohibition by end of
2025.
Statement 2 is wrong because it’s widely used in manufacturing industries. PCBs are carcinogens
leading to varied health problems such as respiratory and skin problems.
97. The answer is (a).
Learning: Burning coal generates fly ash containing pollutants such as zinc, arsenic, and thorium,
which are detrimental to health. Instead of plutonium, uranium or thorium isotopes are the resultant
pollutants.
98. The answer is (a).
A primary pollutant is emitted directly from source point while reaction of such primary pollutants
in the atmosphere leads to the formation of secondary pollutants such as o/one ,md some formof
hydrocarbons.
99. I he answer is (c).
Oxides of nitrogen cause irritant rod haze in congested places and traffic affected regions, thereby
affected living tissues and textiles and some metals also increasing concentration of NO, retards the
process of photosynthesis and is damaging to leaves of plants.
100. The answer is (d)
Pollution by nitrates causes blue baby syndrome. In congested areas higher concentrationsnitrogen
oxides cause irritant red haze. It affects living tissues, textiles and some metals and also causes
respiratory diseases in children.
101. The answer is (d).
Following are the artificial sources ofradioactive pollution.
•Accidents in nuclear power plants and nuclear waste
• Nuclear weapon testing and explosion (nuclear fallout); the fallout contains radioactive substances
such as strontium-90 caesium-137, andiodine-131
• Uranium mining and mining of other radioactive material like thorium (monazite is the ore of
thorium)
• Radiation therapy and direct exposures to radiatorfor diagnostic purposes (e.g. X-rays), chemotherapy,
etc.
The slow nuclear radiations can emanate from a variety of sources viz. nuclear reactors, laboratories,
etc.
The natural sources include cosmic rays from space and terrestrial radiations from radio-nuclides
present in earth’s crust such as radium-224, uranium-238, Thorium-232, potassium-40, and carbon-14.
102. The answer is (a).
Statement 1 is correct as per the statistics given by Central Pollution Control Board (CPCB), an affiliated
organization of Union Environment Ministry.
Statement 2 is incorrect because cities such as Delhi, Pune, and Mumbai have high levels of PM2.S
which is generally toxic in nature and causes respiratory dis eases, in humans and animals alike
103. The answer is (b).
Statement 1: It is incorrect because PM2.5 are particles with a diameter of less than 2.5 micrometers
making it impossible to be visiblethrough the naked eye. One needs electronic microscope to desert
them.
Statement 3: PM2.5 with such small size easily enter circulatory system via throat causing heart and
respiratory problems
Statement 4: AQI includes 8 pollutants along with PM10 and they are PM2.5, sulphur dioxide (SO2),
lead, nitrogen dioxide (NO2), ammonia ozone, and carbon monoxide.
104. The answer is (c).
Statement 1: Black carbon is pure carbon formed due to incomplete combustion of biofuel and fossil
fuels.
Statement 2: Since black carbon has the tendency to absorb heat, being lighter in weight it rises to
greater heights thereby persisting in atmosphere for longer time period.
Statement 3: Black carbon particles quicken meeting of glaciers as they have the tendency to absorb
solarradiation and also terrestrial radiation, heating up glaciers and snow when deposited on it.
105. The answer is (d).
Burning of coal leads to release ofharmful chemicals and pollutants into the atmosphere. It is one
of the major causes for acid rain ire smog formation in regions around thermal power plant. Along
with pollutants mentioned in the options, coal plants also emit arsenic, volatile organic compounds,
cadmium, carbon monoxide, and other heavy metals.
106. The answer is (b).
Learning: Combustion of fossil fuels constitutes around 87% of carbon dioxide emissions from human
source. Rest of the emissions are caused by forests and land use, industrial processes, etc.Coa
produces a major proportion of CO2.
107. The answer is (b).
Photochemical smog is a type of air pollution that is caused due to the reaction of airborne pollutants’
mixtures such as nitrogen oxides and volatile organic compounds in the presence of sunlight Apart
from nitrogen oxides, mixtures may also include duction nitrates and aldehydes, etc. Automobiles,
coal fired power plants aid in the production of these pollutants most common in summer due to high
temperatures.
108. The answer is (d).
Statement 1:This statement is proved right as seen in case of Delhi where, Haryana andPunjab practice
stubble burning further aggravating smog and pollution In the city.
Statement 3: In cold temperatures, volatile organ ic compounds and nitrates remain In the atmosphere
for long period of time further aggravating smog.
109. The answer is (b).
Photochemical smog is a type of air pollution that is caused due to the reaction of air borne pollutants
mixtures such as nitrogen oxides and volatile organic compounds in the presence of sunlight Apart
from nitrogen oxides, mixture may also include peroxyacyl nitrates and aldehydes, etc. Automobiles,
coal fired power plants aid in the pro duction of these pollutants —most common in sum mer due to
high temperatures.
110. The answer is (c).
Statement 1: Fly ash is a fine powder. It is a by-product released during burning of pulverized coal in
thermal power plants. Fly ash contains pollutants such as aluminium and siliceous material, and other
heavy metals such as arsenic, beryllium, barium, cadmium, and chromium. Hence, fly ash is known to
be detrimental to health if the pollutants are present in excess quantities.
Statement 2: Fly ash, along with other heavy metals, also contains siliceous material which causes
lung disease known as silicosis.
111. The answer is (c).
Maharashtra was the first state in the country to adopt a policy on fly ash utilization and the policy
mandates 100% use of fly ash generated from thermal power plants and biogas plants for construction
activities.
• Fly ash is used to make bricks, blocks, tiles, cement, etc.
• The release of fly ash into the atmosphere can be reduced by making it available as a raw material
for construction purposes
112. The answer is (c).
Fly ash is a fine powder. It is a by prod uct produced during burning of pulverized coal in thermal
power plants. Though fly ash has negative impact on environment, if utilized properly it could help in
mitigating environmental consequences and provide certain economic benefits as well. Fly ash can be
utilized to make bricks, cement, and concrete.
Statement 1: The utilization of fly ash will reduce land used for dumping of fly ash in landfills and also
reduce the associated environmental damage caused by pollution of soil and groundwater.
Statement 2: This statement is correct as fly ash canhelp In mitigation of air emissions which are
released during the manufacture of cement. Fly ash can be used as an manufacture of cement and
thereby help in reducing greenhouse gas emission
113. the answer is (c)
Furnace oil is the residual product obtained from distillation of crude oil. It is dark, viscous and heavier
than naphtha and gasoline, making it the heaviest commercial fuel that can be obtained from crude
oil. Hence, statement 1 is correct.
Statement 2 is correct. It has commercial usages and can be used in fertilizer manufacturing, brick
kiln, etc. It is also used in thermal power stations. It is also used for power generation in engines such
as that of marine vehicles, slow speed engines, and gas turbines.
114. The answer is (c).
Particular chemicals are used in fireworks that produce a variety of colours. For example strontium
carbonate gives red colour, sodium nitrate gives yellow colour, and copper chloride gives blue colour
to the fireworks.
Statement 2: Potassium carbonate is highly reactive when mixed with sulphur and it is dangerous
with respect tospontaneity in ignition, it is not safe to use in fireworks and hence the fireworks which
use potassium carbonate are banned in India.
115. The answer is (b).
Statement 1: Arsenic is naturally found in earth’s crust and it’s naturally available in the environment.
Hence, statement 1 is incorrect. The pollution of water can be caused by either natural activities or
anthropogenic activities such as release of industrial waste.
Statement 2: Groundwater at deeper levels have higher concentrations of arsenic due to more
exposure to older sediments and rocks containing arsenic Use of deep tube wells is drawing this high
concentration arsenic groundwater closer to the surface making it a form of groundwater pollution.
Arsenic pollution can be mainly found in Ganga basin and the highest number of people affected with
this arsenic pollution are from West Bengal
116. The answer is (c).
Statement 1: With increase in temperature of water, the level of dissolved oxygen in the water
decreases. This affects the aquatic organisms and the marine ecosystem.
Statement 2: The top layer of water surface with higher temperatures prevents mixing of water
properly. In warm temperature, plants grow at increasedrate and moderately warm temperature
increases primary productivity of plants. I, may also lead to algal blooms which contribute to anaerobic
conditions in deep water.
117. The answer is (a).
Fly ash contains higher concentrations of heavy metals such as arsenic, beryllium, barium cadm.
um, copper, chromium, lead, molybdenum’ mercury, nickel, radium, selenium, thorium, uranium’
vanadium, and zinc. This causes groundwater pollution and soil pollution when fly ash is dumped in
landfills. Examples of such contaminated mines are Raniganj and Jharia coal mines.
118. The answer is (a).
Arsenic contamination is mainlyfound among the Himalayan Rivers. Along with geogenic nature
of river plains of India, anthropogenic activities such as discharge of industrial waste, overuse of
groundwater, burning of coal, and e-waste are also important sources of arsenic contamination.
119. The answer is (d).
Statement 1: Arsenic contamination is caused by anthropogenic and natural activities such as
geological processes, which are reported mainly in West Bengal.
Statement 2: Fluoride concentration is found mainly in semi-arid regions such as in Rajasthan, Gujarat,
and some parts of Haryana and Karnataka.
120. The answer is (a).
Statement 1: Higher concentrations of nutrients such as nitrates cause eutrophication and algal
blooms, resulting in anaerobic condition which leads to loss of aquatic life.
Statement 2: Higher temperatures of water are mostly due to hot water discharge from industries
such as thermal plants and nuclear plants. This results in decrease in the dissolved oxygen in water
and higher temperature of water which affects marine plants and animals.
Statement 3: Presence of larvae eating fish indicates fresh water and not water pollution. Hence,
statement 3 is incorrect.
121. The answer is (b).
Statement 1: Silting of water bodies affects egg laying patterns and excessive silt will damage the eggs
and affect the survival rate of eggs. Hence, statement 1 is incorrect. Statement 2: This statement is
incorrect. Corals are adversely affected by siltation and excessive sedimentation. Corals prefer clear
water than with sediments.
Statement 3: This statement is correct. Soil erosion takes place due to unsustainable and improper
agriculture practices, here the silt acts as a vehicle of transporting pesticides into water bodies due to
water runoff, thus polluting groundwater.
122. The answer is (d).
Light pollution is a type of pollution that arises due to use of artificial light sources especially in the
night environment. This pollution causes visual disturbance. Light pollution affects in the following
ways.
• It affects human health and psychology, changing sleep pattern, visual capacity, and altering Orcadian
rhythms.
• It also affects reproductive cycles of animals that are sensitive to light, especially those which breed
during night times, night blooming flowers, and nocturnal animals. It causes disruption in ecosystems.
• Migration of birds is also affected by the light pollution because many birds use stars as source of
direction or navigation.
123. The answer is (b).
E-waste generation releases certain chemical elements such as beryllium, lead, and mercury into the
environment. Heptachlor is an exception since it’s an insecticide. Plutonium is a radioactive element
that is not used in computer parts or in day to day activities.
124. The answer is (d).
Statement 1: Electronic waste consists of potentially harmful components such as nickel, phosphorous,
lead, beryllium, cadmium, or brominated flame retardants. Such elements when disposed untreated
might cause health risks. Brominated flame retardants are generally used in plastics, electronics, and
industrial uses.
125. The answer is (b).
Learning: The Extended Producer Responsibility (EPR) is a mechanism used by the government to place
the responsibility of disposal of e-waste on the producer or manufacturer. It also includes addition
of all environmental costs associated with a product throughout the product life cycle and after the
life of product (disposal) to the market price of that product. Accordingly, for sound environment
management, the producer has to set up collection systems to meet the cost involved.
126. The answer is (c). Justification: Usage of compost in agriculture helps enhance water holding capacity
of the soil and also replenishes the nutrients depleted from the soil. Compost helps in making roots
stronger as the soilbecomes porous to hold the compost. City compost unlike farmyard manure will
be weed free. It is rich in organic carbon and together with fortification increases the productivity
of agriculture. It helps increase crop yield with bigger size and shelf life. Due to its significance, Solid
Waste Management Rules, 2016 have made it mandatory for fertilizer companies to co-market
compost. Hence, both the statements are correct.
127. The answer is (a).
Justification: The Plastic Waste (Management and Handling) Rules, 2011 were notified for plastic
waste management. In 2016, the government notified the new Plastic Waste Management Rules,
with the following amendments.
• Increase the thickness from 40 to 50 microns for carry bags and a minimum of 50 microns thickness
for plastic sheets.
• Expanding the jurisdiction of plastic waste management from municipal area to rural area.
• It also has the provisions for phasing out non-recyclable multi-layered plastic in two years.
• The rules do not explicitly state anything about thermoset plastic. Hence, statement 3 is wrong.
128. The answer is (c).
Both the statements are correct. Refuse derived fuels are fuels derived from wastes such as municipal
waste and solid waste. Refuse derived fuel must be incinerated at more than 1,000°C to reduce
release of dioxin and furan formation. Hence, monitoring of cement kilns by pollution control board
is needed. Burning of plastic should be the last resort.
Statement 2 is correct. More than 80% of the plastics used are not recycled and are thrown away. It
has led to environmental damage especially to marine animals.
129. The answer is (b).
Statement 2: On World Environment Day, India pledged to phase out single use plastics by 2022.
Statement 1: This is incorrect because it is only 9% not 29%, plastic produced worldwide that is getting
recycled. The remaining is just being burnt or dumped in landfills or oceans.
130. The answer is (c).
The #CleanSeas campaign was launched by the UN Environment with the aim to eliminate major
sources of marine litter such as micro plastics in cosmetic. It urges governments to reduce plastic by
adopting plastic reduction policies.
131. The answer is (b).
India hosted World Environment Day on June 5,2018 with the theme’Beat Plastic Pollution’ to combat
single use plastic pollution. Government of India is committed to perform various activities such as
beach clean-ups and plastic clean-up drives in public places to reduce plastic pollution.
132. The answer is (c).
Both statements are correct.
• The ‘Great Pacific Garbage Patch’ is a collection of marine debris. This patch extends from the West
Coast of North America to Japan and it has the biggest marine trash vortexes in the world.
• This garbage patch is surrounded by’North Pacific Subtropical Gyre’. Since the gyre is in circular
motion debris are drawn to the centre, which is stable and calm and thereby gets trapped.
• These ocean gyres are formed by Earth’s wind patterns and the forces created by rotation of the
planet such as the Coriolis force.
133. The answer is (b).
Statement 1 is incorrect. Micro plastics are small plastic pieces less than 5mm long. They are considered
to be unsafe for consumer products by the Bureau of Indian Standards (BIS). Micro beads were used
in cleansers for the first time in 1972 when they got patented. Due to their abrasive nature, they are
used in toothpastes. Micro plastics pose a potential threat to aquatic life as due to their small size
they are engulfed by the marine life, which leads to bio-magnification in the food chain.
134. The answer is (b).
The Ministry of Environment notified an amendment to the Plastic Waste Management (Amendment)
Rules, 2018, adding multi-layered plastics (MLP) to the list of plastics that it expects to phase out. In
2016, it had added MLP to an exception list notifying that it can have some alternate uses. But, in the
recent amendment, it is phasing out the MLPs that do not have any alternate use. MLP is widely used
in food packaging industry.
Statement 2 is correct. The same amendment issued the prescription of a central system which will
carry the names of producers, brand-owners, and importers. This will allow agencies to better track
and control the usage of plastics.
135. The answer is (d).
Vermicompost is a product of composting by using species of worms. It is the end product of breakdown
of organic matter by earthworms.It s highly degraded in nature and is rich in nitrogen phosphorous,
and potassium, Livestock wastes, processing waste, etc. form the feedstock for vermiform post.
136. The answer is (d).
Bioremediation is a process used to treat contaminated media,including water, soil and subsurface
material, by altering environmental conditions to stimulate growth of microorganisms and degrade
the target pollutants. Bioremediation helps treat oil spills with use of certain species such as algae,
plants, archaea, and bacteria. It helps increase microorganisms number in the oil spilled area. By
metabolizing microbes, hydrocarbons can be broken down.
137. The answer is (b).
Statement 1 is incorrect. Vermicomposting is safe in toilets and is sustainable because it uses less
water in processing excreta leaving behind no fly or odour. This process does not work in regions
having temperature above 35°C as worms would not be protected.
138. The answer is (b).
Learning: Bioleaching is one of the technologies of bioremediation. Other technologies are bioventing,
bio-stimulation, bio-augmentation, land farming, etc. Bioleaching involves extraction of metals from
their ores using living organisms instead of harmful chemicals to degrade organic constituents.
139. The answer is (d).
Bio-pesticide consists of protozoans, fungus and bacterium. Microbial pesticides also include viruses
such as entomopathogenic fungi, or viruses. A biological control being developed for use in the
treatment of plant disease is the fungus Trichoderma.
140. The answer is (c).
justification: Both the statements are correct. Fungi act as bio-fertilizers by competing for nutrients
with pathogens. They compete for space too. In a way they suppress harmful pathogen’s growth
such as weeds. Some fungi help in degradation of insecticides and break them to basic elements and
water. They are also useful in bioremediation, for example, to bio degrade uranium oxides from sites
of radioactive pollution.
141. The answer is (b).
Rhizofiltration is the mechanism of using hydroponically cultivated plant roots to remediate
contaminated water through absorption, concentration, and precipitation of pollutants. Rhizofiltration
involves filtering contaminated groundwater, surface water, and waste water through use of plant
roots to remove toxic substances or excess nutrients. It is similar to the phytoremediation method
used to treat aquatic environment. It is more cost efficient.
142. The answer is (d).
Learning: Bioremediation employs living organisms such as microbes and bacteria to remove
pollutants and toxins from soil and water. They also help in cleaning up of oil spills or contaminated
groundwater. Various bacteria such as mycobacterium, haemo-philus, ralstonia, and rhodococcus and
some other such as pseudomonas and paenibacillus are also used.
143. The answer is (d).
Learning: Bioremediation is a method of treatment of pollutants by using microorganisms. Generally,
it is done in non-polluted environment to break down organic matter. With contamination of site,
microorganisms that are sensitive to pollution would die. Hence organisms that feed on pollutants
can be used along with fertilizers and oxygen. Sites with chemical contamination pose toxicity threat
to microorganisms.
144. The answer is (c).
All the options given in the question except cloth are non-biodegradable in nature. Cotton cloth,
paper, woollen clothes, and wood are bio-degradable.
145. The answer is (a).
Option (a): Silicosis disease is an occupational hazard caused due to inhalation of silica. It affects lungs
and causes shortness of breath, breathlessness, lung tissue damage, etc. The disease goes unnoticed
since silica is non-irritant and odorless. People having occupations in construction industries,
quarrying, etc. are more vulnerable to this disease. It can be chronic in nature and is incurable.
Symptomatic treatment and surveillance is undertaken to lessen the chances of disability.
Option (c): Meningitis is a rare infection that affects the delicate membranes—called meninges that
cover the brain and spinal cord.
146. The answer is (c).
Learning: The instances of blood rain or red coloured rain have been reported since 1896 in parts of
Sri Lanka and Kerala. The latest blood rain report came in 2012. The cause for this kind of rain is due
to the locally found green algaeTrentepohliaannulata. The reason for the red colour is that the spores
of this green algae are carried by air.
147. The answer is (d).
None of the pollutants given in the options are matched correctly with the associated disease.
Minamata disease is caused due to methyl mercury, which got its name from Minamata Bay, Japan
where it was first discovered. Black foot disease is caused due to the pollutant arsenic, which is
leached to the groundwater from rocks and soil. While Blue baby syndrome is caused due to excessive
nitrate pollution. Other pollution associated diseases are
• Itai-itai: Cadmium
• Skeletal fluorosis: Fluorides
• Yokkaichi asthma: Sulfur oxide
• Asbestosis: Asbestos
• Silicosis: Silica
148. The answer is (c).
Statement 1: Methyl mercury and related short chain alkyl and mercurial compounds are most
dangerous to humans, as they accumulate in edible fish tissue. The use of lime helps in reducing
mercury levels in fish.
Statement 2: This statement is correct. Corroded galvanized pipes pollute drinking water by adding
excessive cadmium to it, which could cause renal tubular damage.
149. The answer is (a).
Statement 1: Aerosols affect the rainfall patterns by modifying the size of cloud particles. It in turn
affects absorption and reflection of sunlight thereby affecting the Earth’s energy budget.
Statement 2: Aerosols also have a role in destruction of stratospheric ozone through chemical
reactions such as reactive chlorine destroys ozone in stratosphere.
Statement 3: Aerosols help cool down the planet by scattering the sunlight back in the space.
150. The answer is (d).
The growing number of artificial lights and increased brightness in the night sky cause disruption of
ecosystems, which in turn has certain consequences.
• It affects crop pollination, as light pollution affects nocturnal insects by reducing pollinating activities.
• Light pollution contributes to climate change too, by adding excess heat into the air.
• Urban light installations ‘dramatically alter7 the behaviour of nocturnally migrating birds.
• 99% of people living in the United States and Europe cannot see the Milky Way because of light
pollution.
151. The answer is (c).
The centre’s plan for a graded response to pollution will need to take cognizance of a new pollutant
mentioned in the report—ozone, which is the reason for the alarming rate of deaths. A graded action
plan can help address this problem.
152. The answer is (c).
Air pollutants lead to neurological problems by affecting brain through blood stream. Dementia is one
of the most prevalent pollution led brain diseases. It involves loss of brain cells that affect every day
activities and is found more among Alzheimer’s patients. Delhi and Ontario with worst air pollution
are more vulnerable to such diseases.
153. The answer is (c).
Statement 1: Lead in paints can have adverse impact on health, especially during pregnancy and it
can lead to coordination problems and loss of memory and learning skills, both among adults and
children, etc.
Statement 2: This statement is correct. The symptoms of lead contamination are abdominal pain,
nausea, depression, etc.
154. The answer is (d).
We are giving examples to illustrate the statements given in the question.
Statement 1: Exotic species may disturb the local food chain or create scarcity of competitive resources.
Statement 2: Natural hazards like tsunami or cyclone can uproot coastal mangroves.
Statement 3: In the plants, any disturbance in the native forests such as clearing the forest causes a
change in the species distribution. Such changes may bring competition from secondary forests.
155. The answer is (c).
The Himalayan State Regional Council was constituted by NITI Aayog for the sustainable development
of the Himalayan Region in India. It aims to review and implement the road map for sustainable
development in the region.
156. The answer is (a).
Statement 1: Exotic means introduced from outside. But the oak trees are native to the region.
Statement 2: Decay of organic matter in this case would yield acidic products not alkaline.
Statement 3: Based on statement 1, statement 3 is also wrong, because native trees help maintain the
ecosystem balance rather than hurt the keystone species.
Statement 4: They soak nearly 60% of groundwater of the region.
157. The answer is (a).
Learning: Fossil fuels such as coal, when burnt in power stations produce nitrogen oxides and sulphur
dioxide, which when react with water form acid rain. These lead to formation of acid rain that has
deleterious effect on the overall ecology of the region as well as the human made monuments.
158. The answer is (a).
Justification: The app helps in knowing solar energy potential on monthly and yearly basis, at a
given location with the use of geostationary satellite data.With the help of Digital Elevation Model,
obstruction of sunlight that may be caused due to rough terrain can be calculated.
Statement 2: It is incorrect It can be GPS through smartphone.
Statement 3: It is incorrect. This app needs internet connection to calculate the results and the
complete report can be saved as a PDF file
159. The answer is (d).
All the statements given in thequestion are correct. International Solar Alliance (ISA) is a coalition of
solar resource rich countries lying fully or partially between the Tropic of Cancer and the Tropic of
Capricorn. The alliance was created to specifically address energy needs of the countries by harnessing
solar energy. National Institute of solar energy (NISE), Gurugram houses ISA headquarters. It is an
international intergovernmental organization of 121-member countries and countries lying beyond
tropics can join ISA as ‘partner countries’.
The statement 2 is correct as the coordination between members happens through activities and
programmes on voluntary basis. Statement 3 is also correct. The Private Financing Advisory Network
(PFAN) is a multilateral public private partnership initiated by the United Nations Framework
Convention on Climate Change (UNFCCC) and the Climate Technology Initiative. PFAN identifies and
nurtures promising, innovative, clean, and renewable energy projects by bridging the gap between
investors, clean energy entrepreneurs, and project developers.
160. The answer is (c). Justification: Statements 1 and 3 are correct. To bring down the costs of solar power
production ISA has come up with three tasks of which the two given in these statements are true.
Statement 3 is incorrect because ISA till now has not set any legally binding commitment. It has set
tasks just to increase global demand which can help bring down costs of solar power production.
161. The answer is (c).
Learning: As of 2016, the installed capacity of wind power in India is nearly 27.6 GW, mainly spread
across the south, west, and north regions. India is the 4,hlargest country after China, USA, and Germany
in installed wind power capacity. Tamil Nadu has the maximum capacity of wind power with 35% or
installed capacity followed by Maharashtra.
162. The answer is (c).
Both the statements are correct Bio-methane is produced from fresh organic matter through anaerobic
digestion and is less harmful when produced under controlled conditions. Bio-methane production
has reduced amount of greenhouse gas emissions into the air.
163. The answer is (b).
Background: For the development of biofuels, a national policy on biofuels was announced in the year
2009. Under the policy, oil marketing companies need to sell petrol with 10% of ethanol blending.
Statement 1: Price administration is done based on ethanol availability.
Statement 2: This is an eco-friendly way to convert ethanol from bio-feed stock.
According to BIS specifications, biodiesel is permitted for sale to all consumers by blending it with
high speed diesel.
164. The answer is (b).
Statement 1: Tight gas is produced by hydraulic fracturing of rocks because of its low permeability.
This natural gas is taken from reservoir rocks. To produce tight gas, stimulation is needed by a large
hydraulic fracture treatment and/or it is produced using horizontal wellbores. Through this technique
shale gas, coal-bed methane, and tight reservoirs of carbonate are also produced.
Statement 2: There is a difference between coal-bed methane (CBM) and coal mine methane (CMM).
CBM refers to methane that is found in coal seams and it is formed through the process of coalification.
While CMM is produced from surrounding strata of rocks and coals, when methane is released during
mining activities; it is this methane which is called CMM.
165. The answer is (a). Learning: Biogas is that type of biofuel which is produced from decomposition of
organic waste. It has 75% of methane content making it the largest component of the biogas. When
burnt it leaves no smoke or residue behind. It leaves carbon dioxide and methane when broken down.
Its heating capacity is high and it generates gases such as methane, carbon dioxide, hydrogen, and
hydrogen sulphide during heating.
166. The answer is (c). Learning: Gasification is done using a mixture of coal, water, air and/or oxygen.
Syngas is composed of carbon dioxide, hydrogen, carbon monoxide, water vapour, and methane. It
is also called coal gas as it is a product of coal gasification. It is used in internal combustion engines
because of its combustible nature.
167. The answer is (a). 1
Statement 1: The auto-ignition temperature of CNG is over 500°C. It also has a narrow range of
flammability which makes it safer than pet rol and diesel. CNG mixes well with air and disperses
quickly clearing the area of fire.
Statement 2: It does not contain lead which makes it safer than other fuels.
Statement 3: CNG has a higher calorific value than diesel and petrol. So, statement 3 Is incorrect
Learning: CNG is mainly composed of methane and natural gas that is compressed to acquire standard
atmospheric pressure to 0% of its volume.
168. The answer is (a).
Biofuels are renewable energy sources that are made from organic waste. They have a bigger role
to play in reducing carbon dioxide as they are pollution free, can substitute fossil fuels, and also are
economical in cost. They are currently made ‘only from molasses’ in India that is considered first
generation biofuel. The target has been set for 10% of ethanol blending in petrol by 2022. Twelve bio
refineries have been set up in the country by oil PSUs to enhance the ethanol production capacity. On
the lines of ‘waste to wealth; GobarDhanscheme is focused on the production of bio-CNG.
169. The answer is (b).
Fame India Scheme (Faster Adoption and Manufacturing of (Hybrid &) Electric Vehicles in India) was
formulated by the Department of Heavy Industry as part of the National Electric Mobility Mission
Plan (NEMMP) 2020 to promote manufacturing of hybrid and electric vehicle technology to ensure
sustainable growth of this sector. Phase I was launched in 2015 and Phase II was launched in March
2019.
170. The answer is (a).
Option (d): Liquefied petroleum gases are all heavier than air and will collect in low places when not
confined.
Option (a): CNG is considered safer than other fuels in the event of a spill, because natural gas is
lighter than air and disperses quickly when released.
Learning: CNG is made by compressing natural gas (which is mainly composed of methane, CH4), to
less than 1% of the volume it occupies at standard atmospheric pressure. The cost and placement of
fuel storage tanks is the major barrier to wider/quicker adoption of CNG as a fuel
171. The answer is (a).
Statement 1: It is correct. Flammable ice that has methane being trapped in crystals of water is
also known as methane hydrate. Moderate pressure and low temperature regions are favorable for
methane to be trapped. Methane hydrates are in flammable in nature.
Statement 2: This statement is incorrect. Permafrost appears on top of it. Permafrost is frozen chunks
of ice that often contain carbon and volatile gases.
Statement 3: It Is correct. Most of its reserves are located deep In ocean floors, especially continental
shelves
Statement 4: India, USA, China, and Canada all are looking at it as an alternative energy source.
172. The answer is (c).
Methanol is a non drinking type of alcohol which is used in fuel creation, as a solvent and antifreeze.
It is highly inflammable and volatile in nature.
Statement 1: Methanol can be blended into gasoline to produce an efficient fuel known as methyl
tertiary butyl ether (MTBE) which can have lower emissions than conventional gasoline.
Statement 2: It is correct. Methanol is cheaper than most fossil fuels.
Statement 3: Methanol is also used in biodiesel a renewable type of fuel that can be used in place of
conventional fuel. It is currently being imported from Iran and Saudi Arabia. Efforts are being made to
study automobile standards to use methanol as alternative fuel, as done in China.
173. The answer is (a).
Statement 1: Hydrogen cyanide (HCN) also called prussic acid is highly volatile, colourless, and an
inflammable liquid which is extremely poisonous.
Statement 2: This statement is correct. It acts as a precursor to chemical compounds from
pharmaceutical to polymers.
Statement 3: It acts as a weak acid, when formed as a solution with water, it forms hydrocyanic acid.
Statement 4: It is correct. It is used in astronomical processes and it is detected in atmosphere of
carbon star.
174. The answer is (a).
Statement 1: It can be released when crude oil is brought to the surface. It can be used as a domestic
and industrial fuel.
Statement 2: Russia, Norway, UK, and the Netherlands are the major producers of natural gas.
Statement 3: This statement is incorrect. Natural gas in India is found in offshore areas of Mumbai,
deltas of Godavari and Krishna and also in Jaisalmer region of Rajasthan. Natural gas is also found in
north eastern region of the country, such as Tripura.
Statement 2: It does not contain lead which makes It safer than other fuels.
Statement 3: CNG has a higher calorific value than diesel and petrol. So, statement 3 Is Incorrect.
Learning: CNG is mainly composed of methane and natural gas that is compressed to acquire standard
atmospheric pressure to < 1 % of its volume.
175. The answer is (a).
Biofuels are renewable energy sources that are made from organic waste. They have a bigger role
to play in reducing carbon dioxide as they are pollution free, can substitute fossil fuels, and also are
economical in cost. They are currently made ‘only from molasses’ in India that is considered first
generation biofuel. The target has been set for 10% of ethanol blending in petrol by 2022. Twelve bio
refineries have been set up in the country by oil PSUs to enhance the ethanol production capacity. On
the lines of waste to wealth; GobarDhonscheme is focused on the production of bio-CNG.
176. The answer is (b).
Fame India Scheme (Faster Adoption and Manufacturing of (Hybrid &) Electric Vehicles in India) was
formulated by the Department of Heavy Industry as part of the National Electric Mobility Mission
Plan (NEMMP) 2020 to promote manufacturing of hybrid and electric vehicle technology to ensure
sustainable growth of this sector. Phase I was launched in 2015 and Phase II was launched in March
2019.
177. The answer is (a).
Option (d): Liquefied petroleum gases are all heavier than air and will collect in low places when not
confined.
Option (a): CNG is considered safer than other fuels m the event of a spill, because natural gas is
lighter Tr.an air and disperses quickly when released.
Learning: CNG is made by compressing natural gas (which is maily composed of methane, CH4), to
less than 1% of the volume it occupies at standard atmospheric pressure. The cost and placement of
fuel storage tanks is the major barrier to wider/quicker adoption of CNG as a fuel.
178. The answer is (a)
Statement 1: It is correct. Flammable ice that has methane being trapped in crystals of water is
also known as methane hydrate. Moderate pressure and low temperature regions are favorable for
methane to be trapped. Methane hydrates are inflammable in nature.
Statement 2: This statement is incorrect. Permafrost 1 appears on top of it. Permafrost is frozen
chunks of I ice that often contain carbon and volatile gases. Statement 3: It is correct. Most of its
reserves are 1 located deep In ocean floors, especially continental 1 shelves.
Statement 4: India, USA, China, and Canada all are f looking at it as an alternative energy source.
179. The answer is (c).
Methanol is a non-drinking type of a1cohol which is used in fuel creation, as a solvent and anti-freeze.
It is highly inflammable and volatile in nature.
Statement 1: Methanol can be blended into gasolineto produce an efficient fuel known as methyl
tertiary butyl ether (MTBE) which can have lower emissions than conventional gasoline.
Statement 2:it is correct. Methanol is cheaper than most fossil fuels.
Statement 3: Methanol is also used in biodiesel, a renewable type of fuel that can be used in place of
conventional fuel. It is currently being imported from Iran and Saudi Arabia. Efforts are being made to
study automobile standards to use methanol as alternative fuel, as done in China.
180. The answer is (a).
Statement 1: Hydrogen cyanide (HCN) also called prussic acid is highly volatile, colourless, and an
inflammable liquid which is extremely poisonous.
Statement 2: This statement is correct. It acts as a precursor to chemical compounds from
pharmaceutical to polymers.
Statement 3: It acts as a weak acid, when formed as a solution with water, it forms hydrocyanic acid.
Statement 4: It is correct. It is used in astronomical processes and it is detected in atmosphere of
carbon star.
181. The answer is (a).
Statement 1: It can be released when crude oil is brought to the surface. It can be used as a domestic
and industrial fuel.
Statement 2: Russia, Norway, UK, and the Netherlands are the major producers of natural gas.
Statement 3: This statement is incorrect. Natural gas in India is found in offshore areas of Mumbai,
deltas of Godavari and Krishna and also in Jaisalmer region of Rajasthan. Natural gas is also found in
north eastern region of the country, such as Tripura.
182. Ans. (A)
Sol. Statements 1: This proposal is to be submitted through the concerned forest department of the state
government. Concerned ministries and regulators after giving clearance, will decide the compensation
amount for the loss of forest land. Hence, Statement 2 is incorrect.
183. Ans. (A)
Sol. Statement 1 is correct. Ecosystem approach is a holistic approach to conservation which involves
the application of appropriate scientific methodologies focused on levels of biological organization
that encompass the essential processes, functions, and interactions among organisms and their
environment. It recognizes the importance of cultural diversity and human component as significant
parts of ecosystems. Hence, Statement 2 is incorrect.
184. Ans. (D)
Sol. Environment Impact Assessment (EIA) involves identification and evaluation of potential impacts
of development and projects on the environmental system. It involves all the factors mentioned in
the options for assessment making. A public hearing is scheduled and only after a comprehensive
examination, EIA is prepared and authorization is given as per the official procedure.
185. Ans. (D)
Sol. The factors that are considered to categorize the projects into Category A and Category B arespatial
extent of potential impacts, potential impacts on human health, and potential impact on natural and
manmade resources.
Category A requires prior clearance from the Central Environment Ministry and Category B requires
clearance from State Environment Authority.
186. Ans. (D)
Sol. The discount rate is the rate at which a society as a whole is willing to trade off present for future
benefits; because a rupee received today is considered more valuable than the one received in the
future.
This is considered when weighing the decision to undertake a project with long-term benefits (for
example, wetland protection programmes) versus one with short-term benefits and long-term costs
for example logging forests near aquatic ecosystems.
187. Ans. (C)
Sol. Statements 1, 2, and 4: Suppose a thermal power plant is to be setup in an area. The pattern of
predominant wind flow would affect the distribution of emissions from the plant in the locality. Also,
the waste water of waste products emitted by the plant should not be very close to local water storage
structures. The sanitary fills should not be destroyed by the plant as it will affect waste management
in the area. One can take many examples and justify these statements. So, Statements 1, 2, and 4 are
correct.
Statements 3 and 5: Availability of eateries in the locality and calorie intake of the population are
remote considerations and trivial matters when it comes to an EIA.
188. Ans. (B)
Sol. The harmful emissions that are identified for regulations in different Bharat Stages are car-bon
monoxide (CO), particulate matter (PM), oxides of nitrogen (NOx), oxides of sulphur, and unburnt
hydrocarbons (HC).
189. Ans. (A)
Sol. With a view to document the longterm changes in composition of trace species of the atmosphere
as a result of changing land use pattern, a global programme was commissioned by the World
Meteorological Organization (WMO) called Background Air Pollution Monitoring Network (BAPMoN).
It’s a part of the Global Atmospheric Watch (GAW) Programme. So, Statements 2 and 3 are correct.
India had set up 10 such BAPMoN stations that collect rain water samples for chemical analysis. It also
measures atmospheric turbidity. So, Statement 1 is incorrect.
190. Ans. (B)
Sol. In India, National Air Quality Monitoring Programme (NAMP) is implemented by the Central Pollution
Control Board (CPCB). NAMP is undertaken in India to
• determine the status and trends of air quality;
• ascertain the compliance of National Ambient Air Quality Standards (NAAQS) and identify
non-at- tainment cities;
• understand the natural process of cleaning in the atmosphere; and
• recommend and undertake preventive and corrective measures.
191. Ans. (C)
Sol. Justification: National Clean Air Programme has the following objectives.
• Prevention, control, and abatement of air pollution
• Augmenting the air quality monitoring network across the country
• Strengthening the awareness and capacity building activities
As per the programme, the year 2017 is taken as the base year. Various stakeholders, local bodies,
state governments, and Central Government are involved in it. Statement 2 is incorrect. The tentative
target of 20%-30% reduction of PM2.5 and PM 10 has to be met by 2024.
192. Ans. (D)
Sol. Justification: Both statements are incorrect. National Mission for Clean Ganga (NMCG) comprises two
committees—the Executive Council and the Governing Council, both headed by the Director General
of NMCG. Hence, Statement 1 is incorrect. Statement 2 is incorrect because NMCG can accord project
approval of up to Rs1,000 crores only.
193. Ans. (A)
Sol. The seven-star rating system was launched by the Ministry of Housing and Urban Affairs under the
Swachh Bharat Mission (Urban) based on cleanliness reports. It is for scientific assessment of solid
waste management based on various activities which include collection from door to door, sweeping,
etc.
194. Ans. (B)
Sol. Clean Air Campaign was launched jointly by the Delhi Government and the Ministry of Environment,
Forest and Climate Change (MoEFCC) to send a message for reducing pollution levels in Delhi and
the National Capital Region. It has provisions of remedial measures and also includes penal actions
against polluters.
195. Ans. (D)
Sol. Justification: The’Graded Response Action Plan’ was notified by the Environment Ministry in 2017. It
aims to improve air quality by fighting against air pollution. It is notified for National Capital Region
and Delhi. Hence, Statement 1 is incorrect, because it is only for Delhi region and not for all urban
areas.
196. Ans. (B)
Sol. You will face such questions in Prelims, some are meant to be bouncers, some to be answered
smartly. This was an easy question, if you had solved many such similar ones in the past. Both options
seem correct in the first go. Ecosystem-based management tools are methods and software that help
practitioners incorporate scientific and socioeconomic information into decision making.
Currently, the network is focusing on tools for integrated land-sea planning to minimize the impact of
land use on coastal and marine environments. It is a source of information and management tools on
coastal and marine spatial planning to help with scientific information for decision making. It in turn
helps mitigate harmful impacts of land use on them.
197. Ans. (C)
Sol. The Indian Coral Reefs Monitoring Network (ICRMN) is a programme of the MoEFCC in India.
Strategy for conservation and management is developed by MoEFCC’s National Committee on
Mangroves and Coral Reefs. Besides formulating the ICRM, the MoEFCC supports regional research
and development activities in this regard as well. The objective of the network is to monitor the
biophysical, environmental, and socio-economic changes that natural causes and patterns of resource
utilization entail and use this knowledge for sustainable management of the reefs.
Coral reefs are among the most diverse ecosystems in the world and are rightly referred to as
the’rainforests of the seas’. It is estimated that one-third of all the world’s fish species depend on
coral reefs for their existence. They are considered ‘medicine chests’of the future. Scientists believe
that their organisms could well hold the key to cures for cancer, and coral skeletons are already being
used as bone substitutes in reconstructive bone surgery.
198. Ans. (C)
Sol. Statement 1: Flush toilets that are currently used by railways, make tracks unhygienic by directly
dumping human waste onto the tracks. Bio-toilets will avoid this dumping and keep the tracks clean.
Statement 2: They can be used in tropical, temperate, and Himalayan regions, as well as in glaciers
and sea ports. Bio-toilets are generally maintenance free and have a life of up to 50 years at once.
They do not need to be flushed. This is why they are being installed at large scale in railways.
199. Ans. (B)
Sol. Comprehensive Environmental Pollution Index (CEPI) is developed by CPCB. A nationwide
environmental assessment of industrial clusters was done by CPCB based on CEPI. It was observed
that around 43 industrial clusters that have CEPI greater than 70, on a scale of 0 to 100, have
been identified as critically polluted. It acts as a warning tool and captures various dimensions of
environment including air, water, and land.
200. Ans. (C)
Sol. Statement 1: Ministry of Earth Sciences has introduced SAFAR system to measure air quality of
metropolitan cities.
Statement 2: By assigning ‘poor’ or ‘very poor’ categories, the system helps create awareness among
general public with regard to air quality. For example, the ‘very poor’ category is given for Nev/ Delhi’s
air quality.
201. Ans. (B)
Sol. Justification: The Indian Coast Guard has been nominated as the central coordinating authority for
oil-spill response in the maritime zones of India, whereas the coastal states and Union Territories are
responsible for shoreline clean-up.
202. Ans. (D)
Sol. Global Urban Ambient Air Pollution Database is released by WHO. It monitors air pollution globally in
terms of PM 2.5 levels. It reported that out of the 15 most polluted cities globally, 14 cities are in India
with Kanpur being the most polluted.
203. Ans. (D)
Sol. All the statements given in the question are correct. CPCB is a statutory organization which was
constituted in 1974 under the Water (Prevention and Control of Pollution) Act, 1974. CPCB has also
been entrusted with powers and functions under the Air (Prevention and Control of Pollution) Act,
1981 and also implements the National Air Quality Monitoring Programme.
204. Ans. (D)
Sol. Options (a) and (b) refer to ecosystems and option (c) refers to biosphere. Biodiversity refers to all
kinds of living organisms such as plants, animals, fungi, and other living things in a given area.
205. Ans. (D)
Sol. Statement 1: Naturalized species are species that reproduce naturally in the environments that they
colonize, whether they are introduced intentionally or unintentionally in that environment. They are
species that have adapted to their new environment and reproduce successfully in it.
Statement 2: Exotic species become naturalized if they compete for resources with the native
population and can thrive in the country’s wildernesses by forming stable populations.
206. Ans. (D)
Sol. Only a few Indian mammals have endemism within specified Indian territory, otherwise their
endemism is lower and same is the case with birds. Moreover, more species of birds (than all the
categories given in the options) are the ones that are threatened as per International Union for
Conservation of Nature (IUCN). Endemism is generally high in Indian amphibians and reptiles.
207. Ans. (B)
Sol. Statement 1: At lower altitudes, greater biodiversity is found due to ambient temperature, sufficient
rainfall, and absence of frost. As one moves higher, biodiversity starts shrinking.
Statement 2: At lower latitudes, solar insolation is abundant and temperature conditions are ideal for
growth of vegetation. Higher latitudes suffer from insufficient sunlight, low temperatures, and frost.
Hence, biodiversity is high near the lower latitudes than higher latitudes.
208. Ans. (A)
Sol. Genetic biodiversity refers to the variation of genes within species.The more diversified the species,
the greater is its ability to cope with attacks and other threats from the environment. Hence, increasing
the productivity of that region.
209. Ans. (B)
Sol. Endemism is that ecological state wherein species exist only in that particular geo graphic region.
You should note that a species can be endemic to more than one area, for example, Olive Ridley
SeaTurtles are endemic to Pacific, Atlantic, and Indian oceans.
210. Ans. (A)
Sol. Plankton can be both marine and fresh water organisms.They are non-motile or too small or weak to
swim against the current and exist in a drifting state. The term plankton is a collection of organisms
such as certain algae, crustaceans, coelenterates, mollusks, bacteria, protozoans and even some
representatives from almost every other phylum of animals. Hence, Statement 2 is incorrect.
Statement 1 is correct. They are responsible for 50% of earth’s oxygen. Plankton are an essential part
of the food chain and are responsible for 50% of earth’s oxygen. They act as a source of food for larger
animals and indirectly for humans, whose fisheries depend upon plankton.
211. Ans. (C)
Sol. Highest concentration of plankton are found in higher latitudes, whereas in the tropics and sub-
tropics, their concentration is low. In tropics, thermocline (temperature gradients) and pycnocline
(salinity gradients) limit circulation of nutrients to the surface where planktons grow. However, in
areas where ocean upwelling occurs, nutrients are cycled to the surface and rich plankton grounds
can be found.
212. Ans. (C)
Sol. Statement 1: Phytoplankton are the primary producers and form the foundation o aquatic food web.
They are the primary producers of aquatic food chain.
Statement 2: They act as storehouse of carbon or carbon sink as they help absorb CO2 from the
atmosphere during photosynthesis.
213. Ans. (C)
Sol. Phytoplankton need both nutrients (such as nitrate and phosphate) and sunlight to be able to
photosynthesize. Sunlight is only available in the uppermost layers of the ocean. During the process
of photosynthesis, the nutrients present in the upper layer are quickly used up by phytoplankton and
hence they are not available for long periods under normal circumstances, as a result tropical waters
are very unproductive.
214. Ans. (D)
Sol. They act as storehouse of carbon or carbon sink as they help absorb CO2 from the atmosphere during
photosynthesis. Hence, reduction in phytoplankton may reduce CO2 drawn from atmosphere, which
eventually affects the amount of carbon transferred to deep ocean for long term storage in the form
of fossil deposits after an organism dies. It also affects the aquatic food web as phytoplankton forms
the base of the food web. They are the primary producers and all other organisms in the aquatic food
web are dependent on phytoplankton directly or indirectly.
215 Ans. (B)
Sol. Statement 1:Favourable temperature for seagrass growth ranges from 25°C to 30°C. High temperature
is not favourable for growth of seaweed. Shallow waters can have higher temperatures during sunny
days and hence they are not suitable for growth of seagrass.
Statement 2: Seagrass needs clear, saline water with sufficient sunlight. Fresh water and turbid water
(muddy) will not sustain good seaweed growth.
Statement 3: If the water current is too strong, it can damage seagrass and can even wash away
these weeds by pulling down the stakes. In India, seagrass occurs along the south east coast of Tamil
Nadu and in the lagoons of Lakshadweep Island and Andaman and Nicobar Islands. Seagrass acts as a
means for combating beach erosion through burial in beach dunes and may also be used as a fertilizer
or compost for landscaping.
216. Ans. (B)
Sol. Seagrasses are kind of flowering plants.
Statement 2: It is incorrect: They are not found in Antarctica. Seagrasses grown near shore are lost
due to intense pressure. The grasses provide shelter and food to many sea creatures. They are
commercially important due to their productivity and they are used as fertilizer or compost for
landscaping. They also provide nursery habitat for juvenile fish, which hide from predators among
the stems.
217. Ans. (A)
Sol. Statement 1: It contains all soil nutrients like 1 % of potassium, 0.3% of nitrogen, and 0.1 % of
phosphorus and amino acids.
Statement 2: It is incorrect. Seaweeds are susceptible to coastal pollution and may get contaminated.
With over 12,000 plus varieties, seaweed in the ocean has shown to be a valuable addition to the
organic garden and can be abundantly available for free for those living near the coast.
218. Ans. (D)
Sol. They are macroscopic algae, which means that they have no differentiation of true tissues such as
roots, stems, and leaves.
• Seaweeds are important as food for humans, feed for animals, and fertilizer for plants.
• Seaweeds are used as a drug for goitre treatment and intestinal and stomach disorders.
• Products such as agar-agar and alginates, iodine which are of commercial value, are extracted
from seaweeds.
• By the biodegradation of seaweeds economically important gases such as methane can be
produced in large quantities.
• They are potential indicators of pollution in coastal ecosystem, particularly heavy metal
pollution due to their ability to bind and accumulate metals
strongly.
• Rotting seaweed is a potent source of hydrogen sulphide, a highly toxic gas.

219. Ans. (C)


Sol. The causes for climate change are many. Some of the causes are due to astronomical phenomenon
and others are terrestrial causes. The astronomical causes are the sunspot cycle and related activities
and Milankovitch oscillations. During solar cycle, there is constant increase and decrease in sunspots
on the sun in a cyclical manner. These changes are associated with long-term climate change in the
following ways.
• An increase in sunspots is associated with cooler and wetter weather and greater storminess
occurs.
• A decrease in sunspot numbers is associated with warm and drier conditions.
Another astronomical cause is the Milankovitch oscillations, which are associated with:
• Variations in the earth’s orbital characteristics around the sun
• Wobbling of the earth
• Changes in the earth’s axial tilt
All these changes alter the incoming solar radiation and the amount of insolation, which affects the
climate.
220. Ans. (A)
Sol. China and USA are the countries emitting the most greenhouse gases and they together account
for more than 40% of the global total greenhouse gas emission. Hence, statement 2 is incorrect.
According to the World Meteorological Organization (WMO), the 20 warmest years on record have
been in the past 22 years. Of these warmest years, 2015-18 make up the top four warmest years. If
this trend continues, temperatures may rise by 3-5°C by 2100.
221. Ans. (C)
Sol. Climate justice focuses our attention on people, rather than ice-caps and greenhouse gases. No
world leader should have to plan for evacuation from the land of their ancestors. But the world is
unprepared for climate change and associated challenges, especially with regards to the adaptation
plan and climate change related displacement of people. Climate justice is a moral argument and the
two main arguments of climate justice are:
• Climate justice compels us to understand the challenges faced by those people and
communities most vulnerable to the impacts of climate change.
• Climate justice also informs how we should act to combat climate change.
Principles of climate justice are:
• Protect and respect human rights
• Share benefits and burdens equitably
• Highlight gender equality and equity
• Support the right to development
• Ensure that decisions on climate change are participatory, transparent, and accountable
• Use effective partnerships to secure climate justice
• Harness the transformative power of education for climate stewardship
222. Ans. (C)
Sol. Climate justice focuses our attention on people, rather than ice-caps and greenhouse gases. No
world leader should have to plan for evacuation from the land of their ancestors. But the world is
unprepared for climate change and associated challenges, especially with regards to the adaptation
plan and climate change related displacement of people. Climate justice is a moral argument and the
two main arguments of climate justice are:
• Climate justice compels us to understand the challenges faced by those people and
communities most vulnerable to the impacts of climate change.
• Climate justice also informs how we should act to combat climate change.
Principles of climate justice are:
• Protect and respect human rights
• Share benefits and burdens equitably
• Highlight gender equality and equity
• Support the right to development
• Ensure that decisions on climate change are participatory, transparent, and accountable
• Use effective partnerships to secure climate justice
• Harness the transformative power of education for climate stewardship
223. Ans. (D)
Sol. Justification: After an independent analysis by both NASA and NOAA on global temperatures, it was
concluded that 2016 is the hottest year on record, followed by 2015 and then 2014. Hence, statement
1 is incorrect. In India as well, 2016 was the century’s warmest year for the country, with an increase
of 0.91°C over the long-term average. NASA’s corresponding global figure is 0.99°C. Hence, statement
2 is also incorrect.
224. Ans. (D)
Sol. Statement 1: Coastal flooding occurs due to mean sea level rise as oceans get warmer and water
expands due to global warming.
Statement 2: Tropical disease vectors thrive in higher temperature climates. As temperature rises due
to global warming, their survival becomes more established.
Statement 3: Some of the ecological niches like glacier types may disappear. This will affect plant and
animal ranges, which have already started to shift and trees are flowering sooner. Species that thrive
only in a particular ecosystem, i.e., niche (lower tolerance) will decline sooner.
Statement 4: Disruption of drinking water supplies will happen as glaciers disappear, which give rise
to many perennial rivers.
225. Ans. (A)
Sol. Learning:The glass allows short wave solar radiation into the glass house or greenhouse and
prevents the long wave radiation from going outside the glass house. This leads to an increase in the
temperature inside the glass house as it traps the infrared radiations, which makes the glasshouse
structure warmer than outside. When you enter a car or a bus, during summers, where windows are
closed, you feel more heat than outside. Similarly, in winters also, thevehicles with closed windows
remain warmer than the temperature outside.
226. Ans. (A)
Sol. Option (c): Ionizing radiation is the radiation that carries enough energy to free electrons from atoms
or molecules, thereby ionizing them. It is not relevant here, hence (c) is wrong.
Option (d): High pollution outside may actually keep the vehicle surroundings warm rather than the
opposite. So, (d) is also wrong.
Learning: A greenhouse is made up of glass. The glass allows short wave solar radiation into the glass
house and prevents the long wave radiation from going outside the glass house. This leads to an
increase in the temperature inside the glass house as it traps the infrared radiations, which makes the
glass house structure warmer than outside. When you enter a car or a bus, during summers, where
windows are closed, you feel more heat than outside. Similarly, in winters also, the vehicles with
closed windows remain warmer than the temperature outside.
227. Ans. (D)
Sol. Statement 1: Tetrafluoromethane contributes to the greenhouse effect, hence it is a potent greenhouse
gas. Tetrafluoromethane is very stable. It has very high atmospheric lifetime of 50,000 years, and
greenhouse warming potential of 6500; however, its low amount in the atmosphere restricts the
overall radiative forcing effect.
Statement 2: According to the Intergovernmental Panel on Climate Change, Sulphur hexafluoride
has a global warming potential of 23,900 times that of CO2 and is the most potent greenhouse gas
that it has evaluated. Sulphur hexafluoride is inert in the troposphere and stratosphere and hence, is
extremely long-lived with an estimated atmospheric lifetime of 800-3200 years.
Statement 3: Some perfluoroalkanes have very long atmospheric lifetime and are potent greenhouse
gases. Similar to chlorofluorocarbons, there are environmental concerns with respect to perfluoro
compounds and their history of use, environmental impact, and recommendations for use are
included in the Kyoto Protocol.
228. Ans. (B)
Sol. Justification: At present, the textile industry is the world’s second most polluting industry after the
oil industry. N2O and CO2 are the major gases released during production of textiles. According to the
journal—Nature Climate Change, the total greenhouse gas emissions from textile production currently
stands at 1.2 billion tonnes annually and it is responsible for 10% of the global carbon emissions.
229. Ans. (D)
Sol.
Gas Lifecycle in
Atmosphere
Water vapour Few hours to days
Nitrous oxide Around 114 years
Methane Around 12 years
Sulphur hexafluoride 800-3200 years

230. Ans. (B)
Sol. According to a report by the Standing Committee on Agriculture, it was noted that paddy fields are a
major source of greenhouse gases in the farm sector and the major gases include methane, nitrous
oxide, and carbon dioxide. The committee noted that the reduction of emissions of these greenhouse
gases would help India meet obligations under the International Accord on Climate Change. HFCs
have no natural sources and are known to be emitted through industrial processes. Hence, option (B)
is the answer.
231. Ans. (D)
Sol. Global warming potential (GWP) provides a common unit of measure for analysts to add up emissions
estimates of different greenhouse gases. The time period used for the measure of GWP is 100 years.
The larger the GWP of a given gas, the more that given gas warms the earth compared to CO2 in that
given time. GWP helps policy makers to compare emissions reduction opportunities across sectors
and gases.
232. Ans. (A)
Sol. A number of greenhouse gases are found in the earth’s atmosphere such as water vapour, carbon
dioxide, methane, nitrous oxide, ozone, and chlorofluorocarbons (CFCs). But the contribution of each
gas to the greenhouse effect is different based on the characteristics of each gas and its abundance.
When compared to carbon dioxide over a span of 20-year time frame, the direct radiative effect of
methane is about 72 times stronger than the same mass of carbon dioxide. But the concentration of
methane in the atmosphere is very less when compared to carbon dioxide.
233. Ans. (C)
Sol. Statement 1: Cloud cover plays a highly influential role in the amount of both UV-A and UV-B radiation
reaching the ground. Each water droplet in the cloud scatters or reflects back some of the incoming
UV radiation into space. Hence, the larger the cover of the clouds in sky, the lesser UV reaches the
ground and hence, protects organisms on the earth’s surface.
Statement 2: The penetration of UV-B radiations decreases rapidly as the depth of the water column
increases. Some substances and impurities that are dissolved in water, strongly absorb and scatter the
incoming UV-B radiation and enhance protection of microorganisms, plants, and animals from UV-B.
234. Ans. (B)
Sol. Ozone biosensors record the damage to the organisms due to the exposure of ultraviolet radiations.
An experiment was conducted, where E. coli was used to measure the extent of damage to marine
organisms in the Antarctic Ocean. The results were that damage to organisms was very high at depths
less than 10 meters when compared to 20 and 30 meters of depth.
235. Ans. (D)
Sol. Statement 1: Cloud cover plays a highly influential role in the amount of both UV-A and UV-B radiation
reaching the ground. Each water droplet in the cloud scatters or reflects back some of the incoming
UV radiation into space. Hence, the larger the cover of the clouds in sky, the lesser UV reaches the
ground and hence, protects organisms on the earth’s surface.
Statement 2: Unlike clouds which scatter or reflect the UV radiations, aerosols in the troposphere not
only scatter but also absorb UV-B radiation. Usually in the regions of heavy smoke or dust, aerosol
particles can absorb more than 50% of the radiation.
Statement 3:The penetration of UV-B radiations decreases rapidly as the depth of the water column increases.
Some substances and impurities that are dissolved in water, strongly absorb and scatter incoming
UV-B radiation and enhance protection of microorganisms, plants, and animals from UV-B.
236. Ans. (D)
Sol. The ozone molecule (O3) present in the troposphere, especially the one present at the ground level is
harmful to air quality because of its effects on people and the environment.
Ozone is the main ingredient in ‘smog’. Ground level or bad ozone is a secondary pollutant, which
meansit is not emitted directly into the air, but is created by chemical reactions between volatile
organic compounds (VOC) and oxides of nitrogen (NOx), in the presence of sunlight.
237. Ans. (C)
Sol. Following is the list of ozone depleting substances.
• Chlorofluorocarbons (CFCs)
• Halon
• Carbon tetrachloride (CCI4)
• Bromochloromethane (CH2BrCI)
• Hydrochlorofluorocarbons (HCFCs)
• Hydrobromofluorocarbons (HBFCs)
• Methyl bromide (CH3Br)
• Methyl chloroform (CH3CCI3)
238. Ans. (D)
Sol. Statement 1: UV radiations lead to ageing of skin, cataract, sunburn, and skin cancer in humans.
Statement 2: Phytoplankton in general, cannot tolerate high energy solar radiation such as UV
radiations and it may lead to decreased productivity or even killing of phytoplankton in large numbers.
Statements 3 and 4: Overexposure to UV-B can impair photosynthesis, reduce size, productivity, and
quality in many plant species. UV-B increases plants’ susceptibility to disease by sometimes causing
mutations or damaging plant proteins.
239. Ans. (B)
Sol. Ozone (O3) is a major constituent of atmospheric smog and it is a colourless, reactive oxidant gas.
Ground level ozone is a secondary pollutant, which means it is not emitted directly into the air, but
is created by chemical reactions between volatile organic compounds (VOC) and oxides of nitrogen
(NOx), in the presence of sunlight. Sunlight is the brightest in the afternoon, hence (B) is the correct
answer.
240. Ans. (C)
Sol. Chlorofluorocarbon molecules are several times heavier than air, but they mix with the wind and
are carried to altitudes far above the top of the stratosphere much faster than molecules can settle
according to their weight.
• CFCs are relatively unreactive in the lower atmosphere but in stratosphere, when UV
radiation hits a CFC molecule, it causes one chlorine atom to
break away.
• Lower temperatures favour the formation of polar stratospheric clouds (PSCs). These clouds
serveas a platform for catalytic breakdown of ozone in
stratosphere.
The chlorine atom, then hits an ozone molecule (O3 or three oxygen atoms) and takes one of the oxygen molecules
and forms chlorine monoxide by destroying the ozone molecule and turning it into oxygen (O2).
When an oxygen molecule hits the molecule of chlorine monoxide, the two oxygen atoms join and form an
oxygen molecule. When this happens, the chlorine atom is free and can continue to destroy the
ozone.
241. Ans. (A)
Sol. Global warming is the rise in temperature of the atmosphere of the earth. Global warming results in
the melting of ice caps and polar glaciers and expansion of water, which leads to a rise in the sea level,
causing floods in the coastal areas (for example, Maldives is a low-lying nation and at a threat of being
drowned by the sea level rise). Global warming has also led to massive glacial retreat in the past 50
years.
242. Ans. (A)
Sol. Permafrost is a ground soil which has been at or below the freezing point of water for two or more
years. It is located at high latitudes and some may even occur subsea on the continental shelves
surrounding the Arctic Ocean. In recent times, some portions of the permafrost are exposed to global
warming. Permafrost has a large stock of carbon both as peat and as methane and can also store
carbon, that is, it acts as a carbon sink. Melting of permafrost would raise water levels in the oceans
and increase erosion. Erosion will also increase because when permafrost melts, soil and sediment
are easily washed away by water.
243. Ans. (D)
Sol. Statement 1: Global warming and associated events have led to changes in climate and increased
extreme events such as drought and El Nino, which has led to decrease in productivity of crops and
hence reduced crop production.
Statement 2: Increased temperatures provide a better breeding ground for many pests. Moreover,
as climates ‘migrate’, insects and fungi are moving toward the poles too affecting crops in temperate
regions.
Statement 3: Seafood is a large part of many people’s diet around the world and many marine species
can’t survive in the more acidic environment, for example, planktons which form the basis of food
chain.
244. Ans. (C)
Sol. This is a lateral explanation. Oil extraction in Arctic may release black carbon and methane leading
to global warming and thawing of Arctic ice. It subsequently causes climate change. Some studies
estimate the cost involved as $60 trillion to the world economy. Apart from the usual emission issues
from the Arctic Permafrost, many of the boundary lines in the Arctic remain to be fully demarcated as
under EEZ. Therefore, disputes can arise on resource extraction in the Arctic.
245. Ans. (C)
Sol. According to a report by Intergovernmental Panel on Climate Change (IPCC), the climate change
would hit the poorest countries severely in terms of reducing the agricultural products.
• Due to decrease in water availability, the crop yield would be reduced and may increase the
in- sect/pest incidence because higher temperature is conducive for breeding of pests. So,
both statements 1 and 2 are correct.
• The overall amounts of rainfall may increase or decrease and there can also be shifts in the
timing of the rainfall, which will adversely affect the
agriculture.
246. Ans. (D)
Sol. Option (a): Rice fields are wetlands that release methane which is a GHG that accelerates global
warming.
Option (b): Cattle breeding also releases methane. So, (b) is wrong.
Option (c):Permafrost contains carbon that was trapped since ages, and its release after melting
accelerates global warming.
Option (d): Suspended particles from volcanic ash, when spread in the atmosphere, reduce solar
insolation and help cool off the earth.
247. Ans. (A)
Sol. Justification: Global warming potential (GWP) is a measure of how much energy the emissions of 1
tonne of a gas will absorb over a given period of time, relative to the emissions of 1 tonne of carbon
dioxide (CO2). The larger the GWP, the more that given gaswarms the earth compared to CO2 over that
time period.
248. Ans. (A)
Sol. • The origin of the Polluter Pays Principle can be traced back to the ‘Stockholm
Declaration made at the ‘United Nations Conference on the Human
Environment’ in June 1972. In this declaration, signatory countries were asked to develop
international laws ‘regarding liability and compensation for the victims of pollution and other
environmental damage’.
• According to the principle, those who pollute the environment must be made to pay for the
costs of remedial action and also for compensating victims of environmental damage caused by the
pollution.
• The 1987 report of the ‘Brundtland Commission’ called ‘Our Common Future’, greatly
influenced the Rio Earth Summit of 1992, which, for the first
time, explicitly enshrined the Polluter Pays Principle, which was emphasized in the report.
249. Ans. (A)
Sol. Statement 1: Forest plays a fundamental role in climate change mitigation, by removing CO2 from
the atmosphere and storing it in biomass and soils. Deforestation and forest degradation are a
source of greenhouse gas (GHG) emissions by releasing the stored carbon in the trees and plants.
It is estimated that they account for around 11% of CO2 emissions. Hence, halting deforestation and
forest degradation is a cost-effective action in reducing global GHG emissions.
Statement 2: Rice fields actually emit methane, and more rice farms lead to an increase in the
methane GHG emissions, even if they soak some of the CO2.
250. Ans. (C)
Sol. Carbon sequestration is a process which involves carbon capture and long-term storage of atmospheric
carbon dioxide or other forms of carbon to mitigate or defer global warming. Carbon dioxide (CO2)
is captured naturally or artificially from the atmosphere. Naturally, the carbon is captured through
biological, chemical, and physical processes. Artificial processes involve large-scale, artificial capture
and sequestration of industrially produced CO2 using reservoirs, ocean water, subsurface saline
aquifers, aging oil fields, or other carbon sinks. Geologic carbon sequestration refers to the process of
capturing carbon dioxide (CO2) and storing it in underground geologic formations. The CO2 is usually
pressurized until it becomes a liquid, and then it is injected into porous rock formations in geologic
basins.This method of carbon storage is also sometimes a part of enhanced oil recovery, otherwise
known as tertiary recovery, because it is typically used later in the life of a producing oil well.
Biologic carbon sequestration is the process of storing of atmospheric carbon dioxide (CO2) in
vegetation such as plants and trees, woody products, soils and in aquatic environments especially
phytoplankton.
251. Ans. (A)
Sol. Saltmarshes are one of the key coastal ‘blue carbon’ habitats, recognized for their ability to store
carbon within above- and below-ground biomass and sediments. Carbon sequestration rate of
saltmarshes is 2 to 4 times greater than that of mature tropical forests, with an average annual carbon
sequestration rate of 6 to 8 tonnes of carbon dioxide equivalent per hectare.
252. Ans. (B)
Sol. The Hellisheidi power station is one of the largest geothermal plants in the world located outside
Reykjavik, Iceland. It is Iceland’s main geothermal plant and is located in a volcanic area. The volcano’s
internal heat is harnessed to generate electricity and provide hot water for the city’s heating systems.
Hellisheidi is not just a provider of green energy but also a site for a scientific breakthrough, which is
an experiment to capture carbon dioxide (CO2) and turn it into stone forever.
This experiment is called as CarbFix.The project is an international collaboration led by Reykjavik
Energy, the University of Iceland, Columbia University, and French National Centre for Scientific
Research. This project stands out among CCS projects because the capture of carbon is permanent
andfast.
253. Ans. (D)
Sol. Oceans are one of the largest carbon sinks on earth because of their large geographical coverage and
presence of rich population of phytoplankton and seagrass, which act as carbon sink.
Option (c): The difference in partial pressure of the CO2 between seawater and air facilitates gaseous
exchange.This allows atmospheric CO2 to dissolve in seawater. Carbon dioxide readily dissolves in
water and occurs in three main forms, they are normal carbon dioxide, bicarbonate and carbonate
ions. Hence, oceans provide a huge reservoir of carbon.
254. Ans. (D)
Sol. Oceans store much more carbon than the atmosphere and the terrestrial biosphere. Carboncan be
stored in oceans as dissolved inorganic and organic compounds and also as biomass reserve in the
form of phytoplankton and seagrass. The carbon, however, requires centuries to penetrate into the
deep ocean, because the mixing of the oceans is a rather slow process. However, even more carbon
is stored in the lithosphere in the form of rocks and minerals such as limestone (CaCO3).
255. Ans. (C)
Sol. Learning: Tropical rainforests generally draw in carbon dioxide and breathe out oxygen and are
often called as the ‘lungs of the planet’. The Amazon Forest in South America is the largest tropical
rainforest, covering an area of five and a half million square kilometres and thus the largest forest
lung of the earth.
256. Ans. (A)
Sol. Learning: Carbon Capture Utilization Storage (CCUS) refers to a process of capturing waste carbon
dioxide (CO2) from large point sources and transporting it to a storage site for reusing it. In Carbon
Capture and Storage (CCS), emissions are forced into underground rocks at great cost and no economic
benefit, while CCUS aims at using CO2 emissions by exploiting the resource itself and creating new
markets around it.
257. Ans. (D)
Sol. A number of geo-engineering approaches are utilized to tackle climate change and they are as
explained below.
Statement 1:Lohafex, an Indian-German geo-engineering experiment conducted in 2009, involved
dumping of six tonnes of iron solution into the South Atlantic to encourage plankton to bloom and
trap carbon. When the organisms died, the carbon was sent to the seabed to get deposited.
Statement 2: During volcano eruptions, ash particles and Sulphur-rich gases will be injected into the
atmosphere and form clouds. These clouds will encircle the globe within weeks of the volcanic activity.
These clouds decrease the amount of sunlight reaching the surface of the earth, absorb incoming
solar radiation and scatter it back out into space, thereby lowering the average global temperatures.
Statement 3: Researchers say that cirrus ice clouds are known to have a net warming effect on our
planet because the ice crystals re-emit infrared radiation absorbed from the sun-warmed surface
back down towards ground. So, if we can make cirrus still thinner then we will let out more heat
towards the outer space and cool the globe.
258. The answer is (d).
Justification: Green Climate Fund (GCF) was established in Conference of Parties (COP) 16, Cancun,
as an operating entity of the financial mechanism of the convention under Article 11.The GCF Board
governs the fund and it functions under the guidance of the COP to support projects in developing
country parties and is accountable to COP. Under the UNFCCC Convention, the Special Climate Change
Fund (SCCF) was established in 2001 as a fund to finance projects relating to:
• technology transfer and capacity building;
• adaptation;
• economic diversification;
• energy transport, industry, agriculture, forestry, and waste management.
The Least Developed Countries Fund (LDCF) was established by’194 parties to the United Nations
Framework Convention on Climate Change (UNFCCC)’ in 2001 .The LDCF is managed by the Global
Environment Facility and it supports the world’s most vulnerable countries, especially the least
developed countries, in their efforts to adapt to the effects of climate change.
259. The answer is (c).
Justification: The GCF was established by 194 governments who are parties to UNFCCC and then
included G-20 members also. So, statement 2 is incorrect. It was set up to limit or reduce greenhouse
gas emissions in developing countries. So, statement 1 is incorrect. The GCF supports countries in
implementing theirnationally determined contributions with multiple instruments/mechanisms that
are developed by it and are made available to countries. So, statement 3 is correct.
260. The answer is (b).
Justification: GCF has accredited the National Bank for Agriculture and Rural Development (NABARD)
as the national implementing entity for undertaking climate change related projects in India.
261. The answer is (c).
Justification: GCF was established by 194 governments who are parties to UNFCCC. It was established
in COP 16, Cancun, as an operating entity of the financial mechanism of the convention under Article
11. The GCF Board governs the fund and it functions under the guidance of the COP to support
projects in developing country parties and is accountable to COR
262. The answer is (d).
Justification: The main task of Conference of Parties (COP) is to review the national communications
and emission Inventories submitted by parties to the UNFCCC The COP assesses the effects of the
measures taken by the parties based on the Infor mation submitted by them to COP and assesses the
progress made in achieving the ultimate objective of the UNFCCC.
263. The answer is (a).
Justification: The following table shows the correctly matched pairs of funds and their administrative
bodies.
Fund Administrative Body

SCCF Global Environment Facility


LDCF Global Environment Facility
GCF GCF Board
Adaptation Fund Kyoto Protocol, administered by
(AF) Adaptation Fund Board
264. The answer is (b).
Learning: GCF was established by 194 governments who are parties to UNFCCC. The fund was
established in COP 16, Cancun, as an operating entity of the financial mechanism of the convention
under Article 11. The GCF Board governs the fund and it functions under the guidance of the COP to
support projects in developing country parties and is accountable to COP.
265. The answer is (c).
Justification: UNFCCC has near universal membership with 197 parties. It is the parent treaty of
the 1997 Kyoto Protocol and this protocol has been ratified by 192 parties of the UNFCCC. Hence,
statement 1 is correct.
Statements 2 and 3: The convention has non-binding limits on greenhouse gas emissions for individual
countries. There are no enforcement mechanisms established under this convention.
266. The answer is (a).
Justification: The classification of the parties to the UNFCCC was introduced in Kyoto Protocol.
Statement 1: Under Annex I of the convention, there are over 40 parties including the European
Union and they are classified as industrialized (developed) countries and ‘economies in transition’
(EITs).
Statement 2: Over 20 parties of Annex I are listed in Annex II of the convention and these parties are
required to provide financial and technical supportto the EITs and developing countries to assist them
in reducing their greenhouse gas emissions and manage the impacts of climate change.
Statement 3: The parties under Annex B of the Kyoto Protocol are Annex I Parties with first- or second
round Kyoto greenhouse gas emissions targets.
267. The answer is (d).
Justification: Option (a): The impact of greenhouse gases on climate change is independent of
where in the world they are emitted. These emissions impose a cost on both the present and future
generations, which are not fully recouped from the emitters of these emissions, which historically
were developed nations.
Options (b) and (c): Developed nations must take greater share of the responsibility because
developing countries cannot sacrifice economic development by focusing solely on environmental
conservation. There needs to be a balance based on the principles of sustainable development. This
formed the starting point for a globally coordinated policy action and the need for an international
climate change negotiation regime as in UNFCCC.
268. The answer is (d).
Justification: Statement 2: Principle of equity implies fairness. Its implication can be seen in statement
4.
Statement 4: For example, USA and China have contributed more to GHG emissions historically than
LDCs such as South Sudan or Zimbabwe. So, it is only fair that the larger share of responsibilities
should go to USA and China.
Statement 3: Moreover, not only responsibilities, but also capabilities are different. This implies that
a larger burden cannot be given to LDCs or developing countries. In Rio Declaration, it was stated that
states have common but differentiated responsibilities.
269. The answer is (a).
Justification: Statement 1: Under Article 12 of the Kyoto Protocol, the Clean Development Mechanism
(CDM) allows a country (Annex B Party) with an emission-reduction or emission-limitation commitment
to implement an emission-reduction project in developing countries. Such projects can earn saleable
certified emission reduction (CER) credits, which are each equivalent to one tonne of C02 and they
are counted towards meeting Kyoto targets.
Statement 2: In order to maintain the effectiveness of this protocol, emission savings from the
CDM must be additional, that is, if emission reduction in a project would have happened anyway,
it is notcounted as part of CDM. CDM amounts to an income transfer to non-Annex I countries if
additionally, not considered.
270. The answer is (a).
justification: In 1997, the Kyoto Protocol was adopted to fight global warming by reducing GHGs
emission and it came into effect in 2005- The 1stcommitment period was from 2008-12 and the 2nd
commitment period was from 2013-20. In 2012, the 2nd commitment period was adopted by the
Doha Amendment of the Kyoto Protocol.
271. The answer is (d).
Justification: CO is not a greenhouse gas, so it is not covered under the Kyoto Protocol. Other gases
that were covered are Hydrofluorocarbons (HFCs), Per-fluorocarbons (PFCs), and Carbon dioxide
(CO2).
272. The answer is (a).
Justification: Statement 1: Under Article 12 of the Kyoto Protocol, the Clean Development Mechanism
(CDM) allows a country (Annex B Party) with an emission-reduction or emission-limitation commitment
to implement an emission-reduction project in developing countries. Such projects can earn saleable
certified emission reduction (CER) credits, which are each equivalent to one tonne of CO2 and they are
counted towards meeting Kyoto targets.
Statement 2: CERs can be purchased either from the primary market or secondary market. Primary
market means CERs are purchased from an original party that makes the reduction. In secondary
markets CERs are resold from a marketplace.
273. The answer is (c).
Learning: Under Article 6 of the Kyoto Protocol, the joint implementation allows a country (Annex
B Party) with an emission reduction or limitation commitment under the Kyoto Protocol to earn
emission reduction units (ERUs) from an emission-reduction or emission removal project in another
Annex B Party and these units are counted towards meeting its Kyoto target.
274. The answer is (b).
Justification: Reducing Emissions from Deforestation and Forest Degradation (REDD+) mechanism
was developed under UNFCCC. A financial value for the stored carbon in forests is created by offering
results-based payments and incentives to developing countries to reduce emissions from forested
lands and invest in low-carbon paths to sustainable development. In 1997, the world resources
institute (WRI) established an online platform called Global Forest watch as part of the forest frontier
initiative, to provide data and tools for monitoring forests and to access near real-time information.
275. The answer is (a).
Justification: Wood accounts for roughly 45% of current global renewable energy supply and provides
more energy than solar, hydroelectric, or wind power. The REDD+ mechanism was developed under
UNFCCC. A financial value for the stored carbon in forests is created by offering results-based
payments and incentives to developing countries to reduce emissions from forested lands and invest
in low-carbon paths to sustainable development. The Partnership for Land Use Science (Forest-Plus)
is a joint programme developed by MoEFCC and United States Agency for International Development
(USAID). It was developed to strengthen capacity for REDD+ implementation in India.
276. The answer is (c).
Justification: In response to the UNFCCC decisions on the Bali Action Plan and REDD at COP, UN-REDD
programme was created in 2008.
• UN-REDD programme is different from ‘REDD; which has been developed under UNFCCC.
• UN-REDD programme is a collaborative programme of UNEP, FAO, and UNDP
• UN-REDD programme supports the nationally-led REDD processes and promotes informed and
meaningful involvement of all stakeholders in national and international REDD implementation.
277. The answer is (a).
Justification: A land degradation monitoring tool called ‘Trends.Earth’ was produced as part of the project
‘Enabling the use of Global Data Sources to Assess and Monitor Land Degradation at Multiple Scales’.
This tool is funded by the Global Environment Facility and implemented by Conservation International.
278. The answer is (c).
Justification: The Global Wildlife Programme (GWP) also known as the ‘Global Partnership on
Wildlife Conservation and Crime Prevention for Sustainable Development Programmewas launched
by the Global Environment Facility (GEF) in June 2015. It is a World Bank led global partnership.
The programme promotes wildlife conservation and sustainable development by combatting
illicit trafficking in wildlife. It mainly seeks to reduce both the supply and demand that drives the
illegal wildlife trade,and protect species and their habitats through integrated landscape planning.
In addition, the programme will support integrated landscape management, land use zoning, and
natural resource management best practices.
279. The answer is (c).
Learning: The Global Environment Facility (GEF) was established in 1992 on the eve of Rio Summit
as an independently operating financial organization to provide grants for projects related to climate
change, biodiversity, land degradation, international waters, the ozone layer, and persistent organic
pollutants. The GEF serves as a financial mechanism for:
• UNFCCC
• Convention on Biological Diversity (CBD)
• UN Convention to Combat Desertification (UNCCD)
• Minamata Convention on Mercury
• Stockholm Convention on Persistent Organic Pollutants (POPs)
280. The answer is (c).
Learning: Under GEF’s Country Partnership Programme (CPP), the SLEM Programme was launched
jointly by the Government of India and GEF. The SLEM Programmatic Approach was launched with an
objective to promote sustainable land management and use of biodiversity as well as to maintain the
capacity of ecosystems to deliver goods and services while taking into account climate change. It is a
long-term and strategic arrangement of individual yet interlinked projects aimed at achieving large-
scale impacts on the global environment.
281. The answer is (c).
Justification:The Ecosystem Service Improvement Project will be entirely financed by the World Bank
out of its GEF Trust Fund. The project’s duration is of five years. Indian Council of Forestry Research &
Education (ICFRE) under the National Green India Mission (GIM) will implement the project in Chhat-
tisgarh and Madhya Pradesh. The objectives of the project are
• To improve the livelihoods of forest dependent communities in Central Indian Highlands
• To strengthen the institutional capacity of the community organizations and departments of forestry
to enhance forest ecosystem services
282. The answer is (a).
Justification: Statement 1: SCCF was established in 2001 under GEF, which operates it. This fund was
launched with the aim to finance projects relating to:
• technology transfer and capacity building,
• adaptation,
• economic diversification,
• energy, transport, industry, agriculture, forestry, and waste management.
Statement 2: The amount is US$ 100 billion (hence statement 2 is incorrect), which was a major
demandfrom developing countries for many COP meetings. Gradually, it has been acceded in the
form of new and additional ‘fast-start finance’ (FSF) between 2010-12 to developing countries and
other funds.
283. The answer is (c).
Justification: IPCC was established by UNEP and World Meteorological Organization (WMO) in 1988
with the objective to provide governments at all levels with scientific information that they can use
to develop climate policies. IPCC currently has 195 members, who are either members of the United
Nations or WMO. The IPCC does not conduct its own research, instead thousands of people across
the world contribute to the work of IPCC.
284. The answer is (c).
Justification: Statement 1: According to the IPCC’s Special Report on Global Warming of 1.5°C if the
average global temperature rises by more than 1°C from the present, India could annually expect
conditions like the 2015 heat wave that killed at least 2,000.
Statement 2: The Paris Agreement is considered a landmark agreement as the world agreed to keep
temperatures below 2°C and pursue efforts to limit the temperature increase to 1.5°C above pre-
industrial levels.
285. The answer is (c).
Justification: According to the IPCC’s Special Report on Global Warming of 1.5°C, the five steps to
reach the target of 1.5°C are:
• Global net zero emissions by 2050
• Usage of coal is expected to be reduced to close to zero
• Renewables are estimated to provide up to 85% of global electricity by 2050
• Global emissions of C02 need to decline by 45% from 2010 levels by 2030
• Up to 7 million sq. km of land will be needed for energy crops
286. The answer is (d).
Justification: IPCCs Special Report on Global Warming of 1.5°C differs from previous approaches as it
clearly links lifestyle choices with global warming. The report’s authors say that rapid changes must
take place in four key parts of society:
• land use
• industry
• energy generation
• cities
The findings of this report are
Limiting warming to 1.5°C is a lot more beneficial than limiting warming to 2°C. It really reduces the
impacts of climate change in very important ways
• The unprecedented nature of the changes that are required if we are to limit global warming to 1.5°C
include changes to the way use transportation, changes to the way we manage land, and changes to
energy systems.
287. The answer is (c).
justification: According to a report published in Science by the US National Oceanic and Atmospheric
Administration (NOAA), researchers from NOAA have suggested that the temperatures of the oceans
were being consistently underestimated by the main global climate models. The researchers have
showed that the ocean buoys used to measure sea temperatures tend to report slightly cooler
temperatures than the older ship-based systems.
288. The answer is (a).
Learning: IPCC was established by UNEP and WMO in 1988 with the objective to provide governments
at all levels with scientific information that they can use to develop climate policies. IPCC was later
endorsed by the United Nations General Assembly. IPCC currently has 195 members, who are either
members of the United Nations or WMO. The IPCC does not conduct its own research, instead
thousands of people across the world contribute to the work of IPCC.
289. The answer is (d).
Justification: In COP 21 UNFCCC Paris Climate Conference, ‘Mobilize Your City Initiative’ was launched
to provide support to 100 cities worldwide in 3 years to engage in sustainable urban mobility
planning to reduce greenhouse gas emissions. It is an international initiative supported by the French
and German governments. India is one of the very first countries to benefit from this initiative. In
India 3 cities, Kochi, Nagpur, and Ahmedabad were selected for the project and this project will be
implemented by the Union Ministry of Housing and Urban Affairs. India and France have recently
signed an implementation agreement under this initiative.
290. The answer is (c).
Justification: All the six Indian cities (mentioned in the question) are part of the C40 initiative. The
C40 cities group initiative is a network of the world’s megacities committed to addressing climate
change. Under this initiative, cities collaborate effectively, share knowledge, and drive meaningful,
measurable! and sustainable action on climate change.
291. The answer is (a).
Justification: indigenous and Community Conserved Areas (ICCAs) are areas which are either natural
or modified ecosystems containing significant biodiversity values, ecological services, and cultural
values and are voluntarily conserved and cultural values and are voluntarily conserved. By indigenous
people and local communities, both sedentary and mobile, through customary laws or other effective
means.
Statement 2:ICCAs also include ecosystems which hive0 minimum to substantial human influence as
II as cases of continuation, revival, or modification of traditional practices or new initiatives taken up
by communities in the face of new threats or opportunities.
292. The answer is (c).
Justification: The mission of International Fund for Animal Welfare (IFAW) is to rescue and protect
animals around the world and it is one of the largest animal welfare and conservation charities in the
world. In 1969, a small group of people founded IFAW to stop the commercial hunt for seal pups in
Canada.
293. The answer is (c).
Justification: Statement 2: The Clean Energy Ministerial (CEM) is the initiative of the USA and under
this there are 13 initiatives covering energy efficiency, clean energy supply, and cross cutting areas.
Currently, there are 23 countries as members of CEM.
Statement 3: It does not finance carbon credits.
294. The answer is (d). Learning: The mission of the World Climate Research Programme (WCRP) is to facilitate
the analysis and prediction of Earth system changes and variability, which can be used in increasing
range of practical applications of direct relevance, benefit, and value to society.
295. The answer is (a).
Learning: Commission on Sustainable Development (CSD) was established at Earth Summit to ensure
compliance to the agreements at Rio (particularly the Agenda 21 and Rio Declaration on Environment
and Development). In 2013, High-level Political Forum on Sustainable Development replaced CSD
and it meets every year as part of the ECOSOC meetings and every fourth year as part of the General
Assembly meetings.
296. The answer is (d).
Justification: The rights and responsibilities of nations with respect to their use of the world’s oceans,
establishing guidelines for businesses, the environment, and the management of marine natural
resources have been defined under the Law of the Sea Convention. Some of the important articles of
the UNCLOS are.
• Article 17. Right of innocent passage
•Article 18. Meaning of passage
• Article 19. Meaning of innocent passage
• Article 21. Laws and regulations of the coastal state relating to innocent passage
• Article 22 Sea lanes and traffic separation schemes in the territorial sea
• Article 31. Responsibility of the flag state for damage caused by a warship or other government ship
operated for non-commercial purposes
• Article 32. Immunities of warships and other government ships operated for non-commercial
purposes
297. The answer is (c).
Learning: The first world Parliamentary Forum on Sustainable Development was hosted by Indonesian
House of Representatives on 6-7 September 2017 in Bali and was open to all national parliaments
across the world. The objective of this parliamentary forum was to facilitate the sharing of policy
analysis, experience, best practices from cross-country and cross-sectors, discuss preparedness, and
also deepen the understanding of parliament in each country of SDGs.
298. The answer is (c).
Background: Carbonaceous aerosol emissions lead to air-quality degradation and related health-risks
on local to regional scales and to climate impacts on regional to global scales. They arise by burning
of forest, grasslands, and agricultural residues and from energy use. In South Asia, there is dominance
of small combustion sources (e.g. residential cooking and heating), less-developed industry (e.g. brick
kilns), and vehicular emission. Learning: It was launched under the Climate Change Action Programme
(CCAP). It was launched with the objective to:
• Prepare inventory of the carbonaceous aerosols including black carbon
• Develop national emission factors
• Conduct modelling studies and their impact on regional and global climate
A consortium of 17 leading research institutions of the country are engaged in the task which is
coordinated by IIT Bombay.
299. The answer is (c).
Learning: The New Urban Agenda Is an outcome document of Habitat III Cities Conference or the
United Nations Conference on Housing and Sustainable Urban Development that took place In Quito,
Ecuador. This document was launched to guide the efforts around urbanization of a wide range of
actors—nation states, city, United Nations programmes, and civil society—for the next 20 years laying
the groundwork for policies.
300. The answer is (a).
Learning: Blue label certification is granted by Marine Stewardship Council (MSC), an international
non-profit organization, to the fishery across the world to help create more sustainable seafood
market. The MSC certified products of this sector help in minimizing the impact of fishery on the
whole marine environment to ensure healthy, thriving oceans for the future.
301. The answer is (a).
Justification: ROAM enables countries to define and implement national or subnational contributions
to the Bonn Challenge and to meet existing international commitments under the United Nations
Convention to Combat Desertification, Convention on Biological Diversity, and the United Nations
Framework to Combat Climate Change.
302. T he answer is (a).
Background and Learning: The Arctic is experiencing dramatic changes in climate and patterns of human land
use. Environmental and socio-economic drivers associated with these changes originate both within
and outside of the Arctic system. Change detection and predictive power of these changes is low and
are limited/threatened by the lack of sustained environmental observation time series in northern
high latitudes.
A Circumarctic Environmental Observatories Network (CEON) is a network of observatories and
research sites across the world to provide adequate, diverse, and sustained time series observations.
CEON has the potential to dramatically improve our understanding of the Arctic system and how it
may continue to respond to a variety of environmental and societal changes forecast for the region.
303. The answer is (b).
Learning: Climate and Development Knowledge Network (CDKN) is run by an alliance of private
organizations and think tanks. It is headed by Price-waterhouseCoopers and includes Overseas
Development Institute, a think tank on development and humanitarian issues. It links developing
countries with experts on climate change. Funded by the UK’s Department for International
Development and the Directorate-General for International Cooperation, its aim is to help developing
nations adapt to the consequences of climate change and build capacity for a low-carbon economy.
304. The answer Is (c).
Learning: Future Earth is a 10-year international research programme launched in 2012 at the UN
Conference on Sustainable Development (Rio 20), It was launched with the aim to build knowledge
about the environmental and human aspects ofglobal change and to find solutions for sustainable
development. It aims to increase the impact of scientific research on sustainable development.
Scientific research and synthesis in Future Earth is carried out by a number of international networks
known as ‘core projects’.
305. The answer is (a).
Learning: Transformative Carbon Asset Facility (TCAF) was launched by World Bank to help developing
countries pay for emission reductions and combat climate change.TCAF is a market-based mechanism
specially designed to help developing countries and it would establish the world’s first programmatic
carbon market.
306. The answer is (a).
Justification: Statement 1: World Bank managed BioCarbon Fund Initiative for Sustainable Forest
Landscapes (ISFL), is a multilateral fund which promotes reducing greenhouse gas emissions from the
land sector, by smarter land-use planning, policies and practices, and sustainable agriculture.
Statement 2: In 2011, World Bank launched the Carbon Initiative for Development (Ci-Dev) to build
capacity and develop tools and methodologies to help the world’s poorest countries access carbon
finance, mainly in the area of energy access. The Ci-Dev will build on the infrastructure that has been
created by the Clean Development Mechanism (CDM).
307. The answer is (b).
Learning: Environment Ministry launched theGreen Good Deeds’ campaign to sensitize the people
and students, in particular, about climate change and global warming. The campaign was launched
with the aim to restore and return the clean and green environment to the next generation.
308. The answer is (d).
Justification: Operation IceBridge is aimed at monitoring polar ice changes, but not through a satellite,
instead a fixed-wing aircraft is used for this purpose. Since 2003, NASA was using a satellite, ICESat
(Ice, Cloud, and Land Elevation Satellite), for observing polar ice, but this satellite was retired in 2009
due to a technical malfunction. Therefore, NASA introduced the IceBridgeprogramme which utilizes
an aircraft to make similar measurements.
309. The answer is (a).
Justification: Hydrogen is suitable for meeting the decentralized energy needs of a country’s
population. It can be used for telecom tower for providingback up power in rural areas, vehicular
transportation, and industrial applications. In this light, the National Hydrogen Energy Road Map
(2006) is an industry driven planning process that offers long-term energy solutions to the growing
energy needs of India, while ensuring energy security for the country. There are two major initiatives
under this road map and they are as follows:
• Green Initiatives for Future Transport (GIFT) was initiated with the objective to develop and
demonstrate hydrogen powered IC engine and fuel cell based vehicles ranging from small two/ three
wheelers to heavy vehicles through different phases of development.
• Green Initiatives for Power Generation (GIP) was initiated with the objective to envisage developing
and demonstrating hydrogen powered IC engine/turbine and fuel cell based decentralized power
generating systems ranging from small watt capacity to MW size systems.
310. The answer is (c).
Justification: World Climate Services Programme was launched with the aim
• To improve the availability and access to reliable data
• To define the technical and scientific standards
• To advance the knowledge in the area of climate data management and climate analysis
• To develop activities to support them in countries The scope of the programme is
• Climate data and analysis
• Climate monitoring, watch, and prediction
• Climate system operation and infrastructure
• Climate adaptation and risk management
311. The answer is (a).
Justification: The Global Carbon Project (GCP) was established in 2001. It was initiated with the
objective to develop a complete picture of the global carbon cycle, including both its biophysical and
human dimensions together with the interactions and feedbacks between them. This will be:
• Patterns and variability in the global carbon cycle
• Processes and interactions by anthropogenic and non-anthropogenic activities that determine the
dynamics of the carbon cycle.
• Carbon management: What will be the dynamics of the carbon-climate-human system in the future,
and what points of intervention and windows of opportunity exist for human societies to manage this
system?
312. The answer is (a).
Justification:The plan adopts a landscape approach’ in conservation of all wildlife uncultivated flora
and fauna that have both value for mankind and ecology. It helps in conservation of habitats and
also in recovery of threatened species. Statement 2 is incorrect. It not just focusses on conservation
of wildlife by establishing sanctuaries and national parks, but by focusing on the entire region’s
landscape.
313. The answer is (b).
Justification: Set up in 1992, the detailed role and functions of the National Afforestation and Eco-
development Board (NAEB) are:
• Restoration of degraded forests with proper planning
• Focusing on ecological security by restoring forest cover with natural regeneration
• Sponsoring research on technologies for the development of degraded forest land
• Creating awareness about afforestation and promoting the same through people’s movement
• Monitoring and coordinating action plans regarding plantation of trees, restoration of ecology and
undertaking overall necessary activities regarding the same
314. The answer is (b).
Justification: The Compensatory Afforestation Fund has been established under the Public Account
of India and for each state with the passage of the Compensatory Afforestation Fund Act, 2016. The
Act provides an institutional mechanism to ensure utilization of funds in lieu of forest land that is
diverted for non-forest purpose. The Compensatory Afforestation Fund is raised from the developers
who constructed buildings on forest land.
315. The answer is (a).
Background: Any project proponent, government or private, must apply for forest clearance from
Ministry of Environment and Forests (MoEF), before the conversion of land for mining, building of
dams and industries, construction, etc. This proposal is to be submitted through the concerned forest
depart ment of the state government. When the approval for the clearance of the project is given,
concerned regulators and ministry decides the compensation for the loss of forest land. These orders
were issued by the Supreme Court in the year 2009. Henceforth, the fund was set up under the
chairmanship of the Union Minister of Environment & Forests, which evaluates, monitors, and also
provides technical assistance for afforestation activities.
316. The answer is (c).
Justification: Statement 1: The National Clean Energy Fund (NCEF) was established by the Finance Bill
2010-11.
Statement 2: Any project with innovative methods to adopt clean energy technology and research &
development shall be eligible for funding under the NCEF.
Statement 3: Government assistance under the NCEF is generally either 40% or less of the total
project cost. So, 3 is wrong.
317. The answer is (c).
Justification: Statement 1: NCEF was created as a non-lapsable fund in the year 2010-11 using the
carbon tax which is clean energy cess and is used for research activities. This cess is up to the extent of
40% of the total project cost. The fund was established under the Public accounts under Department
of Expenditure.
Statement 2: The Statement is incorrect. The Cabinet Committee on Economic Affairs approves only
those projects which are above ?300 crores.
Statement 3: Eligibility of projects under NCEF is recommended by an inter-ministerial group, chaired
by the Finance Secretary in Ministry of Finance. After recommendation by the inter-ministerial group,
final approval for the projects below Rs. 50 crores is given by the minister for the concerned project
ministry and between ?150 and ^300 crores is approved together by the minister of nodal ministry
and Minister of Finance.
318. The answer is (b).
Learning: NGT deals with disposal of forest conservation and environmental protection cases. It has
been established under the National Green Tribunal Act, 2010. The tribunal also deals with awards
and compensation for the damages to property and persons. Its sanctioned strength is 20 members
in total with 10 from judiciary and other 10 being experts with doctorate in related fields. Its selection
committee is headed by a sitting judge of the Supreme Court of India, who chooses the members of
NGT. The eligibility criteria for chairman is that either he has to be a retired or a sitting judge of the
High Court or a judge of the Supreme Court.
319. The answer is (b).
justification: The NGT was established by the Central Government under the National Green Tribunal
Act, 2010. NGT deals with the disposal of forest conservation and environmental protection cases as
well as with awards and compensation for the damages to property and persons. The NGT has the
jurisdiction to hear the cases for the following Acts.
• The Water (Prevention and Control of Pollution) Act, 1974
• The Water (Prevention and Control of Pollution) Cess Act, 1977
• The Forest (Conservation) Act, 1980
• The Air (Prevention and Control of Pollution) Act, 1981
• The Environment (Protection) Act, 1986
• The Public Liability Insurance Act, 1991
• The Biological Diversity Act, 2002
Statement 1 is incorrect as matters relating to the Wildlife (Protection) Act, 1972 and the Indian
Forest Act, 1927 are not covered by this tribunal.
320. The answer is (d).
Justification: The Government of India has instituted an ECOMARK Scheme for’labelling of
environment friendly products’. The scheme is being administered by the Bureau of Indian Standards.
So far the following product categories have been identified for coverage under this scheme.
• Soaps and detergents
• Paints
• Paper
• Plastics
• Cosmetics
• Textiles
• Batteries
• Wood substitutes
• Propellants and aerosols
• Food items (edible oils including vanaspati, tea, and coffee)
• Electrical and electronics goods
• Packing/packaging materials
• Lubricating/speciality oils
• Drugs
• Foods preservatives and additives
• Pesticides
• Leather
From March 2000, the Central Pollution Control Board has become the member of Global Eco-labelling
Network (GEN).
321. The answer is (c).
Justification: The National Nature Camps Programme is an initiative of the Ministry of Environ-ment
and Forests in Environment Education, which is aimed at creating greater awareness, and making
children empathetic towards environment.This camping experience is for a child who goes to middle
school. The ‘nature experience’ for children and teachers creates sensitivity towards nature and its
conservation, thus creating a positive impact.
322. The answer is (b).
Justification: Statement 1: It is incorrect. TERI is a not-for-profit, policy research institute working in
the fields of environment, sustainable development, and energy. It was established in 1974.
Statement 2: Oil zapper, a bacterial consortium used in bio-remediation of oil sludge and oil spills, is
developed by TERI. It helps in degradation of oil. Nearly 70% of oil refineries in India use the oil zapper
technology developed by TERI. Oil spills in Kuwait are cleaned with this bio-remediation project by
TERI. TERI has contributed significantly towards rating of green buildings by developing the Green
Rating for Integrated Habitat Assessment (GRIHA) system.
323. The answer is (c).
Explanation: Wildlife Trust of India is an Indian nature conservation organization founded in 1998. It
contributes in conservation of wildlife and its habitat and also towards the welfare of animals and
individuals. It has been credited for various programs such as Human-Animal Conflict, One-horned
Rhino and Bears, and rescuing and rehabilitation of tigers, elephants, and leopards. It is a registered
chanty under Section 12A of the Income Tax Act, 1961. This makes Statement 2 correct.
324. The answer is (a).
Justification: PARIVESH (Pro-Active and Responsive facilitation by Interactive, Virtuous and
Environmental Single-window Hub) is an environmental single-window hub for environment, forest
wildlife, and CRZ clearances from Central, state, and district-level authorities. This was developed by
the Ministry of Environment, Forests and Climate Change and the National Informatics Centre (NIC),
which also provides the technical support.
325. The answer is (b).
Justification: Statement 1:The National Ganga Council is also called National Council for Rejuvenation,
Protection and Management of River Ganga. It replaced the National Ganga River Basin Authority
(NGRBA).
Statements 2 and 3: NMCG was established as a registered society under the Societies Registration
Act, 1860 in the year 2011. It has a two-tier body with Executive Committee to approve projects up
to Rs. 1000 crores and a Governing Council, headed by a Director General (DG). State Programme
Management Groups (SPMGs) act as the implementing arm of NMCG at state level. This structure
attempts to bring allstakeholders on one platform to take a holistic approach towards the task of
Ganga cleaning and rejuvenation.
326. The answer is (a).
Justification: National Water Development Agency was set up as an autonomous society under the
Societies Registration Act, 1860 in the year 1982. It carries out studies for optimum utilization of water
resources. Its functions include: Preparing of feasibility report w.r.t Himalayan rivers development
and peninsular rivers development.
Under the National Perspective Plan for Water Resources Development, it prepares detailed projects
to link rivers.
327. The answer is (a).
Justification: Statement 1 is correct. TERI is a not-for-profit, policy research institute working in the
fields of environment, sustainable development, and energy. It was established in 1974. Statement
2 is incorrect. TERI is not an attached agency of International Centre for Climate Governance (ICCG).
ICCG was founded in 2009 to design policies and governance issues relating to climate.
328. The answer is (a).
Justification: Statement 1: Centre for Science and Environment (CSE) is a not-for-profit Delhi-based
advocacy organization carrying public interest research.
Statement 2: It is a think-tank which addresses climate issues and environment-development issues.
It was established in 1980.
Statement 3: Down to Earth, a fortnightly environment magazine is a prominent publication of CSE.
329. The answer is (a).
Learning: The key national survey organizations of India are GSI (geology), ASI (archaeology), BSI
(botany), FiSI (fisheries), FSI (forests), IIEE (ecology), NIO (oceanography), RGCCI (population survey
and language survey), SI (cartography), and ZSI (zoology). Forest Survey of India (FSI), established in
1981, under the Union Ministry of Environment and Forests, carries monitoring and assessment work.
Among the options mentioned, FSI was established after India’s independence. Zoological Survey of
India was established in 1916, Botanical Survey of India was founded in 1890, and Geological Survey
of India (GSI) was founded in 1851.
330. The answer is (b).
Justification: BSI established in 1890, is a research organization with the objective to explore plant
resources and it carries floristic and taxonomy studies. It works under the Ministry of Environment
and Forests (MoEFCC). It also carries out the following functions.
• It documents data relating to fragile ecosystems protected areas, and Phyto diversity.
• It helps in conservation of species that are critically threatened in botanical gardens.
• It develops the national database on live specimens, Indian plants, etc.
Statement 3 in the question is incorrect. Instead of in-situ conservation, it will be ex-situ conservation
as in-situ conservation is done on the site, not in an external alien environment like the botanical
gardens.
331. The answer is (a).
Justification: India launched LTEO programme to study effects of climate change by assessing
anthropogenic and physical biophysical drivers of ecosystem. Under the programme, eight different
bi-omes will be studied, such as the Western Himalayas, Eastern Himalayas, North-Western Arid
Zone, Central Indian Forests, Western Ghats, Andaman & Nicobar Islands, Jammu & Kashmir, and
Sundarbans. It also assesses the change of structure and function in the natural ecosystems, identifies
patterns and drivers of change in the natural ecosystems by monitoring populations of fresh water
fish, birds, mammals, herbivores and carnivores, animal movements, soil processes in forests and
grasslands, biophysical climatic variables, etc.
332. The answer is (b).
Learning: ICFRE, an apex body in the national forestry research system, has been undertaking the
holistic development of forestry through research, education, and planning on aspects of forestry.
The council deals with the solution-based forestry research in tune with the emerging issues in
the sector, including global concerns such as climate change, conservation of biological diversity,
combating desertification, and sustainable management and development of resources. Challenges
with respect to natural resource management are handled through topical research by the council.
Its other objectives include consultancy services in relevant fields like forestry and also its extension
programmes.
Learning: Forest Survey of India (FSI) also presents the data for national planning, conservation,
and sustainable management of environmental protection, as well for the implementation of social
forestry projects along with research and surveys regarding forests.
333. The answer is (c).
Learning: Facility for Low Carbon Technology Deployment is jointly implemented by the Bureau of
Energy Efficiency (BEE) and the United Nations Industrial Development Organization (UNIDO). It is
supported by the GEF. It’s a five-year project that facilitates low-carbon technologies by increasing
use of energy-efficient technologies, which helps with energy consumption reduction. This in the long
run helps reduce GHG emissions.
334. The answer is (b).
Learning: State/UTs oversee management and conservation of wetlands as they are in physical
possession of the area. They also submit long-term comprehensive Management Action Plans (MAPs)
under the National Wetland Conservation Programme (NWCP) and are receiving Centre’s funding for
the same. Considering wetland degradation, they set forth responsible objectives. Research projects
onconservation of wetlands are sponsored by Ministry of Environment, Forests and Climate Change.
335. The answer is (a).
Learning: ECOLEX is a web-based platform that is operated jointly by FAO, IUCN, and UNEP.The
ECOLEX database includes information on treaties, international soft-law, and other policies that are
not binding in nature. It also provides technical guidance and documents, national legislation, etc. It
provides global information on environmental law. Direct access to abstracts and indexing information
is provided to users.
336. The answer is (b).
Justification and Learning: Statements 1 and 2 are incorrect. OGCI is a bottom-up, voluntary, industry-
driven initiative, comprising of 10 oil and gas companies, on the whole representing one-fifth of the
global oil and gas production. It serves as a platform to bring in technological solutions and climate
change coordination.This is the only initiative to provide a full spectrum on what the sector is currently
doing about climate change.
General Studies & Engineering Aptitude Answer Keys
337.
Ans. (A)
338.
Ans. (C)
339.
Ans. (D)
340.
Ans. (D)
341.
Ans. (B)
342.
Ans. (C)
343.
Ans. (B)
Third statement is advantage regarding solar energy rather than limitation. Therefore third
statement is wrong and 1 and 2 are correct.
344. Ans. (A)
345. Ans. (C)
A fuel cell is a device that converts chemical potential energy (energy stored in molecular bonds) into
electrical energy. A fuel cell uses hydrogen gas (H2) and oxygen gas (O2) as fuel. The products
of the reaction in the cell are water, electricity, and heat.
346. Ans. (A)
Solar cells, also called photovoltaic (PV) cells convert sunlight directly into electricity. It generates
direct current. To operate electrical appliances, the inverters are used to convert DC power into AC
power.
347. Ans. (C)
Biogas is a type of biofuel that is naturally produced from the decomposition of organic
waste. When organic matter, such as food scraps and animal waste, break down in an a n a e r o b i c
environment (an environment absent of oxygen) they release a blend of gases, primarily methane
and carbon dioxide. Because this decomposition happens in an anaerobic environment, the process
of producing biogas is also known as anaerobic digestion.
348. Ans. (B)
Biomass power plants provide electricity as well as manure which can be used in agriculture.
349.
Ans. (B)
Very little maintenance is required to keep solar cell running as there are no moving parts in it.
350. Ans. (C)
• Solar constant is the total radiation energy received on the earth from the Sun per unit
time per unit of area on a theoretical surface perpendicular to the Sun’s rays. It is most
accurately measured from satellites where atmospheric effects are absent.
• The value of the constant is approximately 1.366 kilowatts per square metre.
351. Ans. (A)
Compressed Natural gas is primarily composed of methane, but it also contains ethane, propane,
small amounts of nitrogen, carbon dioxide, hydrogen sulphide and trace amounts of water.
352. Ans. (A)
• REN21 is the global renewable energy policy multi-stakeholder network that connects a
wide range of key actors. Its goal is to facilitate knowledge
exchange, policy development and joint action towards a rapid global transition to renewable
energy.
• It is an international non-profit association and is based in Paris, France.
353. Ans. (A)
Renewable resources are those which can be replenished over time by some natural process, including
farming. These resources have the ability to be renewed over a short period of time. These resources
are inexhaustible source of energy.
354. Ans. (A)
Small hydropower plant is defined as any hydropower project which has an installed capacity up to
25 MW.
355. Ans. (B)
• ITER (International Thermonuclear Experimental Reactor) is the world’s largest experimental
nuclear fusion reactor in southern France which aims to deliver nuclear fusion on a commercial scale,
offering safe, limitless and environmentally responsible energy.
• The project is funded and run by seven member entities--the European Union, India, Japan,
China, Russia, South Korea, and the United States.
356. Ans. (B)
Tides are formed due to gravitational effect of the sun and moon on the earth. This gravitational force
causes a periodic rise and fall of the water level of ocean/sea which can be used to produce
electric power.
357. Ans. (C)
Geothermal energy is relatively inexpensive with respect to other sources of renewable
energy.
358. Ans. (B)
Shale gas is natural gas found within the shale formations. It is colourless, odourless and lighter than
air. It is an unconventional source of methane. Shale gas is now increasingly used in USA.
359. Ans. (C)
• The Jawaharlal Nehru National Solar Mission, also known as National Solar Mission, is one
of the eight key National Mission’s which comprise India’s National Action Plan on Climate Change
(NAPCC). The mission was approved on January 11, 2010 by the government.
• The Mission has set the ambitious target of deploying 20,000 MW of grid connectedsolar
power by 2022, which was revised to 1,00,000 MW by 2022 during June 2015.
360. Ans. (D)
• The International Solar Alliance is a common platform for cooperation among sun rich
countries lying fully or partially between the Tropics of Cancer and Capricorn who are seeking to
massively ramp up solar energy, thereby helping to bend the global greenhouse emissions curve
whilst providing clean and cheap energy.
• The initiative was launched at the UN Climate Change Conference (CoP21) in Paris at the
end of 2015 by the President of France and the Prime Minister of India
361. Ans. (C)
Thorium is fertile material rather than fissile, and can only be used as a fuel in conjunction with a
fissile material such as recycled plutonium. Thorium fuels can breed fissile uranium-233 to be used in
various kinds of nuclear reactors.
362. Ans. (B)
363. Ans. (D)
364. Ans. (C)
Ecosystem is broadly terrestrial and aquatic. Aquatic includes pond, river, lake, wetlands, ocean (Coral
reef, limnetic). While terrestrial ecosystems are forest, grassland (Temperate grassland –Prairies/
Steppers).
365. Ans. (D)
• Roughly, one-quarter of coral reefs worldwide are already considered damaged beyond
repair, with another two-thirds under serious threat.
• Major threats to coral reefs and their habitats include:
(i) Climate change
(ii) Destructive fishing practices
(iii) Overfishing
(iv) Careless tourism
(v) Pollution
(vi) Ocean acidification
366. Ans. (A)
• Wetlands International is the global not-for-profit organisation dedicated to the conservation
and restoration of wetlands.
• Its work ranges from research and community based field projects to advocacy and
engagement with governments, corporate and international policy fora and conventions. It works
through partnerships and is supported by contributions from an extensive specialist expert network
and tens of thousands of volunteers.
367. Ans. (D)
• The Convention on Wetlands, called the Ramsar Convention, is the intergovernmental treaty
that provides the framework for the conservation and wise use of wetlands and their resources. The
Convention was adopted in the Iranian city of Ramsar in 1971 and came into force in
1975.
• The World Wetlands Day (WWD) is observed every year on February 2, to mark the
adoption of the Convention on Wetlands in the city of Ramsar in Iran in 1971.
• The Indo-Gangetic flood plain is the largest wetland system in India, extending from
the river Indus in the west to Brahmaputra in the east. This includes the wetlands of the
Himalayan terai and the Indo-Gangetic plains.
368. Ans. (D)
• Mangroves are a group of trees and shrubs that live in the coastal intertidal zone. There are
about 80 different species of mangrove trees. All of these trees grow in areas with l o w - o x y g e n
soil, where slow-moving waters allow fine sediments to accumulate.
• Mangrove forests stabilize the coastline, reducing erosion from storm surges, currents,
waves, and tides. The intricate root system of mangroves also makes these forests attractive to fish
and other organisms seeking food and shelter from predators.
369. Ans. (C)
In an ecosystem, organisms at higher trophic level receive energy from more than one trophic level.
The best example is man. Humans, for example, are primary consumers when they eat
plants such as vegetables. They are secondary consumers when they eat goat. Therefore, humans
can obtain energy from more than one trophic level.
370. Ans. (D)
Earth Overshoot Day marks the date when humanity’s demand for ecological resources and
services in a given year exceeds what Earth can regenerate in that year. It was previously
known as Ecological Debt Day (EDD).
371. Ans. (C)
Two species can’t occupy the same niche because competition is between organisms that are
struggling to survive in a habitat with limited resources.
372. Ans. (C)
• Food web is an important conceptual tool for illustrating the feeding relationships
among species within a community, revealing species interactions and community structure, and
understanding the dynamics of energy transfer in an ecosystem.
• More the complex food webs, more the stability of the ecosystem.
373. Ans. (B)
An energy pyramid represents the amount of energy at each trophic level and loss of energy at
each transfer to another trophic level. Hence the pyramid is always upward irrespective of t y p e
of ecosystem, with a large energy base at the bottom.
374. Ans. (B)
• The Bombay Natural History Society is a nongovernmental organization, engaged in
conservation and biodiversity research.
• It supports many research efforts through grants and publishes the Journal of the Bombay
Natural History Society. It was established in 1883.
375. Ans. (A)
An ecotone is a transition area between two biomes.It is where two communities meet and integrate.
376. Ans. (D)
• Autotrophs are organisms that can produce their own food by photosynthesis or chemical
energy (chemosynthesis).
• Heterotrophs cannot synthesize their own food and rely on other organisms like plants and
animals for nutrition.
377. Ans. (C)
Ecocline is a gradation from one ecosystem to another, with no abrupt boundary between the two. It
describes an ecotone in which a series of bio communities display a continuous gradient.
378. Ans. (A)
Pyramid of numbers represents the number of organisms in each trophic level. They may be upright
(e.g. Grassland ecosystem), inverted (parasitic ecosystem) or dumbbell shaped (forest ecosystem).
379. Ans. (A)
The Montreux Record is a register of wetland sites on the List of Wetlands of International
Importance where changes in ecological character have occurred, are occurring, or are likely
to occur as a result of technological developments, pollution or other human interference. It is
maintained as part of the Ramsar List.
380. Ans. (D)
Bioremediation is the use of living organisms for the recovery/cleaning up of a contaminated
medium (soil, sediment, air, water). The process of bioremediation might involve the introduction
of new organisms to a site, or the adjustment of environmental conditions to enhance degradation
rates of indigenous fauna.
381. Ans. (C)
• Earth Hour is a worldwide movement organized by the World Wide Fund for Nature
(WWF).
The event is held annually encouraging individuals, communities, and businesses to turn off non-essential
electric lights for one hour (from 8:30 to 9:30 pm) on a specific day towards the end of March.
382. Ans. (C)
The concentration of toxic compound increases with increase of trophic level.
383. Ans. (C)
Eutrophication is the enrichment of an ecosystem with chemical nutrients, typically c o m p o u n d s
containing nitrogen, phosphorus, or both. It causes structural changes to the ecosystem such as.
increased production of algae and aquatic plants depletion of fish species, general deterioration
of water quality and other effects.
384. Ans. (D)
• Energy flows through the trophic levels from producers to subsequent trophic levels is
unidirectional.
• Energy level decreases from the first trophic level upwards due to loss of energy in the
form of heat at each trophic level.
385. Ans. (C)
Primary productivity of following ecosystems:
Man groves:40-100
Grassland: 2-12
Lakes: 2-12
Oceans: Less than 4
386. Ans. (A)
Ecological niche of herbivores is smaller than carnivores, whereas that of omnivores is
largest.
387. Ans. (D)
• International Day for Biological Diversity or World Biodiversity Day is observed on
May 22, every year to mark the day when the OBD was adopted at the Rio Earth Summit in
1992.
• September 16 was designated by the United Nations General Assembly as the
International Day for the Preservation of the Ozone Layer. This designation had been
made to commemorate the date, in 1987, on which nations signed the Montreal
Protocol.
388. Ans. (B)
The aphotic zone is the portion of a lake or ocean where there is little or no sunlight. It is
formally defined as the depths beyond which less than 1% of sunlight penetrates. Most food in this
zone comes from dead organisms sinking to the bottom of the lake or ocean from o v e r l y i n g
waters.
389. Ans. (A)
390. Ans. (D)
• To carry out the complementary activities of biodiversity conservation and sustainable
use of natural resources, biosphere reserves are traditionally organized into three
interrelated zones, known as the core area, the buffer zone, and a transition zone.
• Transition Zone is the large outer area of a reserve where people live and work, using
the natural resources of the area in a sustainable manner.
391. Ans. (C)
TRAFFIC, the wildlife trade monitoring network, is a strategic alliance of IUCN and WWF. I t s
mission is to ensure that trade in wild plants and animals is not a threat to the conservation o f
nature.
392. Ans. (B)
• Buffer zone surrounds or is contiguous to the core area. Here, activities are organized
so they do not hinder the conservation objectives of the core area, but rather help to
protect it. The buffer zone might be an area for
experimental research, or may involve ways to manage natural vegetation,
agricultural land, forests or fisheries to enhance overallqualityofproductionwhile
conserving natural processes and biodiversity.
• Transition zone is known as area of cooperation. It is here that the local
communities, conservationagencies,scientists,civil
associations, cultural groups, businesses and other stakeholders agree to work together to
manage and use the area in a sustainable way that will benefit the people who live there.
393. Ans. (B)
Conserving the animals and plants in their natural habitat is known as In-situ conservation.
394. Ans. (C)
BirdLife International is a global partnership of conservation organisations (NGOs) that
strives to conserve birds, their habitats and global biodiversity, working with people towards
sustainability in the use of natural resources.
395. Ans. (D)
• A biodiversity hotspot is a biogeographic region that is both a significant reservoir of
biodiversity and is threatened with destruction.
• As of now, there are 36 biodiversity hotspot regions in the world. India has four
biodiversity hotspots namely Western Ghats, Eastern Himalayas, Indo-Burma and
Sundaland.
396. Ans. (C)
The MIDORI Prize for Biodiversity is a biennial international prize given by the AEON
Environmental Foundation and the Secretariat of the Convention on Biological Diversity
(CBD) to honour individuals who have made outstanding contributions to the conservation a n d
sustainable use of biodiversity at global, regional or local levels. It was established in 2010.
397. Ans. (A)
• The National Green Tribunal (NGT) is a statutory body that was established in 2010 by
the National Green Tribunal Act. It was set up to handle cases and speed up the cases
related to environmental issues.
• The Tribunal has a mandate to dispose of applications and petitions within a period of
six months. India is the third country in the world- after Australia and New Zealand - to
set up such a body to deal with environmental cases.
• The principal bench of the NGT is located in Delhi, with other benches sitting in
Bhopal, Pune, Kolkata and Chennai.
398. Ans. (B)
• The Red Data Book is a type of a public document, which is created for the
recordings of endangered and rare species including plants, animals, fungi and as well a s
some local subspecies, which are present within the region of the state or country.
• It provides complete information for research, studies and also for monitoring the
programs on rare and endangered species and their habits.
399. Ans. (C)
• The Convention on Biological Diversity (CBD) is an international agreement adopted at
the Earth Summit, in Rio de Janeiro, in 1992.
• It is an international legally-binding treaty with three main goals: conservation of
biodiversity;sustainable use of biodiversity; fair and equitable sharing of the benefits
arising from the use of genetic resources.
• The Secretariat of CBD is located in Montreal, Canada.
400. Ans. (A)
• The Cartagena Protocol on Biosafety to the Convention on Biological Diversity is an
international treaty governing the movements of living modified organisms (LMOs)
resulting from modern biotechnology from one country to another.
• It was adopted on 29 January 2000 as a supplementary agreement to the Convention
on Biological Diversity and entered into force on 11 September 2003.
• India has ratified the CBD and it also enacted the Biological Diversity Act in 2002, to g i v e
effect to the provisions of the CBD.
401. Ans. (D)
402.
Ans. (A)
403. Ans. (D)
404. Ans. (A)
405. Ans. (B)
406. Ans. (B)
407. Ans. (C)
408. Ans. (B)
409. Ans. (B)
410. Ans. (D)
Venturi scrubber can remove particulate matter as well as larger particles. This type of
technology is a part of the group of air pollution control devices collectively referred as ‘wet
scrubbers’. It is designed to effectively use the energy from the inlet gas stream to atomize the
liquid being used to scrub the gas stream.
Note: The term ‘scrubber’ is used for those pollution control devices that use liquid to wash
unwanted pollutants from a gas stream.
411. Ans. (B)
Acid rain has constituents of strong acid (H2SO4 and NNO3).
412. Ans. (D)
Minamata disease is caused by mercury presence (especially water pollution).
413. Ans. (A)
414. Ans. (C)
Biomagnification, also known as bio-amplification or biological magnification, is the i n c r e a s i n g
concentration of a substance, such as a toxic chemical, in the tissues of organisms a t
successively higher levels in a food chain.
415. Ans. (B)
When CO is inhaled into the body it combines with the blood, preventing it from absorbing
oxygen. If a person is exposed to CO over a period, it can cause illness and even death.
416. Ans. (A)
• Fly ash is a by product from burning pulverized coal in electric power generating plants.
During combustion, mineral impurities in the coal (clay, feldspar, quartz, and shale)
fuse in suspension and float out of the combustion chamber with the exhaust gases. As
the fused material rises, it cools and solidifies into spherical glassy particles called fly
ash.
• Fly ash is collected from the exhaust gases by electrostatic precipitators or bag
filters.
417. Ans. (C)
Short-term exposure to tropospheric ozone can temporarily affect the lungs, the respiratory tract,
and the eyes. Long-term exposure to relatively low concentrations of ozone can reduce lung
function.
418.
Ans. (B)
Chemical Oxygen Demand or COD is a measurement of the oxygen required to oxidize
soluble and particulate organic matter in water. The COD test is often used as an alternate to B O D
due to shorter length of testing time.
419. Ans. (A)
• Oil-Zapper refers mixture of five microbial strains- are capable of eating up oil, they
take in the pollutants and give out carbon dioxide without producing any harmful
residues.
• There are five different bacterial strains that are immobilized and mixed with a carrier
material such as powdered corncob. This mixture of five bacteria is called Oil Zapper, •
Oil zapper feeds on hydrocarbon compounds present in crude oil and the hazardous
hydrocarbon waste generated by oil refineries, known as Oil Sludge and converts them
into harmless CO2 and water.
420. Ans. (B)
Catalytic converters use a chemical reaction to change pollutants, like carbon monoxide and other
harmful hydrocarbons, to carbon dioxide, nitrogen and water. Tetraethyl lead would tend to clog
up these converters making them inoperable. Thus, unleaded gasoline became the fuel of choice
for any car with a catalytic converter.
421. Ans. (C)
• Under N.A.M.R, four air pollutants viz., Sulphur Dioxide, Oxides of Nitrogen,
Respirable Suspended Particulate Matter (RSPMIPM1Q) and Fine Particulate Matter
(PM2.5) have been identified for regular monitoring at all the locations.
• The monitoring of meteorological parameters such as wind speed and wind direction,
relative humidity (RH) and temperature were also integrated with the monitoring of air
quality.
422. Ans. (D)
Most of algal blooms are the result of an excess of nutrients (particularly phosphorus and
nitrogen) into waters and higher concentrations of these nutrients in water cause increased
growth of algae and green plants.
423.
Ans. (C)
• The Basel Convention on the Control of Trans-boundary Movements of Hazardous
Wastes and their Disposal was adopted on 22 March 1989 by the Conference of
Plenipotentiaries in Basel, Switzerland, in response to a public outcry following the
discovery, in the 1980s, in Africa and other parts of the developing world of deposits of
toxic wastes imported from abroad.
• The convention does not cover radioactive waste, and is ambiguous about waste that is
destined for recycling rather than disposal.
424.Ans. (D)
The challenge of marking nuclear power safer doesn’t end after the power has been generated.
Nuclear fuel remains dangerously radioactive for thousands of years even after it is no longer
useful in a commercial reactor
425. Ans. (C)
A catalytic converter is a device that uses a catalyst to convert three harmful compounds in c a r
exhaust into harmless compounds. In a catalytic converter the catalyst (in the form of platinum and
palladium) is coated onto a ceramic honeycomb or ceramic beads that are housed in a muffler-
like package attached to the exhaust pipe.
426. Ans. (A)
Ground level ozone and Acid rain are examples of Secondary Pollutants.
427. Ans. (D)
• A bio-toilet is a toilet attached to a fermentation tank containing a liquid consortium of
especially developed bacteria—aerobic or anaerobic.
• The bio-digester tank in every toilet is filled with inoculums containing four types of
bacteria. The water trap system in the toilet prevents air from getting into the tank, the
human waste is processed by anaerobic bacteria and the methane gas is allowed to
escape into the air.
428. Ans. (D)
• Stockholm Convention on Persistent Organic Pollutants is an international
environmental treaty, signed in 2001 and effective from May 2004, that aims to
eliminate or restrict the production and use of persistent organic pollutants (POPs).
• Persistent organic pollutants (POPs) are organic compounds that are resistant to
environmental degradation through chemical, biological, and photolytic processes.
Because of their persistence, POPs bio-accumulate with potential adverse impacts on
human health and the environment.
429. Ans. (A)
• The NAQI measures eight major pollutants, namely, particulate matter (PM 10 and PM
2.5), nitrogen dioxide, sulphur dioxide, ozone, carbon monoxide, ammonia and lead.
• The index is based on real time monitoring, and the health risks are easily identified
through a colour coded system. The risks have been signified through six levels - good,
satisfactory, moderately polluted, poor, very poor, and severe.
• The index is a part of the government’s Swachh Bharat Abhiyan.
430. Ans. (C)
• The Biological or Biochemical Oxygen Demand (BOD) value indicates the amount of
oxygen which is needed for the biological degradation of organic substances in water in
mg O2/L.
• BOD serves as a sum parameter for the assessment of pollution of sewage.
431. Ans. (B)
• A chemical reaction between solar ultraviolet radiation and an atmosphere polluted with
hydrocarbons and oxides of nitrogen causes photochemical smog. This is especially
common from automobile exhaust.
• Smog can happen both during the day and at night, but photochemical smog only
happens in the presence of sunlight.
432. Ans. (C)
Dirty Dozen is a group of 12 highly persistent and toxic chemicals: Aldrin, Chlordane,
DDT, Dieldrin, Endrin, Heptachlor, Hexachlorobenzen, Mirex, Polychlorinated biphenyls,
Polychlorinated dibenzo-p-dioxins, Polychlorinated dibenzofurans, andToxaphen.
433. Ans. (D)
• Phthalates is a group of chemicals that are used to dissolve and carry fragrances, soften
plastics and also as sealants and adhesives. These are commonly found in a variety of
products, including cosmetics, paints, nail polish and children’s toys; and have long
beenat the center of a larger international controversy over their health effects.
• Studies show that high exposures to certain kinds of phthalates can cause cancer,
developmental and sex-hormone abnormalities in infants, and can affect fertility. It can
also affects nervous system.
434. Ans. (D)
• Smog formed in the areas of intense solar radiation is called Brown air.
• Smog formed in the areas of lesser solar radiation is called Grey air.
435. Ans. (C)
• Black carbon and Brown carbon are the light absorbing component of the particulate
matter and are formed by the incomplete
burning of fossil fuels and biomass burning and firewood burning
• The difference between them is Black carbon is emitted by high temperature
combustion like diesel fuel burning and Brown
carbon by low temperature combustion including forest fires, firewood burning.
436. Ans. (D)
The Minamata Convention on Mercury is a global treaty to protect human health and the
environment from the adverse effects of mercury. It was agreed at the fifth session of the
Intergovernmental Negotiating Committee on mercury in Geneva, Switzerland in 2013 and
adopted later that year at a Diplomatic Conference (Conference of Plenipotentiaries), held in
Kumamoto, Japan.
437. Ans. (C)
• Rotterdam Convention on the Prior Informed Consent Procedure for Certain Hazardous
Chemicals and Pesticides in International Trade is an international treaty which
promotes shared responsibility between exporting and importing countries to protect
human health and the environment from
certain banned or restricted hazardous chemicalsandpesticides,andprovides
a mechanism for the exchange of information aboutpotentiallyhazardouschemicals.
• The Convention was adopted on 10 September 1998 in Rotterdam, the Netherlands and
entered into force on 24 February 2004.
438. Ans. (A)
Largest Emitters of Carbon dioxide Gas
I. Per Capita Rank (per person):
1. Qatar (39.7 tonnes of CO2) per year 2. Kuwait (24.7 tonnes)
3. United Arab Emirates (21.8tonnes) 4. Australia (18.6 tonnes)
5. Turkmenistan (17.5 tonnes) 6. Oman (17.5 tonnes)
7. USA (16.1 tonnes) 8. Saudi Arabia (16 tonnes)
9. Canada (15.5 tonnes) 10. Kazakhstan (15.2 tonnes)
21. China (7.7 tonnes) 45. India (1.9 tonnes)
II. In terms of Quantity
1. China (26%) 2. U.S.A (12%)
3. EU (10%) 4. India (7%)
5. Russia (5%) 6. Japan (3%)
7. Brazil (2%) 8. Germany (2%)
9. Indonesia (2%) 10. Canada (2%)
439. Ans. (A)
440. Ans. (D)
441. Ans. (A)
442. Ans. (A)
443. Ans. (C)
444. Ans. (B)
445. Ans. (A)
446. Ans. (C)
UV (Ultra violet) rays once fall on the O2 molecule in stratosphere then O2 molecule is split
into nascent oxygen (O2 = O + O). Nascent oxygen is combined with O2 molecule and forming
ozone (O3) {(O + O2 = O3)} Thus, ozone formation is natural phenomena in stratosphere
unlike the troposphere.
447. Ans. (D)
Methane is emitted during the production and transport of coat, natural gas, and oil. M e t h a n e
emissions also result from livestock and other agricultural practices and by the decay of organic
waste in municipal solid waste landfills.

448.
Ans. (D)
The Montreal Protocol is an international environmental agreement with universal ratification to
protect the earth’s ozone layer y eliminating use of ozone depleting substances (ODS), which
would otherwise allow increased UV radiation to reach the earth, resulting in higher incidence of
skin cancer and eye cataracts, more- compromised immune systems, and negative
effects on watersheds, agricultural lands and forests.
The Montreal Protocol sets binding progressive phase out obligations for developed and
developing countries for all the major ozone depleting substances, including CFCs, halons and
less damaging transitional chemicals such as HCFCs.
449. Ans. (D)
• Global Warming Potential (GWP) is the ability of each greenhouse gas to trap
heat in the atmosphere relative to carbon dioxide (CO2) over a specified time
horizon.
• Often, greenhouse gas emissions are calculated in terms of how much CO2
would be required to produce a similar warming effect oven the chosen time
horizon. This is called the carbon dioxide equivalent (CO2eq) value and is
calculated by multiplying the amount of gas by its associated global warming
potential (GWP).
Among Greenhouse gases, Sulphur Hexafluoride has highest global warming
potential over given specified time.
450. Ans. (C)
• A carbon credit is a financial instrument that allows the holder, usually an
energy company, to emit one ton of carbon dioxide. Credits are awarded to
countries or groups that have reduced their greenhouse gases below their
emission quota.
• Carbon credits can be legally traded in the international market at their current
market price.
451. Ans. (C)
The Dobson Unit is the most common unit for measuring ozone concentration. One Dobson Unit
is the number of molecules of ozone that would be required to create a layer of pure ozone of
0.01 millimeters thick at the standard temperature and pressure.
452. Ans. (B)
Photosynthesis is a process used by plants in which energy from sunlight is used to c o n v e r t
carbon dioxide and water into molecules needed for growth. These molecules i n c l u d e
sugars, enzymes and chlorophyll. Light energy is absorbed by the green chemical
chlorophyll.
453. Ans. (A)
The first commitment period of the Kyoto Protocol applies to the four greenhouse gases, viz.
carbon dioxide, methane, nitrous oxide, Sulphur hexafluoride, and two groups of gases, viz.
hydrofluorocarbons (HFCs) and perfluorocarbons (RFCs).
454. Ans. (B)
• Carbon sequestration describes long-term storage of carbon dioxide or other
forms of carbon to either mitigate or defer
global warming and avoid climate change.
• A carbon offset is a reduction in emissions of carbon dioxide or greenhouse
gases made in order to compensate it with renewable energy generation.
• Green carbon is the carbon stored in the plants and soil of natural ecosystems and
is a vital part of the global carbon cycle. It is basically the carbon removed by
photosynthesis and stored in the plants and soil of natural ecosystems and is a vital
part of the global carbon cycle.
• Blue carbon is generally refers as the carbon captured by the world’s oceans.
455. Ans. (B)
• Kigali Agreement is an amendment to the Montreal Protocol to eliminate planet-
warming HFC (hydrofluorocarbon) gases.
• The amendment will allow the use of ozone-saving Montreal Protocol to phase-
out HFCs, a set of 19 gases in the hydrofluorocarbon family that are used
extensively in the air-conditioning and refrigeration industry. HFCs are not
ozone-depleting but are thousands of times more dangerous than carbon dioxide
in causing global warming
• Developed nations will begin phasing down HFC gases by 2019, and developing
countries will follow suit by 2024.
456. Ans. (C)
457.
Ans. (D)
Environmental impact assessment (EIA) study is required for the new projects regarding
future impacts on the environmental components. It aims to achieve sustainable development
taking into care the resource conservation.
458. Ans. (A)
Environmental science is new and evolving branch of science. Therefore training is required to
have expertise in tackling the environmental issues especially in lower management.
459. Ans. (B)
460. Ans. (B)
EIA has been made mandatory under the Environmental (Protection) Act, 1986 for 29 categories of
developmental activities involving investments of Rs. 50 crores and above.
461.Ans. (C)
Assessment of international funding is not an objective of Environmental Impact Assessment.
462. Ans. (C)
• Screening is the determination of whether an EIA is needed or not, and is a
formal requirement under the EIA Regulations.
• Scoping is the process of determining the content and extent of matters that
should be covered in the environmental
information to be submitted to a competentauthorityorother
decision making body.
463. Ans. (D)
The evaluation of baseline data provides crucial information for the process of evaluating
and describing how the project could affect the biophysical and socio- economic environment.
464.
Ans. (D)
The Major Objectives of EIA are:
(i) to identify and describe elements of community and environment likely to be
affected by the proposed developments.
(ii) to identify and quantify any potential losses or damage to flora, fauna and
natural habitats.
(iii) to identify any negative impacts on sites of cultural heritage and to propose
measures to mitigate these impacts.
(iv) to identify the negative impacts and propose the provision of infrastructure or
mitigation measures so as to minimize pollution, environmental disturbance, etc.
(v) to identify, assesses and specify methods, measures and standards, to be
included in the detailed design, construction and operation of the proposed
developments which are necessary to mitigate these environmental impacts and
reducing them to acceptable levels.
(vi) to investigate the extent of side effects of proposed mitigation measures that
may lead to other forms of impacts.
465. Ans. (A)
Strategic Environmental Assessment (SEA) is the process by which environmental
considerations are required to be fully integrated into the preparation of Plans and Programmes
prior to their final adoption. The objectives of SEA are to provide for a high level of
protection of the environment and to promote sustainable development.
466. Ans. (C)
• The purpose of monitoring is to compare predicted and actual impacts,
particularly if the impacts are either very important or the scale of the impact
cannot be very accurately predicted. The
results of monitoring can be used to managetheenvironment,particularly
to highlight problems early so that action can be taken.
• The range of parameters requiring monitoring may be broad or narrow and will
be dictated by the ‘prediction and mitigation’ stage of the EIA.
467. Ans. (A)
• The Leopold matrix or simple matrix method is the best known matrix
methodology available for predicting the impact of a project on the
environment.
• It is a two dimensional matrix cross-referencing:
(a) the activities linked to the project that are supposed to have an impact on man
and the environment.
(b) the existing environmental and social conditions that could possibly be affected
by the project.
468. Ans. (C)


469. Ans. (A)
The final EIA report is referred to as an Environmental Impact Statement (EIS).
470.
Ans. (A)
471.
Ans. (C)
472.
Ans. (A)
473.
Ans. (D)
• Leadership in Energy and Environmental Design (LEED) is a rating system
devised by the United States Green Building Council (USGBC) to evaluate the
environmental performance of a building and encourage market transformation
towards sustainable design. These standards are backed by the Confederation of
Indian Industry and has been in use in India since 2001.
• Green Rating for Integrated Habitat Assessment (GRIHA) is India’s own rating
system jointly developed by TERI and the Ministry of New and Renewable
Energy, Government of India. GRIHA rating system consists of 34 criteria
categorised in four different sections. Some of them are site selection and site
planning, conservation and efficient utilization of resources, building operation
and maintenance, and innovation
474. Ans. (B)
• Sustainable development is development that meets the needs of the present
without compromising the ability of future
generations to meet their own needs.
• It is the organizing principle for meeting human development goals while at the
same time sustaining the ability of natural systems to provide the natural
resources and ecosystem services upon which the economy and society depend.
475. Ans. (C)
Green building is the practice of creating structures and using processes that are
environmentally responsible and resource-efficient throughout a building’s life-cycle from siting to
design, construction, operation, maintenance, renovation and deconstruction.
476. Ans. (C)
Green Public Procurement (GPP) is a process where public authorities seek to source g o o d s ,
services or works with a reduced environmental impact.
477. Ans. (C)
• The National Mission on Sustainable Habitat (NMSH) envisages a framework to
build urban resilience to climate change, by integrating adaptation and
mitigation aspects into the urban planning process.
• The National Mission for Sustainable Habitat which is a component of the
National Action Plan for Climate Change to broadly cover the following
aspects:
(i) Extension of the energy conservation building code - which addresses the design
of new and large commercial buildings to optimize their energy demand;
(ii) Better urban planning and modal shift to public transport - make long term
transport plans to facilitate the growth of medium and small cities in such a way that
ensures efficient and convenient public transport;
(iii) Recycling of material and urban waste management - a special areas of focus will be
development of technology for producing power form waste.
• The National Mission will include a major R&D programme, focusing on bio-chemical
conversion, waste water use, sewage utilization and recycling options wherever possible.

EES
478. Ans. (B)
As per Noise pollution (Regulation and Control) Rules 2000 of India.
Area Code Area/zone Limits in dB Leq
Day time Night time
A Industrial 75 70
B Commercial area 65 55
C Residential area 55 45
D Silence Zone 50 40
479.
Ans. (D)
50dB and 40dB
480. Ans. (C)
Noise level:

P 
L=20log10  rms 
Where,  Prmso 

Prms = Pressure root mean square value


Prmso = Standard pressure root mean square value
The standard pressure root mean square value is taken as 20 Micropascal.
Explanation:
When the noise level increases 10dB sound energy is increased by a factor of 10.
10dB noise level increases then it will become 10 times more powerful than 1dB.
So, when the 20dB noise level increases then it will become 100 times more powerful than 1dB
A 20dB sound is 10 times lounder than a 10dB sound.
The level of noise in a quiet bedroom, 30dB, is 100 times lounder than 10dB. And 40dB is 1,000 times louder
than 10dB
Hence, 100dB is the correct answer.
481. Ans. (C)
The Biological Diversity Act, 2002
The act was enacted in 2002, it aims at the conservation of biological resources, managing its sustainable use
and enabling fair and equitable sharing benefits arising out of the use and knowledge of biological
resources with the local communities.
482. Ans. (B)
The Paris accords legally bound its signatories collectively to limit greenhouse gas emission to keep
the temperature rise “well below” 2.0 degree Celsius (3.6 Fahrenheit) this century.
But the countries also promised to “pursue efforts” to keep the rise below 1.5C (2.7F), which scientists
say would help to avert some of the most catastrophic effects.
483. Ans. (B)
Paris Accord was the Successor to the Kyoto Protocol, the international climate treaty that expired
in 2020. Agreed in December 2015, the Paris Agreement aims to limit the rise in the average global
surface temperature.
484. Ans. (C)
An environmental management system (EMS) is “a system and database which integrates procedures
and processes for training of personnel, monitoring, summarizing and reporting of specialized
environmental performance information to internal and external stakeholders of a firm”
485.
Ans. (D)

486. Ans. (D)


The five main stages of an EMS, as defined by he ISO 14001 standard are
1.Commitment and Policy – Top management commits to environmental improvement and
establishes the organization’s environmental policy. The policy is the foundation of the EMS.
2.Planning: - An organization first identifies environmental aspects of its operations. Environmental
aspects are those items, such as air pollutants or hazardous waste, that can have negative impacts
on people and/or the environment. As organization then determines which aspects are significant by
choosing criteria considered most important by the organization. For example, an organization may
choose worker health and safety, environmental compliance, and cost as its criteria. Once significant
environmental aspects are determined, an organization sets objectives and targets. An objective is
an overall environmental goal (e.g., minimize use of chemical X). A target is a detailed, quantified
requirement that arises from the objectives (e.g., reduce use of chemical X by 25% by September
2030). The final part of the planning stage is devising an action plan for meeting the targets. This
includes designating responsibilities, establishing a schedule, and outlining clearly defined step to
meet the targets.
3. Implementation: - A organization follows through with the action plan using the necessary
resources (human, financial etc). An important component is employee training and awareness for
all employees (including interns, contractors, etc.). Other steps in the implementation stage include
documentation, following operating procedures, setting up internal and external communication
lines.
4, Evaluation: - A company monitors its operations to evaluate whether objectives and targets are
being met. If not, the company take corrective action.
5.Review: - Top management reviews the results of the evaluation to see if the EMS is working
Management determines whether the original environmental policy is consistent with the
organization’s values. The plan is then revised to optimize the effectiveness of the EMS. The review
stage creates a loop of continuous improvement for a company.
487. Ans. (D)
Basic Elements of an EMS include the following:
Reviewing the organization’s environmental goals
Analysing its environmental impacts and compliance obligations (or legal and other requirements)
Setting environmental objectives and targets to reduce environmental impacts and conform with compliance
obligations
Establishing programs to meet these objectives and targets
Monitoring and measuring progress in achieving the objectives
Ensuring employees environmental awareness and competence
Reviewing progress of the EMS and achieving improvements
488. Ans. (A)
Salient Features of the Act
The Act prohibits the following activities without the prior approval from the National Biodiversity
Authority:
Any person or organization (either based in India or not) obtaining any biological resource occurring in India for
its research or commercial utilization.
The transfer of the results of any research relating to any biological resources occurring in, or obtained from,
India.
The claim of any intellectual property rights on any invention based on the research made on the biological
resources obtained from India.
489. Ans. (D)
The Conference of the Parties (COP) is the supreme body of the United Nations Framework Convention
on Climate Change (UNFCCC), made up of representatives from each country that signed the Paris
Agreement and which meets every year.
490. Ans. (A)
The United Nations Environment Programme (UNEP) is responsible for coordinating responses to
environmental issues within the United Nations system. It was established by Maurice Strong, its first
director, after the United Nations Conference on the Human Environment in Stokholm in June 1972.
The organization also develops international environmental agreements; publishes and promotes
environmental science and helps national governments achieve environmental targets. As a member
of the United Nations Development Group. UNEP aims to help the world meet the 17 Sustainable
Development Goals.
UNEP hosts the secretariats of several multilateral environmental agreements and research bodies,
including. The Convention on Biological Diversity (CBD). The Minamata Convention on Mercury. The
Convention on Migratory Species and The Convention on International Trade in Endangered Species
of Wild Fauna and Flora (CITES).
491.
Ans. (A)
The United Nations Environment Programme (UNEP) is responsible for coordinating response to
environmental issues within the United Nations system. It was established by Maurice Strong, its first
director, after the United Nations Conference on the Human Environment in Stockholm in June 1972.
The organization also develops international environmental agreements; publishes and promotes
environmental science and helps national governments achieve environmental targets. As a member
of the United Nations Development Group, UNEP aims to help the world meet the 17 Sustainable
Development Goals.
492.
Ans. (B)
Air pollution
493.
Ans. (D)
Noise pollution
494.
Ans. (B)
0.00002Pa
495.
Ans. (C)
The SDGs were set up in 2015 by the United Nations General Assembly (UN-GA) and are intended to
be achieved by the year 2030. They are included in UN-GA Resolution called the 2030 Agenda or what
is colloquially known as Agenda 2030.
496.
Ans. (C)
Radiocarbon dating:
It also called carbon-14 method was developed by the American physicist Willard F. Libby in about 1946 and has
proved to be a versatile technique of dating fossils and archaeological specimens from 500 to 50,000
years old.
The method is widely used by Pleistocene geologists, anthropologists, archaeologists, and investigators in related
fields.
Radiocarbon dating or Carbon-14 dating method.
Explanation:
Carbon-14 dating is a method of age determination that depends upon the decay to nitrogen of radiocarbon
(carbon-14 Isotope)
Carbon-14 is continually formed in nature by the interaction of neutrons with nitrogen-14 in the Earth’s
atmosphere; the neutrons required for this reaction are produced by cosmic rays interacting with the
atmosphere.
Radiocarbon present in molecules of atmospheric carbon dioxide enters the biological carbon cycle: It is absorbed
from the air by green plants and then passed on to animals through the food chain.
Radiocarbon decays slowly in a living organism, and the amount lost is continually replenished as long as the
organism takes in air or food and once the organism dies, however, it ceases to absorb carrbon-14, so
that the amount of the radiocarbon in its tissues steadily decreases.
Carbon-14 has a half-life of 5,730 ± 40 years i.e., half the amount of the radioisotope present at any given time
will undergo spontaneous disintegration during the succeeding 5,730 years.
Because carbon-14 decays at this constant rate, an estimate of the date at which an organism died can be made by
measuring the amount of its residual radiocarbon.
497. Ans. (D)
Gamma rays
498. Ans. (A)
Micorhiza- Symbiotic relationship between plant roots and fungi Fungus transfers minerals to a plants
root. Root supply carbohydrates.
499. Ans. (C)
Bioventing in the process of stimulating the natural inside biodegradation of contaminants in soil by
providing air to already existing soil microbes.
500. Ans. (A)
Valid
501. Ans. (B)
Resource recovery
502. Ans. (C)
(1) Osteoporosis: It is bone disease, caused mainly by lack of some hormones, particularly estrogen
in women & androgen in Men.
(2) Minamata diseases: It is neurological syndrome, caused by server mercury poisoning
(3) Blue Baby syndrome: Caused by excess Nitrate in drinking water
503. Ans. (B)
The Kyoto Protocol only binds developed countries, and places a heavier burden on them under
the principle of “common but differentiated responsibility and respective capabilities”, because
it recognizes that they are largely responsible for the current high levels of GHG emissions in the
atmosphere.
Annex & Non-Annex nations
The protocol separated countries into two groups:
Annex-I contained developed nations
Non-Annex I contained developing countries
The Protocol placed emission limitations on Annex I countries only. Non-Annex I nations participated
by investing in projects designed to lower emissions in their countries. For these projects, developing
countries earned carbon credits, which they could trade or sell to developed countries, allowing the
developed nations a higher level of maximum carbon emissions for that period. In effect, this function
helped the developed countries to continue emitting GHG vigorously.
504. Ans. (B)
Twenty years after the Stockholm Conference the United Nations conference on Environment and
Development (UNCED) also known as the “Earth Summit” was held at Rio de Janeiro, Brazil from 3-14
June 1992. This global conference, held on the 20th anniversary of the first international conference
on the Human Environment (Stockholm 1972), brought together policy makers, diplomats, scientists,
media personnel and non-governmental organization (NGO) representatives from 179 countries in a
massive effort to reconcile the impact of human socio-economic activates on the environment and
vice versa.
UNCED outcomes:
A set of 27 universally-applicable Principles to help guide international action on the basis of environmental and
economic responsibility.
The United Nations Framework convention on climate change (UNFCCC) – UNFCCC is an international
environmental treaty adopted and implemented by countries all around the world to address the
issue of climate change.
The convention on Biological Diversity (CBD).
The Statement of Forest Principles
Agenda 21
505. Ans. (C)
Undesirable and unwanted sound
506. Ans. (D)
Sulphide form
507. Ans. (D)
Factors influencing energy consumption
1. Prices of fossil fuels such as coal, oil, gas and petrol etc
2. Public education on problems due to energy wastage
3. Lack of knowledge about benefits that can be gained from using renewable energy
4. Quality of transportation system
5. Poor housing development plans
508. Ans. (B)
Food web or interchange between producers and consumers is meant for increasing the stability of
an ecosystem by providing alternate sources of food.
509.
Ans. (C)
Assessment of national funding is not an objective of Environment Impact Assessment.
510. Ans. (C)
Nuclear energy is a non-renewable source of energy.
511. Ans. (B)
As there are no moving parts, a little maintenance is required to operate the solar panels.
512. Ans. (B)
Under Impact assessment you draw the conclusions that allow you to make better business decisions.
You classify the environmental impacts, evaluate them by what is most important to your company,
and translate them into environmental themes such as global warming or human health.
513. Ans. (C)
The National Environment Policy (NEP) by the Ministry of Environment, Forest and Climate Change
(MOEFCC) aims at mainstreaming environmental concerns into all developmental activities.
514. Ans. (D)
Biomass can be obtained from any of these sources.
515. Ans. (A)
In an EIA, baseline data describes the current socioeconomic, biological and physical conditions of the
immediate area and any area the project is predicted to affect.
516. Ans. (B)
The Bhopal disaster, also referred to as the Bhopal gas tragedy, was a gas leak incident on the night
of 2-3 December 1984 at the Union Carbide India Limited (UCIL) pesticide plant in Bhopal, Madhya
Pradesh, India. It is considered among the world’s worst industrial disasters. Over 500,000 people
were exposed to methyl isocyanate (MIC) gas. The highly toxic substance made its way into and
around the small towns located near the plant.
517. Ans. (C)
The Sustainable Development Goals (SDGs) or Global Goals are a collection of 17 interlinked global
goals designed to be a “blueprint to achieve a better and more sustainable future for all”. The
SDGs were set up in 2015 by the United Nations General Assembly (UN-GA) and are intended to
be achieved by the year 2030. They are included in a UN-GA Resolution called the 2030 Agenda or
what is colloquially known as Agenda 2030. The SDGs were developed in the Post-2015 Development
Agenda as the future global development framework to succeed the Millennium Development Goals
which ended in 2015.
The 17 SDGs are: (1) No Poverty, (2) Zero Hunger, (3) Good Health and Well-being, (4) Quality Education
(5) Gender Equality, (6) Clean Water and Sanitation, (7) Affordable and Clean Energy, (8) Decent
Work and Economic Growth, (9) Industry, Innovation and Infrastructure, (10) Reduced Inequality,
(11) Sustainable cities and Communities, (12) Responsible Consumption and Production, (13) Climate
Action, (14) Life Below Water, (15) Life on Land, (16) Peace, Justice and Strong Institutions, (17)
Partnerships for the Goals.
518. Ans. (C)
Natural resources like wind, tides, solar, biomass, etc. generate energy which is known as “non-
conventional resources”. These are pollution free and hence we can use these to produce a clean
form of energy without any wastage.
519. Ans. (C)
Unreliable supply is a disadvantage of most of the renewable energy sources.
520. Ans. (C)
1, 2 & 4 only
521. Ans. (D)
The option itself is the definition of Bioaugmentation and Bio stimulation.
522. Ans. (B)
Resource Efficiency
523. Ans. (D)
Lead
It is present in petrol, diesel, lead batteries, paints, hair dye products, etc. Lead affects children in
particular. It can cause damage to brain and central nervous system, kidneys and digestive problems,
impaired intelligence and interference with development of RBCs, and in some cases, cause cancer.
Sulphur dioxide (SO2)
It is a gas produced from burning coal, mainly in thermal power plants. Some industrial processes, such
as production of paper and smelting of metals, produce sulphur dioxide. It is a major contributor to
smog and acid rain. Sulfur dioxide can lead to lung diseases, Eye and throat irritation, cough, allergies,
impairs enzyme function in respiratory system. Reduces exchange of gases from lung surface.
Mercury
Industries are the major source of Mercury pollution. It is estimated that around 22% of the yearly
world consumption of mercury is used in electrical and electronic equipment. Mercury is used in
thermostats, sensors, relays, switches, medical equipment, lamps, mobile phones and in batteries.
Mercury can lead to Nervous disorders, insomnia, memory loss, excitability, irritation, tremor,
gingivitis and Minamata disease. The developing foetus is highly vulnerable to mercury exposure.
524. Ans. (D)
The broad objectives of the Ministry of Environment, Forest and climate change are:
Conservation and survey of flora, fauna, forests and wildlife
Prevention and control of pollution
Afforestation and regeneration of degraded areas
Protection of the environment and
Ensuring the welfare of animals
Volcanic eruptions are natural phenomenon and doesn’t fall within the ambit of this ministry
525. Ans. (B)
Carbon credits and carbon markets are a component of national and international attempts to
mitigate the growth in concentrations of greenhouse gases (GHGs). One carbon credit is equal to one
tonne of carbon dioxide, or in some markets, carbon dioxide equivalent gases. The Collins English
Dictionary defines a carbon credit as “a certificate showing that a government or company has paid
to have a certain amount of carbon dioxide removed from the environment”. It is a generic term to
assign a value to a reduction or offset of greenhouse gas emissions. Usually equivalent to one tonne
of carbon dioxide equivalent.
For example consider a business that owns a factory putting out 100,000 tonnes of greenhouse gas
emissions in a year. Its government is an Annex I country that enacts a law to limit the emissions that
the business can produce. So the factory is given a quota of say 80,000 tonnes per year. The factory
either reduces its emissions to 80,000 tonnes or is required to purchase carbon credits to offset the
excess. After costing up alternatives the business may decide that it is uneconomical or infeasible to
invest in new machinery for that year. Instead it may choose to buy carbon credits on the open market
from organizations that have been approved as being able to sell legitimate carbon credits.
The concept of carbon credit originated from Kyoto Protocol, where three flexibility mechanisms
were defined to exchange the carbon credits.
526. Ans. (B)

Taiga or Coniferous Forests


They are cold climate forests found in Canada and Russia. Taiga are world’s largest terrestrial ecosystem
(Biome) accounting for about 29% of the earth’s forests.
Tropical grasslands
Found on either side of the equator and extends till tropics
Grows in the area of low to moderate rainfall
Tropical grasslands are known as Savanna in east Africa, Campos in Brazil and Llanos in Venezuela & Colombia.
Plankton are the diverse collection of organisms that live in large bodies of water and are unable to
swim against a current. The individual organisms constituting plankton are called plankters. They
provide a crucial source of food to many small and large aquatic organisms, such as bivalves, fish and
whales.

527.
Ans. (A)
An ecotone is a transition area between two biomes. It is where two communities meet and integrate.
It may be narrow or wide, and it may be local (the zone between a field and forest) or regional (the
transition between forest and grassland ecosystems). An ecotone may appear on the ground as a
gradual blending of the two communities across a broad area, or it may manifest itself as a sharp
boundary line.
528.
Ans. (A)
In ecology, edge effects are changes in population or community structures that occur at the boundary
of two or more habitats. Area with small habitat fragments exhibit especially pronounced edge effects
that may extend throughout the range. As the edge effects increase the boundary habitat allows for
greater biodiversity.
529.
Ans. (D)
`The Kyoto Protocol is an international agreement linked to the United Nations Framework Convention
on Climate change (UNFCCC), which commits its parties by setting internationally binding emission
reduction targets.
The Kyoto Protocol was adopted in Kyoto. Japan, on 11 December 1997 and entered into force on 16
February 2005. Its first commitment period started in 2008 and ended in 2012. In the first period of
the Protocol (2008-12), participating countries committed to reduce their emissions by an average of
5% below 1990 levels.
During the second commitment period, Parties committed to reduce GHG emissions by at least 18
percent below 1990 levels in the eight- year period from 2013 to 2020; however, the composition of
Parties in the second commitment period is different from the first.
USA has not ratified Kyoto Protocol, Canada withdrew from Kyoto Protocol in 2012.
530.
Ans. (A)
Photochemical Oxidants
* The photochemical oxidants are secondary air pollutants formed by the action of sunlight on
nitrogen oxides and reactive hydrocarbons.
* Photochemical smog is formed in the atmosphere when pre-cursor pollutants including Nitrogen
Oxide (NOX) and Volatile Organic Compounds (VOCs) undergo reaction in sunlight to form smog, of
which ozone is the principal component.
Note:
The most important Secondary components produced by these atmospheric photochemical reactions
are ozone, sulphuric acid (H2SO4), Formaldehydes and peroxyacetyl nitrate (PAN).
531.
Ans. (A)
The Vienna convention for the Protection of the Ozone Layer is often called a framework convention
because it served as a framework for efforts to protect the globe’s ozone layer. The Vienna Convention
was adopted in 198 and entered into force on 22 September, 1988.
532.
Ans. (B)
Solar power, wind power, biomass power
533.
Ans. (C)
Food web is a network of food chains interconnected at various trophic levels, so as to form a number
of feeding alternatives amongst the different organisms of a biotic community. Therefore if the food
web exists and there is extinction of a species in a particular food chain, the damage will be prevented
since alternate source of food will be available.
534.
Ans. (B)
Biome is a very large ecosystem or collection of ecosystems with similar biotic and abiotic factors such
as an entire Rain forest with millions of animals and trees, with many different water bodies running
through them. The Great Barrier Reef along the east coast of Australia is not a continuous reef but
made up of over 2900 reefs and 900 islands. It is the largest reef structure in the world.
In a comparative study of grassland ecosystem and pond ecosystem it may be observed that both
biotic and abiotic components are different.
The term ecosystem was coined by the British ecologist A.G. TANSLEY in 1935. He used it to define a
unit that covers all organisms of a given area as well as their relationship to the inorganic environment.
535.
Ans. (C)
Stratification is the characteristic of biological community. Stratification is the vertical layering or the
phenomenon of having more than one stratum formed by different plants/organisms in the same
place.
536.
Ans. (B)
Biomass provides both power as well as manure which can be used in agricultural fields as source of
nutrients for plants.
537.
Ans. (B)
The Sustainable Development Goals (SDGs) or Global Goals are a collection of 17 interlinked global
goals designed to be a “blueprint to achieve a better and more sustainable future for all”.
The SDGs were set up in 2015 by the United Nations General Assembly (UN-GA) and are intended to
be achieved by the year 2030. They are included in UN-GA Resolution called the 2030 Agenda or what
is colloquially known as Agenda 2030. [2] The SDGs were developed in the Post-2015 Development
Agenda as the future global development framework to succeed the Millennium Development Goals
which ended in 2015.
538.
Ans. (C)
Biogas is produced from the decomposition of organic waste. When organic matter breaks down in an
anaerobic environment it release a mixture of gases, primarily methane and carbon dioxide.
539.
Ans. (C)
National Mission on Sustainable Habitat
The National Mission on Sustainable Habitat was approved by the Prime Minister’s Council for Climate
Change in June 2010. The Prime Minister released India’s first National Action Plan on Climate Change
(NAPCC) outlining existing and future policies and programs addressing climate mitigation. There are
eight National Missions under the National Action Plan, with the National Mission on Sustainable
Habitat being one of them.
The focus of these Missions is to promote understanding of climate change, its adaptation and
mitigation, energy efficiency and natural resource conservation.
Objectives of the Mission
The National Mission on Sustainable Habitat seeks to promote:
* Improvements in energy efficiency in buildings through extension of the energy conservation
building code – which addresses the design of new and large commercial buildings to optimize their
energy demand;
* Better urban planning and modal shift to public transport – make long term transport plans to
facilitate the growth of medium and small cities in such a way that ensures efficient and convenient
public transport;
* Improved management of solid and liquid waste, e.g. recycling of material and urban waste
management – with special focus on development of technology for producing power from waste;
* Improved ability of habitats to adapt to climate change by improving resilience of infrastructure,
community based disaster management, and measures for improving advance warning systems for
extreme weather events;
* Conservation through appropriate changes in legal and regulatory framework.
540.
Ans. (D)
EIA objectives can be categorized in to long term and short term.
The long term objectives include the following:
Conservation and sustainable use of natural resources, Protection and enhancement of the quality of
all life forms and Integration of environmental considerations in development planning processes.
The short-term objectives include the following:
To assess the nature, intensity and duration of impact of proposed development projects; To promote
local community and public participation in the EIA processes of a project; To promote social and
cultural considerations in project design; To provide a methodology for prevention and mitigation of
expected impacts due to the presence of a project.
541.
Ans. (A)
Geothermal energy is a type of renewable energy taken from the Earth’s core. It comes from heat
generated during the original formation of the planet and the radioactive decay of materials.
542.
Ans. (C)
To provide speedy justice & to reduce burden of litigation in higher courts.
543.
Ans. (C)
The Chairperson of the NGT is a retired Judge of the Supreme Court.
544.
Ans. (B)
It sets the standards for emission or discharge of environmental pollutants.
545.
Ans. (A)
Structure of NGT
The Tribunal comprises of the Chairperson, the Judicial Members and Expert Members. They Shall
hold office for term of five years and are not eligible for reappointment.
The Chairperson is appointed by the Central Government in consultation with Chief Justice of India
(CJI)
A Selection Committee shall be formed by central government to appoint the Judicial Members and
Expert Members.
There are to be least 10 and maximum 20 full time Judicial members and Expert Members in the
tribunal.
546.
Ans. (D)
547.
Ans. (A)
Ammensalism, here in this type of relationship one organism is harmed while other one is unaffected.
Algal bloom is rapid increase of algal population, which can lead to death of many species, but algae
do not get any benefit from this
548.
Ans. (A)
A synthetic material used for drainage of liquids
549.
Ans. (D)
All of the mentioned subjects fall within the scope of the Ministry of Environment. Forest and Climate
Change.
550.
Ans. (B)
Kyoto Protocol: It is an international treaty, which extends the 1992 United Nations framework
convention on climate change, whose main aim is to reduce greenhouse gas emissions.
Vienna convention: Vienna convection is related to protection of the ozone layer. It is a multilateral
environmental agreement signed in 1985 that provided frameworks for international reductions in
the production of chlorofluorocarbons due to their contribution to the destruction of the ozone layer,
resulting in an increased threat of skin cancer.
551.
Ans. (B)
Coral Reef Facts

* Coral reef biomes are naturally colourful because of the algae. If the coral reef appears whit, this
means there is a pollution problem.
* Occupying less than one quarter of 1% of the marine environment, coral reefs are home to more
than 25% of all known marine fish species.
* Coral reef ecosystems support a variety of human needs. They are important for fisheries, tourism,
shoreline protection, and yield compounds that are important in the development of new medicines.
* Coral reefs form natural barriers that protect nearby shorelines form the eroding forces of the sea,
thereby protecting coastal dwellings, agriculture land and beaches.
552.
Ans. (A)
Net primary productivity is the weight of the organic matter stored by producers in a unit area/volume
per unit time. NPP is equal to the rate of the organic matter created by photosynthesis minus the rate
of respirations and other losses. Stored biomass is transferred from one trophic level to another
trophic level.
553. Ans. (D)
Micro algae and Phytoplankton are the dominant producers in the world’s oceans and therefore their
biomass is the greatest among all other autotrophs in the world’s oceans.
554. Ans. (B)
Secondary productivity is the rate of storage of organic matter by consumers per unit area per unit
time.
555. Ans. (C)
Ecological niche is a set of conditions and resources that are utilized by organisms most readily.
556.
Ans. (D)
The Convention on Biological Diversity (CBD), known informally as the Biodiversity Convention, is a
multilateral treaty. The convention has three main goals:
* The conservation of biological diversity (or biodiversity)
* The sustainable use of its components
* The fair and equitable sharing of benefits arising from genetic resources.
Its objective is to develop national strategies for the conservation and sustainable use of biological
diversity. The convention was opened for signature at the Earth Summit in Rio de Janerio on 5 June
1992 and entered into force on 29 December 1993.
557. Ans. (A)
Ocean thermal energy conversion (OTEC) is a process or technology for producing energy by
harnessing the temperature differences (thermal gradients) between ocean surface waters and deep
ocean waters.
558. Ans. (C)
Monitoring: Keeping up-to-date with the environmental impacts
The purpose of monitoring is to compare pre- and post-project conditions in the development site. It
also compares the predicted and actual impacts. This is especially important in key impacts like water
quality, air quality, soil fertility, endangered species, etc.
Monitoring is also a way to ensure that the development authorities follow the laws and regulations
set by the government regarding their project. It keeps the “cutting corners” habit of all development
authorities in check.
Accurate baseline data is critical for proper monitoring of the site.
The added bonus of monitoring is that if the EAC had somehow missed a possible impact by the
project, that will be identified during the monitoring process. This is quite conceivable. Considering
the multitude of effects that a single activity can have, it is quite possible that the team missed a few
of the impacts that could be caused.
It also allows the project proponents to alter their activities if they observe considerable environmental
damage. Prediction studies are completely theoretical; there is a high chance of error in them. By
observing results in real-time, appropriate corrections can be made.
Monitoring can be of different types_______
Impact/effect monitoring: This analyses the direct relationship between the environmental impact
and the effect it causes. It includes magnitude extent and intensity.
Baseline monitoring: This is one type of monitoring that is done before the commencement of the
project. Baseline monitoring gives us data that is to be included as pre-project status of the site. For
example, air quality baseline data monitoring requires the team to monitor the air at all strategic
locations on the site for 14 consecutive days prior to the commissioning of the work. This allows them
to obtain dust samples every 24 hours.
Mitigation monitoring: This is the monitoring of the effectiveness of the mitigation measures. It
assesses all mitigation measures, be it implementation, design, technology, etc.
Compliance monitoring: This is an all-encompassing monitoring method where we assess if the
predicted values are in compliance with the actual values. It could include mitigation monitoring and
impact/effect monitoring within it.
559.
Ans. (C)
The International Solar Alliance (ISA) is an action-oriented, member-driven, collaborative platform for
increased deployment of solar energy technologies as a means for bringing energy access, ensuring
energy security, and driving energy transition in its member countries.
The ISA strives to develop and deploy cost-effective and transformational energy solutions powered
by the sun to help member countries develop low-carbon growth trajectories, with particular focus on
delivering impact in countries categorized as Least Developed Countries (LDCs) and the Small Island
Developing States (SIDS). Being a global platform, ISA’s partnerships with multilateral development
banks (MDBs), development financial institutions (DFIs), private and public sector organisations, civil
society and other international institutions is key to delivering the change its seeks to see in the world
going ahead.
The ISA was conceived as a joint effort by India and France to mobilize efforts against climae change
through deployment of solar energy solutions. It was conceptualized on the side-lines of the 21st
Conference of Parties (COP21) to the United Nations Framework Convention on Climate Change
(UNFCCC) held in Paris in 2015. With the amendment of its Framework Agreement in 2020, all
member states of the United Nations are now eligible to join the ISA. At present, 101 countries are
signatories to the ISA Framework Agreement, of which 80 countries have submitted the necessary
instruments of ratification to become full members of the ISA.
560.
Ans. (A)
A Strategic Environmental Assessment (SEA) is a systematic process for evaluating the environmental
implications of a proposed policy, plan or programme and provides means for looking at cumulative
effects and appropriately address them at the earliest stage of decision making alongside economic
and social considerations.
The SEA assesses the extent to which a given policy, plan or programme:
* provides an adequate response to environmental and climate change-related challenges;
* may adversely affect the environment and climate resilience, and
* offers opportunities to enhance the state of the environment and contribute the climate-resilient
and low-carbon development.
Ideally, an SEA should be integrated into the policy, plan or programme preparation process from its
early stages and the Government must have a high degree of ownership Public participation is also
essential for a successful SEA.
Compared with the Environmental Impact Assessment (EIA), SEA provides recommendations at a
strategic level and allows a better control over interactions or cumulative effects. There is no single
approach to SEA, which can take different forms according to the specific needs.
561.
Ans. (A)
Biomass gasification means incomplete combustion of biomass resulting in production of combustible
gases consisting of carbon monoxide (CO), hydrogen ((H2) and traces of methane (CH4). This mixture is
called producer gas.
562.
Ans. (B)
The National Green Tribunal (NGT) is a statutory body that was established in 2010 by the Nation Green Tribunal
Act.
It was set up to handle cases and speed up the cases related to environmental issues.
The Central Pollution Control Board (CPCB), the statutory organization, was constituted in September 1974
under the Water (Prevention and Control of Pollution) Act, 1974. Hence statement 1 is not correct.
Further, CPCB was entrusted with the powers and functions under the Air (Prevention and control of pollution)
Act, 1981.
Principal Functions of the CPCB as spelled out in the Water (Prevention and Control of Pollution) Act, 1974 and
the Air (Prevention and Control of Pollution) Act, 1981.
563.
Ans. (D)
Environmental Impact Assessment
Introduction
Environmental Impact Assessment (EIA) is a process of evaluating the likely environmental impacts of a proposed
project or development, taking into account inter-related socio-economic, cultural and human-health
impacts, both beneficial and adverse.
UNEP defines Environmental Impact Assessment (EIA) as a tool used to identify the environmental, social and
economic impacts of a project prior to decision-making. It aims to predict environmental impacts at
an early stage in project planning and design, find ways and means to reduce adverse impacts, shape
projects to suit the local environment and present the predictions and options to decision-makers.
Environment Impact Assessment in Indi is statutorily backed by the Environment Protection Act, 1986 which
contains various provisions on EIA methodology and process.
History of EIA in India
The Indian experience with Environmental Impact Assessment began over 20 years back. It started in 1976-77
when the Planning Commission asked the Department of Science and Technology to examine the
river-valley projects from an environmental angle.
Till 1994, environmental clearance from the Central Government was an administrative decision and lacked
legislative support.
On 27 January 1994, the Union Ministry of Environment and Forests, under the Environmental (Protection) Act
1986, promulgated an EIA notification making Environmental Clearance (EC) mandatory for expansion
or modernisation of any activity or for setting up new projects listed in Schedule 1 of the notification.
The Ministry of Environment, Forests and Climate Change (MoEFCC) notified new EIA legislation in September
2006.
The notification makes it mandatory for various projects such as mining, thermal power plants, river valley,
infrastructure (road, highway, ports, harbours and airports) and industries including very small
electroplating or foundry units to get environment clearance.
However, unlike the EIA notification of 1994, the new legislation has put the onus of clearing projects on the
state government depending on the size/capacity of the project.
Salient Features of 2006 Amendments to EIA Notification
Environment Impact Assessment Notification of 2006 has decentralized the environmental clearance projects by
categorizing the developmental projects in two categories, i.e., Category A (national level appraisal)
and Category B (state level appraisal)
Category A projects are appraised at national level by Impact Assessment Agency (IAA) and the Expert Appraisal
Committee (EAC) and Category B projects are apprised at state level.
State Level Environment Impact Assessment Authority (SEIAA) and State Level Expert Appraisal Committee
(SEAC) are constituted to provide clearance to Category B process.
After 2006 Amendment the EIA cycle comprises of four stages:
* Screening
* Scoping
* Public hearing
* Appraisal
Category A projects require mandatory environmental clearance and thus they do not undergo the screening
process.
Category B projects undergoes screening process and they are classified into two types.
* Category B1 projects (Mandatorily requires EIA).
* Category B2 projects (Do not require EIA)
Thus, Category A projects and Category B, projects undergo the complete EIA process whereas Category B2
projects are excluded from complete EIA process.
564.
Ans. (A)
Strategic Environmental Assessment (SEA) is the process by which environmental considerations
are required to be fully integrated into the preparation of Plans and Programmes prior to their final
adoption. The objectives of SEA are to provide for a high level of protection of the environment and
to promote sustainable development.
565.
Ans. (C)
Environmental assessment (EA) is the assessment of the environmental consequences (positive and
negative) of a plan, policy, program, or actual projects prior to the decision to move forward with the
proposed action. In this context, the term “environmental impact assessment” (EIA) is usually used
when applied to actual projects by individuals or companies.
The purpose of the assessment is to ensure that decision makers consider the environmental impacts
when deciding whether or not to proceed with a project.
According to a United Nations Environment Program the main advantages and benefits of EIA are:
Improved project design/siting
More informed decision-making
More environmentally sensitive decisions
Increased accountability and transparency during the development process
Improved integration of projects into their environmental and social setting
Reduced environmental damage
A positive contribution toward achieving sustainability.
566. Ans. (B)
Drought, soil erosion & Global warming are few phenomena which gets increased by deforestation

while, decrease in rainfall is also one of the negative impacts of deforestation


only. As the human population has expanded, more and more land has been cleared for agriculture
and other pursuits that degrade the soil and make erosion more likely to occur.
Human activity such as over farming, excessive irrigation, deforestation, and erosion adversely impact the ability
of the land to capture and hold water can lead to the development of drought like conditions over a
period of time. Trees absorb the carbon dioxide and release oxygen. Deforestation reduces the sink
for Carbon dioxide and thus could be one of the reasons for global warming.
Plants absorb water from the soil and evaporates it to form clouds. Deforestation leads to cutting
down of trees. As a result, less water is absorbed from the soil and it disturbs the water cycle. The
formation of clouds becomes difficult which leads to reduced rainfall.
ENVIRONMENTAL ECOLOGY
1. The movement of nutrient through the various components of an ecosystem is called
(A) Bio-geo-chemical cycle (B) Bio-geological cycle
(C) Ecological succession (D) Biological cycle
2. The entire sequence of communities that successively change in a particular area are called:
(A) Ecological succession (B) Sere
(C) Community dynamics (D) Pyramid of biomass
3. Which of the following statement(s) is/are correct in respect of the ‘ecoradicals’?
1. They believe that human societies on earth are moving dangerously closer to the limits of
the earth’s carrying capacity.
2. They call for strict population control.
3. They stress on a less consumption-oriented and waste-producing way of life.
Select the correct answer using the code given below:
(A) 1 only (B) 2 only
(C) 2 and 3 only (D) 1, 2 and 3
4. Food chain is:
(A) Relationship between autotrophic organisms
(B) Exchange of genetic materials between two organisms
(C) Passage of food (and thus energy) from one organism to another
(D) Modern entrepreneur establishment providing food outlets
5. Which one of the following is a terrestrial type of ecosystem?
(A) Limnetic (B) Estuary
(C) Prairie (D) Reefs
6. Which of the following is associated with biological nitrogen fixation?
(A) Red algae (B) Brown algae
(C) green algae (D) Blue-green algae
7. Which one of the following is the well known example of a bio-indicator of air pollution?
(A) Lichens (B) Methyl mercury
(C) Rose plant (D) Sunflower
8. The most biodiversity rich area in India is
(A) Gangetic plain (B) Trans Himalayas
(C) Western Ghats (D) Central India
9. With reference to the Deep Carbon Observatory (DCO) which of the following statements is/are
correct?
1. It is global research programme to outreach role of carbon on earth.
2. It conducts field observations of deep microbial ecosystems.
Select the correct answer using the code given below:
Codes:
(A) 1 only (B) 2 only
(C) Both 1 and 2 (D) Neither 1 nor 2
10. Match List I with List II and select the correct using the codes given below the lists:

List-I List-II
A. Epiphytes 1. Mediterranean
B. Acacia 2. Equatorial
C. Baobab 3. Sahara
D. Cedars 4. Savanna
Codes:
A B C D
(A) 2 3 4 1
(B) 2 3 1 4
(C) 2 4 3 1
(D) 2 4 1 3
11. Which of the following statement is not true?
(A) Species diversity increases from equator towards poles.
(B) Tropics harbour more species than temperate areas.
(C) Greatest biodiversity occurs in Amazonean rain forest.
(D) Species diversity decreases from temperate areas towards poles.
12. Consider the following statements:
1. Most of the world’s coral reefs are in tropical waters.
2. More than one-third of the world’s coral reefs are located in the territories of Australia,
Indonesia and Philippines.
3. Coral reefs host far more number of animal phyla than those hosted by tropical rainforests.
Which of the statement(s) given above is/are correct?
(A) 1 and 2 only (B) 3 only
(C) 1 and 3 only (D) 1, 2 and 3
13. Which of the following statements best describes “carbon fertilization”?
(A) Increased plant growth due to increased concentration of carbon dioxide in the
atmosphere.
(B) Increased temperature of Earth due to increased concentration of carbon dioxide in the
atmosphere.
(C) Increased acidity of oceans as a result of increased concentration of carbon dioxide in the
atmosphere.
(D) Adaptation of all living beings on Earth to the climate change brought about by the
increased concentration of carbon dioxide in the
atmosphere.
14. Which of the following is an example of anthropogenic biome?
(A) Freshwater (B) Grassland
(C) Rainforest (D) Cropland
15. Which one of the following groups of organisms forms a forms chain?
(A) Grass, human and fish (B) Grass, goat and human
(C) Tree, tree cutter and tiger (D) Goat, cow and human
16. NPP (Net Primary Productivity) is highest for:
(A) tropical forests (B) Reefs
(C) Swamps (D) Woodland and shrubland
17. The ecological niche of an organism relates to:
(A) specific habitat of the organism
(B) Symbiotic relationship with other organisms
(C) Diverse occurrence of a species under different conditions
(D) Its Interaction with all other organisms
18. Study of a single species and the environmental factors in its habitat is called:
(A) Genecology (B) Synecology
(C) Autecology (D) Ethology
19. Why does the Government of India promote the use of ‘Neem-coated Urea’ in agriculture?
(A) Release of Neem oil in the soil increases nitrogen fixation by the soil
microorganisms.
(B) Neem coating slows down the rate of dissolution of urea in the soil.
(C) Nitrous oxide, which is a greenhouse gas, is not at all released into atmosphere by crop
fields.
(D) It is a combination of a weedicide and a fertilizer for particular crops.
20. Which of the following is an artificial ecosystem?
(A) Rice field (B) Forest
(C) Grassland (D) Lake
21. In a food chain, man is
(A) producer (B) only primary consumer
(C) only secondary Consumer (D) primary as well as secondary consumer
22. Which one of the following is the best description of the term ‘ecosystem’?
(A) A community of organisms interacting with one another
(B) That part of the Earth which is inhabited by living organisms
(C) A community of organisms together with the environment in which they live
(D) The flora and fauna of a geographical area
23. n India, in which one of the following types of forests is teak a dominant tree species?
(A) Tropical moist deciduous forest (B) Tropical rain forest
(C) Tropical thorn scrub forest (D) Temperate forest with grasslands
24. Plants with breathing roots are abundantly found in:
(A) Mangrove areas (B) Deserts
(C) Tropical savannahs (D) Tundra regions
25. Which one of the following kinds of organisms will be the first one to colonise a bare rock?
(A) Algae alone (B) Ferns
(C) Fungi alone (D) Lichens
26. Two places in India, Rushikulya and Gahirmatha have become well known for:
(A) Mass egg-laying by sea turtles (B) Captive breeding of Gharial
(C) Pre-historic cave paintings (D) Uranium deposits
27. Which one of the following is the correct sequence of a food chain?
(A) Diatoms-Crustaceans-Herrings (B) Crustaceans-Diatoms-Herrings
(C) Diatoms-Herrings-Crustaceans (D) Crustaceans-Herrings-Diatoms
28. Lichens, which are capable of initiating ecological succession even on a bare rock, are actually a
symbiotic association of:
(A) algae and bacteria (B) algae and fungi
(C) bacteria and fungi (D) fungi and mosses
29. Which of the following adds/add carbon dioxide to the carbon cycle on the planet Earth? UPSC CSE
(Pre) 2014
1. Volcanic action
2. Respiration
3. Photosynthesis
4. Decay of organic matter
Select the correct answer using the code given below:
(A) 1 and 3 only (B) 2 only
(C) 1, 2 and 4 only (D) 1, 2, 3 and 4
30. Which one of the following is an appropriate description of mangrove plants?
(A) Large wood-yielding trees of the tropical forests
(B) Plants in marshy areas with breathing roots.
(C) Succulent plants that can thrive in arid/desert areas
(D) Vast grassland areas interspersed with deciduous trees and thorny bushes
31. Which one of the following is the correct sequence of the various stages in the food chain?
(A) Grasshoppers, Snakes, Frogs, Vultures
(B) Grasshoppers, Frogs, Snakes, Vultures
(C) Frogs, Vultures, Snakes, Grasshoppers
(D) Vultures, Snakes, Grasshoppers, Frogs
32. An ecosystem comprises:
(A) only trees, shrubs and herbs in an area
(B) only animals in an area
(C) all plants and animals in an area
(D) all living and non-living things in an area
33. The decomposition and recycling of organic matter from dead organisms is made possible to a large
extent by:
(A) bacteria (B) protozoa
(C) virus (D) none of the above
34. Which one of the following is the largest ecosystem of the world?
(A) Ocean (B) Grassland
(C) Forest (D) Mountains
35. Which of the following leaf modifications occurs/occur in desert areas to inhibit water loss?
1. Hard and waxy leaves
2. Tiny leaves or no leaves
3. Thorns instead of leaves
Select the correct answer using the codes given below:
(A) 1 and 2 only (B) 2 only
(C) 1 and 3 only (D) 1, 2 and 3
36. With reference to the food chains in ecosystems, which of the following kinds of organism is/are
known as decomposer organism/organisms?
1. Virus 2. Fungi
3. Bacteria
Select the correct answer using the codes given below
(A) 1 only (B) 2 and 3 only
(C) 1 and 3 only (D) 1, 2 and 3
37. Which one of the following terms describes not only the physical space occupied by an organism, but
also its functional role in the community of organisms?
(A) Ecotone (B) Ecological niche
(C) Habitat (D) Home range
38. Consider the following organisms: UPSC CSE (Pre) 2013
1. Agaricus 2. Nostoc
3. Spirogyra
Which of the above is/are used as biofertilizer / biofertilizers?
(A) 1 and 3 (B) 2 only
(C) 2 and 3 (D) 3 only
39. Which of the following adds/add nitrogen to the soil?
1. Excretion of urea by animals
2. Burning of coal by man
3. Death of vegetation
Select the correct answer using the codes given below:
(A) 1 only (B) 2 and 3 only
(C) 1 and 3 only (D) 1, 2 and 3
40. Which one of the following is the correct sequence of ecosystems in the order of decreasing
productivity?
(A) Oceans, lakes, grasslands, mangroves
(B) Mangroves, oceans, grasslands, lakes
(C) Mangroves, grasslands, lakes, oceans
(D) Oceans, mangroves, lakes, grasslands
41. With reference to the wetlands of India, consider the following statements:
1. The country’s total geographical area under the category of wetlands is recorded more in
Gujarat as compared to other States.
2. In India, the total geographical area of coastal wetlands is larger than that of inland wetlands.
Which of the statement(s) given above is/are correct?
(A) 1 only (B) 2 only
(C) Both1 and 2 (D) Neither 1 nor 2
42. Consider the following kinds of organisms:
1. Bacteria 2. Fungi
3. Flowering plants
Some species of which of the above kinds of organisms are employed as biopesticides?
(A) 1 only (B) 2 and 3 only
(C) 1 and 3 only (D) 1, 2 and 3
43. Consider the following:
1. Photosynthesis 2. Respiration
3. Decay of organic matter 4. Volcanic action
Which of the above add carbon dioxide to the carbon cycle on Earth?
(A) 1 and 4 only (B) 2 and 3 only
(C) 2, 3 and 4 only (D) 1, 2, 3 and 4
44. In the context of ecosystem productivity, marine upwelling zones are important as they increase the
marine productivity by bringing the
1. decomposer microorganisms to the surface.
2. nutrients to the surface.
3. bottom-dwelling organisms to the surface. Which of the statements given above is/are
correct?
(A) 1 and 2 (B) 2 only
(C) 2 and 3 (D) 3 only
45. With reference to the evolution of living organisms, which one of the following sequences is correct?
(A) Octopus−Dolphin-Shark (B) Pangolin−Tortoise−Hawk
(C) Salamander−Python − Kangaroo (D) Frog − Crab − Prawn
PRACTICE QUESTIONS
46. Consider the following statements:
1. Saprotrophs meet their energy requirements from degrading dead organic matter
2. Fragmentation is a part of decomposition process
Which of the statement(s) given above is/are correct?
(A) 1 only (B) 2 only
(C) Both 1 and 2 (D) Neither 1nor 2
47. Consider the following statements:
1. Ecotone regions witness Edge Effect.
2. Ecotone is a zone of tension.
3. Birds are considered as edge species in the terrestrial ecosystem.
Which of the statement(s) given above is/are correct?
(A) 3 only (B) 2 and 3 only
(C) 2 only (D) 1, 2 and 3
48. With reference to Ecological Niche, consider the following statements:
1. A niche is unique for a species.
2. Two species of a habitat can have identical niches.
Which of the statement(s) given above is/are correct?
(A) 1 only (B) 2 only
(C) Both 1 and 2 (D) Neither 1 nor 2
49. Consider the following statements:
1. The climate determines the boundary of a biome.
2. No two biomes are alike.
3. Aquatic systems are also called biomes.
Which of the statement(s) given above is/are correct?
(A) 1 and 2 only (B) 1 and 3 only
(C) 1 only (D) 3 only
50. Consider the following statements:
1. Food web denotes only part of the food and energy.
2. The removal of any intermediate food chain will have no overall effect on food web.
3. The food preferences of organisms can shift seasonally.
Which of the statements given above are correct?
(A) 1 and 3 only (B) 1 and 2 only
(C) 2 and 3 only (D) 1, 2 and 3
51. With reference to ecological pyramid of numbers, consider the following statements:
1. Pyramid of numbers does not count the variation of size of organisms counted at each
trophic level.
2. Pyramid of numbers completely defines the trophic structure for an ecosystem.
Which of the statement(s) given above is/are correct?
(A) 1 only (B) 2 only
(C) Both 1 and (D) Neither 1 nor 2
52. Consider the following biogeochemical cycles:
1. Phosphorous cycle 2. Nitrogen cycle
3. Oxygen cycle 4. Sulphur cycle
Which of the above are included in sedimentary cycle of the biogeochemical cycle?
(A) 1 and 2 only (B) 2 and 3 only
(C) 1 and 4 only (D) 1, 2, 3 and 4
53. Consider the following statements:
1. Terrestrial ecology is the study of interrelation between organisms and environment on
land.
2. Altitudinal and latitudinal variations do not cause shifts and differences in the climatic
patterns.
Which of the statement(s) given above is/are correct?
(A) 1 only (B) 2 only
(C) Both 1 and 2 (D) Neither 1 nor 2
54. The Brazzaville Declaration is related to:
(A) Reduction in the number of road accidents and deaths by half by 2020.
(B) An agreement signed to protect the world’s largest tropical peatland.
(C) World’s first Insurance market committed to climate risk transparency.
(D) Strengthen the International Protection of Refugees, displaced and stateless persons in
Latin America and the Caribbean.

55. Consider the following statements:


1. The biodiversity is generally high in case of wetlands compared to lakes.
2. Dominant producers in case of wetlands are Macrophytes.
Which of the statement(s) given above is/are correct?
(A) 1 only (B) 2 only
(C) Both 1 and 2 (D) Neither 1 nor 2
56. What is the correct sequence in decreasing order of the albedo value of the following ecosystems?
1. Taiga 2. Tropical evergreen
3. Tropical deciduous 4. Tundra
Select the correct answer using the codes given below:
(A) 1, 4, 3, 2 (B) 4, 1, 2, 3
(C) 4, 1, 3, 2 (D) 1, 4, 2, 3
57. Consider the following statements:
1. Allogenic Succession is caused by the biotic components of an ecosystem.
2. Autogenic Succession is caused by the abiotic components of an ecosystem.
3. In Retrogressive Succession, community becomes complex and contains more species.
Which of the statements given above are correct?
(A) 1 and 2 only (B) 2 and 3 only
(C) 1, 2 and 3 (D) None of the above
58. The environment modified by human activities is called:
(A) Natural environment (B) Anthropogenic environment
(C) Urban environment (D) Modern environment
59. Consider the following statements:
1. The capacity of an ecosystem to self-regulate is known as homeostasis.
2. Homeostatic capacity of ecosystems is unlimited.
Which of the statement(s) given above is/are correct?
(A) 1 only (B) 2 only
(C) Both 1 and 2 (D) Neither 1 nor 2
60. Consider the following statements:
1. The salt content of brackish water is between 5 to 35 parts per thousand (ppt).
2. Water bodies having salt content less than 5 parts per thousand are considered to be
freshwater ecosystems.
Which of the statement(s) given above is/are correct?
(A) Both 1 and 2 (B) 1 only
(C) 2 only (D) Neither 1 nor 2
61. Consider the following statements:
1. Oxygen acts as an important limiting factor for the aquatic ecosystem.
2. Sunlight reaches upto photic zone of the aquatic ecosystems.
3. No respiration takes place in aphotic zone.
Which of the statement(s) given above is/are correct?
(A) 2 only (B) 1 and 3 only
(C) 3 only (D) 1 and 2 only
62. Consider the following statements:
1. The Indo-Gangetic flood plain is the largest wetland system in India.
2. Estuary presents an ecotone between freshwater and marine ecosystem.
Which of the statement(s) given above is/are correct?
(A) 1 only (B) 2 only
(C) Both 1 and 2 (D) Neither 1 nor 2
63. Consider the following statements:
1. Cultural eutrophication occurs as a result of human activities.
2. Oligotrophic lakes have very high nutrient content.
Which of the statement(s) given above is/are correct?
(A) 1 only (B) 2 only
(C) Both 1 and 2 (D) Neither 1 nor 2
64. Consider the following statements:
1. An algal bloom is a rapid increase in the population of algae in freshwater only.
2. Algal bloom can produce toxic effect.
Which of the statement(s) given above is/are correct?
(A) 1 only (B) 2 only
(C) Both 1 and 2 (D) Neither 1nor 2
65. Consider the following statements:
1. Algal bloom result in a change in colour of water.
2. Most of the algal blooms are not harmful.
Which of the statement(s) given above is/are correct?
(A) 1 only (B) 2 only
(C) Both 1 and 2 (D) Neither 1 nor 2
66. With reference to Montreux Record, consider the following statements:
1. Montreux Record highlights only those sites where adverse changes in ecological character
have occurred.
2. Sites may be added or removed from the record only with the approval of the contracting
parties in which they lie.
Which of the statement(s) given above is/are correct?
(A) 1 only (B) 2 only
(C) Both 1 and 2 (D) Neither 1 nor 2
67. Consider the following statements:
1. National Plan for Conservation of Aquatic Ecosystems (NPCA) is a single conservation
programme for both wetlands and lakes.
2. It was formulated in 2015 by merging of the National Lake Conservation Plan and the
National Wetlands Conservation Programme.
Which of the statement(s) given above is/are correct?
(A) 1 only (B) 2 only
(C) Both 1 and 2 (D) Neither 1 nor 2
68. With reference to mangroves, consider the following statements:
1. Mangroves represent a characteristic littoral forest ecosystem.
2. Mangroves along the west coast of India are more luxuriant and considerably diverse.
3. “Mangroves for the Future (MFF)” is an initiative of Ministry of Environment, Forest and
Climate Change.
Which of the statement(s) given above is/are correct?
(A) 1 and 3 only (B) 3 only
(C) 1 only (D) 1, 2 and 3
69. Consider the following statements:
1. Fresh water has higher productivity as compared to Estuaries.
2. Towards the sea coast of the estuaries there are large algae and sea grasses.
Which of the statement(s) given above is/are not correct?
(A) 1 only (B) 2 only
(C) Both 1and 2 (D) Neither 1 nor 2
70. Consider the following statements:
1. Corals have a symbiotic relationship with ‘zooxanthellae’.
2. Corals receive colour from the zooxanthellae living within their tissues.
Which of the statement(s) given above is/are correct?
(A) 1 only (B) 2 only
(C) Both 1and 2 (D) Neither 1 nor 2
71. With reference to Phytoplankton-photosynthetic organisms, consider the following statements:
1. They form an essential part of the Carbon cycle.
2. These are crucial bioindicators.
3. Phytoplankton contributes more than half of the oxygen in the environment.
Which of the statement(s) given above is/are correct?
(A) 1 and 2 only (B) 2 and 3 only
(C) 2 only (D) 1, 2 and 3
72. With reference to phytoplankton, consider the following statements:
1. Phytoplankton does not contain chlorophyll.
2. Algal blooms are observed due to unlimited growth of phytoplanktons.
3. These are regarded as the primary producers of the ocean.
Which of the statement(s) given above is/are correct?
(A) 2 and 3 only (B) 1 and 2 only
(C) 1 and 3 only (D) 3 only
73. With reference to plankton, consider the following statements:
1. Planktons are the marine drifters.
2. Abundance of phytoplankton can quickly affect zooplankton populations.
3. Phytoplankton are found near the surface of water.
Which of the statement(s) given above is/are correct?
(A) 1 and 3 only (B) 1 only
(C) 2 and 3 only (D) 1, 2 and 3
74. Consider the following statements:
1. Phytoplankton are responsible for most of the transfer of carbon dioxide from the
atmosphere to the ocean.
2. Phytoplankton productivity declines due to warming of the ocean surface.
Which of the statement(s) given above is/are correct?
(A) 1 only (B) 2 only
(C) Both 1 and 2 (D) Neither 1 nor 2
75. Consider the following statements regarding distribution of phytoplankton:
1. The highest concentrations of Phytoplankton are found at low latitudes.
2. Winds play an important role in its distribution.
Which of the statement(s) given above is/are not correct?
(A) 1 only (B) 2 only
(C) Both 1 and 2 (D) Neither 1 nor 2
76 In the ‘Carbon Cycle’, the processes by which carbon dioxide is returned to the atmosphere are:
(A) Photosynthesis and Respiration (B) Combustion and Respiration
(C) Photosynthesis and Digestion (D) Decomposition and Nutrition
77. Consider the following statements:
1. In the aquatic ecosystem producers are various species of microscopic algae only.
2. Earthworm and certain soil organisms are detritus feeders that help in the decomposition
of organic matter.
3. Fungi are a saprotrophic decomposer.
Which of the statement(s) given above is/are correct?
(A) 1 and 2 only (B) 2 and 3 only
(C) 3 only (D) All of the above
2. Biodiversity & Its Conservation
78. With reference to India’s biodiversity, Ceylon frogmouth, Coppersmith barbet, Gray-chinned minivet
and White-throated redstart are
(A) Birds (B) Primates
(C) Reptiles (D) Amphibians
79. Which one of the following is not a major cause of Wildlife Extinction?
(A) Loss of natural habitat (B) To make fire in the forest
(C) Illegal commercial trade of wildlife (D) Fast pace of population growth
80. Which of the following statements with regard to Biosphere Reserve is/are correct?
1. The idea of Biosphere Reserve was initiated by UNESCO in 1973-74
2. There are 18 designated Biosphere Reserves in India
3. All Biosphere Reserves in India have been included in world network of Biosphere
Reserves of UNESCO
Select the correct answer using the code given below:
(A) 1 only (B) 2 only
(C) 1 and 2 only (D) 1, 2 and 3
81. Which one of the following is not a reason of decrease in biodiversity?
(A) Large scale deforestation (B) Exploitation of forest produces
(C) Maintaining sacred groves (D) Encroachment in forest areas
82. Consider the following statements:
1. Asiatic lion is naturally found in India only.
2. Double-humped camel is naturally found in India only.
3. One-horned rhinoceros is naturally found in India only.
Which of the statement(s) given above is/are correct?
(A) 1 only (B) 2 only
(C) 1 and 3 only (D) 1, 2 and 3
83. Consider the following statements:
1. Some species of turtles are herbivores.
2. Some species of fish are herbivores.
3. Some species of marine mammals are herbivores.
4. Some species of snakes are viviparous.
Which of the statements given above are correct?
(A) 1 and 3 only (B) 2, 3 and 4 only
(C) 2 and 4 only (D) 1, 2, 3 and 4
84. Consider the following pairs
Wildlife Naturally found in
1. Blue-finned Mahseer : Cauvery River
2. Irrawaddy Dolphin : Chambal River
3. Rusty-spotted Cat : Eastern Ghats
Which of the pairs given above are correctly matched?
(A) 1 and 2 only (B) 2 and 3 only
(C) 1 and 3 only (D) 1, 2 and 3
85. Which of the following organisms belongs to Coelenterata (Cnidaria)?
(A) Planaria and liver fluke (B) Sea urchin And Sea cucumber
(C) Hydra and sea anemone (D) Euplectella and sycon
86. Phreatophytes are the plants adapted to grow in
(A) Moist shaded places (B) Rocky environments
(C) Arid environments (D) Active volcanic lava
87. Which one of the following states of India has the largest forest cover?
(A) Himachal Pradesh (B) Arunachal Pradesh
(C) Karnataka (D) Uttarakhand
88. Which one of the following organisms is dependent on saprophytic mode of nutrition?
(A) Agaricus (B) Ulothrix
(C) Riccia (D) Cladophora
89. Which one of the following pair of animals is warm blooded?
(A) Crocodile and Ostrich (B) Hagfish and Dogfish
(C) Tortoise and Ostrich (D) Peacock and Camel
90. E.K. Janaki Ammal National Award on Taxonomy is administered by the
(A) Ministry of Agriculture and Farmers Welfare
(B) Ministry of New and Renewable Energy
(C) Ministry of Health and Family Welfare
(D) Ministry of Environment, Forest and Climate Change
91. Which of the following statements is not true?
(A) Species diversity increases from equator towards ; poles.
(B) Tropics harbour more species than temperate areas,
(C) Greatest biodiversity occurs in the Amazon rainforest.
(D) Species diversity decreases from temperate areas towards poles.
92. At which of the following places is the Secretariat of the International Solar Alliance (I.S.A.) located at
present?
(A) Paris (B) New Delhi
(C) Gurugram (D) New York
93. Given below are two statements, one is labelled as Assertion (A) and the other as Reason (R).
Assertion (A): Maximum numbers of the species or butterflies are found in tropical countries.
Reason (R): Butterflies cannot tolerate low temperatures. Select the correct answer using the codes
given below:
Codes:
(A) Both (A) and (R) are true and (R) is the correct explanation of (A)
(B) Both (A) and (R) are true but (R) is not the correct explanation of (A)
(C) (A) is true but (R) is false
(D) (A) is false but (R) is true
94. Consider the following:
1. Birds 2. Dust blowing
3. Rain 4. Wind blowing
Which of the above spread plant diseases?
(A) 1 and 3 only (B) 3 and 4 only
(C) 1, 2 and 4 only (D) 1, 2, 3 and 4
95. Why is a plant called Prosopis juliflora often mentioned in news?
(A) Its extract is widely used in cosmetics.
(B) It tends to reduce the biodiversity in the area in which it grows.
(C) Its extract is used in the synthesis of pesticides.
(D) None of the above
96. The National Green Tribunal, setup to deal with the expeditious disposal of cases relating to
environmental protection, was setup under the
(A) 12th Five Year Plan (B) 11th Five Year Plan
(C) 10th Five Year Plan (D) 9th Five Year Plan
97. Which of the following sites does not have coral formation?
(A) Gulf of Cambay/Khambhat (B) Gulf of Mannar
(C) Lakshadweep (D) Andaman and Nicobar
98. In India, Indus Dolphin, one of the world’s rarest mammals, is mainly found in which of the following
(A) Ganga (B) Cauvery
(C) Brahmaputra (D) Beas
99. Which of the following statements about biodiversity hotspots is/are correct?
1. Biodiversity hotspots are identified by the International Union for Conservation of
Nature and Natural Resources (IUCN).
2. Biodiversity hot spots are defined according to their vegetation.
3. In India, Eastern Ghats and Western Himalaya are the biodiversity hotspots.
Select the correct answer using the code given below:
(A) 1 and 2 only (B) 3 only
(C) 1, 2 and 3 (D) 1 only
100. Which of the following statement(s) about the olive ridley turtles is/are correct?
1. They are the smallest and most abundant of all sea turtles found in the world.
2. They live in warm waters of Pacific, Atlantic and Indian Oceans.
3. The Coromandel Coast in India is the largest mass nesting site for the olive ridley
turtles.
Select the correct answer using the code given below.
(A) 1 only (B) 1 and 2 only
(C) 2 and 3 only (D) 1, 2 and 3

101. India’s first National Centre for Marine Biodiversity (NCMB) is located in which city?
(A) Bhavnagar (B) Jamnagar
(C) Mumbai (D) Puducherry
102. Which of the following animals can live for the longest duration without drinking water?
(A) Giraffe (B) Camel
(C) Kangaroo (D) Kangaroo rat
103. Which of the following is not an example of in-situ conservation strategy?
(A) Biosphere reserve (B) Botanical garden
(C) National Park (D) Sacred groves
104. When the World Biodiversity Day is observed?
(A) March, 22 (B) May, 22
(C) June, 23 (D) April, 16
105. When was ‘Project Tiger’ launched in India?
(A) 1968 (B) 1973
(C) 1984 (D) 1993
106. In India, if a species of tortoise is declared protected under Schedule I of the Wildlife (Protection) Act,
1972, what does it imply?
(A) It enjoys the same level of protection as the tiger.
(B) It no longer exists in the wild, a few individuals are under captive protection; and now it is
impossible to prevent its extinction.
(C) It is endemic to a particular region of India.
(D) Both (B) and (C) stated above are correct in this context.
107. The term ‘M-STrlPES’ is sometimes seen in the news in the context of:
(A) Captive breeding of Wild Fauna
(B) Maintenance of Tiger Reserves
(C) Indigenous Satellite Navigation System
(D) Security of National Highways
108. If you want to see ghariaIs in their natural habitat, which one of the following is the best place to visit?
(A) Bhitarkanika Mangroves (B) Chambal River
(C) Pulicat Lake (D) Deeper Beel
109. According to the Wildlife (Protection) Act, 1972, which of the following animals can not be hunted by
any person except under some provisions provided by law?
1. Gharial 2. Indian wild ass
3. Wild buffalo
Select the correct answer using the code given below:
(A) 1 only (B) 2 and 3 only
(C) 1 and 3 only (D) 1, 2 and 3
110. Which one of the following does not pass through the Kaziranga National Park?
(A) The Brahmaputra river (B) The Diphlu river
(C) National Highway No. 37 (D) Railway track of Indian Railway
111. Which of the following are the necessary conditions for the growth of coral reefs?
1. Photic conditions
2. Clean and sediment free water
3. Sea salinity of 6%
4. Tropical sea water with temperature of 20°C to 21°C
Select the correct answer using the code given below:
(A) 1, 2 and 4 only (B) 2 and 4 only
(C) 1 and 3 only (D) 1, 2, 3 and 4
112. Gir National Park is famous for:
(A) Tiger (B) Lion
(C) Migratory birds (D) One-horn rhinoceros
113. The Ministry of Environment, Forest and Climate Change, in November 2016, has constituted a four-
member team for conservation and management of which one of the following lakes?
(A) Dal Lake, Jammu and Kashmir (B) Damdama Lake, Haryana
(C) Loktak Lake, Manipur (D) Chilika Lake, Odisha
114. Which one of the following statements is correct?
(A) The end product of fossil fuels is in the form of electrical energy.
(B) Watershed protection increases the rate of surface run-off of water.
(C) If timber is over-harvested, the ecological functions of the forest are improved.
(D) Rivers change their course during floods and lots of fertile soils are lost to the sea.
115. Besides species diversity and ecosystem diversity, which of the following is included in the term
‘biodiversity’?
(A) Genetic diversity (B) Climatic diversity
(C) Cultural diversity (D) Lingual diversity
116. Which one of the following ecosystems has maximum plant bio-mass?
(A) Tropical deciduous forest (B) Tropical rainforest
(C) Temperate deciduous forest (D) Desert shrubs
117. Biodiversity has maximum danger from
(A) destruction of natural habitats and vegetation
(B) Improper agricultural operations
(C) Climate change
(D) Water pollution
118. Recently, for the first time in our country, which of the following States has declared a particular
butterfly as State Butterfly?
(A) Arunachal Pradesh (B) Himachal Pradesh
(C) Karnataka (D) Maharashtra
119. With reference to ‘Red Sanders’, sometimes seen in the news, consider the following statements:
1. It is a tree species found in a part of South India.
2. It is one of the most important trees in the tropical rain forest areas of South India.
Which of the statement(s) given above is/are correct?
(A) 1 only (B) 2 only
(C) Both 1 and 2 (D) Neither 1 nor 2
120. What is/are unique about ‘Kharai camel’, a breed found in India?
1. It is capable of swimming up to three kilometres in seawater.
2. It survives by grazing on mangroves.
3. It lives in the wild and cannot be domesticated
Select the correct answer using the code given below:
(A) 1 and 2 only (B) 3 only
(C) 1 and 3 only (D) 1, 2 and 3
121. Recently, our scientists have discovered a new and distinct species of banana plant which attains
a height of about 11 meters and has orange coloured fruit pulp. In which part of India has it been
discovered?
(A) Andaman Islands (B) Anaimalai Forests
(C) Maikala Hills (D) Tropical rain forests of northeast
122. In which of the following regions of India are you most likely to come across the ‘Great Indian Hornbill’
in its natural habitat?
(A) Sand deserts of northwest India (B) Higher Himalayas of Jammu and
Kashmir
(C) Salt marshes of western Gujarat (D) Western Ghats
123. The Germplasm is required for the propagation of plants and animals. Germplasm is the: CDS Exam (I)
2016
1. Genetic resource
2. Seed or tissue for breeding
3. Egg and sperm repository
4. A germ cell’s determining zone
Select the correct answer using the code given below:
(A) 1 only (B) 1, 2 and 3 only
(C) 2 and 3 only (D) 2 and 4 only
124. With reference to Loktak lake, consider the following statements:
1. Loktak is a freshwater lake in north-eastern India.
2. Loktak lake carries floating islands that are made up of decaying masses of Vegetation, soil
and organic matter.
Which of the statement(s) given above is/are correct?
(A) 1 only (B) 2 only
(C) Both 1 and 2 (D) Neither 1 nor 2
125. The main reason for the decrease in biodiversity is
(A) habitat pollution (B) introduction of exotic species
(C) over exploitation (D) natural habitat destruction
126. With reference to biodiversity, which of the following statement(s) is/are correct?
1. Species richness and high degree of endemism are important criteria for identifying
biodiversity hot spots.
2. Species diversity is highest in the tropics and decreases down the poles.
3. Biodiversity conservation may be in situ as well as ex situ.
Select the correct answer using the code given below:
(A) 1 only (B) 2 and 3 only
(C) 1, 2 and 3 (D) 1 and 3 only
127. Which one of the following is the national aquatic animal of India?
(A) Saltwater crocodile (B) Olive ridley turtle
(C) Gangetic dolphin (D) Gharial
128. With reference to ‘dugong’, a mammal found in India, which of the following statements is/are
correct?
1. It is a herbivorous marine animal.
2. It is found along the entire coast of India.
3. It is given legal protection under Schedule I of the Wildlife (Protection) Act, 1972.
Select the correct answer using the code given below:
(A) 1 and 2 (B) 2 only
(C) 1 and 3 (D) 3 only
129. From which among the following pairs of species, a small quantity of fine quality wool is obtained in
India?
(A) Pashmina goats and Angora rabbits (B) Pashmina rabbits and Angora goats
(C) Pashmina rabbits and Angora sheep (D) Pashmina goats and Angora
sheep
130. Recently, there was a proposal to reintroduce an animal in India which became extinct in this country
several decades ago. Which one of the following is the animal in question?
(A) Cheetah (B) Clouded leopard
(C) Double-humped camel (D) Musk deer
131. Consider the following animals:
1. Jaguar
2. Salt water crocodile
3. Snow leopard
Which of the above is/are naturally found in India?
(A) 1 and 2 only (B) 3 only
(C) 2 and 3 only (D) 1, 2 and 3
132. Consider the following snakes:
1. Common Krait 2. Russell’s Viper
3. Rock Python
Which of the above is/are poisonous snakes?
(A) 1 and 2 only (B) 3 only
(C) 2 and 3 only (D) 1, 2 and 3
133. Consider the following:
1. Adjutant stork
2. Emperor penguin
3. Rattle snake
Which of the above is/are naturally found in India?
(A) 1 only (B) 2 and 3 only
(C) 1 and 3 only (D) 1, 2 and 3
134. Which one of the following fauna of India is an ape?
(A) Hoolock Gibbon (B) Golden Langur
(C) Lion-tailed Macaque (D) Slow Loris
135. The wild goat Ibex is found in:
(A) Coromandel coast (B) Himalayas
(C) Maikal hills (D) Thar desert
136. With reference to the wildlife of India, what is Dugong?
(A) It is a desert gazelle (B) It is a marine mammal
(C) It is migratory raptor (D) It is a saltwater crocodile
137. Which of the following species is the largest of the ‘toothed-whales’?
(A) Finback whale (B) Blue whale
(C) Sperm whale (D) Humpback whale
138. Which of the following countries has more than 30% of its total area under national parks?
(A) Israel (B) Bhutan
(C) Iceland (D) Gabon
139. Which of the following has been identified as a ‘biodiversity hot-spot’ in India?
(A) Sundarban (B) Western Ghats
(C) Manas (D) Cherapunji
140. Which of the following is not responsible for the increase in the biodiversity of an ecosystem?
(A) Productivity of the ecosystem (B) Intermediate-disturbance
(C) Age of the ecosystem (D) Less number of trophic levels
141. Which one of the following states launched in February 2015 a “dolphin-Census” in the coastal eco-
system and shorelines of the State?
(A) Gujarat (B) Karnataka
(C) West Bengal (D) Odisha
142. Which of the following have coral reefs?
1. Andaman and Nicobar Islands
2. Gulf of Kachchh
3. Gulf of Mannar
4. Sunderbans
Select the correct answer using the code given below:
(A) 1, 2 and 3 only (B) 2 and 4 only
(C) 1 and 3 only (D) 1, 2, 3 and 4
143. Consider the following:
1. Bats 2. Bears
3. Rodents
The phenomenon of hibernation can be observed in which of the above kinds of animals?
(A) 1 and 2 only
(B) 2 only
(C) 1, 2 and 3
(D) Hibernation cannot be observed in any of the above
144. Other than poaching, what are the possible reasons for the decline in the population of Ganges River
Dolphins?
1. Construction of dams and barrages on rivers
2. Increase in the population of crocodiles in rivers
3. Getting trapped in fishing nets accidentally
4. Use of synthetic fertilizers and other agricultural chemicals in crop-fields in the vicinity of
rivers
Select the correct answer using the code given below:
(A) 1 and 2 only (B) 2 and 3 only
(C) 1, 3 and 4 only (D) 1, 2, 3 and 4
145. The Union Ministry of Environment, Forests and Climate change approved in principle a proposal for
Western Ghats Development prepared by K. Kasturirangan led panel. The panel has recommended a
ban on development activities in around 60,000 sq. km ecologically sensitive area spread over
(A) Gujarat, Andhra Pradesh, Tamil Nadu, Maharashtra, Kerala and Karnataka
(B) Gujarat, Maharashtra, Goa, Karnataka, Kerala and Tamil Nadu
(C) Odisha, Maharashtra, Goa, Kerala, Andhra Pradesh and Tamil Nadu
(D) Karnataka, Odisha, Gujarat, Kerala, Maharashtra, and Tamil Nadu
146. Which of the following statements is/are correct?
1. In june 2014, the World Heritage Committee has inscribed the Great Himalyan National
Par Conservation Area (GHNPCA) on the World Heritage List on the basis of UNESCO Guidelines.
2. The Great Himalyan National Park is located in Sikkim.
Select the correct answer using the code given below
(A) 1 only (B) 2 only
(C) Both 1 and 2 (D) Neither 1 nor 2

147. In which of the following States are the famous ‘Living Root Bridges’ found?
(A) Arunachal Pradesh (B) Himachal Pradesh
(C) Kerala (D) Meghalaya
148. The last stronghold of Indian Lions in their natural habitat is located in:
(A) Gujarat (B) Rajasthan
(C) Maharashtra (D) West Bengal
149. Which is biodiversity rich place in India?
(A) Western Ghat (B) Eastern Ghat
(C) Thar Desert (D) Bay of Bengal
150. Biodiversity means
(A) diverse kinds of plants and vegetation.
(B) diverse kinds of animals.
(C) diverse kinds of plants and animals in a particular area.
(D) diverse kinds of exotic plants and animals.
151. In which of the following States is lion-tailed macaque found in its natural habitat?
1. Tamil Nadu 2 Kerala
3. Karnataka 4. Andhra Pradesh
Select the correct answer using the codes given below:
(A) 1, 2 and 3 only (B) 2 only
(C) 1, 3 and 4 only (D) 1, 2, 3 and 4
152. Consider the following animals:
1. Sea cow 2. Sea horse
3. Sea lion
Which of the above is/are mammal/mammals?
(A) 1 only (B) 1 and 3 only
(C) 2 and 3 only (D) 1, 2 and 3
153. Consider the following:
1. Star tortoise 2. Monitor lizard
3. Pygmy hog 4. Spider monkey
Which of the above are naturally found in India?
(A) 1, 2 and 3 only (B) 2 and 3 only
(C) 1 and 4 only (D) 1, 2, 3 and 4
154. Vultures which used to be very common in Indian countryside some years ago are rarely seen
nowadays. This is attributed to:
(A) the destruction of their nesting sites by new invasive species
(B) a drug used by cattle owners for treating their diseased cattle
(C) scarcity of food available to them
(D) a widespread, persistent and fatal disease among them.
155. What is the difference between the antelopes Oryx and Chiru?
(A) Oryx is adapted to live in hot and arid areas whereas Chiru is adapted to live in steppes
and semi-desert areas of cold high mountains
(B) Oryx is poached for its antlers whereas Chiru is poached for its musk
(C) Oryx exists in western India only whereas Chiru exists in north-east India only
(D) None of the statements (A), (B) and (C) given above is correct.
156. Government of India encourages the cultivation of ‘sea buckthorn’. What is the importance of this
plant?
1. It helps in controlling soil erosion and in preventing desertification.
2. It is a rich source of biodiesel.
3. It has nutritional value and is well-adapted to live in cold areas of high altitude.
4. Its timber is of great commercial value.
Which of the statement(s) given above is/are correct?
(A) 1 only (B) 2, 3 and 4 only
(C) 1 and 3 only (D) 1, 2, 3 and 4
157. The 2004 Tsunami made people realize that mangroves can serve as a reliable safety hedge against
coastal calamities. How do mangroves function as a safety hedge?
(A) The mangrove swamps separate the human settlements from the sea by a wide zone in
which people neither live nor venture out
(B) The mangroves provide both food and medicines which people are in need of after any
natural disaster
(C) The mangrove trees are tall with dense canopies and serve as an excellent shelter during a
cyclone or tsunami
(D) The mangrove trees do not get uprooted by storms and tides because of their extensive
root
158. Which one of the following is not a site for in-situ method of conservation of flora?
(A) Biosphere Reserve (B) Botanical Garden
(C) National Park (D) Wildlife Sanctuary
159. A sandy and saline area is the natural habitat of an Indian animal species. The animal has no predators
in that area but its existence is threatened due to the destruction of its habitat. Which one of the
following could be that animal?
(A) Indian wild buffalo (B) Indian wild ass
(C) Indian wild boar (D) Indian gazelle
160. King Cobra is the only snake that makes its own nest. Why does it make its nest?
(A) It is snake-eater and the nest helps attract other snakes
(B) It is a viviparous snake and needs a nest to give birth to its offspring
(C) It is an oviparous snake and lays its eggs in the nest and guards the nest until they are
hatched
(D) It is large, cold blooded animal and needs a nest to hibernate in the cold season
161. Consider the following statements:
1. The Taxes tree naturally found in the Himalayas.
2. The Taxus tree is listed in the Red Data Book
3. A drug called “taxol” is obtained from Taxus tree is effective against Parkinson’s disease
Which of the statements given above is/are correct?
(A) 1 only (B) 1 and 2 only
(C) 2 and 3 only (D) 1, 2 and 3
162. Some species of plants are insectivorous. Why?
(A) Their growth in shady and dark places does not allow them to undertake sufficient
photosynthesis and thus they depend on insects for nutrition.
(B) They are adapted to grow in nitrogen deficient soils and thus depend on insects for
sufficient nitrogenous nutrition.
(C) They cannot synthesize certain vitamins themselves and depend on the insects digested by
them
(D) They have remained in that particular stage of evolution as living fossils, a link between
autotrophs and heterotrophs
163. Which feature of some species of blue-green algae helps promote them as bio-fertilizers?
(A) They convert atmospheric methane into ammonia which the crop plants can absorb
readily
(B) They induce the crop plants to produce the enzymes which help convert atmospheric
nitrogen to nitrates
(C) They have the mechanism to convert atmospheric nitrogen into a form that the crop
plants can absorb readily
(D) They induce the roots of the crop plants to absorb the soil nitrates in larger
quantities
164. Consider the following statements:
1. Biodiversity hotspots are located only in tropical regions.
2. India has four biodiversity hotspots i.e., Eastern Himalayas, Western Himalayas, Western
Ghats and Andaman and Nicobar Islands.
Which of the statements given above is/are correct?
(A) 1 only (B) 2 only
(C) Both 1 and 2 (D) Neither 1 nor 2
165. Consider the following regions:
1. Eastern Himalayas
2. Eastern Mediterranean region
3. North-western Australia
Which of the above is/are Biodiversity Hotspot(s)?
(A) 1 only (B) 1 and 2 only
(C) 2 and 3 only (D) 1, 2 and 3
166. The Panda belongs to the same family as that of:
(A) Bear (B) Cat
(C) Dog (D) Rabbit
167. The marine animal called dugong which is vulnerable to extinction is a/an
(A) Amphibian (B) Bony fish
(C) Shark (D) Mammal
168. Consider the following:
1. Camphor
2. Chicory
3. Vanilla
Which of the above is/are plant product(s)?
(A) 1 and 2 only (B) 3 only
(C) 1 and 3 only (D) 1, 2 and 3
169. In the context of Indian wild life, the flying fox is a:
(A) Bat (B) Kite
(C) Stock (D) Vulture
170. Which one of the following Union Ministries is implementing the Biodiesel Mission (as Nodal
Ministry)?
(A) Ministry of Agriculture
(B) Ministry of Science and Technology
(C) Ministry of New and Renewable Energy
(D) Ministry of Rural Development
171. Which one of the following is not essentially a species of the Himalayan vegetation?
(A) Juniper (B) Mahogany
(C) Silver fir (D) Spruce
172. Consider the following statements:
1. Salt-water crocodile is found in the Andaman and Nicobar Islands.
2. Shrew and tapir are found in the Western Ghats of the Malabar region.
Which of the statement(s) given above is/are correct?
(A) 1 only (B) 2 only
(C) Both 1 and 2 (D) Neither 1 nor 2
173. In which one of the following kinds of organisms is the phenomenon found wherein the female kills
the male after copulation?
(A) Dragonfly (B) Honeybee
(C) Spider (D) Pit viper
174. For which one of the following snakes is the diet mainly composed of other snakes?
(A) Krait (B) Russell’s viper
(C) Rattlesnake (D) King cobra
175. Which one of the following is an Insectivorous plant?
(A) Passion flower plant (B) Pitcher plant
(C) Night queen (D) Flame of the forest
176. Among the following, which one lays eggs and does not produce young ones directly?
(A) Crab (B) Dogfish
(C) Gambusia fish (D) Snail
177. Which one among the following has the maximum number of National Parks?
(A) Andaman and Nicobar Islands (B) Arunachal Pradesh
(C) Assam (D) Meghalaya
178. Among the following, which one is not an ape?
(A) Gibbon (B) Gorilla
(C) Langur (D) Orangutan
PRACTICE QUESTIONS:-
179. Consider the following statements:
1. The overall richness of species is concentrated in equatorial regions and tends to decrease
as one move from equatorial to the Polar Regions.
2. Biodiversity in land ecosystems generally increases with increasing altitude.
Which of the statement(s) given above is/are correct?
(A) 1 only (B) 2 only
(C) Both 1 and 2 (D) Neither 1nor 2
180. Consider the following statements:
1. Beta diversity is measured by counting the number of species in a particular ecosystem
2. Alpha diversity is the species diversity between two adjacent ecosystems.
Which of the statements (s) given above is/are correct?
(A) 1 only (B) 2 only
(C) Both 1 and 2 (D) Neither 1 nor 2
181. To qualify as a biodiversity hotspot, a region must:
1. Have at least 1,500 vascular plants as endemics.
2. Have lost over 70% of the original habitat.
Which of the statement(s) given above is/are correct?
(A) 1 only (B) 2 only
(C) Both 1 and 2 (D) Neither 1 nor 2
182. What are the determining factors of biodiversity?
1. Geographic Isolation
2. Dominance of One Species
3. Availability of Ecological Niches
4. Habitat Stress
Select the correct answer using the code given below:
(A) 1, 2 and 3 only (B) 2, 3 and 4 only
(C) 1, 3 and 4 only (D) 1, 2, 3 and 4
183. With reference to Ortolan bunting, which of the following statement(s) is/are not correct?
1. It is endemic to the Eastern Ghats.
2. The International Union for Conservation of Nature’s (IUCN) red list of threatened species
has placed it in the “Endangered” category.
Select the correct answer using the code given below:
(A) 1 only (B) 2 only
(C) Both 1 and 2 (D) Neither 1 nor 2
184. Which of the following statement(s) is/are correct regarding Cartagena Protocol?
1. It protects biological diversity from the potential risks posed by living modified organisms
resulting from modern biotechnology.
2. It facilitates the exchange of information on living modified organisms between
contracting parties.
Select the correct answer using the code given below:
(A) 1 only (B) 2 only
(C) Both 1 and 2 (D) Neither 1 nor 2
185. Match the following List I with List II:

List-I Species List-II Conservation Status
1. Rare species : Population is small and the location is
confined to limited areas.
2. Near Threatened : Species is widespread and abundant
3. Least Concern : Exhaustive survey have failed to record an
individual species.
Which of the pair(s) given above is/are correctly matched?
(A) 1 only (B) 2 only
(C) 3 only (D) 1, 2 and 3
186. Which of the following species is/are categorized as Critically Endangered?
1. North Atlantic Right Whale
2. European Hamster
3. Madame Berthe’s Mouse Lemur
Select the correct answer using the codes given below:
(A) 1 only (B) 3 only
(C) 2 and 3 only (D) 1, 2 and 3
187. Which of the following are the causes of decline of pollinators?
1. Habitat fragmentation
2. Industrial chemicals
3. Parasites
Select the correct answer using the codes given below:
(A) 2 and 3 only (B) 1 and 3 only
(C) 1 and 2 only (D) 1, 2 and 3
188. Consider the following statements:
1. Intergovernmental Science-Policy Platform Biodiversity and Ecosystem Services (IPBES)
create the International Plant Protection Convention or IPPC in 1952.
2. TRAFFIC is a joint programme of World Wide Fund for Nature (WWF) and the
International Union for Conservation of Nature (IUCN).
Which of the statement(s) given above is/are correct?
(A) 1 only (B) 2 only
(C) Both 1 and 2 (D) Neither 1 nor 2
189. With reference to Biological Diversity, consider the following statements:
1. The species richness is concentrated in equatorial regions and tends to decrease as one
move from equatorial to Polar Regions.
2. In marine ecosystems, species richness is much higher in continental shelves.
Which of the statement (s) given above is/are correct?
(A) 1 only (B) 2 only
(C) Both 1 and 2 (D) Neither 1 nor 2
190. With reference to the Biodiversity Hotspots, which of the following statements is/are correct?
1. The concept of Biodiversity hotspot was put forth by Norman Myers.
2. To qualify as a Biodiversity hotspot a region has to have lost at least 70% of its original
habitat.
Select the correct answer using the codes given below:
(A) 1 only (B) 2 only
(C) Both 1 and 2 (D) Neither 1 nor 2
191. Consider the following statements:
1. India accounts for approximately 2.4% of global biodiversity.
2. Western Himalayas is one of the hotspots identified in India.
Which of the statement(s) given above is/are correct?
(A) 1 only (B) 2 only
(C) Both 1 and 2 (D) Neither 1nor 2
192. Which of the following statement(s) is/are correct?
1. India has 10 bio geographical provinces, 5 biome and 2 bioregion domains.
2. The floral diversity in India is concentrated only the 2 biodiversity hotspots, namely
Eastern Himalayas and Western Ghats.
Select the correct answer using the codes given below:
(A) 1 only (B) 2 only
(C) Both 1 and 2 (D) Neither 1 nor 2
193. Which of the following is the correct order of species in India according to their number from lowest
to highest?
1. Angiosperms
2. Algae
3. Bryophytes
4. Fungi
Select the correct answer using the codes given below:
(A) 1-4-2-3 (B) 3-2-4-1
(C) 1-3-2-4 (D) 2-3-4-1
194. With reference to India-Rhino Vision (IRV) 2020, consider the following statements:
1. The objective of the programmes was to double the Rhino population by 2020.
2. Wild-to-wild translocations were an essential part of IRV 2020.
Which of the statement(s) given above is/are correct?
(A) 1 only (B) 2 only
(C) Both 1 and 2 (D) Neither 1 nor 2
195. With reference to Pygmy Hog, consider the following statements:
1. Pygmy Hog is used as an indicator for the management status of grassland habitats.
2. Pygmy Hogs are found only in the north-eastern state of Assam.
Which of the statement(s) given above is/are correct?
(A) 1 only (B) 2 only
(C) Both 1 and 2 (D) Neither 1 nor 2
196. With reference to Kondana soft-furred rat, consider the following statements:
1. Kondana Rat is an Endangered Species on the IUCN Red List of Threatened Species.
2. It is found only in India.
3. It is a nocturnal burrowing rodent.
Which of the statement(s) given above is/are correct?
(A) 2 only (B) 2 and 3 only
(C) 1 and 3 only (D) 3 only
197. Consider the following statements:
1. Monotremes are egg laying mammals.
2. Monotremes are restricted to Australia and New Guinea.
Which of the statement(s) given above is/are correct?
(A) 1 only (B) 2 only
(C) Both1 and 2 (D) Neither 1 nor 2
198. With reference to Siberian Crane, consider the following statements:
1 They are usually found in wetland areas and are migratory birds.
2. Keoladeo National Park is famous habitat of Siberian Crane.
3. They are placed under the critically endangered category in the International Union for
Conservation of Nature (IUCN) Red list of birds.
Which of the statement(s) given above is/are correct?
(A) 1 and 3 only (B) 2 and 3 only
(C) 1 only (D) 1, 2 and 3
199. Consider the following statements:
1. Bonn Convention is an environmental treaty that provides a global platform for the
conservation and sustainable use of only migratory birds and their habitats.
2. World Migratory Bird Day is an awareness-raising campaign celebrated bi-annually.
Which of the statement(s) given above is/are correct?
(A) 1 only (B) 2 only
(C) Both 1 and 2 (D) Neither 1 nor 2
200. Fejervarya goemchi is a:
(A) Bacteria discovered in Nagaland
(B) A fruit species in Andaman island
(C) Newly discovered frog species in Western Ghats
(D) Newly discovered catfish in Andhra Pradesh
201. Which of the following statement(s) is/are correct with reference to the Indus River Dolphin?
1. It is found only in the main channel of the Indus River.
2. It has been declared aquatic animal of Jammu and Kashmir.
3. It is listed Endangered in the IUCN’s Red List of Threatened Species.
Select the correct answer using the code giving below:
(A) 3 only (B) 1 and 2 only
(C) 2 and 3 only (D) 1 and 3 only
202. Consider the following statements with reference to the Golden langur;
1. It is declared as critically endangered by IUCN.
2. It is found in forests of Assam and Bhutan only.
3. Bornadi Wildlife Sanctuary is India’s first wildlife sanctuary with Golden langur as the
primary species.
Which of the statement(s) given above is/are correct?
(A) 1 and 3 only (B) 2 only
(C) 2 and 3 only (D) All of the above
203. Consider the following statements with reference to Cinereous Vulture:
1. India is home to nine species of vulture including Cinereous Vulture.
2. It is classified as Vulnerable under IUCN Red list.
Which of the statement(s) given above is/are correct?
(A) 1 only (B) 2 only
(C) Both 1 and 2 (D) Neither 1 nor 2
204. With reference to Mammals of India (MaOI) initiative, consider the following statements:
1. It is a repository which contains data on Mammals of India.
2. It is launched by the National Centre for Biological Science (NCBS), Bengaluru.
Which of the statement(s) given above is/are correct?
(A) 1 only (B) 2 only
(C) Both 1 and 2 (D) Neither 1 nor 2
205. Which of the following bird(s) is/are declared Critically Endangered as per IUCN Red List (Red Data
Book)?
1. The White-bellied Heron
2. The Bengal norican
3. Spoon Billed Sandpiper
Select the correct answer using the code given below:
(A) 1 and 3 only (B) 2 only
(C) 3 only (D) 1, 2 and 3
206. Which of the following statements best describes Invasive alien species?
1. Invasive alien species are the accidental/intentional entrants outside of their natural
geographical range.
2. These species can change the ecosystem.
Select the correct answer using the code given below:
(A) 1 only (B) 2 only
(C) Both 1 and 2 (D) Neither 1 nor 2
207. With reference to the Himalayan Yew, which of the following statement(s) is/are not correct?
1. It is used for making anti-cancer drugs.
2. Taxol can only be extracted from its roots.
3. It is a critically endangered species.
Select the correct answer using the code given below:
(A) 1 and 2 only (B) 2 only
(C) 2 and 3 only (D) All of the above
208. Regarding the functions of Seagrass beds, consider the following statements:
1. Seagrass beds form an important nursery habitat.
2. These are considered as ‘biodiversity hotspots’.
3. Seagrass bed protects coasts and coastal communities from strong ocean storms.
Which of the statement(s) given above is/are correct?
(A) 3 only (B) 2 and 3 only
(C) 2 only (D) 1, 2 and 3
209. With reference to Seaweeds, consider the following statements:
1. Seaweeds are the organisms differentiated into roots, stems and leaves.
2. These marine plants are found only in the sub tidal zones of the sea.
3. Seaweed has a significant role in mitigating climate change.
Which of the statement(s) given above is /are correct?
(A) 1and 2 only (B) 2 and 3 only
(C) 1 and 3 only (D) 3 only
210. Free swimming microscopic animals in an aquatic environment are referred to as:
(A) Plankton (B) Periphyton
(C) Benthos (D) Nekton
211. Which of the following statements are correct about mangrove sites?
1. Mangrove plants require mix of saline water and freshwater.
2. Mangrove plants require mudflats to enable it to grow and develop.
3. Mangrove plants are found in the inter-tidal zones of sheltered coasts.
Select the correct answer using the code given below:
(A) 1 and 2 only (B) 2 and 3 only
(C) 1 and 3 only (D) 1, 2 and 3
212. Which of the following species of Rhinos is/are found in India?
1. Greater One-Horned Rhino 2. Javan
3. Sumatran 4. Black Rhino
5. White Rhino
Select the correct answer using the code given below.
(A) 1, 2 and 5 only (B) 2, 3 and 4 only
(C) 1 only (D) 1, 2, 3, 4 and 5
213. Consider the following statements with respect to St. Petersburg Declaration on Tiger Conservation:
1. The resolution was adopted in 2010, by the leaders of 13 tiger range countries (TRCs).
2. Global Tiger Recovery Program aimed at doubling the number of wild tigers by the year
2022.
3. India is one of the tiger range countries as per the Global Tiger Recovery Program.
Which of the statements given above are correct?
(A) 1 and 2 only (B) 2 and 3 only
(C) 1 and 3 only (D) 1, 2 and 3
214. Which of the functional committees work under the National Biodiversity Authority (NBA)?
1. Expert Committee on Normally Traded Commodities.
2. Expert Committee on Agro Biodiversity.
3. Expert Committee on Medicinal Plants.
4. Expert Committee to examine Biological Diversity Rules.
Select the correct answer using the code given below:
(A) 1, 2 and 4 only (B) 2 and 4 only
(C) 2, 3 and 4 only (D) 1, 2, 3 and 4
215. Which of the following is a tribe of herdsmen who live with the lions in the Gir forest in a ‘win-win’
situation and do not see the lions as a threat?
(A) Maldhari (B) Bakarwals
(C) Raikas (D) Bishnoi
216. Consider the following statements with reference to National Wildlife Action Plan:
1. There have been four National Wildlife Action Plans launched in India till date.
2. The last Wildlife Action Plan recognized the concerns relating to climate change impact on
wildlife for 2017-2031.
Which of the statement(s) given above Is/are correct?
(A) 1 only (B) 2 only
(C) Both 1 and 2 (D) Neither 1 nor 2
217. Which of the following statement(s) is/are correct?
1. The principal act of Wildlife Protection Act (WPA), 1972 consists of six schedules for the
protection of wildlife.
2. Rhesus Macaque, Bengal Porcupine and Himalayan Brown Bear are listed in Schedule I.
Select the correct answer using the code given below:
(A) 1 only (B) 2 only
(C) Both 1 and 2 (D) Neither 1 nor 2
218. With reference to National Board for Wildlife (NBWL). consider the following statements:
1. The NBWL is an executive body.
2. It serves as an apex body to review all wildlife related matters and its recommendations
are advisory in nature.
3. Approval of NBWL is mandatory for alteration of boundaries in national parks and
sanctuaries.
4. The Minister of Environment, Forest and Climate Change is the chairman of the NBWL.
Which of the statements given above are correct?
(A) 2 and 3 only (B) 1 and 2 only
(C) 2 and 4 only (D) 1 and 3 only
219. Consider the following pairs:
(Schedule) (Fauna)
1. Schedule III : Hog deer
2. Schedule IV : Indian wild Dog
3. Schedule II : Long-tailed tree mouse
4. Schedule I : Black Buck
Which of the pairs given above are correctly matched?
(A) 1 and 3 only (B) 1 and 4 only
(C) 2 and 3 only (D) 2 and 4 only
220. Consider the following statements:
1. Brown Bear is a critically endangered animal found in India and Nepal.
2. Capped Langur is a vulnerable animal found in India and Bhutan only.
Which of the statement(s) given above is/are correct?
(A) 1 only (B) 2 only
(C) Both 1 and 2 (D) Neither 1 nor 2
221. Which one of the group of animals belongs to the category of Critically Endangered species?
(A) Indian Porcupine, Sambar, Hyena and Forest Owlet
(B) Malabar Civet, Bengal Florican, Hawksbill Turtle and Gharial
(C) Green Turtle, Urial, Kiang and Chiru
(D) Sloth Bear, Pygmy Hog, Mouse Deer and Markhor
222. Consider the following pairs:
List-1 List-Il
(Animals) (Category)
1. Fishing Cat : Vulnerable
2. Asiatic Golden Cat : Critically Endangered
3. Leopard Cat : Endangered
4. Marbled Cat : Near Threatened


(A) (B)
(C) (D)
Which of the pairs given above are correctly matched?
(A) 1 and 4 only (B) 2 and 4 only
(C) 2 and 3 only (D) 2, 3 and 4 only
223. Consider the following statements regarding the Wildlife Crime Control Bureau (WCCB):
1. The Wildlife Crime Control Bureau (WCCB) is a statutory body.
2. The Bureau has its headquarter in New Delhi.
3. It assists and advises the customs authorities in inspection of the consignments of flora
and fauna.
Which of the statement(s) given above is/are correct?
(A) 1 only (B) 1 and 3 only
(C) 1 and 2 only (D) All of the above
224. Consider the following fauna of India:
1. Hispid Hare
2. Golden Langur
3. Chinese Pangolin
Which of the above species is/are Vulnerable as per the IUCN Red list of threatened species?
(A) 1 and 2 only (B) 1, 2 and 3
(C) 2 only (D) None
225. With reference to Sea Cow, consider the following statements:
1. It is legally protected under Schedule I of the Wildlife Protection Act, 1972.
2. India is not the signatory of Conservation of Migratory Species of wild animals (CMS) for
the conservation of Sea Cow.
3. It is an omnivorous marine animal.
Which of the statement(s) given above is/are correct?
(A) 2 only (B) 2 and 3 only
(C) 1 and 3 only (D) 1 only
226. Which among the following is not categorised as Vulnerable?
1. Binturong
2. Clouded Leopard
3. Four horned Antelope
4. Asiatic wild ass
Select the correct answer using the code given below:
(A) 1, 2 and 3 only (B) 2 only
(C) 3 and 4 only (D) 4 only
227. Consider the following statements regarding Joint Forest Management (JFM):
1. The primary and most significant basis for JFM is provided by the Wildlife Protection Act,
1972.
2. It is an approach wherein the state forest departments support local forest dwelling and
forest fringe communities to protect and manage forests.
Which of the statement(s) given above is/are correct?
(A) 1 only (B) 2 only
(C) Both 1 and 2 (D) Neither 1 nor 2
228. Arribada-unique mass nesting is related to which of the following?
(A) Olive ridley turtle (B) Red crowned roofed turtle
(C) Indian Roofed Turtle (D) Asian leaf turtle
229. With reference to Red Crowned Roofed Turtle, consider the following statements:
1. Red Crowned Roofed Turtle is endemic to India.
2. It inhabits shallow marine waters.
3. National Chambal River Gharial Sanctuary in India is the only area with substantial
population of the species.
Which of the statement(s) given above is/are correct?
(A) 1 and 2 only (B) 3 only
(C) 1 and 3 only (D) All of the above
230. With reference to One Horned Rhino, consider the following statements:
1. It is the largest species of rhino in the world.
2. It is listed as Endangered on the IUCN Red List.
3. In India, it is mostly found in Odisha.
Which of the statement(s) given above is/are correct?
(A) 1 only (B) 1 and 2 only
(C) 2 and 3 only (D) 1, 2 and 3
231. Which of the following statements are correct regarding the Indian Star Tortoise?
1. The protection status of the Indian Star Tortoise has been upgraded to Appendix I of CITES.
2. It has been declared Vulnerable by IUCN.
3. This species has been placed in Schedule V of the Wildlife (Protection) Act 1972.
Select the correct answer using the code given below:
(A) 1 and 3 only (B) 2 and 3 only
(C) 1 and 2 only (D) 1, 2 and 3
232. Shailesh Nayak Committee Report is related to which of the following?
(A) Mangrove Restoration (B) Coastal Regulation zone
(C) Wildlife Protection (D) Wetland conservation
233. With reference to Mangrove Restoration, consider the following statements:
1. Mangrove restoration is a management strategy to compensate for degraded and lost
ecosystem goods and services.
2. Artificial Regeneration can help in the control of species composition and distribution.
3. Natural regeneration takes place from direct, freely falling and dispersed mangrove
propagules.
Which of the statement(s) given above is/are correct?
(A) 1 and 2 only (B) 2 only
(C) 2 and 3 only (D) All of the above
234. It is the practice of forestry in areas devoid of tree growth and other vegetation situated in places
away from the conventional forest areas. Which of the following forestry has been described in the
statement?
(A) Recreational forestry (B) Farm forestry
(C) Extension forestry (D) None of the above
235. World Wildlife Day is celebrated on which of the following day?
(A) 10 March (B) 4 March
(C) 3 March (D) 15 March
236. Which one of the following groups of animals belongs to the category of endangered species?
(A) Great Indian Bustard, Cheetal, Blue Bull and Asiatic Wild Ass
(B) Red Panda, Musk Deer, Lion-tailed Macaque, and Irrawaddy Dolphin
(C) Snow Leopard, Swamp Deer, Rhesus Monkey and Saras (Crane)
(D) Kashmir Stag, Blue Bull, Hanuman Langur and Cheetal
237. Consider the following statements:
1. Arctic Tundra extends as a discontinuous belt in the northern hemisphere.
2. Alpine Tundra occurs at high mountains above the treeline.
Which of the statement(s) given above is/are correct?
(A) 1 only (B) 2 only
(C) Both 1 and 2 (D) Neither 1 nor 2
238. Consider the following statements:
1. Nature of soil, climate and local topography determine the distribution of trees and their
abundance.
2. Tropical Rainforest have been called the “world’s largest pharmacy”.
3. Tropical rainforest occurs in areas having high temperature and high humidity.
Which of the statements given above are correct?
(A) 1 and 2 only (B) 2 and 3 only
(C) 1, 2 and 3 (D) 1 and 3 only

239. Which of the following are not the ex-situ mode of conservation?
1. National Parks
2. Reserved Forests
3. Biosphere Reserves
4. Botanical Gardens
Select the correct answer using the code given below:
(A) 1, 2 and 4 only (B) 1, 2 and 3 only
(C) 1 and 3 only (D) 1, 2, 3 and 4
240. Consider the following statements regarding Neelakurinji flower:
1. Munnar is home to the highest concentration of neelakurinji plants in India.
2. It is a tropical plant species found in Asia and Australia.
Which of the statement(s) given above is/are correct?
(A) 1 only (B) 2 only
(C) Both 1 and 2 (D) Neither 1 nor 2
241. Which among the following pairs are correctly matched?
Place Name of Grassland
1. North America Steppes
2. Eurasia Prairies
3. South America Pampas
4. Africa Savanna

Select the correct answer using the codes given below:


(A) 1 and 4 only (B) 1 and 3 only
(C) 2 and 4 only (D) 3 and 4 only
242. Consider the following statements:
1. Hog deer is a vulnerable species in the IUCN Red List.
2. Hog deer is typically found in riverine flooded grasslands of Southeast Asia.
3. Hog deer have been placed under Schedule-V of the Indian Wildlife (Protection) Act, 1972.
Which of the statement(s) given above is/are correct?
(A) 2 only (B) 1 and 2 only
(C) 2 and 3 only (D) 1, 2 and 3
243. Consider the following statements regarding Ganges Dolphin:
1. It has been classified as Critically Endangered in the IUCN’s Red List of Threatened Species.
2. It is found in India and Nepal only.
Which of the statement(s) given above is/are not correct?
(A) 1 only (B) 2 only
(C) Both 1 and 2 (D) Neither 1 nor 2
244. Which of the following species are not extinct?
1. Dodo
2. Passenger Pigeon
3. Tasmanian Tiger
Select the correct answer using the code given below:
(A) 1 and 3 only (B) 2 only
(C) 1 and 3 only (D) None of the above
245. Which of the following statement(s) is/are correct regarding the International Tropical Timber
Organization (ITTO)?
1. It is a non-governmental organisation which was established under the auspices of the
United Nations.
2. It facilitates funding for forestry projects in tropical timber producing countries.
3. Headquarter of the ITTO is located in New York. Select the correct answer using the code
given below:
(A) 2 only (B) 2 and 3 only
(C) 1, 2 and 3 (D) 1 and 3 only
246. With reference to the Moringa, consider the following statements:
1. It is a fast-growing, drought-resistant tree native to the Indian subcontinent.
2. It is used as traditional herbal medicine and also for water purification.
Which of the statement(s) given above is/are correct?
(A) 1 only (B) 2 only
(C) Both 1 and 2 (D) Neither 1 nor 2
247. Consider the following statements;
1. Asian Houbara bustards are endemic to the Indian Subcontinent.
2. Asian Houbara bustards are listed as critically j endangered under IUCN Red List.
Which of the statement(s) given above is/are correct?
(A) 1 only (B) 2 only
(C) Both 1 and 2 (D) Neither 1 nor 2
Protected Area Network
248. Which of the following Protected Areas are located in Cauvery basin?
1. Nagarhole National Park
2. Papikonda National Park
3. Sathyamangalam Tiger Reserve
4. Wayanad Wildlife Sanctuary
Select the correct answer using the code given below:
(A) 1 and 2 only (B) 3 and 4 only
(C) 1, 3 and 4 only (D) 1, 2, 3 and 4
249. Which one of the following protected areas is well-known for the conservation of a sub-species of the
Indian swam-deer (Barasingha) that thrives well on hard ground and is exclusively graminivorous?
(A) Kanha National Park (B) Manas National Park
(C) Mudumalai Wildlife Sanctuary (D) Tai Chhapar Wildlife Sanctuary
250. Among the following Tiger Reserves, which one has largest area under “Critical Tiger Habitat ?
(A) Corbett (B) Ranthambore
(C) Nagarjunasagar-Srisailam (D) Sundarbans
251. The National Chambal Sanctuary does not fall in which of the following States?
(A) Uttar Pradesh (B) Madhya Pradesh
(C) Haryana (D) Rajasthan
252. Which of the following are in Agasthyamala Biosphere Reserve?
(A) Neyyar, Peppara and Shendurney Wildlife Sanctuaries; and Kalakad Mundanthurai Tiger
Reserve
(B) Mudumalai, Sathyamangalam and Wayanad Wildlife Sanctuaries; and Silent Valley
National Park
(C) Kaundinya, Gundla Brahmeswaram and Papikonda Wildlife Sanctuaries; and Mukurthi
National Park
(D) Kawai and Sri Venkateswara Wildlife Sanctuaries; and Nagarjunasagar-Srisailam Tiger
Reserve
253. Which one of the following Forest Acts divided forests of India into reserved, protected and Village
Forests?
(A) Forest Act 1864 (B) Forest Act 1865
(C) Forest Act 1866 (D) Forest Act 1878
254. In which one of the following states is Pakhui Wildlife Sanctuary located?
(A) Arunachal Pradesh (B) Manipur
(C) Meghalaya (D) Nagaland
255. Which of the following National Parks of India are declared as World Heritage by UNESCO?
1. Keoladeo National Park
2. Sundarbans National Park
3. Kaziranga National Park
4. Ranthambore National Park
Select the correct answer using the code given below.
(A) 1 and 2 only (B) 1, 2 and 3 only
(C) 3 and 4 only (D) 1, 2, 3 and 4
256. Dachigam National Park is associated with which of the following?
(A) Musk Deer (B) Golden Oriole
(C) Yellow-throated Marten (D) Hangul or Kashmir stag
257. Which one among the following is the largest tiger reserve of India in terms of area of the core/critical
tiger habitat?
(A) Manas (B) Pakke
(C) Nagarjunasagar Srisailam (D) Periyar
258. Consider the following Wildlife Sanctuaries of India:
1. Shikari Devi
2. Bhadra
3. Similipal
4. Pachmarhi
Which one of the following is the correct order of the above Wildlife Sanctuaries in terms of their
location from south to north?
(A) 1–2–3–4 (B) 2–4–3–1
(C) 2 – 3 – 4 – 1 (D) 3–1–2–4
259. Recently there was a proposal to translocate some of the lions from their natural habitat in Gujarat to
which one of the following sites?
(A) Corbett National Park (B) Kuno Palpur Wildlife Sanctuary
(C) Mudumalai Wildlife Sanctuary (D) Sariska National Park
260. Rani Jhansi maritime National Park was founded in:
(A) 1994 (B) 1995
(C) 1996 (D) 1997
261. Which one of the following is the First National Park established in India?
(A) Chandoli National Park (B) Jim Corbett National Park
(C) Gir Forest National Park (D) Dudhwa National Park
262. Which one among the following is the correct order of tiger reserves situated from North to South in
India?
(A) Corbett - Similipal - Sariska - Periyar (B) Periyar - Sariska - Similipal - Corbett
(C) Corbett - Sariska - Similipal - Periyar (D) Periyar – Similipal – Sariska -
Corbett
263. The Mahatma Gandhi Marine National Park is Located in:
(A) Pirotan Island (B) Rameshwaram
(C) Ganga Sagar Island (D) Port Blair
264. At which of the following places is the Marine National Park located?
(A) Chandipur-on-sea (B) Jamnagar
(C) Puducherry (D) Visakhapatnam
265. Which one of the following is a marine national park?
(A) Bhitarkanika (B) Sundarban
(C) Gahirmatha (D) Gulf of Mannar
266. Match List-I with List-Il and select the correct answer using the code given below the List:
List-I (Wildlife Sanctuary) List-II (State)
A. Mahananda 1. Madhya Pradesh
B. Lakhari Valley 2. Uttar Pradesh
C. Pachmarhi 3. West Bengal
D. Chandra Prabha 4. Odisha


Code:
A B C D
(A) 3 4 1 2
(B) 3 1 4 2
(C) 2 1 4 3
(D) 2 4 1 3
267. Which one of the following is not correctly matched?
(A) Rajaji National Park : Elephant
(B) Periyar National Park : Hangul
(C) Manas National Park : Elephant
(D) Dudhwa National Park : Tiger
268. The first National Park established in India is
(A) Namdapha National Park (B) Corbett National Park
(C) Dudhwa National Park (D) Guinady National Park
269. Consider the following pairs: Protected area
1. Bhitarkanika, Odisha : Salt Water Crocodile
2. Desert National Park, : Great Indian Bustard
Rajasthan
3. Eravikulam, Kerala : Hoolak Gibbon
Which of the pair(s) given above is/are correctly matched?
(A) 1 only (B) 1 and 2 only
(C) 2 only (D) 1, 2 and 3
270. Consider the following statements:
1. The boundaries of a National Park are defined by legislation.
2. A Biosphere Reserve is declared to conserve specific species of flora and fauna.
3. In a Wildlife Sanctuary, limited biotic Interfax permitted.
Which of the statement(s) given above is/are correct?
(A) 1 only (B) 2 and 3 only
(C) 1 and 3 only (D) 1, 2 and 3
271. At present, the number of Ramsar site in India is at:
(A) 51 (B) 42
(C) 37 (D) 60
272. Consider the following statements with respect to National Board for Wildlife (NBWL):
1. National Board for Wildlife is a non-statutory body.
2. The Standing Committee of NBWL is chaired by the Prime Minister.
Which of the statement(s) given above is/are not correct?
(A) 1 only (B) 2 only
(C) Both 1 and 2 (D) Neither 1 nor 2
273. Consider the following statements:
1. No alteration of boundaries of a Sanctuary/National Park can be made without the
recommendations of the National Board for Wildlife.
2. The inclusion of territorial waters in areas to be declared as sanctuaries is provided by the
Wildlife Protection Act, 1991.
Which of the statement(s) given above is/are correct?
(A) 1 only (B) 2 only
(C) Both 1 and 2 (D) Neither 1 nor 2
274. National Parks are declared by State governments under the Wildlife Protection Act, 1972 for the
protection and conservation of outstanding natural fauna, flora, geological formations and natural
scenic. What are the activities prohibited under it?
1. Hunting or capturing fauna
2. Deprivation of any wildlife of its habitat
3. Use of weapons
4. Grazing of any livestock
5. Collection of flora
Choose the correct answer using the code given below:
(A) 1 and 5 only (B) 1, 2 and 4 only
(C) 2 and 5 only (D) All of the above
275. Consider the following statements:
1. Protected areas include wildlife sanctuaries and national parks only.
2. Conservation Reserves were first introduced in the Wildlife (Protection) Amendment Act of
2002.
Which of the statement(s) given above is/are correct?
(A) 1 only (B) 2 only
(C) Both 1 and 2 (D) Neither 1 nor 2
276. Consider the following statements about Sacred Groves:
1. The Garo and Khasi tribes of north eastern India prohibit any human interference in the
sacred groves.
2. Sacred groves are not legally protected in India.
Which of the statement(s) given above is/are correct?
(A) 1 only (B) 2 only
(C) Both 1 and (D) Neither 1 nor 2
277. With reference to the Man and the Biosphere (MAB) Programme, consider the following statements:
1. MAB Programme aims to establish a scientific basis for enhancing the relationship
between people and their environments.
2. MAB programme focuses on sites internationally recognized within the World Network of
Biosphere Reserves.
Which of the statement(s) given above is/are correct?
(A) 1 only (B) 2 only
(C) Both 1 and 2 (D) Neither 1 nor 2
278. Which among the following protected area is considered to be the “learning place for sustainable
development”?
(A) Community reserves (B) Conservation reserves
(C) Wildlife sanctuaries (D) Biosphere reserves
279. Which among the following statement(s) is/are the criteria/criterion to qualify as a biodiversity
hotspot?
1. The area should support at least 1500 vascular plants as endemic species.
2. The area must not have lost more than 70% of its native vegetation.
Choose the correct answer using the code given below:
(A) 1 only (B) 2 only
(C) Both 1 and 2 (D) Neither 1 nor 2
280. Consider the following statements:
1. It is the largest lagoon in India.
2. The first site from Asia to be removed from the Montreux Record.
3. It is a wetland of International importance.
4. It is a home to highly endangered Irrawaddy Dolphin.
Which of the following lake has been referred as the statements given above?
(A) Kolleru lake (B) Pulicat lake
(C) Chilika lake (D) Vembanad lake
281. Which of the following Tiger Reserve(s) is/are located in Karnataka?
1. Dandeli-Anshi
2. Nagarahole
3. Bhadra
4. Kalakad-Mundanthurai
5. Amrabad
Choose the correct answer using the code given below:
(A) 1 and 5 only (B) 1, 2 and 3 only
(C) All of the above (D) None of the above
282. Which of the following Tiger Reserve(s) is/are situated in Eastern India?
1. Pakke Tiger Reserve
2. Nameri Tiger Reserve
3. Buxa Tiger Reserve
4. Nawegaon-Nagzira Tiger Reserve
Select the correct answer using the code given below:
(A) 1 and 4 only (B) 1, 3 and 4 only
(C) 1, 2 and 3 only (D) 4 only
283. With reference to Lion Census in India, consider the following statements:
1. Poonam Avlokan is a new observational exercise performed bi-annually to estimate lion
count.
2. The regular Lion Census is conducted once every five years.
3. The Gir Forests is the only natural habitat of Asiatic lions.
Which of the statement(s) given above is/are correct?
(A) 1 and 3 only (B) 2 and 3 only
(C) 2 only (D) 1, 2 and 3
284. In which of the following states is Loktak Lake situated?
(A) Sikkim (B) Himachal Pradesh
(C) Manipur (D) Meghalaya
285. Which of the following protected areas of Uttar Pradesh is associated with the Census of Otters?
(A) Pilibhit Tiger Reserve (B) Kishanpur Wildlife Sanctuary
(C) Dudhwa National Park (D) Katarniaghat Wildlife Sanctuary
286. With reference to Kaziranga National Park, consider the following statements:
1. Much of the population of Black Rhino is found in Kaziranga.
2. River Diphlu passes through it.
Which of the statement(s) given above is/are not correct?
(A) 1 only (B) 2 only
(C) Both 1 and 2 (D) Neither 1 nor 2
287. Which of the following Wildlife Sanctuary is/are located in Arunachal Pradesh?
1. Pobitora Wildlife Sanctuary
2. Gumti Wildlife Sanctuary
3. Talley Wildlife Sanctuary
4. Rowa Wildlife Sanctuary
Select the correct answer using the code given below:
(A) 1, 2 and 3 only (B) 3 and 4 only
(C) 3 only (D) All of the above
288. Identify and match the following Biosphere Reserves with their respective states:

List I (Biosphere List II (State)
Reserve)
1. Nokrek A. Andhra Pradesh
2. Similipal B. Meghalaya
3. Seshachalam C. Assam
4. Dibru-Saikhowa D. Odisha
Select the correct answer from the code given below:
(A) 1-B, 2-C, 3-A, 4-D (B) 1-A, 2-D, 3-B, 4-C
(C) 1-A, 2-C, 3-B, 4-D (D) 1-B, 2-D, 3-A, 4-C
289. With respect to National Parks and Wildlife Sanctuaries, consider the following statements:
1. National parks are highly restricted areas in comparison to wildlife sanctuaries.
2. Wildlife sanctuaries cannot be upgraded to National Parks.
3. National Parks have clearly marked boundaries.
Which of the statement(s) given above is/are correct?
(A) 1 only (B) 2 and 3 only
(C) 1 and 3 only (D) 1, 2 and 3
290. With reference to Nilgiri Biosphere Reserve, which of the following statement(s) is/are not correct?
1. It is the first biosphere reserve of India.
2. It is habitat for endangered animal species, the Nilgiri Tahr and the Lion-tailed macaque.
3. It covers Agasthyamalai Hills in Kerala.
Select the correct answer using the code given below:
(A) 1 and 3 only (B) 3 only
(C) 2 and 3 only (D) None of the above
291. Which of the following pair(s) is/are correctly matched?
1. Blackbuck National Park − Gujarat
2. Harike wetland − unjab
3. Bhitarkanika National park − Odisha
Select the correct answer from the code given below:
(A) 1 only (B) 1 and 3 only
(C) 2 and 3 only (D) 1, 2 and 3
292. With respect to Kharai Camels, consider the following statements:
1. Gujarat (Kutch) is the only home to Kharai camels.
2. They have the special ability to survive on both dry land and in the sea.
3. They feed on mangroves-salty marine food.
Which of the statement(s) given above is/are correct?
(A) 1 and 2 only (B) 2 and 3 only
(C) 1 only (D) 1, 2 and 3
293. Which of the following National Parks has been designated as the UNESCO World Heritage Site? (A)
Pin Valley National Park (B) Rajaji National Park
(C) Dachigam National Park (D) Great Himalayan Park (GHNP)
294. Consider the following statements:
1. Asiatic Cheetah is listed as vulnerable in IUCN Red listed species.
2. Nauradehi Sanctuary has been identified as the pioneer site for the re-introduction of
foreign cheetahs on an experimental basis.
3. Asiatic Cheetah was officially declared extinct from India.
Which of the statement(s) given above is/are correct?
(A) 1 and 2 only (B) 2 and 3 only
(C) 3 only (D) 1, 2 and 3
295. Consider the following statements with respect to Critically Endangered Great Indian Bustard:
1. The Great Indian Bustard is included in the Appendix I of the CITES.
2. It has been identified as one of the species for the recovery programme under the
Integrated Development of Wildlife Habitats.
3. Its population is mainly confined to Rajasthan and Gujarat.
Which of the statement(s) given above is/are correct?
(A) 2 only (B) 3 only
(C) 1 and 2 only (D) 1, 2 and 3
296. Consider the following statements related to Conservation of Clouded Leopard:
1. It has been added to India’s Recovery Programme for Critically Endangered Species.
2. Dampa Tiger reserve has one of the highest population densities of Clouded Leopards.
3. Dampa Tiger Reserve is in Manipur.
Which of the statements given above are correct?
(A) 1 and 2 only (B) 2 and 3 onl
y
(C) 1 and 3 only (D) 1, 2 and 3
297. Consider the following statements regarding Wildlife Crime in India:
1. Wildlife crimes cannot be investigated by the Central Bureau of Investigation (CBI).
2. Wildlife Crime Control Bureau (WCCB) took up selected wildlife offences with trans-border
ramifications for investigation.
3. State Forest and Police Departments are the primary enforcement agencies with regards
to wildlife crimes
Which of the statements given above are correct?
(A) 1 and 2 only (B) 2 and 3 only
(C) 1 and 3 only (D) 1, 2 and 3
298. With reference to the Western Hoolock Gibbon, consider the following statements:
1. It is the only species of ape found in India.
2. It is listed as Endangered in the IUCN Red List.
3. Hoollongapar Reserve Forest is the first Protected Area ever named after a primate species.
Which of the statement(s) given above is/are correct?
(A) 1 and 3 only (B) 2 and 3 only
(C) 3 only (D) 1, 2 and 3
299. Which of the following statement(s) is/are not correct regarding the United Nations Convention on
the Conservation of Migratory Species (CMS)?
1. It is under the aegis of the United Nations Environment Programme (UNEP).
2. CMS is exclusively established only for the conservation of the terrestrial migratory species.
3. It comprises migratory species which are in danger of extinction throughout all or a
significant portion of their range.
Select the correct answer using the code given below:
(A) 2 only (B) 1 and 2 only
(C) 2 and 3 only (D) None of the above
300. Consider the following criteria:
1. Important Bird Area
2. Ramsar Bird Area
3. UNESCO World Heritage Site
4. UNESCO World Biosphere Reserve
Which of the options given below fulfills all the criteria?
(A) Nandur Madhmeshwar Wildlife Sanctuary
(B) Sandi Bird Sanctuary
(C) Keoladeo National Park
(D) Sunderban Biosphere Reserve
301. To declare an area a Biosphere Reserve, the area must have:
1. Minimally disturbed core area with unique fauna and flora
2. Typical fauna and flora in the core area
3. Rare and endangered species
Which of the statement (s) given above is/are correct?
(A) 1, 2 and 3 (B) 1 and 3 only
(C) 3 only (D) 1 only
302. Consider the following statements regarding Conservation Reserves:
1. Conservation reserves are designated as the protected areas.
2. These areas can be used for subsistence
Which of the statement(s) given above is/are correct?
(A) 1 only (B) 2 only
(C) Both 1 and 2 (D) Neither 1nor 2
303. With reference to National Parks, consider the following statements:
1. Collection of flora and capturing of fauna are not allowed in National Parks.
2. The International Union for Conservation of Nature (IUCN) places National Parks in the
category II of the protected areas.
Which of the statement(s) given above is/are correct?
(A) 1 only (B) 2 only
(C) Both 1 and 2 (D) Neither 1 nor 2
304. Match List-I with List-II and select the correct answer using the code given below the List:

List-I (Wildlife Sanctuary) List-II (State)


A. Mahananda 1. Madhya Pradesh
B. Lakhari Valley 2. Uttar Pradesh
C. Pachmarhi 3. West Bengal
D. Chandra Prabha 4. Odisha



Code:
A B C D
(A) 3 4 1 2
(B) 3 1 4 2
(C) 2 1 4 3
(D) 2 4 1 3

305. Which set of the following biosphere reserves in India is included in the World Network of Biosphere
Reserves?
(A) Gulf of Mannar, Nokrek, Pachmarhi and Similipal
(B) Gulf of Mannar, Khangchendzonga, Nokrek and Seshachalam
(C) Nilgiri, Nokrek, Manas and Ranthambore
(D) Nilgiri, Nokrek, Pachmarhi and Seshachalam
306. Which of the following pairs is/are not correctly matched?
National Park/Wetland Confluence of Rivers
1. Blackbuck National Park - Parvalia and Alang rivers
2. Harike Wetland - Beas and Sutlej rivers
3. Bhitarkanika National Park - Gambhir and Banganga rivers
Select the correct answer from the codes given below:
(A) 3 only (B) 1 and 2 only
(C) 2 and 3 only (D) 1, 2 and 3
307. Consider the following statements about Bandipur Tiger Reserve:
1. It is located at the tri-junction area of states of Karnataka, Tamil Nadu and Andhra
Pradesh.
2. It is located between river Kabini in the north and Tungabhadra in the south.
3. The Bandipur tiger reserve has the single largest Asiatic Elephant population in the world.
Which of the statement(s) given above is/are correct?
(A) 1 and 2 only (B) 2 only
(C) 3 only (D) 2 and 3 only
308. Consider the following statements with reference to Biosphere Reserves:
1. To be designated as a Biosphere Reserve site must contain an effectively protected and
minimally disturbed core area of value of nature conservation.
2. It focuses on conservation of some specific flagship species.
Which of the statements given above is/are not correct?
(A) 1 only (B) 2 only
(C) Both 1 and 2 (D) Neither 1 nor 2
309. Ghodazari Wildlife Sanctuary is located at:
(A) Uttar Pradesh (B) Himachal Pradesh
(C) Maharasthra (D) Gujarat
310. Consider the following statements about Nanda Devi National Park:
1. It is a UNESCO World Heritage Site.
2. The national park is drained by rivers Bhagirathi and Mandakini.
3. The fauna of the national park includes Snow Leopard and Himalayan Musk Deer.
Which of the statements given above are correct?
(A) 1 and 2 only (B) 2 and 3 only
(C) 1 and 3 only (D) 1, 2 and 3
311. Which of the statement(s) given is/are correct about Periyar Tiger Reserve?
1. it is located in the southern region of Western Ghats.
2. Lion-tailed macaque, Nilgiri Tahr and the great Malabar Hornbill are found in the reserve.
3. The reserve is home to three tribal groups namely Gonds, Marati and Naikd
Select the correct answer using the code given below:
(A) 1 and 2 only (B) 2 only
(C) 3 only (D) 1, 2 and 3
312. Consider the following statements:
1. Barasingha is found exclusively in Kanha Tiger Reserve.
2. Barasingha is the state animal of Madhya Pradesh.
3. Barasingha is listed as Vulnerable in the IUCN Red List.
Which of the statement (s) given above is/are correct?
(A) 1 and 2 only (B) 2 only
(C) 3 only (D) 1, 2 and 3
313. With reference to Thane Creek Flamingo Sanctuary, consider the following statements:
1. The areas have been notified as an Eco Sensitive Zone (ESZ).
2. The Thane Creek area is recognized as an “Important Bird Area”.
Which of the statement(s) given above is/are correct?
(A) 1 only (B) 2 only
(C) Both 1 and 2 (D) Neither 1 nor 2
4. Environmental Pollution
314. Which of the following are the reasons/factors for exposure to benzene pollution?
1. Automobile exhaust
2. Tobacco smoke
3. Wood burning
4. Using varnished wooden furniture
5. Using products made of polyurethane
Select the correct answer using the code given below:
(A) 1, 2 and 3 only (B) 2 and 4 only
(C) 1, 3 and 4 only (D) 1, 2, 3, 4 and 5
315. Consider the following statements:
1. Coal ash contains arsenic, lead and mercury.
2. Coal-fired power plants release sulphur dioxide and oxides of nitrogen into the environment.
3. High ash content is observed in Indian coal.
Which of the statement(s) given above is/are correct?
(A) 1 only (B) 2 and 3 only
(C) 3 only (D) 1, 2 and 3
316. With reference to Eutrophication, which of the following statement(s) is/are correct?
1. It decreases dissolved oxygen of water.
2. It is a phenomenon of nutrient enrichment of a water body.
Select the correct answer from the codes given below.
Cades:
(A) 1 only (B) 2 only
(C) Both 1 and 2 (D) Neither 1 nor 2
317. Why is there a great concern about the microbeads that are released into environment?
(A) They are considered harmful to marine ecosystems.
(B) They are considered to cause skin cancer in children.
(C) They are small enough to be absorbed by crop plants in irrigated fields.
(D) They are often found to be used as food adulterants.
318. Consider the following:
1. Carbon monoxide 2. Methane
3. Ozone 4. Sulphur dioxide
Which of the above are released into atmosphere due to the burning of crop/biomass residue?
(A) 1 and 2 only (B) 2, 3 and 4 only
(C) 1 and 4 only (D) 1, 2, 3 and 4
319. Which of the following statements is/are correct about the deposits of ‘methanehydrate’?
1. Global warming might trigger the release of methane gas from these deposits.
2. Large deposits of ‘methanehydrate’ are found in Arctic Tundra and under the sea floor.
3. Methane in atmosphere oxidizes to carbon dioxide after a decade or two.
Select the correct answer using the code given below.
(A) 1 and 2 only (B) 2 and 3 only
(C) 1 and 3 only (D) 1, 2 and 3
320. Consider the following statements:
1. Agricultural soils release nitrogen oxides into environment.
2. Cattle release ammonia into environment.
3. Poultry industry releases reactive nitrogen compounds into environment.
Which of the statement(s) given above is/are correct?
(A) 1 and 3 only (B) 2 and 3 only
(C) 2 only (D) 1, 2 and 3
321. The National Clean Air Programme (NCAP) aims to reduce particulate matter (PM) in at least 102
cities of India by 20% - 30% by 2024. The base year for the comparison of concentration is:
(A) 2015 (B) 2016
(C) 2017 (D) 2018
322. Catalytic converter transforms waste gases from the engines of many cars into carbon dioxide,
nitrogen and water. The catalyst is made of:
(A) platinum and copper (B) molybdenum and copper
(C) platinum and rhodium (D) rhodium and molybdenum
323. Venturi scrubber, a device used to remove particulate matter from the atmosphere, works on the
principle of:
(A) Settling by gravitational force
(B) Removal by centrifugal force
(C) Removal by electrically charged particles
(D) Removal by atomized water vapour
324. Acid rain results when gaseous emissions of Sulfur oxides (SO×) and nitrogen oxides (NO×) interact
with water vapour and:
(A) Moonlight, and are chemically converted to strong acidic compounds such as sulfuric acid
(H2SO4) and nitric acid (HNO3)
(B) Sunlight, and are chemically converted to strong acidic compounds such as sulfuric acid
(H2SO4) and nitric acid (HNO3)
(C) Moonlight, and are chemically converted to weak acidic compounds such as sulfuric acid
(H2SO4) and nitric acid (HNO3)
(D) Sunlight, and are chemically converted to weak acidic compounds such as sulfuric acid
(H2SO4) and nitric acid (HNO3)
325. Which of the following is the well known example of bioindicator of air pollution?
(A) Lichens (B) Methyl Mercury
(C) Rose Plant (D) Sunflower
326. Smog is essentially caused by the atmospheric presence of
(A) Oxygen and ozone (B) Ozone and nitrogen
(C) Oxygen and nitrogen (D) Oxide of nitrogen and sulphur
327. The gas, which is emitted in the paddy fields and increases the earth’s temperature is
(A) Nitrogen (B) Carbon dioxide
(C) Carbon monoxide (D) Methane
328. Which of the following statements about India’s largest Charkha is/are true?
1. It was inaugurated in Noida.
2. It is made of used plastic waste.
Select the correct answer from the codes given below:
Codes:
(A) Only 1 (B) Only 2
(C) Both 1 and 2 (D) Neither 1 nor 2
329. Which of the following has/have shrunk immensely/dried up in the recent past due to human
activities?
1. Aral Sea 2. BlackSea
3. Lake Baikal
Select the correct answer using the code given below:
(A) 1 only (B) 2 and 3
(C) 2 only (D) 1 and 3
330. The term “sixth mass extinction/sixth extinction” is often mentioned in the news in the context of the
discussion of
(A) Widespread monoculture practices in agriculture and large-scale commercial farming with
indiscriminate use of chemicals in many parts of the world that may result in the loss of good
native ecosystems.
(B) Fears of a possible collision of a meteorite with the Earth in the near future in the manner
it happened 65 million years ago that caused the mass extinction of many species including
those of dinosaurs.
(C) Large scale cultivation of genetically modified crops in many parts of the world and
promoting their cultivation in other parts of the world which may cause the disappearance of g o o d
native crop plant sand the loss of food biodiversity.
(D) Mankind’s over-exploitation/misuse of natural resources, fragmentation/loss of natural
habitats, destruction of ecosystems, pollution and global climate change.
331. Which of the following is/are the possible consequence/s of heavy sand mining in river beds?
1. Decreased salinity in the river
2. Pollution of groundwater
3. Lowering of the water-table
Select the correct answer using the code given below.
(A) 1 only (B) 2 and 3 only
(C) 1 and 3 only (D) 1, 2 and 3
332. With reference to agricultural soils, consider the following statements:
1. A high content of organic matter in soil drastically reduce sits water holding capacity.
2. Soil does not play any role in the Sulphur cycle.
3. Irrigation over a period of time can contribute to the salinization of some agricultural lands.
Which of the statement(s) given above is/are correct?
(A) 1 and 2 only (B) 3 only
(C) 1 and 3 only (D) 1, 2 and 3
333. The meaning of ‘Carbon Footprint’ is described by the amount of:
(A) Carbon dioxide released into the atmosphere as a result of the activities of a particular
individual, organization or community.
(B) Greenhouse gases emitted by industries contributing to global warming.
(C) Carbon emissions released by the burning of jet fuel.
(D) Increase in the carbon content of the atmosphere due to the felling of trees.
334. Increased biological oxygen demand is an indication of:
1. Low microbial contamination.
2. Absence of microbial pollution.
3. High level of microbial contamination.
Which of the above statement(s) is/are correct?
(A) 1 only (B) 2 only
(C) 3 only (D) 1, 2 and 3
335. Consider the following statements regarding depletion of the ozone layer:
1. Excessive release of chlorine and bromine in the environment from man-made
compounds, such as chlorofluorocarbons.
2. Occurrence of certain natural phenomena such as sunspots, and stratospheric winds.
3. Degradation of materials by ultraviolet radiation.
4. Major volcanic eruptions.
Which of the above can be categorized as causing ozone depletion?
(A) 1, 2 and 3 only (B) 1, 3 and 4 only
(C) 1, 2 and 4 only (D) 2, 3 and 4 only
336. Directions: The following item consist of two statements one labelled as ‘Statement (I)’ and the other
as ‘Statement (II)’. Examine these two statements carefully and select the answers to these items
using the codes given below:
(A) Both Statement (I) and Statement (II) are individually true, and Statement (II) is the correct
explanation of Statement (I)
(B) Both Statement (I) and Statement (II) are individually true, but Statement (II) is not the
correct explanation of Statement (I)
(C) Statement (I) is true, but Statement (II) is false
(D) Statement (I) is false, but Statement (II) is true
Statement (I): Normally carbon dioxide is not considered an air pollutant.
Statement (II): Carbon dioxide is a constituent of atmospheric air.
337. Sanitary/municipal fills and waste heaps are unavoidably hazardous due to
1. Leachates
2. Emanating gases
3. Rodents and wandering animals
4. Automobile workshops that seem to have an affinity for such neighbourhoods
(A) 1 and 4 only (B) 1 and 2 only
(C) 2 and 3 only (D) 3 and 4 only
338. What is ‘Green House Effect’?
(A) Increase in global temperature
(B) Decrease in global temperature
(C) Increase in sea water temperature
(D) Increase in temperature of rivers and lakes
339. A powerful eye irritant in the smoke is
(A) Ozone (B) Sulphur dioxide
(C) Carbon dioxide (D) Peroxyacetyle nitrate
340. Which among the following is the secondary pollutant?
(A) Smog (B) Carbon dioxide
(C) Carbon monoxide (D) Fly ash
341. Ultraviolet radiation from sunlight causes the reaction that produces which of the following?
(A) CO (B) SO2
(C) O3 (D) Fluorides
342. Incinerators are used for which of the following?
(A) Burning wastes (B) Putting wastes into them
(C) For cutting green trees (D) For making fertilizers
343. Methane is emitted from which of the following?
(A) Paddy fields only (B) Termitary only
(C) Both from (A) and (B) (D) None of the above
344. Which pain killer given to cattle is responsible for near extinction of vultures in India?
(A) Ibuprofen (B) Acetaminophen
(C) Asprin (D) Diclofenac
345. Which of the following rivers of Uttar Pradesh has been declared a ‘Biological Disaster’ due to
environmental pollution?
(A) Yamuna (B) Gomti
(C) Sai (D) Tamsa
346. Which among the following is the secondary pollutant?
(A) Smog (B) Carbon dioxide
(C) Carbon mono-oxide (D) Fly ash
347. In the context of mitigating the impending global warming due to anthropogenic emissions of carbon
dioxide, which of the following can be the potential sites for carbon sequestration?
1. Abandoned and uneconomic coal seams
2. Depleted oil and gas reservoirs
3. Subterranean deep saline formations
Select the correct answer using the code given below:
(A) 1 and 2 only (B) 3 only
(C) 1 and 3 only (D) 1, 2 and 3
348. Biological Oxygen Demand (BOD) is a standard criterion for
(A) Measuring oxygen levels in blood
(B) Computing oxygen levels in forest ecosystems
(C) Pollution assay in aquatic ecosystems
(D) Assessing oxygen levels in high altitude regions
349. In the context of solving pollution problems, what is/ are the advantage/advantages of bioremediation
technique?
1. It is a technique for cleaning up pollution by enhancing the same biodegradation process t
at occurs in nature.
2. Any contaminant with heavy metals such as cadmium and lead can be readily and
completely treated bioremediation using microorganisms.
3. Genetic engineering can be used to create microorganisms specifically designed or
bioremediation.
Select the correct answer using the code given below:
(A) 1 only (B) 2 and 3 only
(C) 1 and 3 only (D) 1, 2 and 3
350. Which one of the following statements about air pollution caused by diesel engines is correct?
(A) It produces large quantity of carbon monoxide at lower and high temperatures.
(B) It produces large quantity of nitrogen oxides at lower and high temperatures.
(C) It produces large quantity of carbon monoxide at lower temperature and nitrogen oxides
at high temperature.
(D) It produces large quantity of nitrogen oxides at lower temperature and carbon monoxide
at high temperature.
351. Which of the following diseases are caused by the consumption of water contaminated by mercury
and nitrate?
(A) Minamata disease and Osteoporosis
(B) Osteoporosis and Blue Baby Syndrome
(C) Minamata disease and Blue Baby Syndrome
(D) Osteoporosis and Minamata disease
352. Leakage of which one of the following gases had caused Bhopal Gas Tragedy in the year 1984?
(A) Methyl isocyanate
(B) Hexamethylene diisocyanate
(C) Isophorone diisocyanate
(D) Isothiocyanate
353. Which one of the following is a major effect of long-term consumption of drinking water containing
little (less than 0.5 ppm) or no fluoride?
(A) cavity of tooth (B) Erosion of a nail
(C) Deformation of bone (D) Mottling of tooth
354. Direction: The following item consists of two statements, Statement I and Statement II. Examine these
two statements carefully and select the correct answer using the code given below:
Code:
(A) Both the statements are individually true and I Statement II is the correct explanation of
Statement I
(B) Both the statements are individually true but Statement II is not the correct explanation of
Statement I
(C) Statement I is true but Statement II is false
(D) Statement I is false but Statement II is true
Statement I: Global warming signifies the rise in global surface temperature.
Statement II: The increase of concentration of greenhouse gases in the atmosphere causes the rise in
global surface temperature
355. One carbon credit is accepted as equivalent to:
(A) 100 kg of carbon (B) 100 kg of carbon dioxide
(C) 1000 kg of carbon (D) 1000 kg of carbon dioxide
356. Which one of the following is not included in the National Air Quality Index?
(A) Sulphur (B) Nitrogen dioxide
(C) Lead (D) Methane
357. Which of the following statements about the Paris Agreement on Climate Change is/are correct?
1. The Agreement recognizes the developing countries’ right to development and their efforts
to harmonize development with the environment, while protecting the interests of the most
vulnerable.
2. The objective of the Agreement ensures that it is mitigation-centric.
Select the correct answer using the code given below:
(A) 1 only (B) 2 only
(C) Both 1 and 2 (D) Neither 1 nor 2
358. The LEED and GRIHA:
1. Are green building rating systems.
2. Issue energy compliance certificate.
3. Refer to Global standards.
4. Are Indian standards under finalization.
Select the correct answer using the codes given below:
(A) 1 and 2 (B) 2 and 3
(C) 3 and 4 (D) 1 and 4
359. The UN Conference on Environment and Development, known as Rio Summit, led to the final
development of which international treaty?
(A) Kyoto Protocol
(B) World Environment Treaty
(C) Environment Treaty of the G8 Nations
(D) Universal Environment Treaty
360. Consider the following statements regarding SMOG:
1. SMOG was coined during the 1950’s to describe a mixture of smoke and fog experienced
in London.
2. The principal pollutants in London SMOG are particulates and sulphur compounds.
3. The London SMOG occurs generally early in the morning on cold wet winter days.
Which of the above statements are correct?
(A) 1, 2 and 3 (B) 1 and 2 only
(C) 1 and 3 only (D) 2 and 3 only
361. Consider the following tertiary treatment methods for treatment of wastewater:
1. Ion-exchange method
2. Reverse osmosis
3. Chemical oxidation method
4. Activated sludge process
Which of the above methods are correct?
(A) 1, 2 and 4 (B) 1, 3 and 4
(C) 2, 3 and 4 (D) 1, 2 and 3
362. Which industrial waste is commonly used in the construction industry?
(A) Fly ash (B) Slag
(C) Sludge (D) Red oxide
363. The Rotterdam Convention deals with:
(A) Reducing nuclear weapon stockpiles (B) Limiting the use of toxic chemicals
(C) Protecting the oceans (D) Banning of human clone
experiment
364. Consider the following statements concerning environmental pollution:
1. Nuclear explosions cause radioactive radiation
2. Earthquakes do not cause Tsunamis.
3. Acid rain is not a major environmental issue.
4. Air pollution has some impact on meteorology.
Which of the above statements are correct?
(A) 1 and 2 (B) 2 and 3
(C) 3 and 4 (D) 1 and 4
365. Which one of the following is not correctly matched?
Green House Gas − Source
(A) Carbon dioxide − Thermal Power Stations
(B) Chloro fluoro carbon − Automobile
(C) Nitrous Oxide − Water logged Paddy fields
(D) Sulphur dioxide − Brick Kilns
366. Which of the following fuels would produce minimum environmental pollution?
(A) Hydrogen (B) Diesel
(C) Kerosene (D) Coal
367. Which one of the following is a cause of acid rains?
(A) Ozone (B) Ammonia
(C) Sulphur dioxide (D) Carbon monoxide
368. On which of the following can you find the Bureau of Energy Efficiency Star Label?
1. Ceiling fans 2. Electric geysers
3. Tubular fluorescent lamps
Select the correct answer using the code given below:
(A) 1 and 2 only (B) 3 only
(C) 2 and 3 only (D) 1, 2 and 3
369. In the cities of our country, which among the following atmospheric gases are normally considered in
calculating the value of Air Quality Index?
1. Carbon dioxide 2. Carbon monoxide
3. Nitrogen dioxide 4. Sulfur dioxide
5. Methane
Select the correct answer using the code given below:
(A) 1, 2 and 3 only (B) 2, 3 and 4 only
(C) 1, 4 and 5 only (D) 1, 2, 3, 4 and 5
370. Which of the following are the key features of ‘National Ganga River Basin Authority (NGRBA)’?
1. River basin is the unit of planning and management.
2. It spear heads the river conservation efforts at the national level.
3. One of the Chief Ministers of the States through which the Ganga flows becomes the
Chairman of NGRBA on rotation basis.
Select the correct answer using the code given below:
(A) 1 and 2 only (B) 2 and 3 only
(C) 1 and 3 only (D) 1, 2 and 3
371. Which of the following best describes/describe the aim of ‘Green India Mission’ of the Government
of India?
1. Incorporating environmental benefits and costs into the Union and State Budgets thereby
implementing the ‘green accounting’.
2. Launching the second green revolution to enhance agricultural output so as to ensure food
security to one and all in the future.
3. Restoring and enhancing forest cover and responding to climate change by acombination
of adaptation and mitigation measures.
Select the correct answer using the code given below.
(A) 1 only (B) 2 and 3 only
(C) 3 only (D) 1, 2 and 3
372. The process of using microbes to treat areas of land or sea that have been contaminated by pesticides,
oil or solvents is known as:
(A) Eutrophication (B) Nitrification
(C) Ammonification (D) Bioremediation
373. Which of the following statement(s) is/are correct?
1. Acid rain reacts with buildings made from limestone
2. Burning of sulphur containing coal can contribute to acid rain
3. Eutrophication is an effective measure to control pollution
Select the correct answer using the code given below:
(A) 1 and 2 only (B) 2 and 3 only
(C) 1 only (D) 1, 2 and 3
374. Which one of the following is most sensitive to environmental change?
(A) Amphibian (B) Reptile
(C) Mammal (D) Insect
375. Which one of the following is associated with the issue of control and phase out of the use of ozone
depleting substances?
(A) Bretton Woods Conference (B) Doha Development Agenda
(C) Geneva II Conference (D) Montreal Protocol
376. The term ‘Bali Action Plan’, is sometimes seen in news in the context of:
(A) Global climate change (B) Global terrorism
(C) Global wildlife trafficking (D) World track
377. Fly ash’, which can be used in making bricks, is emitted from:
(A) Burning of agricultural waste. (B) Burning of municipal solid waste.
(C) Mining of aluminum and iron ores. (D) Thermal power plants.
378. Indiscriminate use of fertilizers have led to
(A) soil pollution (B) water pollution
(C) air pollution (D) All of the above
379. Which one of the following sectors is the largest contributor to carbon dioxide emissions from fuel
consumption in India?
(A) Electricity and heat production
(B) Transport
(C) Manufacturing industries and construction
(D) Others
380. With reference to ‘fly ash’ produced by the power plants using coal as fuel, which of the following
statements is/ are correct?
1. Fly ash can be used in the production of bricks for building construction.
2. Fly ash can be used as a replacement for some of the Portland cement contents of concrete.
3. Fly ash is made up of silicon dioxide and calcium oxide only, and does not contain any
toxic elements.
Select the correct answer using the code given below:
(A) 1 and 2 (B) 2 only
(C) 1 and 3 (D) 3 only
381. What can be the impact of excessive/in appropriate use of nitrogenous fertilizers in agriculture?
1. Proliferation of nitrogen-fixing micro organisms in soil can occur.
2. Increase in the acidity of soil can take place.
3. Leaching of nitrate to the groundwater can occur.
Select the correct answer using the code given below:
(A) 1 and 3 only (B) 2 only
(C) 2 and 3 only (D) 1, 2 and 3
382. Which one of the following is the most appropriate and correct practice from the point of view of a
healthy environment?
(A) Burning of plastic wastes to keep the environment clean
(B) Burning of dry and fallen leaves in a garden or field
(C) Treatment of domestic sewage before its release
(D) Use of chemical fertilizers in agricultural fields
383. Match List I with List II and select the correct answer using the code given below the Lists:

List I List II
A. Chlorofluorocarbon 1. Acid Rain
B. Sulphur dioxide 2. Depletion in Ozone layer in
Atmosphere
C. Lead Compound 3. Harmful for human nervous system
D. Carbon Dioxide 4. Topmost Contribution to greenhouse
gas effect
Code:
A B C D
(A) 4 3 1 2
(B) 4 1 3 2
(C) 2 1 3 4
(D) 2 3 1 4
384. Contribution of the manufacturing sector to environmental degradation primarily occurs during
1. Procurement and use of natural resources
2. Industrial processes and activities
3. Product use and disposal
Select the correct answer using the code given below.
(A) 1 only (B) 2 and 3 only
(C) 1 and 2 only (D) 1, 2 and 3
385. Which of the following is not a gaseous air pollutant?
(A) Oxides of sulphur (B) Oxides of Nitrogen
(C) Hydrocarbon (D) Smoke
386. Mercury is emitted as a pollutant by
(A) Coal based power plants (B) Cold storage facilities
(C) Sugar, paper and jute mills (D) Wineries
387. Consider the following:
1. Solid waste treatment
2. Natural wetlands
3. Ruminant animals
Which of the above can cause methane emissions and thus can contribute to global warming?
(A) 1 only (B) 2 and 3 only
(C) 1 and 2 only (D) 1, 2 and 3
388. In the Earth’s atmosphere, which of the following have the property of absorbing the heat and
contribute to the warming of atmosphere?
1. Carbon monoxide 2. Oxygen
3. Soot 4. Water vapour
Select the correct answer using the code given below:
(A) 1 and 2 only (B) 1, 3 and 4 only
(C) 3 and 4 only (D) 1, 2, 3 and 4
389. With reference to refrigerants, consider the following statements:
1. Hydrofluorocarbons can be the alternatives to ozonedestroying chlorofluorocarbons in
refrigeration.
2. Hydrofluorocarbons do not have any global warming potential.
Which of the statement(s) given above is/are correct?
(A) 1 only (B) 2 only
(C) Both 1 and 2 (D) Neither 1 nor 2
390. The main pollutant in the smoke coming from cigarette is
(A) Carbon monoxide & Dioxicine (B) Carbon monoxide and Nicotine
(C) Carbon monoxide and Benzene (D) Dioxicine and Benzene
391. The problem of water pollution with Arsenic is maximum in
(A) Haryana (B) Rajasthan
(C) Madhya Pradesh (D) West Bengal
392. Which one among the following books is centred around ‘environment’?
(A) The Late, Great Planet Earth (B) Silent Spring
(C) Here I Stand (D) And Then One Day
393. The scientific view is that the increase in global temperature should not exceed 2°C above pre-
industrial level. If the global temperature increases beyond 3°C above the pre-industrial level, what
can be its possible impact/impacts on the world?
1. Terrestrial biosphere tends toward a net carbon source.
2. Widespread coral mortality will occur.
3. All the global wetlands will permanently disappear.
4. Cultivation of cereals will not be possible anywhere in the world.
Select the correct answer using the code given below:
(A) 1 only (B) 1 and 2 only
(C) 2, 3 and 4 only (D) 1, 2, 3 and 4
394. There is some concern regarding the nanoparticles of some chemical elements that are used by the
industry in the manufacture of various products. Why?
1. They can accumulate in the environment, and contaminate water and soil.
2. They can enter the foodchains.
3. They can trigger the production of free radicals. Select the correct answer using the code
given below:
(A) 1 and 2 only (B) 3 only
(C) 1 and 3 only (D) 1, 2 and 3
395. Which of the following are some important pollutants released by steel industry in India?
1. Oxides of sulphur 2. Oxides of nitrogen
3. Carbon monoxide 4. Carbon dioxide
Select the correct answer using the code given below:
(A) 1, 3 and 4 only (B) 2 and 3 only
(C) 1 and 4 only (D) 1, 2, 3 and 4
396. Brominated flame retardants are used in many household products like mattresses and upholstery.
Why is there some concern about their use?
1. They are highly resistant to degradation in the environment.
2. They are able to accumulate in humans and animals.
Select the correct answer using the code given below:
(A) 1 only (B) 2 only
(C) Both 1 and 2 (D) Neither 1 nor 2
397. Arrange the following fuels in the decreasing order of air pollution caused by burning a kilogram of
each of them:
(A) CNG, Petrol, Diesel (B) Diesel, Petrol, CNG
(C) Petrol, Diesel, CNG (D) Diesel, CNG, Petrol
398. Pesticide Treadmill refers to:
(A) Increase in concentration of pesticides in the body of the organism
(B) Increase in doses of pesticides to prevent the resurgence of pests that were being controlled
(C) Development of resistance in plant due to excessive and prolonged use of pesticide
(D) Production of more synthetic pesticides
399. Which of the following ions present in low concentration in drinking water is essential for normal
growth of teeth but harmful to teeth in high concentration?
(A) Aluminium (B) Calcium
(C) Fluoride (D) Chloride
400. Which of the following lamps contains a poisonous gas and therefore should be disposed of safely?
(A) Compact fluorescent lamp (B) Light emitting diode
(C) Neon lamp (D) Halogen Lamp
401. Addition of Ethylene Dibromide to petrol
(A) Increases the octane level of fuel
(B) Helps elimination of lead oxide
(C) Removes the sulphur compound in petrol
(D) Serves as a substitute of tetraethyl lead
402. Which method of water purification does not kill microorganisms?
(A) Boiling (B) Filtration
(C) Chlorination (D) UV-irradiation
403. Nitric Oxide pollution can lead to all of the following except
(A) Leaf spotting in plants
(B) Bronchitis-related respiratory problems in humans
(C) Production of corrosive gases through photochemical reaction
(D) Silicosis in human
404. The gas which is released from landfills and dumpsites in urban areas is:
(A) Oxygen (B) Methane
(C) Nitrogen (D) Carbon dioxide
405. Which of the following are sources of methane, a greenhouse gas?
1. Domestic animals 2. Coal mining
3. Wetlands
Select the correct answer using the code given below:
(A) 1 and 2 only (B) 2 only
(C) 1 and 3 only (D) 1, 2 and 3
406. ‘Green Mufler’ is related to
(A) Soil pollution (B) Air pollution
(C) Noise pollution (D) Water pollution
407. Biodegradable pollutant is
(A) Mercury (B) Sewage
(C) Plastic (D) Asbestos
408. Which one of the following is not a secondary pollutant?
(A) PAN (B) Smog
(C) Sulphur dioxide (D) Ozone
409. World Ozone day is celebrated on
(A) September, 16 (B) April, 21
(C) December, 25 (D) January, 30
410. Which of the following groups of plants can be used as indicators of SO2 pollution of air?
(A) Ferns (B) Mentha
(C) Lichens (D) Hornworts
411. Which one among the following industries is the maximum consumer of water in India?
(A) Engineering (B) Paper and pulp
(C) Textiles (D) Thermal power
412. Due to improper/indiscriminate disposal of old and used computers or their parts, which of the
following are released into the environment as e-waste?
1. Beryllium 2. Cadmium 3 .
Chromium
4. Heptachlor 5. Mercury 6. Lead

7. Plutonium
Select the correct answer using the codes given below:
(A) 1, 3, 4, 6 and 7 only (B) 1, 2, 3, 5 and 6 only
(C) 2, 4, 5 and 7 only (D) 1, 2, 3, 4, 5, 6 and 7
413. Which of the following can be found as pollutants in the drinking water in some parts of India?
1. Arsenic 2. Sorbitol
3. Fluoride 4. Formaldehyde
5. Uranium
Select the correct answer using the codes given below:
(A) 1 and 3 only (B) 2, 4 and 5 only
(C) 1, 3 and 5 only (D) 1, 2, 3, 4 and 5
414. Acid rain is caused by the pollution of environment by
(A) carbon dioxide and nitrogen (B) carbon monoxide and carbon dioxide
(C) ozone and carbon dioxide (D) nitrous oxide and sulphur
dioxide
415. The increasing amount of carbon dioxide in the air is slowly raising the temperature of the atmosphere,
because it absorbs
(A) the water vapour of the air and retains its heat
(B) the ultraviolet part of the solar radiation
(C) all the solar radiations
(D) the infrared part of the solar radiation
416. Consider the following agricultural practices:
1. Contour bunding
2. Relay cropping
3. Zero tillage
In the context of global climate change, which of the above helps/help in carbon sequestration/
storage in the soil?
(A) 1 and 2 only (B) 3 only
(C) 1, 2 and 3 (D) None of them
417. The acidification of oceans is increasing. Why is this phenomenon a cause of concern?
1. The growth and survival of calcareous phytoplankton will be adversely affected.
2. The growth and survival of coral reefs will be adversely affected.
3. The survival of some animals that have phytoplanktonic larvae will be adversely affected.
4. The cloud seeding and formation of clouds will be adversely affected.
Which of the statements given above is/are correct?
(A) 1, 2 and 3 only (B) 2 only
(C) 1 and 3 only (D) 1, 2, 3 and 4
418. Consider the following statements:
Chlorofluorocarbons, known as ozone-depleting substances, are used
1. in the production of plastic foams
2. in the production of tube less tyres
3. in cleaning certain electronic components
4. as pressurizing agents in aerosol cans
Which of the statement(s) given above is/are correct?
(A) 1, 2 and 3 only (B) 4 only
(C) 1, 3 and 4 only (D) 1, 2, 3 and 4
419. Lead, ingested or inhaled, is a health hazard. After the addition of lead to petrol has been banned,
what still are the sources of lead poisoning?
1. Smelting units
2. Pens and pencils
3. Paints
4. Hair oils and cosmetics
Select the correct answer using the codes given below:
(A) 1, 2 and 3 only (B) 1 and 3 only
(C) 2 and 4 only (D) 1, 2, 3 and 4
420. There is a concern over the increase in harmful algal blooms in the sea waters of India. What could be
the causative factors for this phenomenon?
1. Discharge of nutrients from the estuaries.
2. Run-off from the land during the monsoon.
3. Upwelling in the seas.
Select the correct answer from the codes given below:
(A) 1 only (B) 1 and 2 only
(C) 2 and 3 only (D) 1, 2 and 3
421. Human activities in the recent past have caused the increased concentration of carbon dioxide in the
atmosphere, but a lot of it does not remain in the lower atmosphere because of
1. its escape into the outer stratosphere.
2. the photosynthesis by phytoplankton in the oceans.
3. the trapping of air in the polar ice caps.
Which of the statements given above is/are correct?
(A) 1 and 2 (B) 2 only
(C) 2 and 3 (D) 3 only
422. Consider the following:
1. Carbon dioxide 2. Oxides of Nitrogen
3. Oxides of Sulphur
Which of the above is/are the emission/emissions from coal combustion at thermal power plants?
(A) 1 only (B) 2 and 3 only
(C) 1 and 3 only (D) 1, 2 and 3
423. In the Union Budget 2011-12, a full exemption from the basic customs duty was extended to the bio-
based asphalt (bioasphalt). What is the importance of this material?
1. Unlike traditional asphalt, bio-asphalt is not based on fossil fuels.
2. Bioasphalt can be made from non-renewable resources.
3. Bioasphalt can be made from organic waste materials.
4. It is eco-friendly to use bioasphalt for surfacing of the road
Which of the statements given above are correct?
(A) 1, 2 and 3 only (B) 1, 3 and 4 only
(C) 2 and 4 only (D) 1, 2, 3 and 4
424. What are the possible limitations of India in mitigating the global warming at present and in the
immediate future?
1. Appropriate alternate technologies are not sufficiently available.
2. India cannot invest huge funds in research and development.
3. Many developed countries have already set up their polluting industries in India.
Which of the statements given above is/are correct?
(A) 1 and 2 only (B) 2 only
(C) 1 and 3 only (D) 1, 2 and 3
425. Excessive release of the pollutant carbon monoxide (CO) into the air may produce a condition in
which oxygen supply in the human body decreases. What causes this condition?
(A) When inhaled into the human body, CO is converted into CO2
(B) The inhaled CO has much higher affinity for haemoglobin as compared to oxygen
(C) The inhaled CO destroys the chemical structure of haemoglobin
(D) The inhaled CO adversely affects the respiratory centre in the brain
426. Consider the following which can be found in the ambient atmosphere:
1. Soot
2. Sulphur hexafluoride
3. Water vapour
Which of the above contribute to the warming up of the atmosphere?
(A) 1 and 2 only (B) 3 only
(C) 2 and 3 only (D) 1, 2 and 3
427. Due to the irextensive rice cultivation, some regions may be contributing to global warming. To what
possible reason/reasons is this attributable?
1. The anaerobic conditions associated with rice cultivation cause the emission of methane.
2. When nitrogen-based fertilizers are used, nitrous oxide is emitted from the cultivated soil.
Which of the statements given above is/are correct?
(A) 1 only (B) 2 only
(C) Both 1 and 2 (D) Neither 1 nor 2
428. Indiscriminate disposal of used fluorescent electric lamps causes mercury pollution in the environment.
Why is mercury used in the manufacture of these lamps?
(A) a mercury coating on the inside of the lamp makes the light bright white
(B) When the lamp is switched on, the mercury in the lamp causes the emission of ultra-violet
radiations
(C) When the lamp is switched on, it is the mercury which converts the ultra-violet energy into
visible light
(D) None of the statement given above is correct about the use of mercury in the manufacture
of fluorescent lamps
429. Consider the following:
1. Oxides of Hydrogen
2. Oxides of Nitrogen
3. Oxides of Sulphur
Which of the above causes/cause acid rain?
(A) 1 and 2 only (B) 3 only
(C) 2 and 3 only (D) 1, 2 and 3
430. In the middle of the year 2008 the Parliament of which one of the following countries became the first
in the world to enact a Climate Act by passing “The Climate Change Accountability Bill”?
(A) Australia (B) Canada
(C) Germany (D) Japan
431. Consider the following:
1. Rice fields 2. Coal mining
3. Domestic animals 4. Wetlands
Which of the above are sources of methane, a major greenhouse gas?
(A) 1 and 4 only (B) 2 and 3 only
(C) 1, 2 and 3 only (D) 1, 2, 3 and 4
PRACTICE QUESTION
432. Which of the following statements are correct?
1. Ozone is found in the troposphere only.
2. Vehicles and industries are the major sources of ground level ozone emissions.
3. Ozone lowers our resistance to cold and pneumonia.
Select the correct answer using the codes given below:
(A) 1 and 2 only (B) 2 and 3 only
(C) 1 and 3 only (D) 1, 2 and 3
433. Which of the following statement(s) is/are correct regarding Suspended Particulate Matter?
1. Suspended Particulate Matter contain only solid particles.
2. Particulate Matter 2.5 poses greater risk to human health.
Select the correct answer using the codes given below:
(A) 1 only (B) 2 only
(C) Both 1 and 2 (D) Neither 1 nor 2
434. Which of the following affects the level of air pollution?
1. Increasing traffic
2. Increase in population
3. Economic development
4. Industrialization
5. Mining
Select the correct answer using the codes given below:
(A) 1, 2 and 3 only (B) 2, 3, 4 and 5 only
(C) 1, 3, 4 and 5 only (D) 1, 2, 3, 4 and 5
435. Which of the following statement(s) is/are correct?
1. Ground-level ozone is formed through a reaction involving hydrocarbons, nitrogen oxides
and sunlight.
2. Prevalence of heavy smog can result into increase of rickets.
Select the correct answer using the codes given below:
(A) 1 only (B) 2 only
(C) Both 1 and 2 (D) Neither 1 nor 2
436. Which of the following are carcinogenic in nature?
1. Asbestos Dust
2. Tobacco
3. Radioactive pollutants
4. Coal Dust
Select the correct answer using the codes given below:
(A) 1 and 2 only (B) 2, 3 and 4 only
(C) 1, 3 and 4 only (D) 1, 2, 3 and 4
437. Which of the following statement(s) is/are correct?
1. Fly Ash consists of aluminium silicate and silicon dioxide only.
2. Fly Ash contains good pozzolanic properties.
Select the correct answer using the codes given below:
(A) 1 only (B) 2 only
(C) Both 1 and 2 (D) Neither 1 nor 2
438. Which of the following is not a method to abate air pollution?
(A) Cyclone separator (B) Electrostatic Precipitator
(C) Arrester (D) Booms
439. With reference to Hydrochlorofluorocarbon (HCFC)-141b, consider the following statements:
1. It is mainly used as a blowing agent in the foammanufacturing industries.
2. India is the largest exporter of HCFC-141b in the world.
Which of the statement(s) given above is/are correct?
(A) 1 only (B) 2 only
(C) Both 1 and 2 (D) Neither 1 nor 2
440. Which of the following statement(s) is/are correct?
1. Eutrophication reduces dissolved oxygen in water bodies.
2. Water having Dissolved Oxygen (DO) content below 8.0 mg may be considered highly
polluted.
Select the correct answer using the codes given below:
(A) 1 only (B) 2 only
(C) Both 1 and 2 (D) Neither 1 nor 2
441. What do you mean by bioremediation?
(A) It is a method of adding essential vitamins and minerals to foods to increase their
nutritional value.
(B) It is the gradual accumulation of substances such as pesticides or other chemicals in an
organism.
(C) It is the use of microorganisms to degrade the environmental contaminants into less toxic
forms.
(D) It is any concentration of a toxin such as pesticides in the tissues of tolerant organisms at
successively higher levels in a food chain.
442. Consider the following statements:
1. Coliform bacteria acts as an indicator of water pollution.
2. Biochemical Oxygen Demand is the amount of dissolved oxygen used by microorganisms
in the biological process in water.
Which of the statement(s) given above is/are correct?
(A) 1 only (B) 2 only
(C) Both 1 and 2 (D) Neither 1 nor 2
443. Consider the following statements with reference to Fly ash:
1. “ASHTRACK” is a mobile-based application launched to track the monitoring of fly ash
generation.
2. It can increase crop yield.
Which of the statement(s) given above is/are correct?
(A) 1 only (B) 2 only
(C) Both 1 and 2 (D) Neither 1 nor 2
444. Which one of the following is not an example of externalities?
(A) Pollution of air by a cement factory
(B) Health hazard caused to the person due to smoking by himself/herself
(C) Smoke coming out of neighbours vehicle
(D) Increase in land price of adjacent plot due to construction of a road by the
government
445. With reference to the Bharat Stage Emission Standards, consider the following statements:
1. BS-VI fuel will bring down sulphur content by 20 times from the current BS-IV level.
2. The Bharat Stage (BS) regulations are based on the European emission standards.
3. The Central Pollution Control Board implements these standards.
Which of the statements given above are correct?
(A) 1 and 2 only (B) 2 and 3 only
(C) 1 and 3 only (D) 1, 2 and 3
446. Which one of the following gases is released mostly from landfills in urban areas?
(A) Nitrogen (B) Hydrogen
(C) Methane (D) Oxygen
447. If you take a sample of first rain water in a highly polluted city and subject it to a pH test, which of the
following results are you most likely to see?
(A) A low pH value (B) A high pH value
(C) Neutral pH value (D) None of the above
448. With reference to ‘Mosses’, consider the following statements:
1. Mosses act as a bioindicator and a low-cost way to monitor urban pollution.
2. Mosses respond to pollution by changing shape, density or disappearing, allowing
scientists to calculate atmospheric alterations.
Which of the statement(s) given above is/are correct?
(A) 1 only (B) 2 only
(C) Both 1 and 2 (D) Neither 1 nor 2
449. Which of the following statement(s) is/are correct about the Persistent Organic Pollutants (POPs)?
1. These are lipophilic in nature.
2. They are readily soluble in water.
3. Rotterdam Convention aims to eliminate the production and use of Persistent Organic
Pollutants (POPs).
Select the correct answer using the codes given below:
(A) 1 only (B) 1 and 2 only
(C) 2 and 3 only (D) 1, 2 and 3
450. Consider the following statements:
1. Insecticides and pesticides come under the classification of Quantitative pollutants.
2. Ozone is an example of primary pollutant.
Which of the statement(s) given above is/are correct?
(A) 1 and (B) 2 only
(C) Both 1 and 2 (D) Neither 1 nor 2
451. Consider the following statements with reference to National Clean Air Programme (NCAP):
1. It aims to bring down the concentration of Carbon dioxide to World Health Organisation
recommended levels by 2024.
2. All the Metropolitan cities are included under this programme.
Which of the statement(s) given above is/are correct?
(A) 1 only (B) 2 only
(C) Both 1 and 2 (D) Neither 1 nor 2
452. The ‘CleanSeas’ campaign to fight against marine plastic litter is a campaign of which of the following
organizations?
(A) United Nations Environment Programme
(B) International Ocean Institute
(C) Indian Ocean Commission
(D) United Nations Educational Scientific and Cultural Organization
453. Consider the following statements about Biological Oxygen Demand (BOD):
1. It is the amount of dissolved oxygen required by microorganisms present in water to
stabilize inorganic content.
2. The agricultural run-off into the lakes increases its BOD.
Which of the statement(s) given above is/are correct?
(A) 1 only (B) 2 only
(C) Both 1 and 2 (D) Neither 1 nor 2
454. The terms ‘SWAS’, ‘SAFAL’ and ‘STAR- signify which of the following?
(A) Space observation Satellites launched by ISRO
(B) Genetically Modified crops developed by ICAR
(C) Less polluting firecrackers developed by CSIR
(D) None of these
455. Consider the following statements regarding Fly Ash:
1. It is produced as a by-product from coal based thermal power plant.
2. It chemically resembles cement.
3. It enhances water-holding capacity of the land.
Which of the statements given above are correct?
(A) 1 and 2 only (B) 2 and 3 only
(C) 1 and 3 only (D) 1, 2 and 3
456. Which of the following statement(s) is/are correct regarding Acid Rain?
1. The major factors responsible for acid rain are oxides of sulphur and nitrogen.
2. It occurs only in industrial area.
3. It causes an imbalance in the micro-flora of the soil.
Select the correct answer using the codes given below:
(A) 1 only (B) 1 and 2 only
(C) 1 and 3 only (D) 2 and 3 only
457. Which of the following are the in-situ bioremediation techniques?
1. Biopiles 2. Bioaugmentation
3. Bioventing 4. Biosparging
Select the correct answer using the codes given below:
(A) 1, 2, 3, and 4 (B) 2 and 3 only
(C) 1 and 4 only (D) 2, 3 and 4 only
458. With reference to Color Coded Weather Warning by Indian Meteorological Department (IMD),
consider the following statements:
1. Yellow indicates severely bad weather spanning across several days.
2. Orange indicates significant risk to human life.
Which of the statement(s) given above is/are correct?
(A) 1 only (B) 2 only
(C) Both 1 and 2 (D) Neither 1 nor 2
459. Consider the following statements regarding the Comprehensive Environmental Pollution Index
(CEPI):
1. It was carried out by the Ministry of New & Renewable Energy.
2. Its aim was to identify polluted industrial clusters so that appropriate pollution mitigation
measures can be adopted.
Which of the statement(s) given above is/are correct?
(A) 1 only (B) 2 only
(C) Both 1 and 2 (D) Neither 1 nor 2
460. Acid rain affects the growth of the plant. It is due to:
(A) Plant growth is reduced due to the decrease in nutrients available in the soil.
(B) Plant growth is faster due to the increase in nutrients available in the soil.
(C) There is no impact on plant growth due to acid rain.
(D) None of the above
461. According to the World Quality Report- 2020, which of the following capital city was ranked at the first
position in terms of air pollution?
(A) New Delhi (B) Washington D.C.
(C) Doha (D) Dhaka
462. With reference to Clean Seas Campaign, consider the following statements:
1. It was launched by UN Environment Programme (UNEP) in India.
2. The campaign has targeted a time-line of five years (2017-2021).
3. The campaign aims to ban single use of plastic and microbeads.
Which of the statement(s) given above is/are correct?
(A) 1 only (B) 1 and 3 only
(C) 2 and 3 only (D) 3 only
463. Consider the following pairs:
Material Type of acid rain impact
1. Metals Corrosion, tarnishing
2. Paper Embrittlement, discoloration
3. Rubber Cracking
Which of the pair(s) given above is/are correctly matched?
(A) 1 only (B) 1 and 2 only
(C) 2 and 3 only (D) 1, 2 and 3
464. With reference to Coal Gasification, consider the following statements:
1. It is the process of converting coal into syngas.
2. The hydrogen obtained from coal gasification can be used for various purposes such as
powering a hydrogen economy.
3. India has set a target for coal gasification by 2025.
Which of the statements given above are correct?
(A) 2 and 3 only (B) 1 and 2 only
(C) 1 and 3 only (D) 1, 2 and 3
465. With reference to Particulate Matter 2.5, consider the following statements:
1. It is an endocrine disruptor and contributes to diabetes.
2. It is considered in the calculation of the National Air Quality Index.
Which of the statement(s) given above is/are correct?
(A) 1 only (B) 2 only
(C) Both 1 and 2 (D) Neither 1 nor 2
466. Consider the following statements about GOBAR-Dhan Scheme:
1. It is launched by the Ministry of New and Renewable Energy
2. The is implemented as a part of Swachh Bharat Mission (Grameen and Urban).
Which of the statement(s)given above is/are correct?
(A) 1 only (B) 2 only
(C) Both 1 and 2 (D) Neither 1 nor 2
467. Consider the following statements:
1. Blue hydrogen is produced using renewable energy and electrolysis to split water.
2. The by-product of Grey hydrogen is captured and stored.
Which of the statement(s) given above is/are correct?
(A) 1 only (B) 2 only
(C) Both 1and 2 (D) Neither 1 nor 2
468. Stockholm Convention is best describes by which of the following statement?
(A) A global treaty on the Prior Informed Consent Procedure against for certain hazardous
Chemicals and Pesticides in international trade.
(B) A global treaty to protect human health and the environment from persistent organic
pollutants(POPs).
(C) An international treaty designed to protect Indian ocean from anthropogenic emissions
and releases of mercury and mercury compounds.
(D) It is an international treaty for fair and equitable sharing of the benefits arising from the
use of genetic resources.
469. Which of the following statement(s) is/are correct?
1. Carbon farming is the process involved in carbon capture and the long-term storage of
atmospheric carbon dioxide Carbon farming.
2. Carbon sequestration is the process of changing agricultural practices or adopting
agricultural methods that increase the amount of carbon stored in the soil and reduce the
greenhouse gases.
Select the correct answer using the code given below:
(A) 1 only (B) 2 only
(C) Both 1 and 2 (D) Neither 1 nor 2
Solutions
Ecology Solution:
1. Ans. (A)
Sol.
□ The circulation of elements or nutrients from non-living to living and then back
to the non- living components of the ecosystem in a
circular
fashion is known as biogeochemical cycle. The movement of nutrient
elements through the various components of an ecosystem
is also known as nutrient
cycling.


□ Geologic processes, such as weathering, erosion, water drainage, and the subduction

of the continental plates, all play a role in the cycling of elements on Earth.

□ Biogeochemical cycles are of two types:

 Gaseous Nutrient Cycle: Carbon cycle, Nitrogen cycle, Hydrological cycle, etc.

 Sedimentary Nutrient Cycle: Sulphur cycle, Phosphorous cycle.



The reservoir for the gaseous type of nutrient cycle (e.g., nitrogen, carbon cycle)
exists in the atmosphere and for | the sedimentary
cycle (e.g., sulphur and phosphorus j cycle), the reservoir is located in Earth’s crust.
2. Ans. (B)
Sol. □ In ecology, succession is a directional non-seasonal cumulative change in the types
of plant species that occupy a given area through time. It involves the Processes of
colonization, establishment, and extinction, which act on the participating plant species.
□ Most successions contain a number of stages that can be recognized by the collection
of species that dominate at that point in the succession.
□ The entire sequence of communities that successively change in a given area are
called sere(s).The individual transitional communities are termed seral stages or seral
communities.
□ In the successive seral stages, there is a change in the diversity of species of
organisms, increase in the number of species and
organisms as well as an increase in the total biomass.
□ The sequence of seres for a given region is often fully predictable, both with respect
to the general types of population expected at each sere and to seral durations. On land,
for example, a climax stage (final stage) is often represented by a forest community. If
the original physical environment is sand then the sequence of seres includes: soil-
forming organisms (bacteria, lichens, mosses, fungi), annual grasses, perennial grasses,
shrubs, and trees.
3. Ans. (D)
Sol. □ Ecoradicals are those who think that the ecosystem has a limited carrying
capacity which means how large a species population can
become before it overuses the resources available in the ecosystem.
□ Ecoradicals believe that human societies on earth are moving dangerously closer to
the limits of the planet s carrying capacity; they also think that there are no simp e
technological fixes that can take care of the problem. So, many ‘ecoradicals’ call for strict
population control and dramatic changes in modern lifestyles. Hence statements 1 and 2
are correct.
□ It also involved dramatic changes in economic and political organization.
They criticize such advice which calls for environment
protection within a framework of sustainable growth (the Brundtland Report).
Ecoradicals find that this is not at all sufficient. For some, real j sustainability
means abandoning industrial mass production and reverting to some form of
deindustrialized society. Behind such radical ideas lies a world view profoundly
different from the ‘modernist- anthropocentric’ view that is leading in western secular
thinking, i.e., ‘man is above nature’. The ecoradicals
saw the world in a very different way; it puts equal value on humans and nature
as part of one single biosystem and man has no right to exploit nature. Humans
have a duty to live in harmony with nature. Hence statement 3 is correct.
4. Ans. (C)
Sol. □ Food chains and energy flow are the functional properties of ecosystems which make
them dynamic. The biotic and abiotic components of an ecosystem are linked through
them.
□ Transfer of food energy from green plants (producers) through a series of organisms
with repeated eating and being eaten is called a food chain. For example, Grasses →
Grasshopper → Frog → Snake → Hawk/Eagle.
□ Each step in the food chain is called trophic level.
5. Ans. (C)
Sol. □ Ecosystem is broadly classified into terrestrial and aquatic ecosystems.
 The terrestrial ecosystem is the one which involves the land based community of or-
ganisms and their interactions with biotic and abiotic components. For example, forest,
grassland, desert, tundra etc.
 The aquatic ecosystem includes pond, river, lake, wetland, estuary, ocean etc.
□ Limnetic zone is the layer of open water where photosynthesis can occur-
□ An estuary is a partially enclosed body of water, and its surrounding coastal habitats,
where salt water from the ocean typically mixes with fresh water from rivers or streams.
□ Prairies are enormous stretches of flat grassland with moderate temperatures, moderate
rainfall, and few trees. These temperate grasslands are found in North America.
□ Reefs are large underwater structures that are colonised by marine invertebrates called
coral. Coral reefs are the most diverse of all marine ecosystems.
6. Ans. (D)
Sol. □ Nitrogen Fixation process involves conversion of gaseous nitrogen into ammonia, a
form in which it can be used by plants. Atmospheric nitrogen can be fixed by the following
three methods:
 Atmospheric Fixation: Lightening, combustion and volcanic activity help in the fixa-
tion of nitrogen.
 Industrial Fixation: At high temperature and high pressure, molecular nitrogen is bro-
ken into atomic nitrogen which then combines with hydrogen to form ammonia.
 Bacterial Fixation: It is performed by symbiotic bacteria such as Rhizobium in the
root nodules of leguminous plants and Free living or symbiotic bacteria such as Nostoc,
Azotobacter and Cyanobacteria (Blue-Green Algae) that can combine atmospheric or
dissolved nitrogen with hydrogen to form ammonia.
□ Blue-Green Algae are very important for the health and growth of many plants. They
are one of the few groups of organisms that can convert inert atmospheric nitrogen into
an organic form, such as nitrate or ammonia. It is these “fixed” forms of nitrogen which
plants need for their growth, and must obtain from the soil.
□ Cyanobacteria also form symbiotic relationships with many fungi, forming complex
symbiotic “organisms” known as lichens.
7. Ans. (A)
Sol. □ Bioindicators are living organisms such as plants, plankton, animals, and microbes,
which are used to assess the health of the natural ecosystem in the environment. Some of
the examples include:
 Lichens are powerful bioindicators of air quality.
 Frogs are basically influenced by changes that take place in their freshwater and terres-
trial habitats.
 Algal blooms are often used to indicate large increase of nitrates and phosphates in
lakes and rivers.
□ Lichens as Bioindicators
 Lichens is a symbiotic relationship between algae and fungi. The fungus provides shel-
ter for the algae and the algae provides food for the fungi. Lichens do not have roots;
instead they receive all their nutrients from the atmosphere.
 They are sensitive to atmospheric pollution such as nitrogen (N) because they receive
all their nutrients and water from wet and dry atmospheric deposition (fall out). Nitro-
gen deposition can increase the load of nutrients. Too much nitrogen can harm and kill
the algae s chlorophyll, which is used to produce sugars feeding it and the fungi.
 Certain species of lichen are more tolerant of nitrogen than others. Scientists monitor
lichen communities. If an increase in nitrogen tolerant species in combination with a
decrease in nitrogen sensitive species occurs this may indicate an increase in atmo-
spheric nitrogen deposition.
 Lichens are the “canaries in the coal mine” of nitrogen deposition. A shift in their spe-
cies composition and/ or their health exemplifies the potential beginning of ecosystem
decline due to nitrogen deposition.
8. Ans. (C)
Sol. □ The Western Ghats are internationally recognized as a region of immense global
importance for the conservation of biological diversity, besides containing areas of high
geological, cultural and aesthetic values.
□ A chain of mountains running parallel to India’s western coast, approximately
30-50 km inland, the Ghats traverse the States of
Kerala, Tamil Nadu, Karnataka, Goa, Maharashtra and Gujarat. It is interrupted by
Palghat Pass.
□ Older than the Great Himalayan mountain chain, the Western Ghats are home to four
tropical and subtropical moist broadleaf forest ecoregions - the North Western Ghats moist
deciduous forests, North Western Ghats montane
rain forests, South Western Ghats moist deciduous forests, and South Western Ghats
montane rain forests.
□ These mountains act as a rain barrier during southwest monsoon, leading to ample
rainfall and source to 58 major rivers.
□ A significant characteristic of the Western Ghats is the exceptionally high level of
biological diversity and endemism. This mountain chain is recognized as one of the world’s
eight ‘hottest hotspots’ of biological diversity along with Sri Lanka. It is also a UNESCO World
Heritage Site.
□ The forests of the Western Ghats include some of the best representatives of non
equatorial tropical evergreen forests in the world. At least 325 globally threatened (IUCN Red
Data List) species occur in the Western Ghats.
9. Ans. (C)
Sol. □ The Deep Carbon Observatory (DCO) is a
global research program designed to transform understanding of carbon’s role in Earth. To
complement this research, the DCO’s infrastructure includes public engagement
and education, online and offline community support, innovative data management,
and novel instrumentation development. Hence, statement 1 is correct.
□ The Deep Carbon Observatory’s research considers the global carbon cycle beyond
Earth’s surface. It explores high-pressure and extreme temperature organic synthesis,
complex interactions between organic molecules and minerals, conducts field observations
of deep microbial ecosystems and of anomalies in petroleum geochemistry, and constructs
theoretical models of lower crust and upper mantle carbon sources and sinks. Hence,
statement 2 is correct.
□ Considerable amounts of life forms, including 70% of bacteria and archaea on
Earth, comprising up to 23 billion tonnes of carbon, live up to at least 4.8 km (3.0 mi)
deep underground, including 2.5 km (1.6 mi) below the seabed, according to a ten-year
Deep Carbon Observatory project.
10. Ans. (C)
Sol.
Vegetation Features
Epiphytes □ Climate: Equatorial
(A-2) □ An epiphyte grows on the surface of a plant and derives its moisture
and nutrients from the air, rain, water or from debris accumulating
around it.
Acacia □ Climate: Savanna
(B-4) □ Acacias are well adapted to deserts and tropical areas. Therefore,
they have a wide distribution and are native to Australia, South
Africa, the Southwestern United States, and South and Central
America.
□ Large, umbrella-shaped acacias of the African Savanna are often
used as a landmark.
Baobab □ Climate: Sahara
(C-3) □ Baobabs are deciduous trees ranging in height from 5 to 20 meters.
The African baobab, the oldest known angiosperm tree is known as
the “Tree of Life,”.
□ The species is found throughout the drier regions of Africa and
features a water-storing trunk.
Cedars (D-l) □ Climate: Mediterranean
□ Cedar, any of four species of ornamental and timber evergreen
conifers of the genus Cedrus (family Pinaceae), three native to
mountainous areas of the Mediterranean region and one to the
western Himalayas.
11. Ans. (A)
Sol. □ Species diversity is higher at the equator than at the poles. In biological terms, this is
referred to as the latitudinal diversity gradient (LDG), in which the number of species increases
from the poles to the Equator.
□ Ecological factors commonly are used to account for this gradation. Higher temperatures,
greater climate predictability, and longer growing seasons all conspire to create a more
inviting habitat, permitting a greater diversity of species.
□ Tropical rainforests are the richest habitat of all, tropical grasslands exhibit more diversity
than temperate grasslands, and deserts in tropical or subtropical regions are populated by a
wider range of species than are temperate deserts.

\
12. Ans. (D)
Sol. □ Coral reefs are large underwater structures composed of the skeletons of corals, which
are marine invertebrate animals. The coral species extract Calcium Carbonate from seawater to
create a hard, durable exoskeleton that protects their soft, sac-like bodies. Besides other
conditions, reef-building corals require warm ocean temperatures (20 to 28°C). Hence, most
of the coral reefs are found in tropical waters. Hence, statement 1 is correct.
□ Reef-building corals are scattered throughout the tropical and subtropical Western Atlantic
and Indo-Pacific oceans, generally within 30°N and 30°S latitudes. More than three- quarters
of the world’s reef-building coral species can be found within the waters of “Coral Triangle”, the
area covering the reefs of Northern Australia, Indonesia, Philippines and Papua New Guinea
lending from Indonesia in the west to the Solomon Elands in the east, and the Philippines in
the north. Hence, statement 2 is correct.
□ Coral reefs have much more animal phyla than those found in tropical rainforests. 32 of
the 34 recognized animal Phyla are found in coral reefs compared to 9 in tropical rainforests.
Hence, statement 3 is correct.
13.
Ans. (A)
Sol. □ Carbon Fertilization can be defined as increased rate of photosynthesis due to the large
amount of carbon dioxide in the atmosphere that has resulted from rising anthropogenic
emissions.
□ It is the artificial enrichment of the atmosphere of greenhouses with carbon dioxide, an
essential nutrient for plants and vegetables.
□ Due to the increased rate of photosynthesis, it improves production/growth level of plant,
both from a qualitative and from a quantitative point of view. It is particularly suitable for cold
climates and can be used for growing practically all types of vegetables (asparagus, c e l e r y ,
lettuce, tomatoes, aubergines etc.), greenhouse fruit (strawberries) and ornamental plants.
14.
Ans. (D)
Sol. □ The word biome is a short form of biological home. It may be defined as a large natural
eco-system wherein we study the total assemblage of plant and animal communities.
□ Whereas, anthropogenic biome also known as Anthrome or Human Biomes, are the
globally significant ecological patterns created by sustained interactions between humans and
ecosystems, including urban, village, cropland, rangeland and semi natural anthrome.
□ The term was originated by Erle Ellis and Navin Ramankutty in 2007 in their paper-
Frontiers in Ecology and the Environment as a more comprehensive way to integrate humans
into global ecology.
15. Ans. (B)
Sol. □ Food chain is the sequence of transfer of matter and energy in the form of food from
organism to organism.
□ A food chain starts with producers and ends with top carnivores. Among the above given
options, the correct sequence is grass that is eaten by goats which, in turn, is eaten by
human beings.
16.
Ans. (A)
Sol. □ In an ecosystem the energy stored at the producer level is called primary productivity.
Energy stored at the consumer level is called secondary productivity.
□ Gross primary productivity (GPP) is the total amount of solar energy captured and stored
in the form of organic substances by the green plants.
□ Net primary productivity (NPP) is the amount of organic substances left in the plant after
its own metabolism i.e., NPP = GPP - Respiration.
□ Highest value of net primary productivity in terrestrial environments occurs in tropical
rainforests; the lowest occurs in deserts. While in aquatic environments, the highest NPP
occurs in estuaries, algal beds, and reefs.
17.
Ans. (A) Sol.
□ The functional characteristics of a species in its habitat are referred to as “niche” in that
common habitat. Hence, the niche is the set of biotic and abiotic conditions in which a
species is able to persist and maintain stable population sizes.
□ The term niche means the sum of all the activities and relationships of a species by which
it uses the resources in its habitat for its survival and reproduction
18.
Ans. (C)
Sol. □ Autecology, also known as Species Ecology, is
an approach in ecology that seeks to explain thedistributionandabundanceof
species by studying interactions of individual organisms with their environments.
□ Synecology, also known as Community Ecology, is the branch of ecology dealing with the
relations between natural communities and their environments.
□ Genecology is a branch of ecology, which studies genetic variation of species and
communities compared to their population distribution in a particular environment.
□ Ethology is the scientific study of animal behaviour, usually with a focus on behaviour
under natural conditions, and viewing behaviour as an evolutionarily adaptive trait.
19.
Ans. (B)
Sol. □ Ordinary Urea: It is a fertiliser used to provide Nitrogen to the soil, which is necessary for
the development of plants. Only 30-40% of Nitrogen present in the urea is utilised by crops. T h e
rest gets degraded. Ordinary urea gets converted to Ammonium Carbamate. Some of this
gets converted to Ammonia Gas in a process called ammonia volatilisation, while the rest of the
Ammonium Carbamate undergoes a chemical transformation and Nitrates are formed.
Some of these are absorbed by the plants. The rest is either leached into the underground water or
are denitrified to gaseous Nitrogen and Nitrous Oxide under anaerobic conditions (absence
of Oxygen).
□ Neem Coated Urea: Neem has properties that check nitrogen loss at each stage. It
slows down the process of nitrate formation and hence,
excess nitrate is not available for denitrification. Thus, it helps in countering the degradation of
soil and underground water and also any subsequent air pollution by slowing down the
rate of dissolution of urea in the soil.
20. Ans. (A)
Sol. □ An artificial ecosystem is a human made system of plants, animals, and people living in an
area together with their surroundings.
□ The zoos, aquariums and botanical gardens are examples of artificial ecosystems which are
maintained with the objective of conserving biodiversity.
□ Rice-field is an artificial ecosystem because the fields are grown by men/ women for
agricultural needs. They do not occur naturally. Modern agricultural practices with large-
scale intensive farming have significantly accelerated the homogenization of the paddy fi e l d
ecosystem.
21. Ans. (D)
Sol. □ A food chain refers to the order of events in an ecosystem, where one living organism eats
another organism, and later that organism is consumed by another larger organism. The fl o w
of nutrients and energy from one organism to another at different trophic levels forms a
food chain.
□ A food chain shows us how every living organism is dependent on other organisms for
survival. The food chain explains the path of energy flow inside an ecosystem.
□ Ina food chain, man is the primary consumer as he eats vegetables (herbivores) as well as
secondary consumer as he eats meats and fishes (carnivores).
22. Ans. (C)
Sol. □ An ecosystem can be visualized as a functional unit of nature, where living organisms
interact among themselves and also with the surrounding physical environment. It varies
greatly in size from a small pond to a large forest or a sea. The ecosystem is defined as a
community of organisms together with their living environment.
□ An ecosystem generally consists of biotic components (animals, plants, bacteria, etc.) and
abiotic components including minerals, climate, soil, water, sunlight, etc., linked through major
forces of the flow of energy and geochemical cycles within the ecosystem.
23. Ans. (A)
Sol. □ Moist deciduous forests are found throughout India except in the western and the north-
western regions. The trees found here are tall with broad trunks and have firm roots which h o l d
them to the ground. Tall trees shed their leaves in the dry season and a layer of shorter trees
and shrubs grow under the canopy of tall trees
□ Dominant tree species are sal and teak, along with mango, bamboo, and rosewood.
24.
Ans. (A)
Sol. □ Mangrove forests only grow at tropical and subtropical latitudes near the equator because
they cannot withstand freezing temperatures. These trees grow in areas with low-oxygen soil,
where slow-moving waters allow fine sediments to accumulate.
□ They characteristically have prop roots or exposed supporting roots. Many species
characterize respiratory or knee roots. Such roots project above the mud and have small
openings (lenticels) through which air enters, passing through the soft spongy tissue to the roots
beneath the mud.
□ This tangle of roots allows the trees to handle the daily rise and fall of tides, which means
that most mangroves get flooded at least twice per day. The roots also slow the movement o f
tidal waters, causing sediments to settle out of the water and build up the muddy bottom.
□ Mangrove forests stabilize the coastline, reducing erosion from storm surges, currents,
waves, and tides. The intricate root system of mangroves also makes these forests attractive to
fish and other organisms seeking food and shelter from predators.
25.
Ans. (D)
Sol. □ A lichen is not a single organism. Rather, it is a symbiosis between different organisms, a
fungus and an algae or cyanobacterium. Lichens are the first organisms to colonise a bare rock.
□ They have the ability to colonize a wide range of surfaces and can be found growing in
almost all parts of the terrestrial world, from the ice-free polar areas to the tropics, from
tropical rainforests to those desert areas free of mobile sand dunes.
□ The surfaces on which lichens grow vary from the natural (such as soil, rock, wood, bone)
to the man-made (for example, bitumen, concrete, glass, canvas, metal, etc.).
□ Referred to as the pioneer species, spores of lichen are spread onto a land of rocks. Then,
the rocks are broken down into smaller particles. Organic matter gradually accumulates,
favouring the growth of herbaceous plants like grasses, ferns and herbs.
26.
Ans. (A)
Sol. □ The Olive ridley turtles are the smallest and most abundant of all sea turtles found in
warm waters of the Pacific, Atlantic and Indian oceans.
□ Rushikulya river and Gahirmatha Marine Sanctuary in Odisha are known for mass nesting
site (Arribada) of Olive Ridley turtles.
□ The species is listed as Vulnerable in the IUCN Red List, Appendix 1 in CITES, and
Schedule 1 in Wildlife Protection Act, 1972.
27.
Ans. (A)
Sol. □ The food chain is defined as the relation between organisms of different trophic levels
which are connected to each other for food or energy. In a food chain the flow of energy or f o o d
is unidirectional and in a linear sequence.
□ First, plants capture solar energy and then, food is transferred from the producers to
decomposers.
□ Diatoms are single celled photosynthesizing algae found in seas and oceans.
□ Animals like crab, shrimps, lobsters, etc., are crustaceans and they eat diatoms.
□ Herrings are fish and they eat crustaceans.
□ Thus, Diatoms-Crustaceans-Herrings forms the correct food chain.
28.
Ans. (B)
Sol. □ Lichen is not a single organism. Rather, it is a symbiosis between different organisms - a
fungus and an alga or cyanobacterium. Cyanobacteria are sometimes referred to as ‘blue- green
algae’, though they are quite distinct from the algae.
□ Lichens are amongst the first organisms to colonize the barren surfaces (e.g., road cuttings,
rock outcrops and volcanic ash) and prepare these areas for later plants by trapping moisture and
windblown organic debris and then contributing to the organic deposits when t h e y
themselves die and decay.
29.
Ans. (C)
Sol. □ The Carbon cycle is a process in which Carbon travels from the atmosphere into organisms
and the Earth and then back into the atmosphere. In the atmosphere, Carbon is attached to
Oxygen and is present in the form | of a gas called Carbon Dioxide (CO2).
□ Volcanic eruptions besides discharging lava, rocks and volcanic ash; emits water vapour,
Carbon Dioxide, Sulphur Dioxide, Hydrogen Sulphide, Hydrochloric Acid’ Carbon Monoxid,e
etc. Hence, 1 is correct.
□ In the process of respiration in animals and plants, Carbon Dioxide is released. Hence, 2 is
correct.
□ In decay of organic matter, microorganisms (decomposers) act upon crop and animal
residue to produce CO2, water and nutrients. Hence, 4 is correct.
□ Through the process of photosynthesis, Carbon Dioxide from the air is used by the plants
to produce Oxygen and carbohydrates. Hence, 3 is not correct.
30.
Ans. (B)
Sol. □ Mangroves are the plant communities occurring in intertidal zones along the coasts of
tropical and subtropical countries.
□ Since, mangroves grow in marshy conditions, where soil is poor in nutrient and oxygen
content, they develop specialed roots called pneumatophores or breathing roots for the
purpose of breathing.
31. Ans. (B)
Sol. □ The sequence in which organism feed on each other, forms a food chain and each step in
the food chain is called trophic level.
□ A food chain starts with producers and ends with top carnivores. Among the above given
options, the correct sequence of the food chain is Grasshoppers, Frogs, Snakes, Vultures
because grasshopper is eaten by frog, which is eaten by snake and which is the source of food
for vultures.
32. Ans. (D)
Sol. □ An ecosystem consists of a community of organisms and their physical environment
interacting as an ecological unit. It involves the interaction between both biotic factors (plants,
animals, and other organisms) and abiotic factors (include rocks, temperature, and humidity) in
a given area.
□ The biotic and abiotic components are considered to be linked through nutrient cycles and
energy flows.
□ It acts as a functional unit of nature and varies greatly in size from a small pond to a large
forest or a sea.
33. Ans. (D)
Sol. □ Decomposition is the process by which dead organic matter are broken down into simpler
organic or inorganic matter such as carbon dioxide, water, simple sugars and mineral salts.
□ Fungi are major decomposers in ecosystems because they produce specific enzymes that
ensures dead plants and animals are broken down into smaller molecules that can be used by o ther
members of the ecosystem. Without fungi, decaying organic matter would accumulate in t h e
forest.
34. Ans. (A)
Sol. □ An ecosystem consists of a community of organisms and their physical environment
interacting as an ecological unit. It involves the interaction between both biotic factors (plants,
animals, and other organisms) and abiotic factors (include rocks, temperature, and humidity) in
a given area.
□ Oceans are the largest existing ecosystem on our planet. Covering 70% of the Earth’s
surface, producing half of the world’s oxygen and storing 50 times more carbon dioxide than our
atmosphere. It is home to more than 2 million estimated species and provides for a sixth of all
animal protein humans consume.
35. Ans. (D)
Sol. □ Adaptations are special features that allow a plant or animal to live in a particular place or
habitat.
□ Leaf modification is also a special feature that allows a plant to inhibit water loss in desert
area. Some leaf modifications are as follows:
Hard and waxy coating on the leaf; hence, 1 is correct.
Compound leaves;
Small leaf size; hence, 2 is correct.
Hair or fur on leaves;
Spines or thorns. Hence, 3 is correct.
36. Ans. (B)
Sol. □ Decomposers breakdown dead organic material into water, carbon dioxide, minerals and
other simple chemicals which can again be consumed by green plants.
□ Like herbivores and predators, decomposers are heterotrophic, meaning that they use
organic substrates to get their energy, carbon and nutrients for growth and development.
Viruses infect cells and use components of the host cell to make copies of themselves. They are not
decomposers. Hence, 1 is not correct.
The primary decomposers of litter in many ecosystems are fungi. Hence, 2 is correct.
Bacteria are other important decomposers apart from fungi. Hence, 3 is correct.
37. Ans. (B)
Sol. □ Ecological Niche: It refers to the physical position and functional role of species within a
community. A species’ niche includes all of its interactions with the biotic and abiotic factors of its
environment. Biotic factors are living things, while abiotic factors are non-living things. It is
advantageous for a species to occupy a unique niche in an ecosystem because it reduces t h e
amount of competition for resources that species will encounter.
□ Ecotone: It is a transition zone of vegetation between different plant communities, such as
forest and grasslands, where two communities meet and integrate.
□ Habitat: It is a place or environment where a plant or animal naturally or normally lives
and grows.
□ Home Range: The home range of an animal is the area where it spends its time; it is
the region that encompasses all the resources which the
animal requires to survive and reproduce.
38. Ans. (B)
Sol. □ Biofertilizers are products containing carrier based (solid or liquid) living microorganisms
which are agriculturally useful in terms of nitrogen fixation, phosphorus solubilisation or
nutrient mobilization to increase the productivity of soil or crop. Classification of biofertilizers based
on microorganism include:
Bacterial Biofertilizers: Rhizobium, Azospirilium, Azotobacter, Phosphobacteria, Nostoc, etc. Hence,
2 is correct.
Fungal Biofertilizers: Mycorrhiza, Penicillium sp, Aspergillus awamori.
Algal Biofertilizers: Blue Green Algae (BGA) and AzoIla.
Actinomycetes Biofertilizer: Frankia.
□ Agaricus is an edible fungus and is commonly known as mushroom. It is a saprophytic
fungus found growing on soil humus, decaying litter on forest floors, in the fields, lawns, wood
logs and manure piles. Hence, 1 is not correct.
□ Spirogyra is a large genus of freshwater green algae found in shallow ponds, ditches
and amongst vegetation at the edges of large lakes, generally growing free-floating. It is valued
for human consumption, and is known as an important source of natural bio-active compounds
for antibiotic, antiviral, antioxidant, anti-inflammatory, and cytotoxic purposes. Hence, 3 is not
correct.
39. Ans. (C)
Sol. □ Nitrogen is a naturally occurring element that is essential for growth and reproduction in
both plants and animals. It is found in amino acids that make up proteins in nucleic acids, that
comprise the hereditary material and life’s blueprint for all cells and in many other organic and
inorganic compounds.
□ Nitrogen also comprises about 78% of the Earth’s atmosphere.
□ Mammals, including humans, are the primary producers of urea. Because they secrete urea
as the primary nitrogenous waste product, they are called ureotelic animals. These wastes
add nitrogen to the soil. Hence, 1 is correct.
□ Coal combustion produces Oxides of Carbon (COx), Oxides of Sulphur (Sox), Oxides of
Nitrogen (NOx) and a variety of byproducts, including fly-ash, flue gas and scrubber sludge.
However, it does not directly add nitrogen to the soil. Hence, 2 is not correct.
□ When plants and animals die, the Nitrogen compounds in the organic matter re-
enter the soil where they are broken down by microorganisms,
known as decomposers. This decomposition produces Ammonia, which goes through the
nitrification process, i.e., nitrifying bacteria in the soil convert Ammonia into Nitrite (NO2) and
then into Nitrate (NO3). Hence, 3 is correct.
40. Ans. (C)
Sol. □ Productivity is the rate of biomass production or the amount of food energy produced
or obtained or stored by a particular trophic level per unit area in a unit time. It can also be
defined as the energy accumulated in plants by photosynthesis. The unit of productivity is g m /
m2/ year or kcal/m2/year.
Primary Productivity is the productivity at the producer level. It can be termed as the amount of
organic matter produced by the plants from solar energy in a given area during a given period of time.
Secondary Productivity is the productivity at the consumer level. It reflects only the utilisation of food
for the production of consumer biomass. It can be referred to as the net rate of increase in biomass
of heterotrophs.
□ Of all the given ecosystems, mangrove ecosystem has highest productivity followed by
grassland, lakes, and oceans in decreasing order.
41. Ans. (A)
Sol. □ According to ‘National Wetland Inventory and Assessment’ report prepared by the Space
Applications Centre; Gujarat (34.74 hectares) leads in wetland area compared to other states
followed by Andhra Pradesh, Uttar Pradesh, West Bengal and Maharashtra. Hence, statement 1 is
correct.
□ The total area of inland wetlands is 69% of total wetlands while it is only 29% for coastal
wetlands. Hence, statement 2 is not correct.
42. Ans. (D)
Sol. □ Biopesticides are certain types of pesticides derived from natural materials such as
animals, plants, bacteria, and certain minerals. For example, canola oil and baking soda have
pesticidal applications and are considered biopesticides.
Bacterial pesticides are used to control pests, pathogens and weeds by a variety of mechanisms. They
might act as competitors or inducers of host resistance in plants or they work by inhibiting growth,
feeding, development or reproduction of a pest or pathogen. Hence, 1 is correct.
Fungal biopesticides can be used to control plant diseases as well as some pests and weeds. Fungi
are a diverse group of organisms and can be found in almost every environment on earth. Two of the
most common commercial biopesticides are Trichoderma species and Beauveri abassiana. Hence, 2
is correct.
Botanical pesticides are eco-chemicals isolated from parts of the plants such as leaves, roots, barks,
fruits, seeds or seed kernels. By nature, several higher plants have the ability to synthesize and
produce numerous secondary metabolites which will be unpalatable to insect pests. Azadirachta
indica commonly called as Neem is a most promising plant, used effectively and extensively for insect
pest management program. Hence, 3 is correct.
43.
Ans. (C)
Sol. □ The Carbon cycle is the process in which Carbon travels from the atmosphere into
organisms and the Earth and then back into the
atmosphere, where, Carbon is attached to Oxygen in a gas called Carbon Dioxide (CO2).
□ Through the process of photosynthesis, Carbon Dioxide from the air is used by the
plants to produce carbohydrates. Hence, 1 is not correct.
□ In the process of respiration in animals and plants, Carbon Dioxide is released. Similarly, in
the decay of organic matter, the CO2 is released. Hence, 2 and 3 are correct.
□ Volcanic eruptions besides discharging lava, rocks and volcanic ash; emit Water Vapour,
Carbon Dioxide, Sulphur Dioxide, Hydrogen Sulphide, Hydrochloric Acid, Carbon Monoxide e t c .
Hence, 4 is correct.
44. Ans. (B)
Sol. □ Upwelling is the process through which cold and nutrient rich water from the bottom of
the ocean comes above the surface of the ocean by the action of the surface wind. It replaces
the warmer, usually nutrient depleted, surface water. Hence, statement 2 is correct.
□ This is related to Phytoplanktons and has nothing to do with the decomposer
microorganisms. Hence, statement 1 is not correct.
□ The bottom dwelling organisms or benthos, are not brought to the surface due to
upwelling. Hence, statement 3 is not correct.
□ The upwelling sites give rise to the great fishing grounds of the world because these areas
become rich in nutrients and have high biological productivity.
45. Ans. (C)
Sol. □ Sequence of Evolutionary Stages: Single cellular -> Multi cellular -> Fishes -> Amphibians
-> Reptiles → Birds → Mammals.
Octopus (Molluscs) - Dolphins and Whales (Mammals)-Shark (Fish)
Pangolin (Mammal)-Tortoise (Reptile) - Hawk (Bird).
Salamander (Amphibian)-Python (Reptile) - Kangaroo (Mammal). This sequence follows the correct
sequence of evolutionary stages.
Frog (Amphibian)-Crab (Crustaceans)-Prawn (Crustaceans).
PRACTICE SOLUTION:
46.
Ans. (C)
Sol. □ Decomposers also known as Saprotrophs or detritivores, secretes digestive enzymes that
breakdowns the complex organic matters, dead and waste materials into simple, inorganic
materials like carbon dioxide, water and nutrients, which are subsequently absorbed by them.
□ Saprotrophs meet their energy and nutrient requirements by degrading dead organic
matter or detritus. Hence, statement 1 is correct.
□ The important steps in the process of decomposition are fragmentation, leaching,
catabolism, humification and mineralization. Detritivores
(e.g., earthworm) break down detritus into smaller particles. This process is called
fragmentation. They help in the decomposition of organic matter. Hence, statement 2 is correct.
□ Decomposers, such as bacteria and fungi (mushrooms), feed on waste and dead matter,
converting it into inorganic chemicals that can be recycled as mineral nutrients for plants to u s e
again.
47. Ans. (D)
Sol. □ Ecotone is a transition area between two adjacent ecological communities. It has the
conditions intermediate to the adjacent ecosystems due
to which ecotone is a zone of tension. Hence, statement 2 is correct.
□ Sometimes the number of species and the population density of some of the species is
much greater in ecotone than either community known as the edge effect. Hence, statement 1 is
correct.
□ Organisms in this zone are known as edge species. In the terrestrial ecosystems edge effect
is especially applicable to birds. For example, the density of some birds is greater in the
mixed habitat of the ecotone between the forest and the desert. Hence, statement 3 is correct.
Therefore, option (d) is the correct answer.
48. Ans. (A)
Sol. □ The functional characteristics of a species in its habitat are referred to as “niche”. Hence,
the niche is the set of biotic and abiotic conditions in which a species is able to persist and
maintain stable population sizes. It is a description of all the biological, physical and chemical
factors that a species needs to survive, stay healthy and reproduce.
□ The term niche means the sum of all the activities and relationships of a species by which
it uses the resources in its habitat for its survival and reproduction.
□ A niche is unique for a species while many species share the habitat. No two species in a
habitat can have the same niche. This is because if two species occupy the same niche, they
will compete with one another until one is displaced. For example, a large number of different
species of insects may be pests of the same plant but they can co-exist as they feed on different
parts of the same plant. Hence, statement 1 is correct and statement 2 is not correct.
49. Ans. (A)
Sol. □ Ecosystems are of two major types i.e., terrestrial and aquatic. Terrestrial ecosystem can
be further classified into biomes. A biome is a plant and animal community that covers a
large geographical area. The boundaries of different biomes on land are determined mainly
by climate most important of which is temperature and precipitation. Hence, statement 1 is
correct.
□ A biome can be defined as the total assemblage of plant and animal species interacting
within specific conditions. These include rainfall, temperature, vegetation, animal life, humidity
and soil conditions. They are distinct from habitats, because any biome can comprise a variety
of habitats. Hence, no two biomes are alike. Hence, statement 2 is correct.
□ Some of the major biomes of the world are forest, grassland, desert and tundra biomes.
However, aquatic ecosystems are not referred to as biome though they are divided into distinct
life zones such as fresh water ecosystem, marine ecosystem, estuaries, mangroves and coral
reef. The salinty, temperature, sunlight penetration are some of the factors which distinguishes
various aquatic ecosystems. Hence, statement 3 is not correct.
50. Ans. (C)
Sol. □ Complex network of inter-connected food chains is known as a food web. Food chain
denotes only one part of the food and energy flow, thus represents an isolated relationship which
rarely occurs in an ecosystem. However, food web provides all possible transfers of energy and
nutrients among the organisms in an ecosystem. Food webs are more realistic models of
energy flow through an ecosystem. Hence, statement 1 is not correct.
□ The food web provides more than one alternative for food in an ecosystem unlike food
chain which provides only one food source and
therefore, the chance of survival is better in the food web. For example, the grass is the food source
for rabbit or goat. Similarly, a herbivore may be a food source for many different carnivores.
□ In food web, if any of the intermediate food chain is removed, the succeeding links of the
chain will be affected largely, however it will not affect the overall food web, as alternative f o o d
is available for most of the organisms, which increases their chance of survival. Hence,
statement 2 is correct.
□ Food availability and preferences of food of the organisms may shift seasonally. Hence,
statement 3 is correct.
51. Ans. (A)
Sol. □ The pyramid of numbers depicts the relationship in terms of the number of producers,
herbivores and the carnivores at their successive trophic levels.
□ The pyramid of numbers does not take into account the fact that the size of organisms
being counted in each trophic level can vary. Hence, statement 1 is correct.
□ It is very difficult to count all the organisms in a pyramid ; of numbers so the pyramid of
numbers does not completely define the trophic structure for an ecosystem. Hence, statement 2
is not correct.
52. Ans. (C)
Sol. □ Biogeochemical cycles are intricate processes that transfer, change and store chemicals in
the geosphere, atmosphere, hydrosphere, and biosphere. In its simplest form, cycling describes
the movement of elements through various forms and their return to their original state.
Biogeochemical cycles can be classed as gaseous and sedimentary.
□ Sedimentary cycles follow weathering of rocks and erosion of minerals along with its
circulation in the environment and back to earth crust. In this cycle, elements move from land
to water to sediment. The main reservoir for the sedimentary cycle is located in Earth’s crust. It
includes Sulphur cycle, Phosphorous cycle, Iron cycle, Calcium cycle and other earthbound
elements. Hence, 1 and 4 are correct.
□ In a gas cycle, elements move through the atmosphere and the main reservoirs are the
atmosphere and the ocean. It includes Carbon cycle, Nitrogen cycle, Hydrological cycle,
Oxygen cycle etc. Hence, 2 and 3 are not correct.
□ The function of the reservoir is to meet with the deficit which occurs due to imbalance in
the rate of inflow and outflow.
53. Ans. (A)
Sol. □ Terrestrial ecology is the study of land-based ecosystems, their populations and
communities of plants, animals, and microbes, their
interactions with the atmosphere and with streams and groundwater, and their role in the
cycling of energy, water, and the major biogeochemical elements such as carbon and nitrogen.
Hence, statement 1 is correct.
□ Due to variations in topographic features of valleys, mountains and slopes, certain
differences occur. These differences are reflected in both
material and biotic diversities. Altitudinal and latitudinal variations cause shifts and
differences in the climatic patterns. Hence, statement 2 is not correct.
54. Ans. (B)
Sol. □ Brazzaville Declaration was signed to protect the Cuvette Centrale region in the Congo
Basin, the world’s largest tropical peatlands, from unregulated land use and prevent its drainage
and degradation.
□ The Democratic Republic of Congo (DRC), the Republic of Congo and Indonesia jointly
signed the Brazzaville Declaration that promotes better management and conservation of this
globally important carbon store. The Declaration was signed on the sidelines of the Third
Conference of Partners of the Global Peatlands Initiatives (GPI), that took place in Brazzaville,
Republic of Congo. Peatlands are wetlands that contain a mixture of decomposed organic
material, partially submerged in a aver of water, lacking oxygen. They are home to a variety of
species, but their high carbon content makes them uniquely vulnerable to incineration if they a r e
drained.
55. Ans. (C)
Sol. □ The biodiversity is generally high in the case of wetlands compared to lakes. They support
high concentrations of birds, mammals, reptiles, amphibians, fish and invertebrate species.
Hence, statement 1 is correct.
□ Dominant producers in case of wetlands are Macrophytes such as cattails, hydrilla, water
hyacinth and duckweed, compared to lakes where phytoplankton are dominant. Hence,
statement 2 is correct.
56. Ans. (C)
Sol. □ Albedo is the amount of light or radiation that a surface is able to reflect back. White
surfaces such as snow reflect the most and thus have the highest albedo while darker surfaces such
as concrete or granite have a low albedo.
□ Tundra is found in the higher latitudes where there is a lot of snow therefore it has the
highest albedo out of the given options.
□ Taiga is a sub arctic biome and has a lower albedo than Tundra.
□ Tropical deciduous forests shed their leaves at a same time so there is less absorption of
solar energy, required for photosynthesis, thus have higher albedo than tropical evergreen
forests.
□ Tropical evergreen forests have the lowest albedo out of the above options as they do not
shed their leaves at a same time, means they absorb relatively more solar energy as compared
to other given biomes.
□ So, the correct sequence of the following ecosystems in decreasing order of their albedo
value is as follows: Tundra > Taiga > Tropical deciduous > Tropical evergreen
57. Ans. (D)
Sol. □ Allogenic Succession: It is caused by the abiotic components of an ecosystem. The
succession can be brought about in a number of ways,
such as volcanic eruptions; meteor or comet strike; flooding; drought; earthquakes and non-
anthropogenic climate change. For example, in Cornwall, England, observations on the estuary
of the Fai river suggest that the deposition of silt may be causing an allogenic succession
from salt marsh to woodland. Hence, statement 1 is not correct.
□ Autogenic Succession: It is caused by the biotic components of an ecosystem. The plants
themselves (biotic components) cause succession to occur for instance light captured by the
leaves; production of detritus; water and nutrient uptake; nitrogen fixation, anthropogenic
climate change. These aspects lead to? gradual ecological change in a particular spot of land
known as a progression of inhabiting species. Autogenic succession can be viewed as a
secondary succession because of pre-existing plant life. Hence, statement 2 is not correct.
□ Progressive Succession: It is a succession where the community becomes complex and
contains more species and biomass over time.
□ Retrogressive Succession: It is a succession where the community becomes simplistic and
contains fewer species and less biomass over time. Some retrogressive successions are
allogenic in nature. For example, the introduction of grazing animals results in degenerated
grasssland. Hence, statement 3 is not correct.
58. Ans. (B)
Sol. □ The term anthropogenic designates an effect or object resulting from human activity.
Human impact on the environment or anthropogenic impact on the environment includes changes
in the biophysical environments, ecosystems, biodiversity, and natural resources.
□ The impacts caused directly or indirectly by humans, include global warming,
environmental degradation (such as ocean acidification), mass
extinction and biodiversity loss, ecological crisis, and ecological collapse.
59. Ans. (A)
Sol. □ Ecosystems arc capable of maintaining their state of equilibrium. They can regulate their
own species structure and functional processes. This capacity of ecosystem of self-regulation is
known as homeostasis. In ecology the term applies to the tendency for a biological system to resist
changes.
□ For example, in a pond ecosystem if the population of zooplankton increased, they would
consume large number of the phytoplankton and as a result soon phytoplankton would be of
short supply to them as food. As the number zooplankton is reduced because of starvation,
phytoplankton population start increasing, thus maintaining an equilibrium. Hence, statement 1
is correct.
□ Ina homeostatic system, negative feedback mechanism is responsible for maintaining
stability in an ecosystem. However, homeostatic capacity of ecosystems is not unlimited as well as
not everything in an ecosystem is always well regulated. Hence, statement 2 is not correct.
60. Ans. (A)
Sol. □ Regular supply of water on earth is maintained through its circulation in the atmosphere.
Precipitation of water vapour in the form of rain, snow, dew, hail etc. is the main source of water
in the environment. Water vapours present in the atmosphere, in turn, come from the water
bodies such as lakes, streams, oceans, ponds, moist earth as well as from living organisms.
□ Of the total water present on the earth, oceans have 97% and only 3% of total water is
available as fresh water.
□ Water on earth is classified into following three types:
Fresh water: It is inland water and its salt content is less than 5 ppt or 0.5%. Examples: lakes, pools,
ponds etc. Hence, statement 2 is correct.
O Marine water: It occurs in seas and oceans with salt content of more than 35 ppt or 3.5%.
O Brackish water: Its salt content is more than 5 ppt but less than 35 ppt. For example, estuaries,
salt marshes and salt lakes. A lot of underground water in Rajasthan, Gujarat, Haryana and Punjab is
brackish. Hence, statement 1 is correct.
61. Ans. (D)
Sol. □ Aquatic ecosystems refer to plant and animal communities occurring in water bodies.
□ Sunlight and oxygen are the most important limiting factors for aquatic ecosystems
whereas moisture and temperature for terrestrial ecosystems. Hence, statement 1 is correct.
□ On the basis of light penetration and plant distribution the aquatic ecosystems are divided
into two zones:
Photic Zone: Upper most layer of aquatic ecosystems upto which light penetrates. Both photosynthesis and
respiration activity take place in this zone. Hence, statement 2 is correct.
Aphotic Zone: Lower layer of the aquatic ecosystems where light penetration and plant growth is restricted. Only
respiration takes place in this zone. Hence, statement 3 is not correct.
62. Ans. (C)
Sol. □ The Indo-Gangetic flood plain is the largest wetland system in India, extending from the
river Indus in the west to Brahmaputra in the east. This includes the wetlands of the Himalayan
terai and the Indo-Gangetic plains Hence, statement 1 is correct.
□ An estuary is a place where a river or a stream opens ntot e sea. It is a partially enclosed
coastal area at the mouth of the river where Its fresh water carrying fertile silt and runoff from
the land mixes with the salty sea water. It represents an ecotone between freshwater and
marine ecosystem and shows a variation of salinity due to mixing of seawater with fresh water.
Hence statement 2 is correct.
□ Estuaries are very dynamic and productive ecosystems since the river flow, tidal range
and sediment distribution is continuously changing in them. Examples of estuaries are river
mouths, coastal bays, tidal marshes and lagoons.
63. Ans. (A)
Sol. □ Eutrophication is a process when a water body becomes overly enriched with minerals
and nutrients which induce excessive growth of algae or algal bloom. This process also results
in oxygen depletion of the water body.
□ Eutrophication is of two types:
Natural Eutrophication: It occurs naturally over centuries as lakes get older and is filled in with
sediments and nutrients. It occurs when the flow of nutrients is on a seasonal basis.
Cultural Eutrophication: Human activities have accelerated the rate and extent of eutrophication
through both point-source discharges and non-point loadings of limiting nutrients, such as nitrogen
and phosphorus, into aquatic ecosystems with dramatic consequences for drinking water sources,
fisheries, and recreational water bodies. Hence, statement 1 is correct.
□ Oligotrophic lakes have very low nutrient content, thus low productivity and clear water,
with drinking water quality. Mesotrophic lakes have moderate nutrient content and Eutrophic
lakes are highly nutrient rich. Hence, statement 2 is not correct.

64. Ans. (B)


Sol. □ An algal bloom is a rapid increase or accumulation in the population of algae in freshwater
or marine water systems, recognized by the discoloration in the water from their pigments.
Hence, statement 1 is not correct.
□ All algal blooms are not harmful. Most blooms, in fact, are beneficial because the tiny
plants are food for animals in the ocean. In fact, they are the major source of energy that fuels
the ocean food web.
□ Harmful algal blooms, or HABs, occur when colonies of algae grow out of control while
producing toxic or harmful effects on people, fish, shellfish, marine mammals, and birds. HABs
are caused due to ‘overfeeding’. This occurs when nutrients from point and non- point
sources release into nearby water bodies and build up at a rate that ‘overfeeds’ the algae that e x i s t
normally in the environment. Hence, statement 2 is correct.
□ HABs are often associated with large-scale marine mortality events and have been
associated with various types of shellfish poisonings.
People often get sick from eating shellfish containing toxins produced by these algae.
Airborne HABs toxins may also cause breathing problems and, in some cases, trigger asthma
attacks in susceptible individuals.
65. Ans. (C)
Sol. □ An algal bloom is a rapid increase or accumulation in the population of algae in freshwater
or marine water systems and are recognized by the discoloration in the water from their
pigments.
□ Algal bloom results in colour change of water. Algal bloom can be of any colour but most
common ones are red and brown referred to as red or brown tides. Hence, statement 1 is
correct.
□ All algal blooms are not harmful. Most blooms, in fact, are beneficial because the tiny
plants are food for animals in the ocean. In fact, they are the major source of energy that fuels
the ocean food web. But some produce toxins that affect the aquatic ecosystems. The toxin
can even make the surrounding air difficult to breathe known as Harmful Algal Blooms (HABs).
Hence, statement 2 is correct.
66.
Ans. (B)
Sol. □ Montreux Record is a register of wetland sites on the list of Wetlands of International
Importance where changes in ecological character have occurred, are occurring, or are likely to
occur as a result of technological developments, pollution or other human interference. It is
maintained as part of the Ramsar List. Hence, statement 1 is not correct.
□ Sites may be added to and removed from the record only with the approval of the
contracting parties in which they lie. Chilika lake in Odisha
was added to Montreux Record, but removed later on. Hence, statement 2 is correct.
67. Ans. (C)
Sol. □ Wetlands are shallow water-bodies, transitional between terrestrial and aquatic systems,
with high biodiversity and productivity whereas lakes are relatively deep, perennial water
bodies where substantial growth of macrophytes is undesirable.
□ National Plan for Conservation of Aquatic Ecosystems (NPCA) is a single conservation
programme for both wetlands and lakes. Hence, statement 1 is correct.
□ NPCA aims to provide an integrated and scientific framework for protection,
conservationand sustainable management of wetlands in the
country.
□ It is a centrally sponsored scheme, implemented by the Union Ministry of Environment,
Forests and Climate Change.
□ It was formulated in 2015 by merging of the National Lake Conservation Plan and the
National Wetlands Conservation Programme. Hence, statement 2 is correct.
□ NPCA seeks to promote better synergy and avoid overlap of administrative functions.
68. Ans. (C)
Sol. □ Mangroves represent a characteristic littoral (near the seashore) forest ecosystem. These
forests grow in sheltered low lying coasts, estuaries, mudflats, tidal creek backwaters (current l e s s ,
coastal waters held back on land), marshes and lagoons of tropical and subtropical r e g i o n s .
Hence, statement 1 is correct.
□ They are distributed over the east and west coast and islands of Andaman and Nicobar.
Since mangroves are located between the land and sea, they represent the best example of the
ecotone.
□ Mangroves along the east coast are more luxuriant and considerably diverse due to the
presence of nutrient rich deltas formed by the rivers Ganga, Mahanadi, Godavari, Krishna and
Cauvery. Hence, statement 2 is not correct.
□ “Mangroves for the Future (MFF)” is a regional initiative, being coordinated by United
Nations Development Programme (UNDP) and International Union for the Conservation of
Nature (IUCN). Hence, statement 3 is not correct.
□ MFF is a unique partner-led initiative to promote investment in coastal ecosystem
conservation for sustainable development.
69. Ans. (A)
Sol. □ An estuary is a place where a river or a stream opens into the sea. It is a partially enclosed
coastal area at the mouth of the river where its fresh water carrying fertile silt and runoff from
the land mixes with the salty sea water.
□ Estuaries are very dynamic and productive ecosystems since the river flow, tidal range
and sediment distribution is continuously changing in them. Estuaries are richer in nutrients
than fresh waters or marine waters therefore; they are highly productive and support abundant
flora and fauna. The seashore provides feeding and breeding ground for a number of birds also.
Hence statement 1 is not correct.
□ Towards the sea coast of the estuaries there are large algae and sea grasses. Near the mouth
of the rivers and deltas there are mangrove forests. Hence, statement 2 is correct.
□ All the plants and animals in the estuaries are subjected to variations in salinity to which
they are adapted (osmoregulation).
70. Ans. (C)
Sol. □ Corals are made up of genetically identical organisms • called polyps. These polyps have
microscopic algae called zooxanthellae living within their tissues. The corals and zooxanthellae
have a mutualistic (symbiotic) relationship. Hence, statement 1 is correct.
□ The coral provides the zooxanthellae with the compounds necessary for photosynthesis.
□ In return, the zooxanthellae supply the coral with organic products of photosynthesis, like
carbohydrates, which | are utilized by the coral polyps for synthesis of their I calcium c a r b o n a t e
skeletons.
□ In addition to providing corals with essential nutrients, zooxanthellae are responsible for
the unique and beautiful colors of corals. Hence, statement 2 is correct.
□ Corals are also called the “rainforests of the seas”.
71. Ans. (D)
Sol. □ Phytoplanktons are tiny microscopic floating plants found in water bodies.
□ They are regarded as one of the Earth’s most critical organisms as they contribute more
than half of the oxygen in the environment. Hence, statement 3 is correct.
□ Phytoplankton is the base of several aquatic food webs. In a balanced ecosystem, they
provide food for a wide range of sea creatures.
□ They are important bioindicators regulating life in oceans. Their abundance determines
the overall health of the ocean ecosystem. Hence, statement 2 is correct.
□ As phytoplanktons are photosynthetic organisms, they convert inorganic carbon (of the
atmosphere and seawater) into organic compounds, making them an important part of the
carbon cycle of the Earth. Hence, statement 1 is correct.
72. Ans. (A)
Sol. □ Phytoplanktons (microalgae) are photosynthetic organisms, similar to terrestrial plants as
they contain chlorophyll and require sunlight in order to live and grow. These are buoyant in
nature and float in the upper part of the ocean, where sunlight penetrates the water. Hence,
statement 1 is not correct.
□ Phytoplankton is the base of several aquatic food webs. They are known as primary
producers of the ocean— the organisms that form the
base of the food chain. In a balanced ecosystem, they provide food for a wide range of sea
creatures including jellyfish, shrimp and snails. Hence, statement 3 is correct.
□ Due to high availability of the nutrients, phytoplankton may grow out of control and form
harmful algal blooms (HABs). These blooms can produce extremely toxic compounds that have
harmful effects on marine organisms, birds, and even humans. Hence, statement 2 is correct.
73. Ans. (D)
Sol. □ Plankton are the marine drifters, organisms which are carried along by tides and currents.
An organism is considered plankton if it is carried by tides and currents, and cannot swim well
enough to move against these forces. Hence, statement 1 is correct.
□ Plankton are usually microscopic, often less than one inch in length, but they also include
larger species like some crustaceans and jellyfish.
□ Scientists classify plankton in several ways, including by size, type, and how long they
spend drifting. But the most basic categories divide plankton into two groups i.e., phytoplankton
(plants) and zooplankton (animals).
□ Phytoplankton are microscopic plants of the sea that perform photosynthesis to convert
the sun’s rays into energy to support them, and they take in carbon dioxide and produce
oxygen. Phytoplankton are found near the surface of water as they need the sun’s energy for
photosynthesis. Hence, statement 3 is correct.
□ Zooplankton are typically the microscopic animals found near the surface in aquatic
ecosystems. Just like phytoplankton, zooplankton are generally weak swimmers and usually j u s t
drift along with the currents.
□ As many zooplankton species eat phytoplankton, shifts in timing or abundance of
phytoplankton can quickly affect zooplankton populations, which then affect species along the
food chain. Hence, statement 2 is correct.
74. Ans. (C)
Sol. □ The ocean biology and climate depends largely on the phytoplankton, any alteration in
their productivity could have a significant influence on fisheries, the human food supply and
biodiversity. It will also affect the pace of global warming.
□ Many models of ocean chemistry and biology predict that as the ocean surface warms in
response to increasing atmospheric greenhouse gases, phytoplankton productivity will decline.
Productivity is expected to drop because as the surface waters warm, the water column
becomes increasingly stratified; there is less vertical mixing to recycle nutrients from deep
waters back to the surface. Because larger plankton require more nutrients, they have a greater
need for the vertical mixing of the water column. Hence, statement 2 is correct.
□ Phytoplankton are responsible for most of the transfer of carbon dioxide from the
atmosphere to the ocean Carbon dioxide is consumed
during photosynthesis, and the carbon is incorporated in the phytoplankton.
□ Most of the carbon is returned to near-surface waters when phytoplankton are eaten or
decompose, but some falls into the ocean depths. Hence, statement 1 is correct.
75. Ans. (A)
Sol. □ Phytoplanktons are microscopic organisms that live in watery environments, both salty
and fresh.
□ Phytoplankton thrives along coastlines and continental shelves, along the equator in the
Pacific and Atlantic Oceans, and in high-latitude areas.
□ Phytoplanktons are most abundant in high latitudes and in upwelling zones along the
equator and near coastlines, where nutrient levels are high. Upwelling is a process in which deep,
cold water rises toward the surface. Hence, statement 1 is not correct.
□ Winds play a strong role in the distribution of phytoplankton because they drive currents
that cause deep water, loaded with nutrients, to be pulled up to the surface. Hence, statement 2
is correct.
76. Ans. (C)
Sol. □ Carbon cycle is the continuous exchange of carbon between the atmosphere and
Earth through different processes such as photosynthesis,
respiration, burning of fossil fuels, decomposition, etc.
□ It is available from the following three main sources -atmosphere, oceans (hydrosphere),
limestone, coal and petroleum.
□ The atmosphere contains about 0.03% to 0.04% carbon dioxide in free state. Green plants
use this carbon dioxide to synthesize food by the process of photosynthesis. The atmospheric
carbon taken in by the plants is transferred to animals in the form of food. From both plants a n d
animals, it is then passed on to the decomposers after their death.
□ The processes by which carbon dioxide is returned to the atmosphere are as follows:
By the process of combustion, i.e. burning of fuels like wood, coal, petroleum, etc.
By the process of respiration in plants, animals and decomposers
□ Carbon is the main constituent of the living matter. It is found in carbohydrates, fats,
proteins and nucleic acids that make up the living cell.
77. Ans. (B)
Sol. □ Primary producers (autotrophs) are basically green plants, certain bacteria and algae
that carry out photosynthesis. They are self-nourishing.
They synthesise carbohydrates from simple inorganic raw materials like carbon dioxide and
water in the presence of sunlight by the process of photosynthesis for themselves, and
supply indirectly to other non-producers (consumers).
□ In the terrestrial ecosystem, producers are basically herbaceous and woody plants, while in
the aquatic ecosystem; producers include bacteria, phytoplankton, and microscopic algae.
Hence, statement 1 is not correct.
□ Consumers (heterotrophs or phagotrophs) are incapable of producing their own food
(photosynthesis). They depend on organic food derived from plants, animals or both.
□ Decomposers are small consumers like bacteria, fungi and worms that cause the decay of
dead organisms. Some decomposers like fungi are Saprotrophic, which means they take in food
by absorbing dissolved organic substances that were products of organic decay. Hence,
statement 3 is correct.
□ Earthworms and certain soil organisms (such as nematodes, and arthropods) are detritus
feeders and help in the decomposition of organic matter and are called detritivores. Hence,
statement 2 is correct.
Solution Capter -2
78. Ans. (A)
Sol. □ The above mentioned species constitute the part of the Avian Ecology.
□ Ceylonfrogmouth is a grey-brown nocturnal bird species that is found in the Western
Ghats and Sri Lanka’s forested habitats.
It is distinguished by its wide, hooked bill with slit-like nostrils and a large head with eyes facing
forward.
With destruction of forest cover, this species is likely compelled to now take refuge in cashew
plantations.
□ Coppersmith barbet, also called crimson-breasted barbet and coppersmith, is an Asian
barbet with crimson forehead and throat. It is known for its metronomic call that sounds
similar to a coppersmith striking metal with a hammer.
It is a resident bird in the Indian subcontinent and parts of Southeast Asia.
□ White-throated redstart is a species of bird in
the Muscicapidae family.
It is found in Bhutan, China, India, Myanmar, and Nepal.
□ Grey-Chinned Minivet is a species of bird in the Campephagidae family.
It is found in Bangladesh, Bhutan, Cambodia, China, India, Indonesia, Laos, Malaysia, Myanmar, Nepal,
Taiwan, Thailand and Vietnam.
Its natural habitat is subtropical or tropical moist lowland forests.
79. Ans. (D)
Sol. □ Extinction means complete disappearance of a species, that is, not a single member of the
extinct species is found on the earth. It is an irreversible loss and is called biological e x ti n c ti o n .
Before a species goes biologically extinct, it goes through stages of local and ecological extinction.
□ Extinction occurs when species are diminished because of environmental forces
like habitat fragmentation, global change, natural disaster,
overexploitation of species for human use or because of evolutionary
changes in their members such as genetic inbreeding, poor reproduction, decline in population
numbers.
□ Other Reasons for Extinction
Endemism,
Restricted food source,
Pollution,
Migration,
Predation, competition, and disease.
80. Ans. (B)
Sol. □ The idea of Biosphere Reserve was initiated by UNESCO in 1975 as a part of UNESCO’s
‘Man and Biosphere Programme’, dealing with the conservation of ecosystems and the
genetic resources contained therein. Hence, statement 1 is not correct.
□ There are eighteen designated and notified Biosphere Reserves in India such as Nilgiri,
Nanda Devi, Nokrek, Great Nicobar, Gulf of Mannar, Manas, Sunderbans, etc. Hence, statement 2 is
correct.
□ 12 of the 18 biosphere reserves in India have been included in world network of Biosphere
Reserves of UNESCO.
81. Ans. (C)
Sol. □ In order to maintain biodiversity, some patches of forest are left untouched. These types of
forests are regarded as “sacred groves”. It is one among the various methods of biodiversity
conservation.
□ Sacred groves contribute in biodiversity protection whereas large scale deforestation,
exploitation of forest produces and encroachment in forest areas contribute in decrease in
biodiversity.
82. Ans. (A)
Sol. □ Asiatic lions were once found in regions ranging from Persia (Iran) to Eastern India.
By late 1890s range of Asiatic lions got restricted to the Gir forest range of Gujarat, India. Hence, statement 1 is
correct.
With the continuous efforts of State and Union Government, the population ofAsiatic lions increased from 50 in
the late 1890s to over 500 at present.
IUCN status of Asiatic Lion is Endangered.
□ The double-humped camel is a native of the Gobi Desert and is found on a vast expanse
of cold-desert areas across Mongolia, China, Kazakhstan, Turkmenistan, Uzbekistan and
parts of Afghanistan. Hence, statement 2 is not correct.
IUCN status of the double-humped camel is Critically Endangered.
□ The One-horned rhinoceros is found in North-Eastern India and the Terai grasslands of
Nepal. Hence, statement 3 is not correct.
IUCN status of the One-horned rhinoceros is Vulnerable.
83.
Ans. (D)
Sol. □ Green sea turtles are adapted to mostly vegetarian diet of sea grasses and algae. As adults,
these are the only predominantly herbivorous sea turtles, although they are carnivorous from
hatching until juvenile size. Hence, statement 1 is correct.
□ Surgeonfish and parrotfish are two species of fish often seen feeding on reef algae. Hence,
statement 2 is correct.
□ Manatees, sometimes called sea cows, are large mammals that live in the warm sea
waters. They live in shallow coastal areas and feed on sea vegetation. Hence, statement 3 is
correct.
□ Snakes that are viviparous nourish their young ones through a placenta and yolk sac. Boa
constrictors and green anacondas are two examples of viviparous snakes. Hence, statement 4
is correct.
84.
Ans. (C)
Sol. □ The Western Ghats region is famous for freshwater biodiversity. Mahseer, belonging to
the genus Tor, is of great cultural value. The Mahseer community comprises two varieties—a
“blue-finned” fish and an “orange-finned, hump backed” fish and are naturally found in
Cauvery river. It is listed as critically endangered in IUCN Red List. Hence, pair 1 is correctly
matched.
□ Irrawaddy Dolphins are found in coastal areas in South and Southeast Asia, and in the
three rivers- Ayeyarwady (Myanmar), Mahakam (Indonesia) and the Mekong. In India,
Irrawaddy Dolphin is found in Chilika lake. It is listed as critically endangered in IUCN Red List.
Hence, pair 2 is not correctly matched.
□ Forests of the Eastern Ghats are home to some endangered and lesser-known mammal
species like the Fishing Cat, Rusty Spotted Cat, Indian Pangolin, etc. Rusty Spotted Cat is listed
as near threatened in IUCN Red List. Hence, pair 3 is correctly matched.
85. Ans. (C)
Sol. □ Phylum Cnidaria (Coelenterata) is a group made up of more than 9,000 living species.
These are mostly marine animals including the corals, hydras, jellyfish, Portuguese man-
of-war, sea anemones, sea pens, sea whips, and sea fans.
□ There are two major body forms among the Cnidaria - the polyp (Hydra and Sea anemone)
and the medusa (jellyfish).
□ The phylum Cnidaria is made up of four classes:
Hydrozoa (hydrozoans)
Scyphozoa (scyphozoans)
Anthozoa (anthozoans)
Cubozoa (cubozoans)
86. Ans. (C)
Sol. □ Phreatophytes are the desert plants or the plants of arid environments, with long deep
roots that are capable of reaching the water table.
□ Some of the traits of Phreatophytes include:
rapid vertical root growth;
vigorous vegetative regeneration;
high hydraulic conductance;
high rates of biomass production; and
low water use efficiency.
□ Phreatophytes’ deep root system helps them to reach saturation zones to access water
during extended periods of drought. This is the reason why they are considered to be
hydro- ecological plant type.
87. Ans. (B)
Sol. □ The Ministry of Environment, Forests and Climate Change releases the India State of
Forest Report (ISFR). It is a biennial publication of the Forest Survey of India (FSI), an
organization under the Ministry of Environment, Forest, and Climate Change.
□ Forest cover includes all lands having trees more than one hectare in an area with tree
canopy density of more than 10%, irrespective of ownership, legal status of the land and
species composition of trees.
□ Largest forest cover in India in decreasing order: Madhya Pradesh > Arunachal Pradesh >
Chhattisgarh > Odisha > Maharashtra.
□ Forest cover as percentage of total geographical area in decreasing order as per ISFR 2019:
Mizoram (85.41%) > Arunachal Pradesh (79.63%) > Meghalaya (76.33%) > Manipur (75.46%) >
Nagaland (75.31%).
88. Ans. (A)
Sol. □ In saprophytic nutrition the organisms obtain their food from dead and decaying organic
matter of dead plants, dead animals and other decomposing organic matter. The organisms which
are dependent on saprophytic mode of nutrition are known as saprophytes.
□ Fungi, bread molds, some protists and many bacteria are saprophytic in nutrition. The
saprophytic organisms like fungi releases digestive enzymes in their surrounding medium t o
convert the complex organic molecules such as sugars in simple forms such as glucose.
□ This simple food is then absorbed through the body surface, and utilized for various
activities by fungus.
Agaricus is a saprophytic mushroom that grows on humus soil, decaying litter, wood logs and manure
piles. It grows best in moist and shady places and is commonly seen during rainy season.
The genus Agaricus comprises the most edible and widely consumed species. A. bisporus (A.
brunnescence; white mushroom), A. campestris (field mushroom) are common edible mushrooms.
Some species of Agaricus are poisonous and some may cause gastrointestinal disturbances in some
persons (e.g., A. placomyces, A. silvaticus).
□ Ulothrix is a genus of filamentous green algae (family Ulotrichaceae) found in marine and
fresh waters.
□ Riccia is a genus of liverworts in the order Marchantiales. These plants are small and
thalloid, that is not differentiated into root, stem and leaf.
□ Cladophora is a genus of reticulated filamentous Ulvophyceae (Green Algae).
89.
Ans. (D)
Sol. □ Warm-blooded animals are those animal species that have the ability to regulate and
maintain relatively constant internal body temperature. Generally, homeothermic species
maintain a stable body temperature by regulating metabolic processes.
□ The ability to maintain an internal temperature distinguishes these (warm-blooded
animals) from coldblooded, or poikilothermic animals, which usually have about the same
temperature as their environment. Warm-blooded animals are able to remain active in
situations in which cold-blooded ones cannot.
□ As crocodiles and tortoise belong to the reptile family they are not warm-blooded. Fishes
are also cold blooded and cannot regulate their temperature. Therefore, the only plausible pair is
peacock and camel.
90.
Ans. (D)
Sol. □ Taxonomy is the science of identification, classification and naming of living organisms.
Taxonomic work involves study of morphological characteristics and phylogenetic relationship
of organisms which is essential for applied biological sciences, such as medicine, agriculture,
forestry and fisheries.
□ In 1999, the Ministry of Environment, Forest and Climate Change initiated E.K. Janaki
Ammal National Award in the honor of late Prof. E.K. Janaki Ammal to encourage young
students and scholars to work in this field of science.
□ Late Prof. Janaki Ammal did taxonomic work of outstanding merit and excellence,
particularly in the area of cytotaxonomy and has been a source of inspiration to many young
scientists.
91. Ans. (A)
Sol. □ Species diversity is higher at the equator than at the poles. In biological terms, this is
referred to as the latitudinal diversity gradient (LDG), in which the number of species i n c r e a s e s
from the poles to the Equator. This ranks among the broadest and most notable biodiversity
patterns on Earth.
□ The pattern of species-rich tropics (e.g. lush rainforests teeming with ‘Life’) relative to
species-poor temperate and polar areas (e.g. barren polar deserts), has been recognized for o v e r
a century.
□ Ecological factors are commonly used to account for this gradation. Higher temperatures,
greater climate predictability, and longer growing seasons all conspire to create a more
inviting habitat, permitting a greater diversity of species. Tropical rainforests are the richest
habitat of all, tropical grasslands exhibit more diversity than temperate grasslands, and
deserts in tropical or subtropical regions are populated by a wider range of species than a r e
temperate deserts.
□ As an ecosystem, the Amazon is one of the most biodiverse places on earth. Over 3 million
species live in the rainforest, and over 2,500 tree species (or one-third of all tropical trees that
exist on earth) help to create and sustain this vibrant ecosystem.
92. Ans. (C)
Sol. □ The International Solar Alliance (ISA) is an initiative jointly launched by the India and
France on 30th November 2015 in Paris on the side-lines of the Conference of the Parties (COP-
21), with 121 solar resource-rich countries fully or partially between the Tropic of Cancer and of
Capricorn as prospective members.
□ Objective: To collectively address key common challenges to the scaling up of solar
energy in member countries and to mobilise over $1 trillion investment for the deployment o f
solar energy affordable costs.
□ Headquarters: National Institute for Solar Energy Gurugram (Haryana), India
93. Ans. (A)
Sol. □ Butterflies are the adult flying stage of certain insects belonging to an order or group
called Lepidoptera. Like all other insects, butterflies have six legs and three main body parts:
head, thorax (chest or mid-section) and abdomen (tail end). They also have two antennae a n d
an exoskeleton. The butterfly life cycle has four stages: egg, larva, pupa, and adult.
□ Butterflies are cold-blooded and cannot withstand winter conditions in an active state.
Few can fly in temperatures below 60 degrees. Although some species of butterflies will live
through freezing temperatures, they do not fly when it is cold. If they are wet in freezing
temperatures, they will die. Hence, Reason (R) is true.
□ Butterflies are distributed worldwide except Antarctica, totalling some 18,500 species.
Butterfly activities are closely controlled by weather and many species are constrained by
climate. As Butterflies are cold-blooded and cannot withstand winter conditions, maximum
numbers of the butterflies are found in tropical countries Hence, Assertion (A) is true.
94. Ans. (D)
Sol. □ Extensive and severe epidemics of plant diseases are mostly the results of wind blowing
including dust blowing which causes transmission of the pathogens. Hence, 2 and 4 are
correct.
□ Splashing rain drops mostly transmit the foliar diseases from leaf to leaf, from shoot to
shoot and even from plant to plant in case of closely spaced crops. Hence, 3 is correct.
□ Birds play a minor role in disease transmission in plants. It is mainly by way of dispersal
of seeds of higher flowering plant parasites, etc. Hence, 1 is correct.
95. Ans. (B)
Sol. □ Prosopis juliflora is a thorny evergreen tree with a large, flat-topped crown and open
canopy; it can grow to about 5-10 metres tall. It is a multi-purpose tree providing food,
medicines and many commodities; and is widely cultivated in many areas of the dry tropics for its
wide range of uses.
□ Range Countries or areas for Prosopis juliflora are Northern and Western South America -
Colombia, Ecuador, Venezuela and through Central America to Mexico.
□ Environmental conditions for its growth are light, sandy or gravelly soils in arid or semi-
arid areas where there is subterranean moisture. It is commonly found on over-grazed land.
□ It is an exotic species which was introduced in India to meet the needs of fuel and wood
for rural poor and to restore degraded lands. The plant turned invasive as it is threatening l o c a l
plant species and also affecting the nesting success of birds.
96. Ans. (B)
Sol. □ The National Green Tribunal, established in 2010 was set up under the 11th Five Year P l a n
(2007-2012) under the National Green Tribunal Act 2010. It is a specialized body equipped
with the necessary expertise to handle environmental disputes involving multi- disciplinary
issues.
□ The tribunal works for effective and expeditious disposal of cases relating
to environmental protection and conservation of forests and
other natural resources including enforcement of any legal right relating to environment and
giving relief and compensation for damages to persons and
property and for matters connected therewith or incidental thereto.
97. Ans. (A)
Sol. □ Coral Reefs often called the ‘rainforests of the sea’ are J one of the most diverse marine
ecosystems on the Earth.
□ Coral reefs in India are present in the areas of Gulf of Kutch, Gulf of Mannar, Andaman &
Nicobar, Lakshadweep I Islands and Malvan.
□ Coral reefs play an important role in marine ecosystem and support the habitats of flora a n d
fauna in the sea. They enable the formation of associated eco-systems which allow the formation
of essential habitats, fisheries and livelihoods.
□ Coral reef formation is not seen in the Gulf of Khambat/ | Cambay.
98.
Ans. (D)
Sol. □ Indus river dolphins are believed to have originated in the ancient Tethys Sea. When the
sea dried up approximately 50 million years ago, the dolphins were forced to adapt to its only
remaining habitat rivers.
□ Today, they can only be found in the lower parts of the Indus River in Pakistan and in R iver
Beas, a tributary of the Indus River in Punjab, India.
□ They have adapted to life in the muddy river and are functionally blind. They rely on
echolocation to navigate, communicate and hunt prey including prawns, catfish, and carp.
99. Ans. (A)
Sol. □ The British biologist Norman Myers coined the term “biodiversity hotspot” in 1988 as
a biogeographic region characterized both by exceptional levels of plant endemism and by
serious levels of habitat loss. Biodiversity hotspots are identified by the International Union f o r
Conservation of Nature (IUCN). Hence, statement 1 is correct.
□ To qualify as a biodiversity hotspot, an area must meet two strict criteria:
Contain at least 1,500 species of vascular plants found nowhere else on Earth (known as “endemic” species).
Have lost at least 70% of its primary native vegetation. Thus, Biodiversity hotspots are defined according to their
vegetation. Hence, statement 2 is correct.
□ There are four biodiversity hotspots in India:
Himalaya: Includes the entire Indian Himalayan region (and that falling in Pakistan, Tibet, Nepal, Bhutan, China
and Myanmar).
Indo-Burma: Includes entire North-eastern India, except Assam and Andaman group of Islands (and Myanmar,
Thailand, Vietnam, Laos, Cambodia and southern China).
100.
Ans. (B)
Sol. □ The Olive ridley turtles are the smallest and most abundant of all sea turtles found in the
world, inhabiting warm waters of the Pacific, Atlantic and Indian oceans. Hence, statements 1
and 2 are correct.
□ They are best known for their unique mass nesting called Arribada. It is a mass-nesting
event when thousands of turtles come ashore at the same time to lay eggs on the same.
□ The enormous number of Olive ridleys nest in Odisha (namely at three river mouths:
Dhamara, Devi and Rushikulya), India on the Utkal Plains and not on the Coromandel Coast. Hence,
statement 3 is not correct.
□ The species is listed as Vulnerable in the IUCN Red List, Appendix I in CITES, and
Schedule I in Wildlife Protection Act, 1972.
□ Olive-ridleys face serious threats across their migratory route, habitat and nesting beaches
due to human activities such as unfriendly turtle fishing practices, development and exploitation
of nesting beaches for ports, etc.
101. Ans. (B)
Sol. A National Centre for Marine Biodiversity (NCMB) is situated in Jamnagar, Gujarat. The centre has the
goal of safeguarding the biodiversity of the coastal area. It is the first research institute set up in a
public private partnership in the country.
102. Ans. (D)
Sol. □ Giraffe: It will go up to 3 weeks without drinking water, When it drinks, it has been known
to down 12 gallons in one sitting.
□ Camel: It can go up to seven months in the desert without drinking water. During such a
time, it may lose nearly half of its body weight.
□ Kangaroo: It needs very little water to survive and is capable of going for months without
drinking at all. It usually rests in the shade during the day and comes out to eat in the late
afternoon and night when it is much cooler. It eats mostly grass. It needs very little water t o
survive.
□ Kangaroo Rat: This little animal can last longer without drinking water than any other. It
goes its whole life (3-5 years) without drinking any.
They almost require no water and their diet consists of mostly dry seeds. The survival of Kangaroo rats depends
entirely on the metabolised water, obtained from seeds that are eaten.
103. Ans. (B)
Sol. □ The process of protecting an endangered plant or animal species in its natural habitat is
commonly known as in-situ conservation.
□ The national parks, wildlife sanctuaries and biosphere reserves are some of the examples
of in-situ conservation. This method of conservation allows animals to flourish in their
natural habitat and food chain and offers more mobility to the animals. It is suitable for the
conservation of animals that are found in abundance.
□ Ex-situ conservation, also known as off-site conservation, is the relocation of endangered
or rare species from their natural habitats to protected areas equipped for their protection a n d
preservation that imitate their natural habitats, e.g. zoo, aquarium, botanical garden, etc.
It offers less mobility to the animals as it is smaller in area than the area of in-situ conservation. This
method of conservation is suitable for the animals which are not found in abundance.
104. Ans. (B)
Sol. □ The United Nations has proclaimed May 22nd, the International Day for Biological
Diversity (IDB) to increase understanding and awareness of biodiversity issues. When first
created by the Second Committee of the UN General Assembly in late 1993,29th December
(the date of entry into force of the Convention of Biological Diversity), was designated the
International Day for Biological Diversity.
□ In December 2000, the UN General Assembly adopted 22nd May as IDB, to commemorate
the adoption of the text of the Convention on 22nd May 1992 by Nairobi Final Act of the
Conference for the Adoption of the Agreed Text of the Convention on Biological Diversity.
□ Themes of last few International Day for Biological Diversity (IDB) are:
2021 - We’re part of the solution
2020 - Our solutions are in nature
2019 - Our Biodiversity, Our Food, Our Health
2018 - Celebrating 25 Years of Action for Biodiversity
2017 - Biodiversity and Sustainable Tourism.
105.
Ans. (B)
Sol. □ Project Tiger is an ongoing centrally sponsored scheme of the Ministry of Environment,
Forests and Climate Change providing central assistance to the tiger states for tiger conservation
in designated tiger reserves.
□ The Government of India has taken a pioneering Initiative for conserving its national
animal, the tiger, by launching the “Project Tiger” in 1973. From 9 tiger reserves since its formative
years, the Project Tiger coverage increased to 51 at present, spread out in 18 of tiger range
states. This amounts to around 2.21% o geographical area of our country.
□ The tiger reserves are constituted on a core/buffer strategy. The core areas have the legal
status of a national park or a sanctuary, whereas the buffer or peripheral areas are a mix of forest
and non-forest land, managed as a multiple use area. The Project Tiger aims to foster an
exclusive tiger agenda in the core areas of tiger reserves,
with an inclusive people oriented agenda in the buffer.
□ The National Tiger Conservation Authority (NTCA) is a statutory body of the Ministry of
Environment, Forests and Climate Change with an overarching supervisory / coordination role,
performing functions as provided in the Wildlife (Protection) Act, 1972.
106. Ans. (A)
Sol. □ Wildlife Protection Act, 1972 (WPA) provides for 6 schedules and each Schedule
guarantees varying degrees of protection to animals and plants. Animals include insects,
reptiles, fishes, birds and mammals.
□ Schedule I and part II of Schedule II lists animals which are provided with absolute
protection. Offences against animals of these schedules are prescribed with the highest
penalties.
□ Schedule III and Schedule IV also protect some animals, but in case of offences against
these animals, the penalties are lower as compared to Schedule I and part II of Schedule II.
□ Schedule V animals are called “vermin” which can be hunted. According to Section 62 of
WPA, the Central Government may, by notification, declare any wild animal other than those
specified in Schedule I and part II of Schedule II to be vermin for any area and for such period
as may be specified therein.
□ Schedule VI was inserted via amendment to WPA in 1991 to prohibit cultivation, collection,
extraction, trade, etc, of plants and its derivatives in the list.
□ Tiger is enlisted in Schedule I of WPA, 1972. Thus, if a species of tortoise is declared
protected under this Schedule, it will enjoy the same level
of protection as that of the tiger.
107. Ans. (B)
Sol. □ M-STrlPES (Monitoring System for Tigers: Intensive Protection and Ecological Status) w a s
developed by the National Tiger Conservation Authority and the Wildlife Institute of India in 2010.
□ It is a platform where modern technology is used to assist effective patrolling, assess
ecological status and mitigate human-wildlife conflict in and around tiger reserves.
□ The M-STrlPES program uses Global Positioning System (GPS), General Packet Radio
Services (GPRS), and remote sensing, to collect information from the field, create a database
using modern Information Technology (IT) based tools, analyses the information using
Geographic Information System (GIS) and statistical tools to provide inferences that allow ti g e r
reserve managers to better manage their wildlife resources.
108. Ans. (B)
Sol. □ National Chambal Sanctuary, also called the National Chambal Gharial Wildlife
Sanctuary, is a 5,400 sq km tri-state, namely Madhya Pradesh, Rajasthan and Uttar Pradesh
protected area in northern India for the protection of the critically endangered gharial, the r e d -
crowned roof turtle and the endangered Ganges river dolphin.
□ The gharial (Gavialis gangeticus), also known as the gavial, and fish-eating crocodile is a
crocodilian in the family Gavialidae, and is native to the northern part of the Indian subcontinent.
□ The global wild gharial population is estimated at fewer than 235 individuals, which are
threatened by loss of riverine habitat, depletion of fish resources, and entanglement in fishing
nets. As the population has declined drastically since the 1930s, the gharial is listed as
critically endangered on the IUCN Red List.
109. Ans. (D)
Sol. □ Gharial, Indian wild ass and Wild buffalo all are listed under Schedule I of the Wildlife
(Protection) Act 1972. The Wildlife (Protection) Act 1972 prohibits hunting of any animal
enlisted in Schedule I of the Act except under some provisions provided by law. Hence, 1, 2
and 3 are correct.
□ Further, the section 11 of the Act states that the Chief Wildlife Warden may, if he is
satisfied that any wild animal specified in Schedule I has become dangerous to human life or is
so disabled or diseased as to be beyond recovery, by order in writing and stating the reasons
therefore, permit any person to hunt such animal or cause such animal to be hunted. The killing
or wounding in good faith of any wild animal in defence of oneself or of any other person shall
not be an offence.
110. Ans. (D)
Sol. □ Kaziranga National Park is a stretch known for tall elephant grass, dense tropical broadleaf
forests crisscrossed by the Brahmaputra river and the Diphlu river. In addition to this, the park
also passes through the NH-37.
□ Till now the railway track of Indian railways does not run through the Kaziranga national
park, Kamakhya temple and the Manas National Park, all in Assam.
111 Ans. (A)
Sol. □ Coral reefs are large underwater structures composed of the skeletons of colonial marine
invertebrates called coral. These are some of the most diverse ecosystems in the world,
therefore, regarded as the “the rainforests of the sea” for their biodiversity. Coral polyps a r e
the animals primarily responsible for building reefs and can take many forms.
□ Corals need certain conditions to survive including:
Warm water temperature: Reef-building corals require warm water conditions to survive.
Corals generally live in water temperatures of 68-90° F or 20-32° C. However, different corals living in
different regions can withstand various temperature fluctuations. Hence, 4 is correct.
Sunlight: Corals need to grow in shallow water where sunlight can reach them. Corals rarely develop
in water deeper than 165 feet (50 meters). Hence, 1 is correct.
Clean and clear water: Corals need clean and clear water that lets sunlight pass through, they do not
thrive well when the water is opaque. Corals are sensitive to pollution and sediments. Hence, 2 is
correct.
Saltwater: Corals need saltwater to survive and require a certain balance in the ratio of salt to water
(average salinity of the ocean should be 3.5%). Hence, 3 is not correct.
112. Ans. (B)
Sol. □ Gir National Park, was established in 1965 at the Saurashtra region of Gujarat. It is the
only place where endangered Asiatic lions are found in its natural habitat.
□ The National park is also home to “Maldhari Community” a pastoralist community who
have lived in symbiotic relationship with the lions.
113. Ans. (C)
Sol. □ In 2016, the Ministry of Environment, Forest and Climate Change constituted a four-
member committee for conservation of Loktak lake in Manipur and also assessed steps
required to make it a UNESCO World Heritage site.
□ Loktak lake situated in Manipur, is the largest freshwater lake in the north-eastern region
of the country.
□ The lake is also famous for Keibul Lamjao National Park, the only floating national park in
the world and hosts endangered Manipur Brow Antlered Deer, locally known as Sangai.
114. Ans. (D)
Sol. □ The fossil fuels are non-renewable sources of energy. The end product of fossil fuels can
be in the form of chemical energy (light and heat), Mechanical energy (running of machine) or
electrical energy (generation of electricity).
□ Watershed is defined as any surface area from which runoff resulting from rainfall is
collected and drained through a common point. Watershed protection is a means of protecting a
lake, river, or stream by managing the entire watershed that drains into it. It aims to rejuvenate
streams, rivers, lakes, wetlands and bays. Hence, the rate of surface run- off is controlled or
minimized.
□ Over harvesting of timber results in decrease of ecological function of the forest such
as carbon storage, nutrient cycling, water and air purification, and maintenance of wildlife
habitat.
□ When a river flows at a relatively greater speed it erodes and carries with it silt and
sediments which are deposited in the floodplain when the flow of the river slows down.
However, during the flood the silt causes the level of the riverbed to rise. As a result, the
natural longitudinal (straight) course of the river is disturbed. Therefore, the river searches for a
lateral path (left or right). As a result, it changes its course. Further, while changing its course at
the time of flood it starts eroding the new paths as the flow speed is higher and drains the
sediments into the sea as a result lots of fertile soils are lost.
115. Ans. (A)
Sol. □ Biodiversity is a term used to describe the various diversities and varieties of life present
on Earth. This includes human beings, plants, animals, bacteria.
□ It plays an important role in the way ecosystems function and in the services they provide.
□ Biodiversity includes three main types:
Diversity within species (genetic diversity);
Diversity between species (species diversity) and;
Diversity between ecosystems (ecosystem diversity).
116. Ans. (B)
Sol. □ Plant biomass is the weight of living plant material contained above and below a unit of
ground surface area at a given point in time.
□ Biomass contains stored chemical energy from the sun. Plants produce biomass through
photosynthesis. Biomass can be burned directly for heat or converted to renewable liquid and
gaseous fuels through various processes.
□ The Tropical rainforest has the maximum plant biomass value, around 400 to 700 metric
tons per hectare, greater than most temperate forests and substantially more than other
vegetation with fewer or no trees.
117. Ans. (A)
Sol. Biodiversity is defined as the variety and variability of life on Earth. It is subject to many threats.
The greatest threat is due to destruction of natural habitats and vegetation. Population increase
and various other anthropogenic activities have increased the threat to biodiversity. Other threats
to biodiversity like Improper agricultural operations, Climate change, water pollution, invasive alien
species etc., have also threatened its sustainability.
118. Ans. (D)
Sol. □ Maharashtra became the first State in the country to have a ‘State Butterfly’. It declared
the Blue Mormon (Papilio polymnestor) as the State Butterfly.
□ It is the second largest butterfly in India after the Troides minos commonly known as the
Southern Birdwing.
□ It is found only in Sri Lanka, Western Ghats of Maharashtra, South India and coastal belts
of the country.
□ It has velvet and black wings with bright blue spots.
119. Ans. (A)
Sol. □ Red Sanders has a highly restricted distribution in the southeastern portion of Indian
peninsula to which it is endemic. Hence, statement 1 is correct.
□ Red Sanders (Pterocarpus santalinus) occurs in the forest formation which is classified as
Southern Tropical Dry Deciduous Forests. Hence, statement 2 is not correct.
□ The tree is endemic to several districts in Andhra Pradesh and some parts of Tamil Nadu
and Karnataka.
□ It is known for its rich hue and therapeutic properties. It is high in demand across Asia,
particularly in China and Japan, for use in cosmetics and medicinal products as well as for
making furniture, woodcraft and musical instruments.
□ In the 1980s, the Union government recommended inclusion of Red Sanders in Appendix
II of CITES. It was listed in Appendix II of CITES in 1995, and subsequently exports of Red Sanders
has been prohibited since 2004.
120. Ans. (A)
Sol. □ Kharai camel is found in coastal parts of the Kachchh region of Gujarat. These camels are
well adapted to both dry-lands as well as coastal ecosystems.
□ They have an excellent swimming capacity in sea water.
Hence, 1 is correct.
□ They graze mainly on mangrove and other saline species.
Hence, 2 is correct.
□ Kharai camel can thrive on high saline water and tolerate high total dissolved solids.
□ Kharai camel is one of the most preferred choices of grazers in the arid coastal region of
Kachchh. People consume its milk, while male calves are sold for economic returns. Hence, 3
is not correct.
121. Ans. (A)
Sol. □ A new species of banana, Musa idandamanensis was discovered by a team of scientists
from the Botanical Survey of India (BSI) from a remote Krishna Nalah tropical rain forest o n
the Little Andaman Islands.
□ Its flowers are cylindrical in shape compared to the conical shape of regular banana
species.
□ It is about 11 metres high, whereas as the usual banana species is about three to four
metres high.
□ The species is edible and very sweet. The fruit pulp is orange in colour, distinctive from
the white and yellow colour of regular bananas
122. Ans. (D)
Sol. □ Great Indian Hornbills are large and wide-ranging birds and most species are dependent
on tropical forest habitats that contain large and tall trees.
□ India has nine Hornbill species, of which four are found in the Western Ghats − Indian
Grey Hornbill (endemic to India), Malabar Grey Hornbill (endemic to the Western Ghats),
Malabar Pied Hornbill (endemic to India and Sri Lanka) and endangered Great Indian Hornbill.
□ India also has one species that has one of the smallest ranges of any Hornbill - the
Narcondam Hornbill, found only on the island of Narcondam in Andaman Sea.
123. Ans. (B)
Sol. □ Germplasm are living genetic resources such as seeds or tissues that are maintained for
the purpose of animal and plant breeding, preservation, and other research uses. These
resources may take the form of seed collections stored in seed banks, trees growing in
nurseries, animal breeding lines maintained in animal breeding programs, egg and sperm
repository or gene banks, etc. Hence, 1, 2 and 3 are correct.
□ Germplasm collections can range from collections of wild species to elite, domesticated
breeding lines that have undergone extensive human selection. Germplasm collection is
important for the maintenance of biological diversity and food security. Hence, 4 is not
correct.
124. Ans. (C)
Sol. □ Loktak Lake is not only the largest freshwater lake in northeast India, but also the only
floating national park in the world. Hence, statement 1 is correct.
□ Loktak Lake is a home to unique floating islands called “phumdis”. These
circular landmasses are made of vegetation, soil, and organic matter (at different stages of
decomposition) that have been thickened into a solid form. Like an iceberg, most of the mass
of phumdis lies below the water surface. During the dry season, when water levels drop, the
living roots of the islands can reach the lakebed and absorb nutrients. Hence, statement 2 is correct.
□ Speckled across the Loktak Lake, the several thousand phumdis and its surrounding waters
are vital for irrigation, drinking water, food supplies, thus the lake has been referred as the
“lifeline of Manipur” state. Thousands of fishermen make their livelihood in the waters,
catching about 1,500 tons (6.6 million pounds) of fish every year.
125. Ans. (D)
Sol. □ The main cause for loss of biodiversity can be attributed to the influence of human beings
on the world s ecosystem. Human beings have deeply altered the environment and have
modified it, by exploiting the species directly, for example by fishing and hunting, changing the
biogeochemical cycles and transferring species from one area to another of the planet.
□ Habitat destruction is a major cause for biodiversity loss. Species which are physically large
and those living in forests or oceans are more affected by habitat destruction.
126. Ans. (C)
Sol. □ A biodiversity hotspot is a region with a high level of endemic species that is under threat
from humans. The term hotspot was introduced in 1988 by Norman Myers. The criteria for
determining a hotspot are:
The area should support >1500 endemic species.
It must have lost over 70 % of the original habitat.
Hence, statement 1 is correct.
□ The overall richness of species is concentrated in equatorial regions and tends to decrease
as one moves from equatorial to the Polar Regions. In addition, biodiversity in land e c o s y s t e m s
generally decreases with increasing altitude. Hence, statement 2 is correct.
□ Biodiversity Conservation is the planned management of biotic resources to retain the
balance in nature and retain the diversity. Biodiversity conservation is in-situ as well as ex- situ
which are as follows:
In-situ (on-site) Conservation: It includes the protection of plants and animals within their natural
habitats or in protected areas.
Ex-situ (off-site) Conservation: It includes protection of plants and animals outside their natural
habitats. These include botanical gardens, zoo and gene banks, seed bank, tissue culture and
cryopreservation. Hence, statement 3 is correct.
127. Ans. (C)
Sol. □ Ganges river Dolphin or Gangetic Dolphin is the National Aquatic Animal of India. The
Ganges river dolphin was officially discovered in 1801. It inhabits parts of the Ganges, M e g h n a
and Brahmaputra rivers in India, Nepal, Bhutan and Bangladesh, and the Karnaphuli River in
Bangladesh.
□ It is listed as endangered in IUCN Red List and has been included in the Schedule I of the
Wildlife (Protection) Act, 1972.
□ The main factors for decline in population of the species are poaching and habitat
degradation due to declining flow, heavy siltation, construction of barrages causing physical
barrier for this migratory species.
128. Ans. (C)
Sol. □ Dugong is a sea grass eating marine mammal found in shallow coastal waters of the Indian
and Western Pacific Oceans. It is also called Sea Cow. If there is not enough sea grass to eat,
then the dugong does not breed normally. They are hunted for their meat and oil. Hence,
statement 1 is correct.
□ It is found only in coasts along the Gulf of Kuchchh, Gulf of Mannar and Andaman and
Nicobar Islands. Hence, statement 2 is not correct.
□ Dugongs are threatened by sea grass habitat loss or degradation because of coastal
development or industrial activities that cause water pollution. It is protected under Schedule I of
Wildlife Protection Act, 1972. Hence, statement 3 is correct.
129. Ans. (A)
Sol. □ Changthang, the highest permanently inhabited plateau in the world is home to an
extremely hardy and exotic breed of goat- the Changra, or Pashmina goat. The luxurious fi b e r
of the goat is known as Pashmina, the softest and most valuable type of cashmere wool in the
world.
□ Angora fibre is the downy coat produced by the Angora rabbit. It is reared for the long
fibres of its coat, called Angora Wool.
130. Ans. (A)
Sol. □ The Supreme Court has allowed the reintroduction of African Cheetahs from Namibia into
the Indian habitat after hearing a plea brought by the National Tiger Conservation A u t h o r i t y
(NTCA).
□ The wildlife experts have identified three regions which have the potential
to support cheetah populations. The Nauradehi
and Kuno-Palpur Wildlife Sanctuaries in Madhya Pradesh and the Shahgarh Landscape
in Rajasthan have been declared potentially suitable for the reintroduction of the cheetah.
□ In 1952, the Asiatic Cheetah was officially declared extinct from India.
131. Ans. (C)
Sol. □ Jaguar: They are naturally found from northern Mexico southward to northern Argentina.
The largest know population of Jaguar exists in the Amazon rainforest. It is listed as “Near
Threatened” on the IUCN Red List of Threatened Species. Hence, 1 is not correct.
□ Saltwater crocodile: The saltwater crocodile is the largest of all crocodilians, and the
largest reptile in the world.
Apart from the eastern coast of India, the saltwater crocodile is extremely rare on the Indian
subcontinent A large population is present within the Bhitarkanika Wildlife Sanctuary of Odisha while
smaller populations occur throughout the Sundarbans. It is listed as “Least Concern” on the IUCN Red
List of Threatened Species. Hence, 2 is correct.
□ Snow leopard: They inhabit mountainous regions of central and southern Asia. In India,
their geographical range encompasses a large part of the western Himalayas including,
Ladakh, Himachal Pradesh, Uttarakhand, and Sikkim and Arunachal Pradesh in the eastern
Himalayas. Hence, 3 is correct.
The snow leopard is listed as “Vulnerable” on the IUCN Red List of Threatened Species.
132. Ans. (A)
Sol. □ Common Krait: Popularly known as the Indian Krait, it is a highly venomous snake
commonly found throughout Peninsular India. Common Krait is responsible for high frequency of
snakebites in India. Hence, 1 is correct.
□ Russell’s Viper: It is a highly venomous terrestrial snake of the family Viperidae. Being
found from India to Taiwan and Java, it is the prime reason behind snakebite deaths within i t s
range because it often exists in farmlands where human contact and rodent prey are a b u n d a n t .
Hence, 2 is correct.
□ Rock Python: It is one of two subspecies of Python molurus that exist. The Indian python
is found in Pakistan, Nepal and Sri Lanka apart from India. Similar to all boas, Rock Python is
not venomous and kills prey by constriction, wrapping around them. Hence, 3 is not correct.
133. Ans. (A)
Sol. □ Adjutant stork: It is a typical scavenger with naked pink skin on the head and neck. In
India, the Greater Adjutant is confined to the northeastern state of Assam. It is listed as
“Endangered” on the International Union for the Conservation of Nature (IUCN) Red List o f
Threatened Species. Hence, 1 is correct.
□ Emperor penguin: These are largest of all penguins and are found in Antartica. They are
listed as “Near Threatened” as per the IUCN Red List of Threatened Species. Hence, 2 is n o t
correct.
□ Rattle snake: It is characterized by a segmented rattle at the tip of the tail that produces
a buzzing sound when vibrated. They inhabit the region from southern Canada to central
Argentina. They are listed as “Least Concern” as per IUCN Red List of Threatened Species.
Hence, 3 is not correct.
134. Ans. (A)
Sol. □ The tailless Hoolock Gibbon is the only ape found in India. The primate inhabits forests
in north- east India and has two types; the Western Hoolock Gibbon and the Eastern Hoolock
Gibbon.
□ As per the International Union for Conservation of Nature (IUCN) Red List; Western
hoolock gibbon is listed as Endangered whereas Eastern hoolock gibbon is listed as Vulnerable.
□ In India, both the species are listed on Schedule I of the Indian (Wildlife) Protection
Act,1972.
□ Being found in all the states of the north-east, restricted between the south of the
Brahmaputra river and east of the Dibang river, the western hoolock gibbon has a much wider
range.
□ The eastern hoolock gibbon can be seen in specific pockets of Arunachal Pradesh and
Assam in India.
135. Ans. (B)
Sol. □ Ibex is a sturdy wild goat that belongs to the mountains of Europe, Asia and north-eastern
Africa. The Indian Ibex, particularly found in the Himalayas, is larger and has a longer beard and
horns.
□ The Ibex has the ability of crossbreeding with domestic goats and with wild goats. It has
very good climbing skills but avoids deep snow. If the lower branches are sturdy enough to ca r r y
its weight, it can climb trees to browse foliage.
□ Siberian Ibex is a species of wild goat, distributed in diverse habitats, ranging from cold
deserts, rocky outcrops, steep terrain, high-land flats and mountain ridges to low mountains a n d
foothills.
□ In Asia, Ibex is distributed in the Montane habitats, ranging in elevations from 500 m to
6,700 m in countries like India, Kazakhstan, Tajikistan, Mongolia, Pakistan, Southern Siberia and
China. Its IUCN status is least concerned.
□ Earlier the Himalayan Ibex (Capra sibirica hemalayanus) was regarded as a subspecies of
the Siberian Ibex (Capra sibirica) but study by scientists of the Zoological Survey of India (ZSI)
has proved that Himalayan Ibex is a distinct species from the Siberian Ibex.
□ The Himalyan Ibex is distributed mainly in the trans-Himalayan ranges of the Union
Territories of Ladakh and Jammu and Kashmir and Himachal Pradesh.
136. Ans. (B)
Sol. □ Dugong is a species of sea cow, found throughout the warm latitudes of the Indian and
western Pacific Oceans. Similar to all sea cows, the dugong is herbivorous, primarily grazing o n
sea grasses and, therefore, spending most of its time in seagrass beds. It never enters in
freshwater and is, therefore, the only exclusively marine mammal that is herbivorous.
□ They are listed as Vulnerable on the IUCN Red List and are protected in India under
Schedule I of the Wildlife Protection Act, 1972.
□ According to a 2013 survey report of the Zoological Survey of India (ZSI), there were only
about 200 dugongs in the Gulf of Mannar in Tamil Nadu, the Andaman and Nicobar Islands a n d
the Gulf of Kutch in Gujarat.
□ In 2020, India hosted the 13th Conference of Parties (CoP) of the Convention on the
Conservation of Migratory Species of Wild Animals (CMS), an environmental treaty under t h e
aegis of the United Nations Environment Programme (UNEP) for conservation and management
of Dugongs, and many other species.
□ World Dugong Day is celebrated on 28th May, every year.
137. Ans. (C)
Sol. □ The Sperm whale (Physeter macrocephalus) is the largest of the toothed whales and is also
the largest toothed predator.
□ The species is protected by a whaling moratorium and is currently listed as ‘vulnerable’ by
the International Union for the Conservation of Nature and Natural Resources (IUCN).
138. Ans. (A)
Sol. □ There are over forty national parks in Israel ranging from archaeological sites such as
Caesarea National Park and Masada to sites of natural beauty such as the Carmel National Park.
□ According to the latest data, Zambia has the highest percentage of its total area under
national parks at 32% followed by Israel at 30%.
139. Ans. (B)
Sol. □ Biodiversity is referred to as the variation of plant and animal species in a particular
habitat. India is known for its rich biodiversity and has around 24.56% of the geographical
area covered by forests and trees. The Western Ghats and Eastern Himalayas are identified
as biodiversity hotspots in India.
□ The Western Ghats are present along the western edge of peninsular India and covers
most of the deciduous forests and rainforests. As per UNESCO, it is home to at least 325
globally threatened flora, fauna, bird, amphibian, reptile and fish species.
140.
Ans. (B)
Sol. □ Biodiversity boosts ecosystem productivity, where each species, no matter how small, has
an important role to Play.
□ Productivity refers to the rate of generation of biomass in the ecosystem. More the
productivity of the ecosystem, the more diverse it is.
□ The intermediate disturbance in an ecosystem is basically a hypothesis that states that
moderate levels of disturbance can create conditions that foster greater species diversity than
low or high levels of disturbance. It does not help in increasing the biodiversity of the ecosystem.
□ The older an ecosystem, the more diverse it is in its nature. It is because they are more
resistant to change exhibiting the importance of the age of the ecosystem. Also, the limited
number of trophic levels in an ecosystem tends to increase biodiversity as there is less
competition between the levels, less energy loss; which increases the biodiversity.
141.
Ans. (D)
Sol. □ The first ever Census of dolphins was conducted during the year 2015 along the coasts of
Odisha.
□ Apart from Chilika and Bhitarkanika, the two coastal ecosystems, the survey was
conducted off the coast in Bhadrak, Balasore, Berhampurand Puri.
□ The dolphin census carried out for the first time at the Bhitarkanika wildlife sanctuary
recorded at least six species of dolphins in the ecosystem: Hump Back, Irrawaddy, Finless
Porpoise, Bottle Nose, and Pan Tropical Spotted dolphins.
142.
Ans. (A)
Sol. □ India with its coastline extending over 7,500 kilometres and subtropical climatic
conditions has very few coral reef areas.
□ The major coral reef formations in India are;
Gulf of Mannar; hence, 3 is correct.
Palk Bay,
Gulf of Kuchchh; hence, 2 is correct.
Andaman and Nicobar Islands; hence, 1 is correct.
Lakshadweep Islands
□ While the Lakshadweep reefs are atolls, all the others are fringing reefs. Patchy coral is
present in the intertidal areas of the central west coast of the country.
□ Coral reefs require clean and clear water, warm surface water and sunlight to survive.
Since, most of these requirements are not met in Sunderbans region, coral reefs are not found
here. Other disadvantage to reef growth are the heavy monsoonal rains and the high human
presence on the coastline. Hence, 4 is not correct
143. Ans. (C)
Sol. □ Hibernation is a way for many creatures to survive cold, dark winters without having to
forage for food or migrate to somewhere warmer. Instead, they turn down their metabolism t o
save energy.
□ Animals practicing hibernation in winter:
Bats; hence, 1 is correct.
Bears; hence, 2 is correct.
Rodents; hence, 3 is correct.
Alpine Marmot
Snail
Garter Snake
Common Poorwill
Common Box Turtle
Bumblebees
□ Animals in hot climates also undergo a form of hibernation called aestivation. This works
in a similar way and enables them to survive extreme heat, drought or lack of food.
144. Ans. (C)
Sol. □ Habitat of Ganges River Dolphins is freshwater mainly the Ganges-Brahmaputra-Meghna
and Karnaphuli-Sangu river systems of Nepal, India, and Bangladesh. They are essentially blind.
They hunt by emitting ultrasonic sounds, which bounces off of fish and other prey, e n a b l i n g
them to “see” an image in their mind. They are frequently found alone or in small groups, and
generally a mother and calf travel together. Calves are chocolate brown at birth and then have
grey-brown smooth, hairless skin as adults. Females are larger than males and give birth once
every two to three years to only one calf.
□ As per the study conducted by WWF-lndia, the causes of decline in population of Gangetic
River Dolphins are:
Construction of dams and barrages on the rivers; hence, 1 is correct.
Dolphins getting trapped in fishing nets; hence, 3 is correct.
Use of synthetic fertilizers and other industrial pollutants around the vicinity of rivers; hence, 4 is
correct.
□ The increased population of crocodiles in the rivers has not been cited as a reason for the
decline in population of Ganges River Dolphins. Hence, 2 is not correct.
145.
Ans. (B)
Sol. □ K. Kasturirangan led panel, recommended that instead of the total area of Western Ghats,
only 37% (i.e. 60,000 sq. km.) of the total area to be brought under Ecologically Sensitive Areas
(ESA) and complete ban on mining, quarrying and sand mining in ESA.
□ The Ministry of Environment, Forests and Climate Change by approving in principle
recommendations of K. Kasturirangan has banned development activities in around 60,000
sq. km ecologically sensitive area spread over Eco-Sensitive Zones of Western Ghats, it
include Kerala, Tamil Nadu, Karnataka, Goa, Maharashtra and Gujarat, a part of Western Ghats
Development Programme.
146.
Ans. (A)
Sol. □ Great Himalayan National Park (GHNP) was constituted in 1984 and was formally notified
as a national park in 1999. It is located in Kullu District of Himachal Pradesh, India, in the f a r
Western Himalayas, characterized by high alpine peaks, alpine meadows and riverine f o r e s t s .
Hence, statement 2 is not correct.
□ GHNP was awarded UNESCO World Heritage Site status in 2014, in recognition of its
outstanding significance for biodiversity conservation. The park protects over 1,000 plant
species, 31 mammal species and 209 bird species, as well as amphibians, reptiles and insects.
Hence, statement 1 is correct.
147.
Ans. (D)
Sol. □ Living root bridges (also known as Jing Kieng Jri) are the aerial bridges that are built by
weaving and manipulating the roots of the Indian rubber tree
□ They have been serving as connectors for generations in the Indian state of Meghalaya.
□ It spans between 15 and 250 feet and is built over centuries.
□ The bridges are primarily a means to cross streams and rivers. They have also become
world-famous tourist attractions. The two most popular tourist spots are- Riwai Root Bridge and
Umshiang Double Decker Bridge.
□ They have three main properties:
They are elastic,
The roots easily combine, and
The plants grow in rough and rocky soils.
148.
Ans. (A)
Sol. □ Gir National Park, established in 1965 at the Saurastra region of Gujarat, is the only place
where endangered Asiatic lion is found in its natural habitat.
□ The Asiatic Lion (also known as the Persian Lion or Indian Lion) is a member of the
Panthera Leo subspecies that is restricted to India.
□ Its previous habitats consisted of West Asia and the Middle East before it became extinct
in these regions.
□ Asiatic lions are slightly smaller than African lions.
□ The most striking morphological character, which is always seen in Asiatic lions, and rarely
in African lions, is a longitudinal fold of skin running along its belly.
□ Threats: Its vulnerability to unpredictable events such as a plague or a natural disaster,
poaching and locals near the Gir National Park killing the lions in retaliation for attacks on
livestock.
□ Protection Status:
IUCN Red List: Endangered
CITES: Appendix I
Wildlife (Protection) Act 1972: Schedule I
□ The Gir National Park and Wildlife Sanctuary is located in the Junagadh district of Gujarat.
The Gir Forests is the largest compact tract of dry deciduous forests in the semi-arid western part of
India.
The Gir Forests forms a unique habitat for many mammals, reptiles, birds and insect species along
with a rich variety of flora.
Gir is often linked with “Maldharis” who have survived through the ages by having symbiotic
relationship with the lion.
149.
Ans. (A)
Sol. □ The Western Ghats is a chain of mountains running parallel to India’s western coast,
traversing the States of Kerala, Tamil Nadu, Karnataka, Goa, Maharashtra and Gujarat.
□ The Western Ghats include a diversity of ecosystems ranging from tropical wet evergreen
forests to montane grasslands containing numerous medicinal plants and important genetic
resources such as the wild relatives of grains, fruit and spices. They also include the unique shola
ecosystem which consists of montane grasslands interspersed with evergreen forest patches.
□ Though it covers just 6 % of the land area of India, the Western Ghats contain more than
30% of all plant, fish, herpeto-fauna, bird, and mammal species found in India.
□ Due to its high level of biological diversity and species endemism it is recognized by
UNESCO as one of the world’s eight ‘hottest hotspots’ of biological diversity along with Sri Lanka
and has been added to a list of World Heritage Sites by the United Nations.
150. Ans. (C)
Sol. □ Biodiversity is a term used to describe the various diversities and varieties of life present
on Earth. This includes human beings, plants, animals, bacteria. It plays an important role in the
way ecosystems function and in the services they provide.
□ According to the 1992 United Nations Earth Summit, biological diversity is the variability
among living organisms from all sources, including, interalia, terrestrial, marine, and other
aquatic ecosystems, and the ecological complexes of which they are a part. This, includes
diversity within the species, between species and ecosystem.
151. Ans. (A)
Sol. □ The Lion-tailed Macaque (Macaco silenus), or the Wanderoo, an Old-World monkey, is
endemic to the Western Ghats of South India. It prefers the upper canopy of primary tropical
evergreen rainforest, but may also be found in the monsoon forests in hilly areas.
□ This species is endemic to the Western Ghats hill ranges -the Kalakkadu Hills north to
Anshi Ghat, in the states of Karnataka, Kerala and Tamil Nadu. The IUCN Red list has enlisted it in
the endangered category. Hence, 1, 2 and 3 are correct.
152. Ans. (B)
Sol. □ Sea cow is a sea-grass eating marine mammal which is found in the Indian Ocean on
the western part of the Pacific Ocean. They are the only extant species of the family
“Dugongidae”. IUCN status of Sea cow is Vulnerable. Hence, 1 is correct.
□ Sea horses are tiny fish that are named for the shape of their head, which looks like the
head of a tiny horse. Sea horses are not mammals as they are classified as fish, in the “genus
Hippocampus” family. IUCN status of Sea horses is Vulnerable. Hence, 2 is not correct.
□ Sea lions are the marine or aquatic mammals as they give live births, breathes air, and
suckles its young with milk like all other mammals. They are found primarily in the Pacific waters.
Hence, 3 is correct.
153. Ans. (A)
Sol. □ Star tortoise: It is a medium sized species of tortoise found in the dry and arid forests
of both India and Sri Lanka. It is found across the Indian sub-continent, more specifically,
the Indian star tortoise is found in the Central and southern parts of India. It is listed as least
concern in the IUCN Red list, and is placed in Schedule IV of the Wildlife Protection Act, 1972.
Hence, 1 is correct.
□ Monitor lizard: It is native to Africa, Asia and Oceania. They are the Reptilia class species
of Varanidae families found distributed over the Indian subcontinent. Monitor Lizard is
placed in Schedule I (Part II) of the Indian Wildlife Protection Act, 1972. Hence, 2 is correct.
□ Pygmy Hog: It is a critically endangered suid, previously spread across Bhutan, India and
Nepal, but now only found in India (Assam). The IUCN Red list enlists it into the critically
endangered category, whereas it is placed in Schedule I (Part I) of the Wildlife Protection Act,
1972. Hence, 3 is correct.
□ Spider Monkey: They are found in tropical forests from Southern Mexico through Central
and South America to Brazil. Spider monkeys are New World monkeys belonging to the genus
Ateles, subfamily Atelinae and family Atelidae. The IUCN Red list enlists it as an endangered
species. Hence, 4 is not correct.
154. Ans. (B)
Sol. □ In the early 1990s, the Gyps vultures of India and South Asia were among the most
abundant large raptors in the world.
□ Within a decade, the populations of three species, white-rumped
vulture (Gyps- bengalensis), Indian vulture (Gyps-indicus),
and slender-billed vulture (Gyps-tenuirostris) had declined so precipitously that all
three are considered critically endangered.
□ According to research, the cause of the decline is ‘diclofenac’, a non-steroidal anti-
inflammatory drug used to treat livestock on which vultures feed. Due to diclofenac drug,
vultures suffer from renal failure and die.
155. Ans. (A)
Sol. □ Oryx is a mammal of the Bovidae family of Oryx genus. It is found in shrubland, desert
and savanna region. Under IUCN Red list, its status is extinct in wild. It is used for livestock
farming and ranching. Overhunting and habitat loss, including competition with d o m e s ti c
livestock have been the main reasons for the extinction of the wild population. It was prized by
local people for their meat and hide. Their thick hide was used for ropes, bags, shoes and shield
coverings and they were targeted by trophy hunters for their horns.
□ Chiru is a mammal of the Bovidae family of panthalops genus. It is found in the grassland,
artificial/terrestrial, and in extreme cold climatic regions. It is found in Qinghai-Tibet Plateau,
its entire range lies within China. A small number occurs seasonally in North- Eastern Ladakh,
in the extreme north of India and the species formerly occurred in a small area of Northwest
Nepal. Under IUCN Red list, its status is near threatened. It is used for livestock farming and
ranching. Commercial scale hunting of chiru for shahtoosh in the late 1980s and 1990s led to
a severe decline in numbers.
156. Ans. (C)
Sol. □ ‘Sea buckthorn’ is a shrub of the Himalayas. Every part of the sea buckthorn plant - fruit,
leaf, twig, root and thorn has been traditionally used for medicine, nutritional supplements,
firewood and building fences.
Hence, statement 3 is correct.
□ Hardy, drought-resistant and tolerant to extreme temperatures from −43°C to 40°C, the
plant has an extensive root system which can fix atmospheric nitrogen, making it ideal for
controlling soil erosion and preventing desertification. Hence, statement 1 is correct.
□ It is not a source of biodiesel. Hence, statement 2 is not correct.
□ It is a shrub, so it is not a source of timber. Hence, statement 4 is not correct.
157. Ans. (D)
Sol. □ Mangroves are the plant communities occurring in inter-tidal zones along the coasts of
tropical and subtropical countries.
□ The various ways in which mangroves act as a reliable safety hedge against coastal
calamities include:
Reduction in the wind and swell waves as they Pass through mangroves, thus lessening wave damage
during storms.
Reducing the flooding impacts of storm surges occurring during major storms (cyclones, typhoons or
hurricanes).
Reduction in the flood extent in low lying areas, Narrower mangrove belts, hundreds of meters wide,
will still be able to reduce wind speed, the impact of waves on top of the surge and flooding impact
to some degree.
□ Wide areas of mangroves can reduce tsunami heights, helping to reduce loss of life and
damage to property in areas behind the mangroves.
□ The dense and extensive roots of mangroves help to bind and build soils. The above
ground roots slow down water flows (thus reducing the vulnerability of coastal land during
storms), enhance the deposition of sediments and reduce erosion.
□ Over time mangroves can actively build up the soils, increasing the thickness of the
mangrove soil, which may be critical as sea level rise accelerates.
158. Ans. (B)
Sol. □ The conservation without displacing the affected organism is in-situ conservation, while
conserving the organism in an artificial habitat by displacing it from its natural habitat is e x -
situ conservation.
□ Biosphere Reserve, National Park and Wildlife Sanctuary are examples of in-situ
conservation methods.
□ In a Botanical Garden, plants are bred in a protected environment far from their natural
home, especially for research and conservation purposes. So, it is an example of ex-situ
conservation.
159.
Ans. (B)
Sol. □ Indian Wild Ass/Khur was once widely distributed across the arid region of North-West
India (including present day Pakistan) and westwards through much of Central Asia towards
Syria.
However, the range is now restricted to the Little Rann of Kuchchh, Wild Ass Sanctuary in Gujarat. The
species is currently under severe threat from anthropogenic activities.
The area has a large reserve of table salt and its extraction is a serious threat to the habitat of the wild
ass.
Its IUCN status is near threatened. It is a Schedule I animal under Wildlife Protection Act, 1972.
□ The Indian Gazelle or Chinkara is considered sacred by the Bishnoi community of
Rajasthan which explains the large population of this animal there.
In India, the chinkara is found in the plains and low hills of Punjab, Rajasthan, Central India and the
Deccan Plateau.
One of the specialties of this animal is that they can go without water for days and practically survive
on dew and water obtained from leaves and plants. Its IUCN status is least concern.
□ The Indian Wild Buffalo is the state animal of Chhattisgarh and is a large species of bovine
native to the Indian Subcontinent and South East Asia.
In India, majority of its population lives in North - East India. The wild buffalo is mainly found in the
alluvial grasslands, marshes, swamps and river valleys.
They are generally found in areas that have plenty of water holes and resources. The IUCN has listed
Asiatic wild buffalo as endangered.
□ Indian Wild Boar does not face any existential threat. It is even declared vermin in a few
states. They are widely distributed across India, Nepal and Sri Lanka.
160. Ans. (C)
Sol. □ King cobras are venomous snakes that are found in South and Southeast Asia. They can
grow up to 18 feet long which makes it the longest venomous snake in the world. They are
threatened by habitat destruction and have been listed as vulnerable in the IUCN Red List since
2010.
□ Being an oviparous (who lays eggs) reptile, it makes nests to lay eggs and guard them.
161.
Ans. (B)
Sol. □ Himalayan yew (Taxus wallichiana) belongs to the family Taxaceae and is found in India
as an evergreen tree ,n the temperate Himalayas at altitudes between 1800 and 3300 m and in the
hills of Meghalaya and Manipur at an altitude of 1500 m. Hence, statement 1 is correct.
□ It is currently classified as endangered by the IUCN m its Red Data Book. Hence,
statement 2 is correct.
□ The plant holds an important place in traditional medicine and its products are used by
the local populations for treating common infections. It received wide attention because its
leaves and bark were found to be the prime source of taxol, a potent anticancer drug which has a
unique property of preventing the growth of cancerous cells and is used in the treatment of
breast and ovarian cancers. Hence, statement 3 is not correct.
162. Ans. (B)
Sol. □ Insectivorous plants are plants that eat insects. Some plants trap and consume insects to
fulfil their nitrogenous need. Also known as carnivorous- they are adapted to grow in places such as
acidic bogs and rock outcroppings where the soil is thin or poor in nutrients, especially in Nitrogen.
□ They develop bright coloured leaf modifications of different shapes for luring, trapping,
and digesting tiny creatures. Modified leaves may look like a pitcher, a flower with nectar
oozing out. Their common occurrence is m the marshy and also in extreme desert c o n d i ti o n s
where soils are deficient nutritionally. A protein rich diet of insectivorous plants helps them to
grow fast under very poor soil conditions.
□ Same examples of insectivorous plants are: Venus flytrap (Dionaea muscipula), Floating
bladderwort (Utricularia gibba), and Tropical Pitcher plant (Nepenthes khasiana)
163. Ans. (C)
Sol. □ Cyanobacteria or blue-green algae is an example of a bio-fertilizer, a type of organic
fertilizer which contains living organisms and harnesses naturally occurring inputs like solar
energy, nitrogen, and water to ensure soil fertility and plant growth.
□ Blue green algae are photoautotrophic microbes. They have specialised cells which utilises
solar energy to reduce atmospheric N2 into Ammonia. Ammonia is used by plants for growth and
increased production.

164. Ans. (D)


Sol. □ The British biologist Norman Myers coined the term “biodiversity hotspot” in 1988 as
a biogeographic region characterized both by exceptional levels of plant endemism and
by serious levels of habitat loss. To qualify as a hotspot, a region must meet two strict criteria
− it must contain at least 1,500 species of vascular plants (which is more than 0.5% of the
world’s total) as endemics, and it has to have lost at least 70% of its original habitat.
□ There are currently 36 recognized biodiversity hotspots. While most of them are in tropical
regions, some like Eastern Australian temperate forests, Succulent Karoo (South Africa), etc.,
are outside tropical regions. Hence, statement 1 is not correct.
□ India hosts 4 biodiversity hotspots - the Himalayas, the Western Ghats, the Indo-Burma
region and the Sundaland (includes Nicobar group of Islands). Hence, statement 2 is not
correct.
165. Ans. (B)
Sol. □ To qualify as a biodiversity hotspot, an area must meet two strict criteria:
Contain at least 1,500 species of vascular plants found nowhere else on Earth (known as “endemic”
species).
Have lost at least 70% of its primary native vegetation.
□ The Eastern Himalayas extend from eastern Nepal across northeastern India, Bhutan,
Tibet Autonomous Region to Yunnan in China and northern Myanmar. It is widely considered a
biodiversity hotspot that contains exceptional freshwater biodiversity and ecosystems that are of
vital importance to local and regional livelihoods. Hence, 1 is correct.
□ The Eastern Mediterranean Region (Eastern Turkey) is known as the Mediterranean Basin
biodiversity hotspot and is identified as one of the world’s 36 biodiversity hotspots, earth’s m o s t
biologically rich yet threatened areas. Hence, 2 is correct.
□ North Western Australia is not a biodiversity hotspot. South Western Australia is a
biodiversity hotspot. Hence, 3 is not correct.
166. Ans. (A)
Sol. □ The Giant Panda or simply Panda is a species of bear that is found in the mountains of
central and western China. The DNA of Panda is related to bears and because of this reason t h e y
have been placed in the bear family Ursidae.
□ They live mainly in bamboo forests and subsist entirely on bamboo. Their IUCN status is
vulnerable.
167. Ans. (D)
Sol. □ The dugong is a species of sea cow (Dugong dugong) found throughout the warm
latitudes in the Indian and western Pacific Oceans.
□ It is a medium-sized marine mammal and one of four living species of the order Sirenia,
which also includes three species of manatees.
168. Ans. (D)
Sol. □ Camphor is extracted by distilling the leaves and bark of Cinnamomum camphora, a native
to China, Taiwan, I southern parts of Japan, Korea, and Vietnam. It is also synthetically
produced. Camphor oil is extracted by steam from the chipped wood, root stumps and
branches of the camphor tree. Hence, 1 is correct.
□ Chicory is the roasted and ground root of the cultivated plant species, Chicorium Intybus,
subspecies Sativum. It is popular as an alternative to coffee for reducing caffeine intake a n d
may be linked to several health benefits, including reduced inflammation, decreased blood sugar
and improved digestive health. Hence, 2 is correct.
□ Vanilla comes from orchids of the genus Vanilla. While the major species of vanilla
orchids are now grown around the world, they originally came from Mesoamerica, i n c l u d i n g
parts of modern-day Mexico and Guatemala. Hence, 3 is correct.
169. Ans. (A)
Sol. □ Flying Fox is large, fruit eating bat that lives in warm climates. They are called flying foxes
because their furry faces look like foxes. Bats are the only group of mammals capable o f
sustained flight.
□ The Indian flying fox (Pteropus giganteus), also known as the greater Indian fruit bat, is
a species of flying fox found in South Asia. It is one of the largest bats in the world. It is
of interest as a disease vector, as it is capable of transmitting several viruses to humans. It is
nocturnal and feeds mainly on ripe fruits, such as mangoes and bananas, and nectar.
□ They are often regarded as vermin due to their destructive tendencies towards fruit farms,
but the benefits of its pollination and seed propagation often outweigh the impacts of its fruit
consumption.
170. Ans. (D)
Sol. □ The Government of India launched the National Policy on Biofuels in December 2009.
The biofuel policy encouraged the use of renewable energy resources as an alternate fuels t o
supplement transport fuels (petrol and diesel for vehicles) and proposed a target of 20%
biofuel blending (both biodiesel and bioethanol) by 2017. Ministry of Rural development w a s
designated as the Nodal Ministry for this Mission.
□ The government launched the National Biodiesel Mission (NBM) identifying Jatropha
curcas as the most suitable tree-borne oilseed for biodiesel production. Jatropha is a plant o f
Latin American origin, which is now widespread throughout arid and semi-arid tropical
regions of the world.
□ Bio fuel is any hydrocarbon fuel that is produced from an organic matter (living or once
living material) in a short period of time (days, weeks, or even months).
□ Bio fuels may be solid, liquid or gaseous in nature.
Solid: Wood, dried plant material, and manure
Liquid: Bioethanol and Biodiesel
Gaseous: Biogas.
171. Ans. (B)
Sol. □ Himalayan vegetation is characterised by elevation and precipitation. It can be classified
into Tropical, Subtropical, Temperate, and Alpine.
□ Juniperus recurva, commonly named the Himalayan Juniper, is a Juniper native to the
Himalayas, from northern Pakistan to western Yunnan in southwestern China. It grows at
3,000-4,000 m altitude.
□ Silver Fir is a Himalayan pyramidal tree, growing up to 50 m or taller and is spread in
Himachal Pradesh, Jammu and Kashmir, Uttarakhand to Nepal, at altitudes of 2800-4000 m.
□ Picea smithiana or Morinda Spruce is a spruce native to the Western Himalaya and
adjacent mountains, ranging from northeast Afghanistan, northern Pakistan, India to central Nepal.
It grows at altitudes of 2,400-3,600 m. The name Morinda derives from the tree’s name,
in Nepali.
□ Swietenia species, commonly known as genuine mahogany is native to South America,
West Indies and Mexico. It was introduced in India in 1990s in the face of imposition of t h e
regulation /permit requirement by CITES on the trading of American mahogany (Swietenia
macrophylla). The predominant Indian mahogany species is Swietenia mahagoni. It has
immense commercial value owing to its durability.
172. Ans. (A)
Sol. □ Saltwater Crocodile is popularly called saltie . It is a hypercarnivorous apex predator
widely distributed in saline and brackish waters, but is also suited to freshwater condition. I n
India, it is protected under Schedule I of the Wildlife Protection Act, 1972 and is found in
Odisha, West Bengal and Andaman and Nicobar Islands. They are capable of uni- hemispheric
sleep, meaning they can sleep by putting one half of the brain at rest and other in an alert
state. Hence, statement 1 is correct.
□ Asian house shrew is a commensal and adaptable mammal. It is a rapid coloniser and
threatens many plant and animal species. Asian house shrew is found in most parts of India.
□ Tapirs are large mammals that look like wild hogs with anteater snouts. They are mainly
found in South America. However, Malayan tapirs are found in Burma, Thailand, Malaya a n d
Sumatra. They are not found in Malabar and the Western Ghats region. Hence, statement 2 is
not correct.
173. Ans. (C)
Sol. □ Sexual cannibalism is the phenomenon when a female cannibalizes (consuming all or part
of an individual of the same species) her mate after copulation.
□ It is characterized primarily by members of most arachnid orders (of which Spiders
belong), as well as several insect orders. Black Widow Spider and Australian Redback Spider are a
notable example of this practice.
□ Male reverse cannibalism is rare. Example: Micaria sociabilis genus of spider.
174. Ans. (D)
Sol. □ King Cobra (Ophiophagus hannah) is world’s longest venomous snake and its diet
comprises of other snakes such as rat snakes (most common prey), pythons, and venomous
snakes like true cobras, the Krait, Malabar pit viper and hump-nosed pit viper by following t h e i r
odour trails.
□ Its generic name Ophiophagus stands for snake-eater in Greek. It is enlisted as vulnerable
in the IUCN Red List owing to habitat destruction. It is the national reptile of India. It is
endemic to India and is spread throughout Southeast Asia.
175. Ans. (B)
Sol. □ Plants which derive their nutrient requirement (mainly Nitrogen and Phosphorus) from
trapping and consuming animals or protozoans, typically insects and other arthropods are called
carnivorous or insectivorous plants. They follow the carnivorous mode of nutrition as their soil
has low-nutrients and lacks in Nitrogen.
□ Pitcher plants (or pitfall traps) are carnivorous plants whose prey-trapping mechanism
features a deep cavity filled with liquid, known as a pitfail trap. It preys on J foraging, flying or
crawling insects like flies by attracting them to its cavity formed by the cupped leaf, often by
visual lures such as anthocyanin pigments and nectar bribe. Its slippery sides ensures that insects/
prey cannot escape.
176. Ans. (C)
Sol. □ The western mosquitofish (Gambusia affinis) and eastern mosquitofish (Gambusia
holbrooki) are a species of freshwater fish, known commonly as mosquitofish or by its
generic name, Gambusia, or by the common name gambezi.
□ It survives in all kinds of water, thereby it is widely used to curtail the mosquito menace.
□ Each Gambusia consumes over 250-300 mosquito larvae a day
□ It was first detected in Italy in 1931 that Gambusia consumes mosquito larvae.
□ Larvicidal fish such as gambusia and lebistes (guppies) are stated to be very effective in
controlling the breeding of mosquitoes without disturbing the ecological balance.
177. Ans. (A)
Sol. □ Under the Wildlife Protection Act (WPA), 1972, an area, whether within a sanctuary or
not, can be notified by the state government to be constituted as a National Park, by reason of its
ecological, faunal, floral, geomorphological, or zoological association or importance, needed for
the purpose of protecting and propagating or developing wildlife therein or its environment.
□ No human activity is permitted inside the National Park except for the ones permitted by
the Chief Wildlife Warden of the state under the conditions given in Chapter IV, WPA, 1972.
□ There are 104 existing National Parks in India covering an area of 40501.13 km2, which is
1.23% of the geographical area of the country (National Wildlife Database, May, 2019).
□ Andaman and Nicobar Islands have nine National Parks; Assam has five and both
Meghalaya and Arunachal Pradesh have two each.
□ Madhya Pradesh has the largest share of the National Park, i.e., Ten. Tenth National Park
was added in 2011. But as per the given options, Andaman and Nicobar have the maximum
national parks.
178. Ans. (C)
Sol. □ Apes (Hominoidea) are a branch of Old-World tailless simians native to Africa and
Southeast Asia. They are divided into Greater Apes and Lesser Apes. The Greater Apes
family is Hominidae, with subfamily comprising of gorillas, hominoids and chimps, whereas
Lesser Apes belong to family of Hylobatidae. Examples - Bonobos, Pygmy Chimpanzee,
Gibbon, Orangutan etc.
□ Monkeys and apes are both primates, which means they both are parts of the
human family tree. The quickest way to tell the difference
between a monkey and an ape is by the presence or absence of a tail. Almost all monkeys have
tails, but apes do not. Their bodies are different in other ways too - monkeys are generally
smaller and narrow-chested, while apes are larger and have broad chests and shoulder joints that
allow them to swing through trees.
□ Gray langurs or Indian langurs are a group of Old World monkeys native to the Indian
subcontinent constituting the entirety of the genus Semnopithecus. Gray langurs are fairly
terrestrial, inhabiting forests, open light wooded habitats, and urban areas of the Indian
subcontinent. Most species are found at low to moderate altitudes, but the Nepal gray langur a n d
Kashmir gray langur are found up to 4,000 m in the Himalayas.

PRACTICE SOLUTION:-
179. Ans. (A)
Sol. □ Biodiversity or “biological diversity,” generally refers to the variety and variability of life
on Earth. The number and variety of plants, animals and other organisms that exist is known a s
biodiversity.
□ Generally biodiversity is considered at three levels:
Genetic diversity: Genetic diversity refers to the variety of genes contained within species of plants,
animalsand micro-organisms. New genetic variation in individuals occurs by gene and chromosomal
mutation, and in organisms with sexual reproduction may be spread across the population by
recombination.
Species diversity: Diversity at the species level. It can be measured in the terms of species richness,
species abundance and taxonomic diversity.
Ecosystem diversity: Ecosystem diversity is the variety of habitats found in an area.
Ecosystem diversity can be described for a specific geographical region, or a political
entity such as a country or a state. Distinctive ecosystems include landscapes such as
forests, grasslands, deserts, mountains, etc., as well as aquatic ecosystems such as rivers,
lakes, and the sea. Ecosystems are most natural in wilderness areas.
□ The overall richness of species is concentrated in equatorial regions and tends to decrease
as one move from equatorial to the Polar Regions. In addition, biodiversity in land e c o s y s t e m s
generally decreases with increasing altitude. Hence, statement 1 is correct and statement 2 is
not correct.
180. Ans. (D)
Sol. □ The number and variety of plants, animals and other organisms that exist is known as
biodiversity.
□ Measurement of Biodiversity
Alpha Biodiversity: It is measured by counting the number of species in a particular ecosystem. It is
the diversity that exists within a community, which is also called local diversity. Hence, statement 2 is
not correct.
Beta Diversity: It is the species diversity between two adjacent ecosystems and is obtained by
comparing the number of species unique to ecosystems. It is the relative richness of different species
along a gradient from one habitat to another habitat within a community, which is mainly because of
variations in physical conditions of habitats. It gives a quantitative measure of the diversity of
communities that experience changing environments. Hence, statement 1 is not correct.
Gamma Diversity: It refers to the total species richness over a large area or region.lt is a measure
of the overall diversity of the different ecosystems within a region. It is the product of a diversity of
component ecosystems and the diversity between component ecosystems.
181. Ans. (C)
Sol. □ A biodiversity hotspot is a region with a high level of endemic species that is under threat
from humans. The term hotspot was introduced in 1988 by Norman Myers.
□ To qualify as a biodiversity hotspot, a region must meet two strict criteria:
It must have at least 1,500 vascular plants as endemics that mean, it must have a high percentage
of plant life found nowhere else on the planet. A hotspot, in other words, is irreplaceable. Hence,
statement 1 is correct.
It must have lost over 70% of the original habitat. In other words, it must be threatened. Hence,
statement 2 is correct.
□ Around the world, 36 areas qualify as hotspots. They represent just 2.4% of the Earth’s
land surface, but they support more than half of the world’s plant species as endemics, i.e.,
species found nowhere else and nearly 43% of bird, mammal, reptile and amphibian species a s
endemics.
182. Ans. (D)
Sol. Factors Determining the Degree of Biodiversity
□ Habitat Stress: Diversity is low in habitats under any stress like harsh climate of pollution.
Hence, 4 is correct.
□ Geographic Isolation: Isolation can lead to special species located in that area only. If a
species on an island disappears due to random events, it cannot be easily replaced. Hence, 1
is correct.
□ Dominance of One Species: Dominance of one species doesn’t allow others to grow and
flourish. Hence, 2 is correct.
□ Availability of Ecological Niches: A complex community offers a variety of niches than a
simple species promotes greater diversity. Hence, 3 is correct.
□ Edge Effect: Ecotone (transition between one type ecosystem to another) regions have
great^ biodiversity.
□ Geological History: Old and stable ecosystems like Amazon forests have greater diversity as
they are more resistant to change.
183. Ans. (C)
Sol. □ “Ortolan bunting” is a bird which breeds from Mongolia to Europe and migrates to Africa
via the Middle East. In summers it moves to Western Europe and winters in Africa and
tropical areas. It is not endemic to Eastern Ghats of India. Hence, statement 1 is not correct.
□ The International Union for Conservation of Nature’s (IUCN) red list of threatened species h a s
placed it in the “Least Concern” category. Hence, statement 2 is not correct.
184. Ans. (C)
Sol. On 29th January 2000, the Conference of the Parties to the Convention on Biological Diversity adopted
a supplementary agreement to the Convention known as the Cartagena Protocol on Biosafety. It
entered into force on 11th September 2003. The Protocol:
□ Seeks to protect biological diversity from the potential risks posed by living modified
organisms resulting from modern biotechnology. Hence, statement 1 is correct.
□ Establishes an advance informed agreement (AIA) procedure to ensure that countries
are provided with the information necessary to make informed decisions before agreeing to
the import of such organisms into their territory.
□ Contains reference to a precautionary approach and reaffirms the precaution language in
Principle 15 of the Rio Declaration on Environment and Development.
□ Establishes a Biosafety Clearing-House to facilitate the exchange of information on living
modified organisms and to assist countries in the implementation of the Protocol. Hence,
statement 2 is correct.
185. Ans. (A)
Sol. □ Rare Species (RS): Rare Species are those species whose population is small and the
location is confined to limited areas. The rare species, in times to come may become v u l n e r a b l e
and finally may be endangered. These species need to be provided special care and attention
in conservation programmes. Hence, pair 1 is correctly matched.
□ Near Threatened (NT): A species is near threatened when it has been evaluated against
the criteria but does not qualify for Critically Endangered or Vulnerable now, but is close to
qualifying for or is likely to qualify, for a threatened category in the near future. Hence, pair 2
is not correctly matched.
□ Least Concern (LC): A species is Least Concern when it has been evaluated against the
criteria and does not qualify for Critically Endangered, Endangered, Vulnerable or Near
Threatened. Widespread and abundant species are included in this category. Hence, pair 3 is
not correctly matched.
□ Extinct Species (EC): These are the species which are not found after exhaustive survey
and search of known or likely areas, where they occur. A species may be extinct from a local
area, region, country, continent or the entire world. In other words, a species is extinct when
there is no reasonable doubt that the last individual has died. A species is presumed as
extinct, when exhaustive survey has failed to record an individual species.
186. Ans. (D)
Sol. □ Recent update (July 2020) shows that 33 lemur species are Critically Endangered, with
103 of the 107 surviving species threatened with extinction, mainly due to deforestation (for
shifting cultivation, fuel wood and charcoal) and hunting in Madagascar. Thirteen lemur
species have been pushed to higher threat categories as a result of intensifying human
pressures.
□ The North Atlantic Right Whale (Eubalaena glacialis) has been moved from Endangered to
Critically Endangered on the IUCN Red List. Hence, 1 is correct.
□ Climate change appears to be exacerbating the threats to North Atlantic Right Whales.
Warmer sea temperatures have likely pushed their main prey species further north during
summer, into the Gulf of St. Lawrence, where the whales are more exposed to accidental
encounters with ships and also at high risk of entanglement in crab-pot ropes.
□ The European Hamster (a Rodent), once abundant across Europe and Russia, is now listed
as Critically Endangered. If nothing changes, the species is expected to go extinct within t h e
next 30 years. Hence, 2 is correct.
□ Among those newly listed as Critically Endangered are Verreaux’s Sifaka (Propithecus
verreauxi) and Madame Berthe’s Mouse Lemur (Microcebus berthae), the smallest primate in the
world, both of which were previously listed as Endangered. Hence, 3 is correct.
187. Ans. (D)
Sol. □ At least one-third of the world’s agricultural crops depend upon pollination provided by
insects and other animals, and agricultural production throughout the world is threatened b y
declining populations of pollinators. The major contributors to this decline in pollinator
populations are:
Habitat fragmentation,
Agricultural and industrial chemicals,
Parasites and diseases,
Introduction of alien species.
Hence, 1,2 and 3 are correct.
□ The scope of the pollinator crisis is international both in the distribution of pollinators, the
crops they service and the threats to them. We need to take immediate international action t o
secure sustained pollinator services in agricultural ecosystems.
188. Ans. (B)
Sol. □ Food and Agriculture Organisation (FAO) created the International Plant Protection
Convention or IPPC in 1952. Hence, statement 1 is not correct.
This international treaty organization works to prevent the international spread of pests and plant
diseases. The maintenance of lists of plant pests, tracking of pest outbreaks, and coordination of
technical assistance between member nations are among its junction.
□ The TRAFFIC, the Wildlife Trade Monitoring Network, is a leading non-governmental
organisation working on wildlife trade in the context of both biodiversity conservation and
sustainable development. It is a joint program of World Wildlife Fund (WWF) and the International
Union for Conservation of Nature (IUCN). Hence, statement 2 is correct.
TRAFFIC works in co-operation with the Secretariat of the Convention on International Trade in
Endangered Species of Wild Fauna and Flora (CITES).
It was established as a specialist group of the IUCN Species Survival Commission in 1976.
TRAFFIC focuses on leveraging resources, expertise and awareness of the latest globally urgent species
trade issues such as tiger parts, elephant ivory and rhino horn.
189. Ans. (C)
Sol. □ Biodiversity or “biological diversity,” generally refers to the variety and variability of life
on Earth. The number and variety of plants, animals and other organisms that exist is known a s
biodiversity.
□ The 1992 United Nations Earth Summit defined “biological diversity” as “the variability
among living organisms from all sources, including, ‘inter alia’, terrestrial, marine, and other aquatic
ecosystems, and the ecological complexes of which they are part: this includes
diversity within species, between species and of ecosystems”.
□ Species diversity is not evenly distributed across the globe. The overall richness of species
is concentrated in equatorial regions and tends to decrease as one move from equatorial to the
Polar Regions. Hence statement 1 is correct.
□ In addition biodiversity in land ecosystems generally decreases with increasing altitude.
The other factors that influence biodiversity are the amount of rainfall and nutrient level in soil.
□ In marine ecosystems, species richness tends to be much higher in continental shelves.
Hence statement 2 is correct
190. Ans. (C)
Sol. □ A biodiversity hotspot is a region with a high level of endemic species that is under threat
from humans. The concept of Biodiversity hotspot was put forth by Norman Myers in 1988. Hence,
statement 1 is correct.
□ To qualify as a biodiversity hotspot, a region must meet two strict criteria:
O Species endemism: It must have at least 1,500 vascular plants as endemics, which is to
say, it must have a high percentage of plant life found nowhere else on the planet. A hotspot,
in other words, is irreplaceable.
° Degree of threat: It must have lost at least %70 of its original habitat. In other words, it
must be threatened. Hence, statement 2 is correct.
□ Each biodiversity hotspot represents a remarkable umverse of extraordinary floral and
faunal endemicity struggling to survive in rapidly shrinking ecosystems
□ 36 biodiversity hotspots have been identified in the world. These hotspots are
characterized by posing exceptionally high biodiversity.
While hotspots are spread all over the world, the majority are forest areas and most of them are
located in the tropics.
□ The hottest hotspots: Some hotspots are much richer than others in terms of their numbers
of endemics.
□ The eight hottest hotspots of the world are:
Madagascar
Philippines
Sundaland
Brazil’s Atlantic Forest
Caribbean
Indo-Burma
Western Ghats/Sri Lanka
Eastern Arc and Coastal Forests of Tanzania/Kenya
191.
Ans. (D)
Sol. □ Biodiversity or “biological diversity,” generally refers to the variety and variability of life
on Earth. The number and variety of plants, animals and other organisms that exist is known a s
biodiversity. Biodiversity is unevenly distributed across the geographical regions of the earth.
Some regions of the world are very rich in biodiversity Such as “mega diversity zones” also
know as, “hot-spots’
□ India is one of 17 mega-biodiverse countries in the world. With only 2.4 percent of the
earth’s land area, it accounts for 7-8 percent of the global diversity. Home to 96,000 species o f
animals, 47,000 species of plants and nearly half the world’s aquatic plants, India’s management
of its natural resources is crucial for protecting globa1 biodiversity. Hence, statement 1 is not
correct.
□ Among the 36 hotspots of the world, 4 are found in Indian subcontinent namely Western
Ghats, Indo-Burma, Sundaland and eastern Himalayas. These areas of the country are exceptionally
rich in flowering plants reptiles, amphibians, butterflies and some species of mammals. Western
Himalayas is not a hotspot, but is rich biodiverse region. Hence, statement 2 is not correct.
192.
Ans. (D)
Sol. □ India has a total geographical area of about 329 million hectares with a coastline of over
7500 km. The ecological diversity of the country is enormous, ranging from sea level to t h e
highest mountainous ranges in the world; hot and arid conditions in the northwest to cold arid
conditions in the trans-Himalayan region; tropical wet evergreen forests in Northeast India and
the Western Ghats; mangroves of Sundarbans and fresh water aquatic to marine ecosystems.
□ India has 12 bio geographical provinces, 5 biomes and 3 bioregion domains. The country
supports a diverse array of habitats or ecosystems such as forests, grasslands, wetlands,
coastal, marine and desert and each with rich and unique floristic diversity. Hence, statement 1 is
not correct.
□ These biological attributes are further enhanced by the geographic location of the country
at the confluence of three major global biogeographic realms, viz. Indomalesian, Eurasian a n d
Afro-tropical, thus allowing the intermingling of floristic elements from these regions as well and
making it one of the 17 megadiversity countries in the world, recognized by the W o r l d
Conservation Monitoring Centre in 2000.
□ The floral diversity in India is majorly concentrated in the 4 biodiversity hotspots, out of
36 biodiversity hotspots recognized in the world, namely:
Eastern Himalayas,
Western Ghats (and Sri Lanka),
Northeast India and Andaman Islands (Indo-Burma) and
Nicobar Island (Sundaland)
Hence, statement 2 is not correct.
□ These floristically significant areas exhibit exceptional concentration of endemic species
and are also experiencing loss of habitat with higher occurrence of threatened plant species.
193.
Ans. (B)
Sol. □ Bryophytes: The less known group of plants, comprising about 2500 species, is the second
largest group of green plants in India, next only to the angiosperms. They usually inhabit
narrow ecological niches with preference for damp and shady conditions. The vast areas in
the Himalayas and Peninsular India with abundant precipitation and high humidity are richer
in bryophytes.
□ Algae: Represented by over 6500 species in ca. 666 genera, they are found growing in
a variety of habitats ranging from fresh water, marine, terrestrial and to soil. Of which 1924
species are endemic to the country.
□ Fungi: Fungi range from microscopic organisms to huge solid bodies. Approximately
14,500 species in 2300 genera are found in India.
□ Angiosperms: The largest group of green plants in India. The angiospermic flora of India
is further characterized by high endemism, which is next only to Australia. Recent estimate
accounts a total of 17926 species of angiosperms in the country.
194.
Ans. (B)
Sol. □ Launched in 2005, Indian Rhino Vision (IRV) 2020, was an ambitious effort to increase
the population of greater one-horned rhinos at least from 2,000 to 3,000 spread over seven
protected areas in the Indian state of Assam by the year 2020. Hence, statement 1 is not
correct.
□ Seven protected areas are Kaziranga, Pobitora, Orang National Park, Manas National Park,
Laokhowa wildlife sanctuary, Burachapori wildlife sanctuary and Dibru Saikhowa wildlife
sanctuary.
□ Wild-to-wild translocations were an essential part of IRV 2020 - moving rhinos from
densely populated parks like Kaziranga National Park to ones in need of more rhinos like Manas
National Park. Hence, statement 2 is correct’.
It is collaborative effort between various organisations, including the International Rhino
Foundation, Assam’s Forest Department, Bodoland Territorial Council, World Wide Fund - India,
and the US Fish and Wildlife Service.
195.
Ans. (C)
Sol. □ Pygmy Hog (Porcula salvania) is world’s smallest wild pig. Due to a small wild population,
estimated less than 250 individuals, and a highly restricted range, the pygmy hog is c u r r e n t l y
listed as ‘Critically Endangered’ on the IUCN Red List of Threatened Species.
□ It is restricted to the reserve forest belts of the Manas Wildlife Sanctuary and the Barnadi
Wildlife Sanctuary in Assam, India. Hence, statement 2 is correct.
□ It was nearly wiped out; nonetheless, a watchful breeding programme in captivity and
security concerns were successful in conserving the Pygmy Hog population.
□ It is an indicator species (animals, plants or microorganisms used to monitor changes in
our environment) of the management status of grassland habitats Its presence reflects the health
of its primal habitat, the tall, wet grasslands of the region Current, It is protected by Schedule
I of the Wildlife (Protection) Act, 1972. Hence statement 1 is correct.
196.
Ans. (B)
Sol. □ Kondana soft-furred rat is a critically endangered (CR) species on the IUCN Red List of
Threatened Species, known from a single locality - Sinhgad in the northern Western Ghats, India.
Hence, statement 1 is not correct and statement 2 is correct.
□ The species has been reported to occur in tropical and subtropical dry deciduous forests
and tropical scrub. It is listed in the Schedule V (considered as vermin) of the Wildlife (Protection)
Act, 1972.
□ It is a nocturnal burrowing rodent. It sometimes builds nests also. Hence, statement 3 is
correct.
197.
Ans. (C)
Sol. □ Monotremes, a subdivision of mammals, are egg laying rather than giving birth to young
ones. There are only five living monotreme species, Duck-billed platypus and four species o f
Echidna. Hence, statement 1 Is correct.
□ Monotremes are restricted to Australia and New Guinea. Hence, statement 2 is correct.
□ From a neurobiological perspective, monotremes are quite intriguing because it is thought
that they may have retained features of brain organization that were present in the first mammals.
198.
Ans. (D)
Sol. □ Siberian Crane is a migratory and aquatic bird that uses wetlands for feeding, nesting,
roosting and other behavioral displays, where one can find shallow fresh water along with g o o d
visibility. Hence, statement 1 is correct.
□ The International Union for Conservation of Nature (IUCN) Red List of Birds has listed
Siberian crane as a Critically endangered species. Hence, statement 3 is correct.
□ Siberian Crane is now only found in one main population in East Asia, in addition to a few
birds still existing in the Western/Central population.
□ Keoladeo National Park, Rajasthan is famous habitat of Siberian Crane. Hence, statement
2 is correct.
□ Habitat loss, especially due to changing hydrology caused by water diversions and
conversion of wetlands, hunting, trapping and poisoning, pollution, and environmental
contamination are the major threats to Siberian Crane.
199.
Ans. (B)
Sol. □ A migratory species is one that cyclically and predictably crosses one or more national
jurisdictional boundaries due to factors like food, temperature, shelter, etc.
□ World Migratory Bird Day (WMBD) is an awareness-raising campaign highlighting the
need for the conservation of migratory birds and their habitats. It is an effective tool to help r a i s e
global awareness of the threats faced by migratory birds, their ecological importance, and the need
for international cooperation to conserve them.
□ The World Migratory Bird Day is celebrated bi-annually on the second Saturday in May
and in October. Hence, statement 2 is correct.
□ Convention on the Conservation of Migratory Species of Wild Animals (CMS) also known
as the Bonn Convention is an environmental treaty of the United Nations that provides a
global platform for the conservation and sustainable use of terrestrial, aquatic and avian
migratory animals and their habitats. Hence, statement 1 is not correct.
200.
Ans. (C)
Sol. □ Fejervarya goemchi is a new species of frog found in the highland plateaus of the Western
Ghats parts of Goa. The new species is named after the historical name of the state of Goa
where the species is discovered.
□ A combination of geographic distribution range, morphology and molecular methods has
been used by the scientists to describe the new species.
201.
Ans. (A)
Sol. □ Indus river dolphin also known as Bhulan is one of the only four river dolphin species and
subspecies in the world that are found in freshwater. Others include the Chinese river dolphin,
Ganges river dolphin and Amazon River dolphin.
□ Indus river dolphin is found in the main channel of the Indus River in Pakistan and in the
Beas River in India.
Hence, statement 1 is not correct.
□ The Punjab Government has declared the Indus River dolphin, as the state’s aquatic
animal. Hence, statement 2 is not correct.
□ It is listed endangered in the IUCN’s Red List of Threatened Species. Its international trade
is prohibited as it is listed under Appendix I of the Convention on International Trade in
Endangered Species (CITES). Hence, statement 3 is correct.
□ Indus dolphin like Ganges River dolphin are functionally blind and relies on echolocation
to navigate, communicate and hunt prey in muddy river water.
202.
Ans. (B)
Sol. □ Golden Langur (Trachypithecus geei), found only in a few forest patches of Assam and
Bhutan, is placed among world’s 25 endangered primates. Hence, statement 2 is correct.
□ Golden langurs can be most easily recognized by the color of their fur, after which they are
named.
□ It has been noted that their fur changes colors according to the seasons as well as
geography (region they live in). The color of the young also differs from adults in that they a r e
almost pure white.
□ They are highly dependent on trees, living in the upper canopy of forests. They are also
known as leaf monkeys.
□ Protection Status
IUCN List of Threatened Species: Endangered; hence, statement 1 is not correct.
Convention on International Trade in Endangered Species of Wild Fauna and Flora (CITES): Appendix I
Wildlife Protection Act, 1972 : Schedule I
□ Chakrashila is India’s first wildlife sanctuary with Golden langur as the primary species.
Hence, statement 3 is not correct.
□ Moreover, the Mahatma Gandhi National Rural Employment Guarantee Act (MGNREGA)
for the first time since its inception in 2005, has made the rare golden langur as its first non-
human beneficiary in a reserve forest in western Assam’s Bongaigaon district.
203.
Ans. (A)
Sol. □ Cinereous vulture is a dark brown and broad-winged species with a slightly wedge-shaped
tail. The bald head and neck are bluish gray, with a fluffy collar which is lighter in older bird.
□ Cinereous vulture (Aegypius monachus) migrates from the mountainous regions of
Europe and Asia to warmer places, including India.
□ India is home to nine species of vultures. Out of nine, three species (Red-headed vulture,
Slender billed vulture and Long billed vulture) are listed as critically endangered. Hence,
statement 1 is correct.
□ Cinereous vulture is classified as Near Threatened under the IUCN Red List. Hence,
statement 2 is not correct.
□ In many countries, this bird is called ‘monk vulture’, because of its upright standing neck
feathers that resemble the hood of a monk.
204.
Ans. (C)
Sol. □ Mammals of India (MaOI) is an online, peer- reviewed, freely-accessible portal launched
in 2018 by the scientists and researchers from the National Centre for Biological Sciences
(NCBS), Bengaluru. Hence, statement 2 is correct.
□ The portal has come up with a new citizen-science repository on Indian mammals. It will
make the basic information freely available to any mammal enthusiasts and will cater to t h e
research needs of the community. Hence, statement 1 is correct.
205.
Ans. (D)
Sol. □ The White-bellied Heron
The White-bellied Heron (Ardea insignis) is a critically endangered bird endemic to South Asia. Hence, 1 correct.
Its range extends from eastern Nepal to north Myanmar through Bhutan and northeast India, Bangladesh.
□ The Bengal Florican
The Bengal Florican (Houbaropsis bengalensis) is a critically endangered bustard found in the terai grasslands in
India. Hence, 2 is correct.
It is listed under Schedule I of the Wildlife Protection Act of India, 1972, which provides the species the highest
degree of protection in the country.
□ Spoon Billed Sandpiper
Spoon Billed Sandpiper (Calidris pygmaea) is a critically endangered species that stays in close proximity to wet
and muddy areas near or in water. Hence, 3 is correct.
It requires highly specialized breeding habitat, a constraint that has always kept its population scarce. India is
home to some of the last existing wintering grounds of this species.
206.
Ans. (C)
Sol. □ Invasive alien species are animals, plants, fungi and microorganisms introduced,
accidentally or intentionally, outside of their natural habitat. They reproduce rapidly, out-
compete native species for food, water and space, and are one of the main causes of global
biodiversity loss. Hence, statement 1 is correct.
□ They are often introduced as a result of the globalisation of economies through the
movement of people and goods.
□ These species can cause severe ecological effects on tb* environment, they invade. As they
lack natural predator in their new environments, it eventually leads to increa^ in their a b u n d a n c e
and spread.
□ These invasive species can have various impacts on the environment including:
Diseases,
Preying on native species,
Alteration of food chains, and
Change in ecosystems, for example, altering soil composition or creating habitats that encourage wildfires.
Hence, statement 2 is correct.
207.
Ans. (C)
Sol. □ The Himalayan Yew (Taxus wallichiana) is a medicinal plant (tree) found in various parts
of Himachal Pradesh and Arunachal Pradesh. In India, this evergreen tree is found at altitudes
between 1800 and 3300 m above mean sea level.
□ It has been used by the native populations for treating common cold, cough, fever, and
pain. Its uses are described in Ayurveda and llnani medicine.
□ The bark, needles, twigs and roots of the tree are the prime source of ‘taxol’, a potent anti-
cancer drug. Moreover, it also possesses many other biological activities. Hence, statement 1
is correct and statement 2 is not correct.
□ Taxol is a highly substituted polyoxygenated cyclic diterpenoid characterized by the taxane
ring.
□ The species is under great threat due to illegal cutting and other anthropogenic pressures.
The Himalayan Yew , is endangered as per the IUCN status and is at the risk of extinction, i n
the Himalayas. In the last one decade, thousands of yew trees have dried up in various parts
of Himachal Pradesh and Arunachal Pradesh. Hence, statement 3 is not correct.
208.
Ans. (D)
Sol. □ Seagrass beds are often called nursery habitats because the leafy underwater canopy
they create provides shelter for small invertebrates (like crabs and shrimp and other types of
crustaceans), small fish and juveniles of larger fish species. Hence, statement 1 is correct.
□ It is because of the wide variety of different species that live amongst the grasses that
seagrass beds often form important “biodiversity hotspots”. Hence, statement 2 is correct.
□ The dense root systems found on most seagrass beds secure the seabed, preventing soft
sediments from being washed onto coral reefs and other sensitive ecosystems and p r o v i d i n g
some protection to coasts and coastal communities from strong ocean storms. H e n c e ,
statement 3 is correct.
209.
Ans. (D)
Sol. □ “Seaweed” is the common name for countless species of marine plants and algae that
grow in the ocean as well as in rivers, lakes, and other water bodies.
□ They are the primitive, marine non-flowering algae without root, stem and leaves that play
a major role in marine ecosystems. Hence, statement 1 is not correct.
□ Seaweeds are found mostly in the intertidal region, in shallow and deep waters of the sea
and also in estuaries and backwaters. Hence, statement 2 is not correct.
□ Large seaweeds form dense underwater forests known as kelp forests, which act as
underwater nurseries for fish, snails and sea urchins.
□ Seaweed has a significant role in mitigating climate change By afforesting 9% of the ocean
with seaweed, it is possible to sequester 53 billion tons of carbon dioxide annually. Hence, there
is a proposal termed as ocean a forestation’ for farming seaweed to remove carbon. H e n c e ,
statement 3 is correct.
□ Seaweed provides many benefits to land-dwellers, notably those of the human variety
including:
Seaweed is chock-full of vitamins, minerals, and fiber, and can be tasty.
Many seaweeds contain anti-inflammatory and anti-microbial agents.
Some also possess powerful cancer-fighting agents.
It is used as an ingredient in preparing toothpaste, cosmetics and paints.
210.
Ans. (D)
Sol. □ Plankton: An organism is considered plankton if it is carried by tides and currents, and
cannot swim well enough to move against these forces. Plankton are usually microscopic, o ften
less than one inch in length, but they also include larger species like some crustaceans and
jellyfish.
□ Periphyton: It is a complex mixture of autotrophic (algae, bacteria, phytoplankton) and
heterotrophic (microbes) microorganisms submerged in a matrix of organic detritus (waste o r
debris). It covers most submerged substrates, ranging from sand to macrophytes to rock.
□ Benthos: Animals that live on the sea floor are called benthos. Most of these animals lack
a backbone and are called invertebrates. Typical benthic invertebrates include sea
anemones, sponges, corals, sea stars, sea urchins, worms,
bivalves, crabs, and many more.
□ Nekton: Nekton refers to the actively swimming (free swimming) aquatic organisms in
a body of water. Nekton are heterotrophic and have a large size range, such as fish, squid,
octopus, sharks, and marine mammals.
211.
Ans. (D)
Sol. □ Mangroves are a special type of vegetation, found in the inter-tidal regions along the
coasts of tropical and subtropical countries (area where the ocean meets the land between high
and low tides), where freshwater and saltwater intermixes, in the bays, estuaries, creeks, a n d
lagoons. They are the salt-tolerant variety of plants, which can survive in harsh c o n d i ti o n s .
Hence, statements 1 and 3 are correct.
□ Mangroves grow in sheltered low lying coasts, estuaries, mudflats, tidal creek backwaters
(current less, coastal waters held back on land), marshes and lagoons.
□ Mudflats or tidal flats refer to land near a water body that is regularly flooded by tides
and is usually barren (without any vegetation). Mudflats and mangroves together constitute an
important ecosystem. Hence, statement 2 is correct.
□ Mangrove forests perform multiple ecological functions such as production of woody
trees; provision of habitat, food, and spawning grounds for fin-fish and shellfish; provision of
habitat for birds and other valuable fauna; protection of coastlines and accretion of sediment to
form new land.
□ Mangrove forests are the warriors in the fight against climate change, as they have the
ability to store vast amounts of carbon.
212.
Ans. (C)
Sol. □ There are five different species of rhinos including Black Rhino, White Rhino, Javan
Rhino, Greater One-horned rhino and Sumatran Rhino. Each one has different characteristics,
behaviours, and personalities.
□ There are three species of rhino in Asia—greater one-horned, Javan and Sumatran. Javan
and Sumatran Rhino are critically endangered but the greater one-horned (or Indian) rhino is
vulnerable.
□ Among the three species found in Asia, only the Greater One-Horned Rhino (Indian rhino)
is found in India, which is the largest of the rhino species. In India, these are mainly found in
Assam, West Bengal and Uttar Pradesh.
213.
Ans. (D)
Sol. □ St. Petersburg Declaration was adopted In November 2010, by the leaders of 13 tiger
range countries (TRCs) assembled at an International Tiger Forum in St. Petersburg, Russia.
Hence, statement 1 is correct.
□ The resolution’s implementation mechanism is called the Global Tiger Recovery Program
(GTRP) whose overarching goal was to double the number of wild tigers from about 3,200 t o
more than 7,000 by 2022. Hence, statement 2 is correct.
□ The tiger range countries that are part of the GTRP are Bangladesh, Bhutan, Cambodia,
China, India, Indonesia, Malaysia, Myanmar, Nepal, Russia, Thailand, and Vietnam. Hence,
statement 3 is correct.
214.
Ans. (D)
Sol. □ The National Biodiversity Authority (NBA) is a statutory body, established by the Central
Government in 2003 to implement India’s Biological Diversity Act (2002).
□ The NBA with its headquarters in Chennai, Tamil Nadu, delivers its mandate through a
structure that comprises the Authority, Secretariat, State Biodiversity Boards (SBBs), BMCs
(Biodiversity Management Committees) and Expert Committees.
□ Functional committees under National Biodiversity Authority (NBA) include:
Expert Committee on Agro Biodiversity.
Expert Committee on Normally Traded Commodities.
Expert Committee on Access and Benefit Sharing.
Expert Committee on Medicinal Plants.
Expert Committee to examine Biological Diversity Rules, user country measures under Nagoya Protocol and
develop sector specific guidelines on Access and Benefit Sharing (ABS). Hence, 1, 2, 3 and 4 are correct.
215.
Ans. (A)
Sol. □ Maldhari, a tribe of herdsmen have survived through the ages by having symbiotic
relationships with the lions in the Gir forest in a ‘win-win’ situation and they do not see the l i o n s
as a threat. These are religious pastoral communities living in Gir. Their settlements are called
“nesses”.
□ Raikas are tribes of Rajasthan region. One group of Raikas is known as camel herders and
another group known for rearing sheep and goat.
□ Bakarwals are mostly Muslim nomadic tribes living in the Pir Panjal and Himalayan
mountains of South Asia.
□ The Bishnoi tribe belong to Rajasthan. They are known for agricultural development and
resource conservation.
216.
Ans. (B)
Sol. □ In order to have concerted approach to protection, conservation and management of
wildlife throughout the country, in 1982, a decision was taken to prepare a National W i l d l i f e
Action Plan. Accordingly, the first National Wildlife Action was adopted in 1983, which
outlined the strategies and action points for wildlife conservation. It was implemented from
1983 to 2001
□ The Plan was revised and a new Action Plan was put in place in for the period 2002-2016.
□ The Union Ministry of Environment, Forests, and Climate Change (MoEFCC) announced
the third National Wildlife Action Plan for 2017-2031.
□ This Plan is unique as this is the first time India has recognized the concerns relating to
climate change impact on wildlife. Hence, statement 1 is not correct and statement 2 is
correct.
It also stressed on integrating actions that need to be taken for its mitigation and adaptation into
wildlife management planning processes.
Further, the Plan adopts a “landscape approach” in conservation of all wildlife that have an ecological
value to the ecosystem and to mankind irrespective of where they occur.
The Plan also gives special emphasis to recovery of threatened species and aquatic ecosystems. It also
emphasizes the importance of integrated management of coastal and marine ecosystems in India.
Under five themes, there are 103 conservation actions and 250 projects in the Action Plan.
217.
Ans. (A)
Sol. The Wildlife Protection Act (WPA), 1972 consists of a six schedules list which gives different kind of
protection to wildlife. Hence, statement 1 is correct.

Schedule Animals
Schedule Tiger, lion, elephants, lion tailed macaque, one horned rhinoceros,
I great Indian bustard, narcondam hornbill, nicobarmegapode,
blackbuck, etc.
Schedule Rhesus macaque, dhole, Bengal porcupine, king cobra, flying squirrel
II Himalayan brown bear. Hence, statement 2 is not correct.
Schedule Hyena, hog deer, goral, sponges, barking deer, etc.
III
Schedule Mongooses, vultures, etc.
IV
Schedule Animals that can be hunted are included in this Schedule. For example,
V Mice, rat, common crow, flying fox, etc.
Schedule It Includes list of plant species which cannot be cultivated; collection,
VI extraction, trade are prohibited. Red Vanda, Blue Vanda, Kuth,
Pitcher plant, bed domes cycad and ladies slipper orchid plants
are listed.
218.
Ans. (A)
Sol. □ The National Board for Wildlife (NBWL) is a statutory organisation constituted under the
Wildlife Protection Act, 1972. Hence, statement 1 is not correct.
□ Theoretically, the board is “advisory” in nature and advises the Central Government on
framing policies and measures for conservation of wildlife in the country-Also, serves as a n
apex body to review all wildlife related matters. Hence, statement 2 is correct.
□ The NBWL has power to review all wildlife related matters and approve projects in an
around national parks and sanctuaries. No alternation of boundaries in national parks and
wildlife sanctuaries can be done without approval of the NBWL. Hence, statement 3 is
correct.
□ The NBWL is chaired by India’s Prime Minister and its vice-chairman is Minister of
Environment, Forest and Climate Change. Hence, statement 4 is not correct.
219.
Ans. (B)
Sol. □ The Wildlife Protection Act, 1972 is an Act of the Parliament of India enacted for
protection of plants and animal species. It has six schedules which give varying degree of
protection to the flora and fauna.
□ The correctly matched pairs are as follows:

Schedule Fauna
Schedule III Hog deer; hence, pair 1 is
correctly matched.
Schedule Long-tailed tree mouse; hence,
pair 2 is not correctly
matched.
Schedule II Indian wild Dog; hence, pair
3 is not correctly
matched.
Schedule I 5 Black Buck; hence, pair 4 is
correctly matched.

220.
Ans. (D)
Sol. □ Brown Bear is a least concerned animal found in India (Jammu and Kashmir, Himachal
Pradesh, Uttarakhand), Nepal, Pakistan and China. India has four species of bears i.e., Asiatic Black,
Sloth, Sun and Himalayan Brown Bear. All Indian Bear species are listed under Appendix I in
CITES and Schedule I of the Wildlife (Protection) Act, 1972. Hence, statement 1 is not correct.
□ Capped Langur is a vulnerable animal found in India (Arunachal Pradesh, Assam,
Manipur, Meghalaya, Mizoram, Nagaland, Tripura), Bhutan, Bangladesh and Myanmar. It i s
listed under Appendix I in CITES and Schedule I of the Wildlife (Protection) Act, 1972. H e n c e ,
statement 2 is not correct.
221.
Ans. (B)
Sol. □ IUCN Status of the species
Critically Endangered: Malabar Civet, Bengal Florican, Hawksbill Turtle and Gharial.
Endangered: Forest Owlet, Green Turtle, Pygmy Hog, Mouse Deer
Vulnerable: Sambar, Urial, Sloth Bear
Least Concerned: Indian porcupine (Hystrix indica), Hyena, Kiang
Near Threatened: Chiru, Markhor
222.
Ans. (A)
Sol.
Species Protection Status
Fishing cat IUCN Red List: Vulnerable. Despite multiple threats,
the Fishing Cat was down listed to
“Vulnerable” from “Endangered” in the
IUCN Red List species assessment.
CITES: Appendix II
Indian Wildlife Protection Act, 1972:
Schedule I
Hence, pair 1 is correctly matched.
Asiatic IUCN Red List: Near Threatened
Golden cat CITES: Appendix I
Indian Wildlife Protection Act, 1972:
Schedule I
Hence, pair 2 is not correctly matched.
Leopard cat IUCN Red List: Least concern
CITES: Appendix II
Indian Wildlife Protection Act, 1972:
Schedule I
Hence, pair 3 is not correctly matched.
Marbled cat IUCN Red List: Near threatened
CITES: Appendix I
Indian Wildlife Protection Act, 1972:
Schedule I
Hence, pair 4 is correctly matched.
223.
Ans. (D)
Sol. □ Wildlife Crime Control Bureau is a statutory multidisciplinary body established by the
Government of India in 2007, under the Ministry of Environment and Forests, to combat
organized wildlife crime in the country. Hence, statement 1 is correct.
□ The Bureau has its headquarters in New Delhi. Hence, statement 2 is correct.
□ It is mandated to collect and collate intelligence related to organized wildlife crime
activities and to disseminate the same to the State and
other enforcement agencies for immediate action so as to apprehend the criminals.
□ It advises the Government of India on issues relating to wildlife crimes having national and
international ramifications, relevant policy and laws.
□ It also assists and advises the customs authorities in inspection of the consignments of
flora and fauna as per the provisions of the Wild Life Protection Act, Convention on
International Trade in Endangered Species of Wild Fauna and
Flora (CITES) and Export Import (EXIM) Policy governing such an item. Hence, statement
3 is correct.
224.
Ans. (D)
Sol. Hispid Hare and Golden Langur are listed as Endangered species while Chinese Pangolin is listed as
Critically Endangered in the IUCN List. All three species are protected under Schedule I category of
Wildlife Protection Act, 1972.
225.
Ans. (D)
Sol. □ Dugong also called ‘Sea Cow’ is one of the four surviving species in the Order Sirenia and
it is the only existing species of herbivorous mammal that lives exclusively in the sea
including in India. Hence, statement 3 is not correct.
□ They are listed as Vulnerable on the IUCN Red List and are protected in India under
Schedule I of the Wild I Life Protection Act, 1972. Hence,
statement 1 is correct.
□ According to a 2013 survey report of the Zoological Survey of India (ZSI), there were only
about 200 dugongs in the Gulf of Mannar in Tamil Nadu, the Andaman and Nicobar Islands a n d
the Gulf of Kutch in Gujarat.
□ India has signed non-legally binding Memorandums of Understanding (MoU) with CMS on
the conservation and management of Siberian Cranes (1998), Marine Turtles (2007), Dugongs
(2008) and Raptors (2016). Hence, statement 2 is not correct.
□ World Dugong Day is celebrated on 28th May, every year.

226.
Ans. (D)
Sol. □ Binturong, Clouded Leopard and Four horned Antelope are listed as Vulnerable in IUCN
Red list.
□ Asiatic wild ass is also called Indian Wild Ass, Ghor Khar or Ghud Khur. It
is predominantly found in the Little Rann of Kutch and its
surrounding areas in Gujarat. It is also found in southern Pakistan, Afghanistan, and
southeastern Iran. Its preferred environment is saline deserts like Rann of Kutch, arid grasslands and
shrublands. It is a species of ‘Near Threatened’ on the IUCN red list of threatened
animals.
227.
Ans. (B)
Sol. □ Joint Forest Management (JFM) is an approach and program initiated in the context of the
National Forest Policy of 1988 wherein state forest departments support local forest dwelling
and forest fringe communities to protect and manage forests and share the costs and
benefits from the forests with them. Hence, statement 2 is correct.
□ Communities organize themselves into a JFM Committee to protect and manage nearby
forests, guided by locally prepared byelaws and micro plans. The key element in JFM is that
communities have the power to manage the use of forests by members and also exclude
nonmembers. Thus, involvement of communities in conservation of forests and wildlife is of
paramount interest.
□ The primary and most significant basis for JFM is provided by the National Forest Policy of
1988. Hence, statement 1 is not correct.
228.
Ans. (A)
Sol. □ Olive ridley turtles are best known for their unique mass nesting called Arribada, where
thousands of females come together on the same beach to lay eggs. They lay eggs over a
period of five to seven days in conical nests.
□ The species is recognized as Vulnerable by the IUCN Red list. They are also protected
under CITES Appendix I and Schedule -1 of the Indian Wildlife (Protection) Act, 1972.
□ They are extensively poached for their meat, shell and leather, and their eggs. However,
the most severe threat they face is the accidental killing through entanglement in trawl nets a n d
gill nets due to uncontrolled fishing during their mating season around nesting beaches.
229.
Ans. (C)
Sol. □ Red Crowned Roofed Turtle is one of the 24 species endemic to India, is characterised
by the bright colours such as red, yellow, white and blue on the faces and necks of the males.
Hence, statement 1 is correct.
□ It is a freshwater turtle species found in deep flowing rivers with terrestrial nesting sites, which
is native to India, Bangladesh and Nepal. Hence, statement 2 is not correct.
□ Currently in India, the National Chambal River Gharial Sanctuary is the only area with s u b s t a n ti a l
population of the species, but even this Protected Area and habitat is under threat. The threats
include:
Loss or degradation of habitat
Sand mining and growing of seasonal crops
Drowning by illegal fishing nets
Poaching and illegal trade
□ Conservation Status
The International Union for Conservation of Nature (IUCN): Critically Endangered
Wildlife Protection Act (WPA): Schedule I
The Convention on International Trade in Endangered Species of Wild Fauna and Flora (CITES): Appendix
II
230.
Ans. (A)
Sol. □ The One Horned Rhino is the largest species of Rhino in the world found commonly in
India, Nepal, Bhutan, and Pakistan. Hence, statement 1 is correct.
□ It is listed as Vulnerable on the IUCN Red List and is protected under the Schedule I of the
Wildlife Protection Act. Hence, statement 2 is not correct.
□ In India, it is found in the state of Assam with two thirds of its population concentrated
in Kaziranga National Park Other areas where it is found are Manas National Park, Pobitora
Reserve Forest, Orang National Park, Laokhovva Reserve Forest, etc. Hence, statement 3 is n o t
correct.
□ Assam has a considerable number of rhinos in four protected areas, i.e., Pobitora Wildlife
Reserve, Rajiv Gandhi Orang National Park, Kaziranga National Park, and Manas National Pa r k .
About 2,400 of them are in the Kaziranga National Park and Tiger Reserve (KNPTR).
□ The five rhino range nations (India, Bhutan, Nepal, Indonesia and Malaysia) have signed
a declaration ‘The New Delhi Declaration on Asian Rhinos 2019’ for the conservation and
protection of the species.
231.
Ans. (C)
Sol. □ Indian Star Tortoise is a medium-sized tortoise endemic to the Indian subcontinent, more
specifically in the Central and Southern parts of India, in West Pakistan and in Sri Lanka.
□ The species faces two threats one is loss of habitat to agriculture another is illegal
harvesting for the pet trade. According to the Wildlife Crime Control Bureau, 90% of the trade
of Star Tortoise occurs as part of the International pet market.
□ In the IUCN Red List, it has been placed in the Vulnerable category. Hence, statement 2 is
correct.
□ This species has received protection as a Schedule IV list species of the Wildlife
(Protection) Act 1972. This list is for species that are not endangered. It includes protected
species but the penalty for any violation is less compared to the Schedules I and II. Hence,
statement 3 is not correct.
□ India’s proposal to upgrade the protection status of Star Tortoise from Appendix II to
Appendix I was approved by the Convention on International Trade in Endangered Species o f
Wild Fauna and Flora (CITES). Hence, statement 1 is correct.
232.
Ans. (B)
Sol. □ Shailesh Nayak committee was constituted in June 2014, and it submitted its report in
January 2015
□ The committee recommended relaxation on the terms set up by the Coastal Regulation
Zone (CRZ) 2011 notification. The major objective behind the recommendations was to boost
tourism, port construction and real estate.
□ The committee suggested diluting the regulatory powers of the Central Government in
the coastal areas, except for those activities which require environmental clearances, all other
activity should fall under the ambit of state and local planning bodies.
□ Based on the recommendations of Shailesh Nayak committee, the suggestions were given b y
the coastal states and union territories, and the CRZ 2018 notifications were issued.
233.
Ans. (D)
Sol. □ Mangrove restoration is a management strategy to compensate for degraded and lost
ecosystem goods and services. It has the potential to increase the mangrove resource base,
provide employment to local populations, Protect fragile tropical coastlines and enhance
biodiversity and fisheries productivity. Hence, statement 1 is correct.
□ Restoration Is recommended when an ecosystem has been altered to such an extent that it
can no longer selfcorrect and/or self-rencw.
□ Typology of Mangrove Restoration
Natural Regeneration
This approach uses natural occurring mangrove propagules (or seeds) to restock degraded sites. In this
case, regeneration is from direct, freely falling and dispersed mangrove propagules, where
species composition of the regenerated forest depends on the species types and combinations of
the adjacent forest from where propagules are dispersed. Hence, statement 3 is correct.
Important biological and physical factors that determine success for natural regeneration include forest
conditions, tides and soil stability.
Artificial Regeneration
Artificial regeneration entails direct planting of desired propagules and saplings (of less than 1.2 m high), and
rarely the use of small trees (of up to 6 m high) of chosen species at the designated restoration site.
Use of propagules and nursery-raised saplings is the most common method of mangrove restoration.
There are several advantages of using artificial regeneration, for example, the species composition and
distribution can be controlled, genetically improved stocks can be introduced and, pest infestation
can be controlled. Hence, statement 2 is correct.
234.
Ans. (C)
Sol. □ Agro forestry focuses on the wide range of trees grown on farms and other rural areas. It is
a collective name for land-use systems involving trees combined with crops and/or animals
on the same unit of land.
□ Types of Agro-forestry in non-forest areas:
Recreational forestry: It is also known as Aesthetic forestry which is defined as the practice of forestry with the
objective of developing or maintaining a forest of high scenic value (by raising flowering trees and
shrubs) for the urban and rural population.
Extension forestry: It is the practice of forestry in areas devoid of tree growth and other vegetation situated
in places away from the conventional forest areas with the object of increasing the area under tree
growth. For example, Mixed forestry, Shelter Belts, Linear Strip Plantations.
Farm forestry: It is defined as the practice of forestry in all its aspects in and the around the farms or village lands
integrated with other farm operations. All in all, it promotes commercial tree growing.
235.
Ans. (C)
Sol. □ March 3 is chosen as World Wildlife Day as the Convention on International Trade in
Endangered Species of Wild Fauna and Flora (CITES) was adopted on this date in 1973.
□ It was proclaimed by the United Nations General Assembly (UNGA) in 2013.
236.
Ans. (B)
Sol. □ The International Union for Conservation of Nature (IUCN) Red List is a critical indicator
of the health of the world’s biodiversity.

Species Current IUCN Status


Great Indian Bustard Critically Endangered
Musk Deer Endangered
Red Panda Endangered
Asiatic Wild Ass Near Threatened
Kashmir Stag Critically Endangered
Cheetal Least Concern
Blue Bull Least Concern
Snow Leopard Vulnerable
Rhesus Monkey Least Concern
Saras (Crane) Vulnerable
Lion Tailed Macaque Endangered
Hanuman Langur Least Concern
Irrawaddy Dolphin Endangered

237.
Ans. (B)
Sol. □ Tundra ecosystems are treeless regions found in the Arctic and on the tops of mountains,
where the climate is cold and windy, and rainfall is scant. Tundra lands are covered with s n ow
for much of the year, but summer brings bursts of wildflowers. The types of Tundra include Arctic
Tundra and Alpine Tundra.
The arctic tundra is a continuous belt restricted to the high latitudes of the northern hemisphere in a belt around
the Arctic Ocean. It occupies the northern fringe of Canada, Alaska, European Russia, Siberia and
island group of Arctic Ocean. Hence, statement 1 is not correct.
Alpine tundra are located at very high elevations atop mountains, above the treeline (the line or altitude above
which no trees grow). These windswept areas are characterized by an abundance of barren rocks or
thin soils. In the well-drained soil, the growing season is only 180 days. Hence, statement 2 is correct.
238. Ans. (C)
Sol. □ Forests cover about 20% of land on the earth. Forests are large areas supporting rich
growth of trees. Depending on the climate and type of trees, they are generally grouped into:
Tropical Rainforests
Tropical Deciduous Forests
Temperate Deciduous Forests
Boreal or North Coniferous Forests
Mediterranean Forests (Chaparral)
□ Forest ecosystems include complex assemblages of different kinds of biotic communities.
The nature of soil, climate and local topography determine the distribution of trees and their
abundance. Hence, statement 1 is correct.
□ Tropical Rainforest
Rainforests play an important role in maintaining biological diversity, sequestering and storing carbon, global
climate regulation, disease control, and pollination. They have been called the “world’s largest
pharmacy”, because over one quarter of natural medicines have been discovered within them. Hence,
statement 2 is correct.
Tropical rainforest occurs in areas having high temperature and high humidity (above 200 cm o rainfall per year).
Such forests are found on both sides of equator, especially in the Western Coast of India South East
Asia, some parts of Africa, especial Congo Basin and South America especially Amazon Basin. Hence,
statement 3 is correct.
239.
Ans. (B)
Sol. □ In-situ conservation or on-site conservation refers to situations where the object is
maintained in its natural habitat, within the community, of which it forms a part. It is one o f
two basic conservation strategies alongside ex-situ consecration.
Article 8 of the Convention on Biological Diversity (CBD) specifies in-situ conservation as the primary conservation
strategy, and states that ex-situ measures should play a supportive role to reach conservation targets.
□ In-situ management approaches can either be targeted at the population of selected species (species-
centred) or whole ecosystems (ecosystem-based). The established natural habitats are:
National Parks;
Sanctuaries;
Biosphere Reserves;
Reserved Forests;
Natural Reserves. Hence 1, 2 and 3 are not correct.
□ Ex-situ (off-site) conservation is a set of conservation techniques involving the transfer of a target
species away from its native habitat.
The main purposes of ex-situ collections are the rescue and preservation of threatened genetic material and the
breeding of species for reintroduction in cases where a species’ continued survival in its native habitat
is threatened.
Zoological Parks or Botanical gardens, Seed banks, Horticulture are important centres for ex-situ conservation.
Hence, 4 is correct
240.
Ans. (C)
Sol. □ Munnar is home to the highest concentration of Neelakurinji plants in the country - spread
over 3000 hectares. The mass flowering of neelakurinji happens only once in 12 years - each
shrub reproduces once in its lifetime and dies after flowering. It takes another 12. years for the
seeds to sprout again and grow up to 30 to 60 centimetres high, for another glorious bloom.
Hence, statement 1 is correct.
□ The Neelakurinji plant belongs to the genus Strobilanthes, which is a tropical plant species
found in Asia and Australia. There are about 450 species of Strobilanthes in the world, of which
146 are found in India and of them, about 43, in Kerala. Hence, statement 2 is correct.
□ The mass flowering Neelakurinji provides a feast for butterflies, honeybees and other
insects. The purple flowers hold a large amount of nectar, which attracts the eastern honeybees
(Apis cerana).
Therefore, option (c) is the correct answer.
241.
Ans. (D)
Sol. □ Grasslands are areas dominated by grasses. They occupy about 20% of the land on the
earth’s surface.
□ Grasslands occur in both tropical and temperate regions where rainfall is not enough to □
support the growth of trees.
□ Grassland at different places:

Place Name of Grassland


North America Prairies. Hence, pair 1 is not
correctly matched.
Eurasia Steppes. Hence, pair 2 is not
correctly matched.
Pampas. Hence, pair 3 is
South America correctly matched.
Africa Savanna. Hence, pair 4 is
correctly matched.
Australia Downs
South Africa Veld

242.
Ans. (A)
Sol. □ Hog deer is a small deer with dark brown to yellow brown fur and white-tipped tails. This
species is typically found in riverine flooded grasslands of Southeast Asia. Hence, statement 2 is
correct.
□ The IUCN Red List of Threatened Species is the world’s most comprehensive inventory of
the global conservation status of plant and animal species. In the IUCN Red List, hog deer h a s
been listed as an endangered species. An endangered species is an animal or plant that i s
considered at risk of extinction. Hence, statement 1 is not correct.
□ Animals that are declared vermin by the Environment Ministry are placed under Schedule-
V of the Wildlife Protection Act, 1972. Once declared vermin, a species can be hunted or culled
without restriction. Hog deer has been declared a protected species under the Wildlife
Protection Act, 1972 and has been placed under Schedule-1 of the Act. Hence, statement 3 is
not correct.
243.
Ans. (C)
Sol. □ The Ganges River dolphin, or Susu, inhabits the Ganges-Brahmaputra-Meghna and
Karnaphuli-Sangu river systems of Nepal, India, and Bangladesh. Hence, statement 2 is n o t
correct.
□ The Gangetic dolphin is among the four “obligate” freshwater dolphins - the other three
are the Baiji now likely extinct from the Yangtze river in China, Bhulan of the Indus in
Pakistan and the Boto of the Amazon River in Latin America.
□ It has been classified as Endangered in the International Union for Conservation of
Nature’s Red List of Threatened Species. Hence, statement 1 is not correct.
244.
Ans. (D)
Sol. □ As per the International Union for Conservation of Nature’s (IUCN) red list of threatened
species, a taxon is considered extinct when there is no reasonable doubt that the last individual has
died.
□ Dodo: The Dodo which was endemic to Mauritius is an extinct species. It was a flightless
bird and the last sighting of one was in 1662. Hence, 1 is not correct.
□ The Passenger Pigeon: The birds’ traditional habitats were the large forests of eastern
North America. Due to widespread hunting and trapping of the birds it led to its extinction.
Hence, 2 is not correct.
□ Tasmanian Tiger: Thylacines (Thylacinus cynocephalus) known as Tasmanian tigers due
to their stripes, were the largest modern carnivorous marsupial according to the Smithsonian
Institution. They once existed across the Australian continent but due to their habitat destruction,
they were declared a protected species in 1936, the same year the last known specimen
died. Hence, 3 is not correct.
245.
Ans. (A)
Sol. □ The International Tropical Timber Organization (ITTO) is an intergovernmental
organisation for promoting the sustainable management and conservation of tropical forests a n d
trade in tropical forest resources. India is a member of ITTO. ITTO was established under
the provisions of the International Tropical Timber Agreement (ITTA), 1983. It was e s t a b l i s h e d
under the auspices of the United Nations in 1986. Hence, statement 1 is not correct.
□ ITTO began operations in late 1986 and its headquarter is located in Yokohama, Japan.
Hence, statement 3 is not correct.
□ Its members manage about 80 per cent of the world’s tropical forests and are responsible
for 90 per cent of the global tropical timber trade. It facilitates funding for forestry projects i n
tropical timber producing countries. Hence, statement 2 is correct.
□ It also facilitates discussion, consultation and international cooperation on issues relating
to the International trade and utilization of tropical timber and fhe sustainable management of its
resource base.
246.
Ans. (C)
Sol. □ Moringa (Moringa oleifera) is a fast-growing, droughtresistant tree native to the Indian
subcontinent. Hence, statement 1 is correct.
□ It is widely cultivated for its young seed pods and leaves, used as vegetables and for
traditional herbal medicine. It is also used for water purification. Hence, statement 2 is
correct.
□ It has various healthy compounds such as various vitamins, important elements such as
Iron, Magnesium etc. and is extremely low on fats and contains no cholesterol.
□ Globally, the demand for Moringa products, such as Moringa Leaf Powder and Moringa
Oil, Moringa as a nutritional supplement and in food fortification has been witnessing h e a l t h y
growth.
□ Its usage has been well received among the global consumers for its nutritional, medicinal,
culinary uses.
247
Ans. (D)
Sol. □ Bustards are large, terrestrial birds that belong to several species, including some of the
largest flying birds.
□ The International Union for Conservation of Nature (IUCN) recognises two distinct
species of Houbara Bustard found in:
North Africa (Chlamydotis undulata) and
Asia (Chlamydotis macqueenii). Hence, statement 1 is not correct.
□ The population of the Asian Houbara Bustards extends from northeast Asia, across central
Asia, the Middle East, and the Arabian Peninsula to reach the Sinai desert (Egypt).
□ After breeding in the spring, the Asian Bustards migrate south to spend the winter in
Pakistan, the Arabian Peninsula and nearby Southwest
Asia.
□ Poaching, unregulated hunting along with degradation of its natural habitat are the major
reasons of its decline.
□ Conservation Status
IUCN Status: Vulnerable
Convention on International Trade in Endangered Species of Wild Fauna and Flora (CITES): Appendix I
Convention on the Conservation of Migratory Species of Wild Animals (CMS): Appendix II. Hence, statement 2
is not correct.
3. Protected Area Network
Solution:
248.
Ans. (C)
Sol. □ Nagarhole National Park
This park is also known as Rajiv Gandhi National Park and is located in the two districts of Karnataka, namely
Mysore and Kodagu.
The park was established as a wildlife sanctuary in 1955 and was upgraded into a national park in 1988. The park
was declared as the 37th Tiger reserve in 1999.
Kabini, a tributary of the Cauvery River, is the largest river draining the park. Hence, 1 is correct.
□ Papikonda National Park
This park is spread over 1012.86 square kilometres area in East and West Godavari districts of Andhra Pradesh.
The National Park was termed as a reserve forest in 1882, a wildlife sanctuary in 1978, and as a national park
from 2008.
The park lies along the left and right banks of the river Godavari and cuts through the Papikonda hill range of
Eastern Ghats. Hence, 2 is not correct.
□ Sathyamangalam Tiger Reserve
Sathyamangalam Wildlife Sanctuary and Tiger Reserve is a protected area and Tiger Reserve along the Western
Ghats in the Erode district of the state of Tamil Nadu.
In the northern part of Erode district, Palar river flows and drains into Cauvery river. Hence, 3 is correct.
□ Wayanad Wildlife Sanctuary
Located in Kerala, Wayanad Wildlife Sane. (WWS) is an integral part of the Nilgiri Bioshere Reserve. It was
established in 1973.
It is contiguous to the tiger reserves of Nagarhola and Bandipur of Karnataka and Mudumalai of Tamil Nadu.
Kabini river (a tributary of Cauvery river) flows through the sanctuary. Hence, 4 is correct.
249.
Ans. (A)
Sol. □ Hard ground swamp deer or Barasingha (Rucervusduvaucelii), the state animal of Madhya
Pradesh, is seeing a revival in the Kanha National Park and Tiger Reserve (KNPTR).
□ Swamp Deer was close to extinction in the Kanha National Park. However, with the
conservation efforts, the population currently numbers around 800.
□ The deer is endemic to the Kanha National Park and Tiger Reserve on the Maikal Range
of Satpura Hills. Measures like captive breeding and habitat improvement were used for the
conservation of the species.
250.
Ans. (C)
Sol. □ Critical Tiger Habitats (CTH), also known as core areas of tiger reserves, are identified
under the Wild Life protection Act, 1972 based on scientific evidence that “such areas are
required to be kept as in-violative for the purpose of tiger conservation, without affecting
the rights of the Scheduled Tribes or such other forest-dwellers”
□ The CTHs are notified by the state government in consultation with the expert committee
constituted for the purpose.
□ Area of the core/critical tiger habitat:
0 Corbett (Uttarakhand): 821.99 sq.kms
O Ranthambore (Rajasthan): 1113.36 sq.kms
□ Sundarbans (West Bengal): 1699.62 sq.kms
□ Nagarjunsagar Srisailam (part of Andhra Pradesh) 2595.72 sq. kms
251.
Ans. (C)
Sol. □ National Chambal Sanctuary, also called the National Chambal Gharial Wildlife
Sanctuary, is located along the Chambal River on the tri-junction of Rajasthan, Madhya
Pradesh and Uttar Pradesh. It is known for critically endangered gharial (small crocodiles), t h e
red-crowned roof turtle and the endangered Ganges river dolphin.
□ It was first declared as a Protected Area in Madhya Pradesh in 1978 and now constitutes
a long narrow eco-reserve co-administered by the three states. Within the sanctuary the
Chambal River cuts through mazes of ravines and hills with many sandy beaches along its
banks.
252.
Ans. (A)
Sol. □ The Agasthyamalai Biosphere Reserve is located in the Western Ghats on the border of the
districts of Kollam and Thiruvanthapuram. Consisting mostly of tropical forest, the site is home
to 2,254 species of higher plants, including about 400 that are endemic.
□ It is also a unique genetic reservoir of cultivated plants, I in particular cardamom, jamun,
nutmeg, pepper and plantain. I
□ Three Wildlife Sanctuaries - Shendurney, Peppara and Neyyar, and the Kalakad
Mundanthurai Tiger Reserve are located in the site.
253.
Ans. (D)
Sol. □ The Forest Act of 1878 classified forests in India into three categories i.e., reserved,
protected and village forests.
□ The Act attempted to regulate the collection of forest produce by forest dwellers and some
activities declare: as offences and imprisonment and fines were impose: in this policy to
establish the state control over forests.
□ Types of Forests
Reserved Forests: These are the most restricted forests and are constituted by the State Government on any
forest land or wasteland which is the proper of the Government.
Protected Forests: The State Government is empowered to constitute any land other than reserved forests as
protected forests over which the Government has proprietary rights and the power to issue rules
regarding right and the power to issue rules regarding the use of such forests.
Village Forests: These are the one in which the State Government may assign to ‘any village community the
rights of Government to or over any land which has been constituted a reserved forest’.
254.
Ans. (A)
Sol. □ Pakhui Wildlife Sanctuary Iles In the foothills of the Eastern Himalaya in the East Kameng
district of Arunachal Pradesh. It was declared as a sanctuary in 1977 and a tiger reserve (also
known as Pakke finer Reserve) in 2002.
□ At least 40 mammal species occur in the Pakhui Wildlife Sanctuary.
Large Cats: Tiger, Leopard and Clouded Leopard
Canids: Wild Dog and Asiatic Jackal
Herbivore Species: Elephant, Barking Deer, Gaur and Sambhar
Commonest Monkeys: Rhesus and Assamese Macaques, and Capped Langur
The site is also home to as many as sixteen species of civets, weasels and mongooses.
255.
Ans. (B)
Sol. □ A World Heritage Site is a place that is listed by UNESCO for its special cultural or
physical significance. The list of World Heritage Sites is maintained by the international
‘World Heritage Programme’, administered by the UNESCO World Heritage Committee.
□ This is embodied in an international treaty called the Convention concerning the
Protection of the World Cultural and Natural Heritage,
adopted by UNESCO in 1972.
□ India has 38 world heritage sites that include 30 Cultural properties, 7 Natural properties
and 1 mixed site.
□ India’s Natural World Heritage Sites are mentioned below:
Great Himalayan National Park Conservation Area (2014)
Kaziranga National Park (1985)
Keoladeo National Park (1985)
Manas Wildlife Sanctuary (1985)
Nanda Devi and Valley of Flowers National Parks (1988, 2005)
Sundarban National Park (1987)
Western Ghats (2012)
256.
Ans. (D)
Sol. □ Dachigam National Park is located 22 km from Srinagar in Jammu and Kashmir. It covers
an area of 141 sq km. The name stands for ‘ten villages’, which is in memory of the ten
villages that were relocated in order to create the park.
Dachigam was initially established to ensure the supply of clean drinking water to Srinagar city. A protected area
since 1910, it was declared as a national park in 1981. The park is best known as the home of the
Hangul or Kashmir stag.
Dachigam National Park is the only habitat area where Kashmir stag is found.
□ Kashmir Stag locally known as Hangul is a sub-species of the European red deer. It is the
only sub-species of red deer in India.
First identified by Alferd Wagner in 1844, the species is believed to have traveled all the way from Bukhara in
Central Asia to Kashmir.
The animal is classified as ‘critically endangered’ by the International Union for Conservation of Nature (IUCN).
The challenges that Hangul faces include poaching, threats from insurgency and the border conflict between
India and Pakistan.
257.
Ans. (C)
Sol. □ Manas tiger reserve in Assam has a core/critical tiger habitat area of 840 sq. km.
□ Pakke tiger reserve in Arunachal Pradesh has a core/ critical tiger habitat area of 683 sq.
km.
□ Nagarjunasagar Srisailam in Andhra Pradesh, is the largest tiger reserve with a core/critical
tiger habitat area of 2595 sq. km.
□ Periyar tiger reserve in Kerala has a core/critical tiger habitat area of 881 sq. km.
258.
Ans. (C)
Sol. □ Bhadra located in Karnataka,
□ Similipal located in Odisha,
□ Pachmarhi located in the state of Madhya Pradesh,
□ Shikari Devi is located in Himachal Pradesh in the north.
259.
Ans. (B)
Sol. □ A committee constituted by the Ministry of Environment, Forest and Climate Change has
recommended the relocation of Asiatic Lions from Gir forests, Gujarat to the Kuno-Palpur
sanctuary as it was found to be the most suitable site on account of its area, shape and
vegetation.
□ Kuno-Palpur is situated in the Sheopur district of Madhya Pradesh. It was notified as a
sanctuary in the year 1981.
□ The protected region has a core area of 345 sq. km., and an outer buffer area of 890 sq.
km. They both constitute the wildlife division, created in the year 2002 to provide an
alternative home to the endangered Asiatic Lion. The River Kuno divides the sanctuary from
north to south.
260.
Ans. (C)
Sol. □ Rani Jhansi Marine National Park, notified as a national park in 1996 is located in the
Andaman and Nicobar Islands in the Bay of Bengal. The park spreads across three Islands
namely John Lawrence Island, Henry Lawrence Island and Outram Island.
□ The national park is known for its coral reefs and marine life. Vegetation consists of
terrestrial moist forests and mangroves whereas fauna consists of crocodiles, dugong and fruit-
eating bat species.
261.
Ans. (B)
Sol. Established in 1936 as Hailey National Park in Uttarakhand, Jim Corbett National Park is the first
national park in India. The park is also known as the place where Project Tiger was fin launched in
1973.
262.
Ans. (C)
Sol. The correct order of tiger reserves from North to South in India is Corbett (Uttarakhand) – Sariska
(Rajasthan) – Similipal (Odisha) - Periyar (Kerala)
263.
Ans. (D)
Sol. □ India’s marine national parks and water sanctuaries ar. hotspots of rich biodiversity and
habitat to various marine organisms like Olive Ridley sea turtles, saltwater crocodile among
others.
□ Mahatma Gandhi Marine National Park of India i$ situated in Andaman and Nicobar island
and lies about 29 km., from Port Blair. It is also known as Wandoor Marine National Park. It is an
important breeding ground fortuities.
□ Jolly Buoy and Red skin Islands are famous islands of Mahatma Gandhi Marine National
Park.
264.
Ans. (B)
Sol. □ There are 42 islands on the Jamnagar coast in the Marine (Gulf of Kutch) National Park,
most of them surrounded by reefs. Major ecosystem types available are coral reefs, m a n g ro v e s ,
mudflats, creeks, Sea grass, and Sea weeds, estuaries, sandy strands to saline grasslands, marshy
areas and rocky shores. All ecosystems support rich marine life and bird life.
□ The Marine National Park was established in 1982 under the provisions of the Wildlife
(Protection) Act, 1972 of India. Marine (Gulf of Kutch) National Park in Jamnagar, is the fi r s t
National Marine Park of India.
□ A diverse range of marine habitat that support 52 species of corals, seven kinds of
mangroves and species of birds such as painted stork, darter and black-necked Ibis habitat t h e
archipelago of islands. The area is also inhabited by colonies of dolphins, green sea turtles and
the endangered whale shark.
265.
Ans. (D)
Sol. □ Marine Protected Areas in India are the area defined under IUCN guidelines. They limit
anthropogenic activities and exploitation of resources in these areas.
□ Bhitarkanika is India’s second largest mangrove ecosystem after the Sunderbans.
Bhitarkanika is home to the largest congregation of the endangered Salt Water Crocodile in t h e
country.
□ The Sundarban National Park is located in the south-east of Kolkata in the District of West
Bengal and forms part of the Gangetic Delta. The site is intersected by a complex network o f
tidal waterways, mudflats and small islands of salt-tolerant mangrove forests, and presents
an excellent example of ongoing ecological processes.
□ Gahirmatha Marine Sanctuary is a marine wildlife sanctuary located in Odisha. It is known
for the world’s largest rookery of Olive Ridley sea turtles.
□ The Gulf of Mannar Marine National Park was established in 1982 under the provisions of
the Wildlife (Protection) Act, 1972. It spreads along 560 km., stretch between Rameswaram a n d
Tuticorin. It is lying within the biosphere of Mannar.

266.
Ans. (A)
Sol. □ Mahananda Wildlife Sanctuary: It is located on the west bank of river Teesta in the
southern part of Darjeeling district in West Bengal.
□ Lakhari Valley Wildlife Sanctuary: It is situated in the state of Odisha. It was established
in the year 1985. The place receives a huge amount of rainfall every year due to its proximity to
the Bay of Bengal. The sanctuary is full of rich tropical forest, valleys, hills, water sources etc. It is
one of the elephant populated sanctuaries in India.
□ Pachmarhi Wildlife Sanctuary: It is located in Madhya Pradesh in the biogeographical region
of the Deccan Peninsula and the Biotic Province of Central India. UNESCO designated it a
biosphere reserve in 2009.
□ Chandra Prabha Wildlife Sanctuary: It is situated in the eastern region of Uttar Pradesh.
The forest is named after the Chandraprabha River; meaning ‘The Luminescence of Moon’. A
variety of wild animals like Black Duck, Cheetal, Sambhar, Nilgai, Wild Boar, Porcupine and Indian
Gazelle, etc., are found here.
267.
Ans. (B)
Sol. □ Periyar National Park: It is a protected area located in the districts of Idukki and
Pathanamthitta in Kerala and was declared as a national park in 1982. Though the national park
is famous for elephants, species such as Gaur, Wild Pigs, Sambar, Barking Deer, rare l i o n - t a i l e d
macaque, the Nilgiri Langur, can also be spotted.
□ Rajaji National Park: It is located in the state of Uttarakhand and was declared as a national
park in 1983. The park is famous for elephants and for its conservation and habitat protection, a
Centrally Sponsored Scheme Project Elephant is going on. Along with elephants Cheetal, Barking
deer, Sambar deer, Wild Boar, antelopes such as the Nilgai, Goral etc., can also be seen in this park.
□ Manas National Park: It is located in the state of Assam and was declared as a National Park
in 1990. The park is is home to a great variety of wildlife, including many endangered species, such
as the tiger, pygmy hog, Indian rhinoceros and Indian elephant.
□ Dudhwa National Park: It is a protected area in Uttar Pradesh located on the India-Nepal
border. It stretches mainly across the Lakhimpur Kheri and Bahraich districts. Tigers, one-
horned rhinoceros and barasingha (swamp deer) are the major attractions of the Dudhwa
National Park.
268.
Ans. (B)
Sol. Established in 1936 as Hailey National Park in Uttarakhand, Jim Corbett National Park is the first
national park in India. The park is also known as the place where Project Tiger was first launched in
1973.
269.
Ans. (B)
Sol. □ Bhiterkanika National Park is the core area of the Bhiterkanika Wildlife Sanctuary located
in the Kendrapara district in the state of Odisha. It was designated as national park in 1998 a n d
as a Ramsar site by the state of Odisha in 2002. The national park is home to saltwater
crocodile (Crocodylus porosus), Indian python, king cobra, black ibis, darters and many other
species of flora and fauna. It hosts a large number of mangrove species. | The national park
and wildlife sanctuary are inundated by the rivers Brahmani, Baitarani, Dhamra, Pathsala. i
Hence, pair 1 is correctly matched.
□ Desert National Park is located in the Thar Desert and is situated near to Jaisalmer,
Rajasthan. The major wildlife found in the park are chinkara, desert fox, blackbuck, Bengal fox,
spiny-tail lizard, sand fish, desert monitors, j chameleons, etc. The main.attraction of the national
park is the Great Indian Bustard which is an endangered bird species. Hence, pair 2 is correctly
matched.
□ Eravikulam National Park, located in Kerala, holds the largest viable population of the
endangered Nilgiri Tahr and is a famous habitat
of Neelakurinji, which blooms j once in 12years.ApartfromTahr,theparkis
an abode i of other little known fauna such as nilgiri marten (endemic), ruddy mongoose,
small clawed otter, dusky j striped squirrel etc.
□ Hoolock Gibbons are found in several states of North-East including Assam, Arunachal
Pradesh, Manipur, Meghalaya, Tripura and Nagaland. They are also spotted
270.
Ans. (C)
Sol. □ National Parks in India are declared by notification of the State Governments by reason of
their ecological, faunal, floral, geomorphological, or zoological association or importance. T h e
notification specifies the limits of the area notified as a National Park. Further, no alteration of
the boundaries of a National Park can be made except on a resolution passed by the Legislature of
the State. Hence, statement 1 is correct.
□ Limited biotic interference like scientific research, tourism, photography, transaction of
lawful business, etc., are allowed in the Wildlife Sanctuary only after the permission of the Chief
Wildlife Warden. Hence, statement 3 is correct.
□ Biosphere reserves is sites established by countries and recognized under UNESCO’s Man
and the Biosphere (MAB) Programme to promote sustainable development based on local
community efforts and sound science. The purpose of the formation of the biosphere reserve is
to conserve in situ all forms of life, along with its support system, in its totality, so that it could
serve as a referral system for monitoring and evaluating changes in natural ecosystems. Hence,
statement 2 is not correct.
271.
Ans. (B)
Sol. □ World Wetlands Day is celebrated every year on 2nd February. In 2021, the Ramsar
Convention on Wetlands would also complete its 50 years. The global treaty popularly
known as Ramsar Convention was adopted in 1971 and came into force in 1975 following
negotiations.
□ The main objective of the treaty is to promote and wise use of all wetlands through
regional action and international cooperation. Currently, the treaty has been accepted by
171 nations including India.
□ The treaty came into force on February 01, 1982 in India.
□ In 2020, five new sites have been added into the list taking the total Ramsar sites numbers
to 42 spreading over 10814.38 sq. km., area.
272.
Ans. (C)
Sol. □ Wildlife Protection Act, 1972 is one of the most robust acts on wildlife protection. It
provides for the protection of wild animals, birds and plant species of the country. This
ensures the ecological and environmental security.
□ National Board for Wildlife (chaired by the Prime Minister) is a statutory Board
constituted officially in 2003 under the Wildlife Protection Act, 1972. The board is ‘advisory’ in
nature and can only advise the Government on policy making for conservation of wildlife.
Hence, statement 1 is not correct.
□ It is an important body because it serves as an apex body for the review of all wildlife-
related matters and for the approval of projects in and around national parks and sanctuaries.
□ The Standing Committee of NBWL approves all the projects falling within protected
wildlife areas or within 10 km., of them. It is chaired by the Minister of Environment Forest and
Climate Change. Hence, statement 2 is not correct.
273.
Ans. (C)
Sol. □ A protected area is a clearly defined geographical space, recognised, dedicated and
managed, through legal or other effective means, to achieve the long term conservation of
nature with associated ecosystem services and cultural values.
□ India has systematically designated its Protected Areas in four legal categories — National
Parks, Wildlife Sanctuaries, Conservation Reserves and Community Reserves under the
Wildlife (Protection) Act, 1972.
□ India has a network of 981 Protected Areas including 104 National Praks, 566 wildlife
Sanctuaries, 97 Conservation Reserves and 214 community Reserves covering a total of
1,71,921 km2 of geographical area of the country which is approximately 5.03%.
□ No alteration of boundaries of a Sanctuary/National Park can be made without the
recommendations of the National Board for Wildlife (constituted under the Wildlife P r o t e c ti o n
Act, 1972). There is a Standing Committee of the board which approves all the projects
falling within protected wildlife areas or within 10 km., of them. Hence, statement 1 i s
correct.
□ The Wildlife (Protection) Amendment Act, 1991 provided for the inclusion of territorial
waters in areas to be declared as sanctuaries for the protection of off-shore marine flora a n d
fauna. Hence, statement 2 is correct.
274.
Ans. (D)
Sol. □ National Parks are declared by State governments under the Wildlife Protection Act, 1972
for the protection and conservation of outstanding natural fauna, flora, geological f o r m a ti o n s
and natural scenic.
□ There are 104 existing national parks in India covering an area of 43,716 km2, which is
1.33% of the geographical area of the country (National Wildlife Database, December 2020).
□ Following activities are prohibited under the National Parks including:
Hunting, killing or capturing of fauna; hence, 1 is correct.
Deprivation of any wild animal of its habitat; hence, 2 is correct.
Destruction and collection of flora; hence, 5 is correct.
Use of weapons; hence, 3 is correct.
Grazing of any livestock; hence, 4 is correct.
□ No human activity is permitted inside the national park, except for the ones permitted
by the Chief Wildlife Warden of the state. It is the authority who shall control, manage and
maintain all sanctuaries and for that purpose, within the limits of any sanctuary.
275.
Ans. (B)
Sol. □ Protected areas are those in which human occupation or at least the exploitation
of resources is limited. There are several kinds of protected areas, which vary by level of
protection depending on the enabling laws of each country or the regulations of the international
organizations involved.
□ The term “protected area” also includes Marine Protected Areas, Conservation Reserves
and Community Reserves. Hence, statement 1 is not correct.
□ A marine protected area (MPA) is essentially a space in the ocean where human
activities are more strictly regulated that the surrounding
waters, similar to park we have on land regulated than the surrounding waters e -i Strictly we
have on land.
□ Conservation reserves and community reserves in India are terms denoting protected areas
of India which typically act as buffer zones to or connectors and migration corridors between
established national parks wildlife sanctuaries and reserved and protected forests of India.
□ These protected area categories were first introducted in the Wildlife (Protection)
Amendment Act of 2002 which is the amendment to the
Wildlife Protection Act of 1972. These categories were added because of reduced protection in
and around existing or proposed protected areas due to private ownership of land, and land use.
Hence, statement 2 is correct.
276.
Ans. (A)
Sol. □ Sacred groves comprise of patches of forests or natural vegetation that are usually
dedicated to local folk deities or tree spirits. These spaces are protected by local communities
because of their religious beliefs and traditional rituals that run through several generations.
□ Sacred groves have been legally protected under ‘community reserves’ in the Wildlife
(Protection) Amendment Act, 2002. Hence, statement 2 is not correct.
□ In India, there are over lakh sacred groves across different states called by different names
like Kaavu in Malayalam, Koyilkaadu in Tamil, Orans in Rajasthan, Devarakaadu in Karnataka,
and Sernas in Madhya Pradesh.
□ In general, no one is permitted to cut any tree or plant, kill animals and birds, or harm any
form of life in this area.
□ The Gond tribe of Central India prohibits the cutting of trees but allows fallen parts to be
used. The Garo and Khasi tribes of north-eastern India prohibit any human interference in the
sacred groves. Hence, statement 1 is correct.
277.
Ans. (C)
Sol. □ The MAB programme was launched in 1971 and is an intergovernmental scientific
programme that aims to establish a scientific basis for enhancing the relationship between
people and their environments. Hence, statement 1 is correct.
□ It combines the natural and social sciences, economics and education to improve human
livelihoods and the equitable sharing of benefits and safeguards natural and managed
ecosystems, thus promoting innovative approaches to economic development that are
socially and culturally appropriate and environmentally sustainable.
□ The World Network of Biosphere Reserves of the MAB Programme consists of a dynamic
and interactive network of sites of excellence. It promotes North-South, South-South and
South-North-South collaboration and represents a unique tool for international cooperation
through the exchange of experiences and know-how, capacity-building and the promotion of b e s t
practices among Biosphere Reserves. Hence, statement 2 is correct.
□ It contributes to the 2030 Agenda and the Sustainable Development Goals (SDGs)
through participatory dialogue; knowledge sharing; poverty reduction and human well-being
improvements; respect for cultural values and society’s ability to cope with change.
□ The World Network of Biosphere Reserves currently counts 714 sites in 129 countries all
over the world, including 21 transboundary sites.
278.
Ans. (D)
Sol. □ Biosphere reserves are ‘learning places for sustainable development’. These are
representative parts of natural and cultural landscapes extending over large areas of terrestrial or
coastal/marine ecosystems or a combination thereof and representative examples of
biogeographic zones/provinces.
□ They are sites for testing interdisciplinary approaches to understanding and managing
changes and interactions between social and ecological systems, including conflict p r e v e n ti o n
and management of biodiversity. Each site promotes solutions reconciling the conservation of
biodiversity with its sustainable use.
□ Biosphere Reserves involve local communities and all interested stakeholders in planning
and management. They integrate three main functions including:
Conservation of biodiversity and cultural diversity;
Economic development that is socio-culturally and environmentally sustainable;
Logistic support, underpinning development through research, monitoring, education and training.
□ The idea of the biosphere reserve was initiated by UNESCO in 1974 under the Man and
the Biosphere (MAB) Programme, with the objective of obtaining international cooperation
for the conservation of the biospheres.
279.
Ans. (A)
Sol. □ Biodiversity Hotspots are the Earth’s most biologically rich, yet threatened terrestrial
regions. These are the regions with a high level of endemic species that is under threat from
humans.
□ The Concept of Biodiversity Hotspots began in 1988, when British ecologist Norman
Myers published a seminal paper identifying 10 tropical forest “hotspots.” These regions were
characterized both by high degree of plant endemism and habitat loss.
□ To qualify as a biodiversity hotspot, an area must meet two strict criteria:
The area should support at least 1500 endemic species (found nowhere else on the Earth) of vascular plants.
Hence, statement 1 is correct.
It must have lost at least 70 percent of its primary native vegetation. Hence, statement 2 is not correct.
□ Around the world, 36 areas qualify as biodiversity hotspots. They represent just 2.4% of
Earth’s land surface, but they support more than half of the world’s plant species as endemics,
i.e., species found no place else and nearly 43% of bird, mammal, reptile and amphibian
species as endemics.
280.
Ans. (C)
Sol. □ Chilika Lake is a brackish water lake and a shallow lagoon j with estuarine character
spread across the districts of Puri, Khurda and Ganjam in the state of Odisha in eastern India.
□ It is the largest lagoon in India and counted amongst the largest lagoons in the world. It
is the largest wintering ground for migratory waterfowl found anywhere on the Indian sub-
continent. A number of islands are present in the lagoon, prominent among which are
Krushnaprasad, Nalaban, Kalijai, Somolo and Birds Islands.
□ It is the hotspot of biodiversity in the country and some rare, vulnerable and endangered
species listed in the IUCN Red List of threatened Animals inhabit the Lake area for atleast p a r t
of their life cycle. The list of animals includes a number of rare, threatened and endangered
species including the Barakudia limbless skink.
□ Chilika lake has been designated as a “Ramsar Site”, i.e., a wetland of International
Importance due to its rich biodiversity.
□ It is an assemblage of marine, brackish and freshwater ecosystem, that support amazing
biodiversity. It is a home to highly endangered Irrawaddy dolphin.
□ The lake was included in the Montreux Record (Threatened list) in 1993 by Ramsar
Secretariat due to the change in the ecological character of the lake\ecosystem. Subsequently,
however, due to successful; restoration of the lake ecosystem by Chilika Development
Authority it was removed from the Montreux Record in 2002 (first site from Asia).
281.
Ans. (B)
Sol. □ Project Tiger was launched by the Government of India in the year 1973 to save the
endangered species of tiger in the country. It is a Centrally Sponsored Scheme of the Ministry
of Environment, Forests and Climate Change to provide central assistance to the tiger
States for tiger conservation in designated tiger reserves in India. The project is administered
by the National Tiger Conservation Authority (NTCA).
□ India has 70 percent of world’s tiger population and is tirelessly working with all 13 tiger
range countries towards nurturing the tiger.
□ According to the Tiger Census Report, 2019, the Tiger population has substantially
increased from 2,226 in 2014 to around 2,967 in 2019, wherein, Madhya Pradesh saw the
highest number of tigers (526) followed by Karnataka (524) and Uttarakhand (442).
□ The International Tiger Day celebrated on 29th July is an annual event marked to raise
awareness about tiger conservation.

Tiger Reserve State


Dandeli-Anshi Karnataka, hence, 1 is correct.
Nagarahole Karnataka, hence, 2 is correct.
Bhadra Karnataka, hence, 3 is correct.
Tamil Nadu, hence, 4 is not
Kalakad- Mundanthurai correct.
Amrabad Telangana, hence, 5 is not correct.

282.
Ans. (C)
Sol. □ Pakke Tiger Reserve (PTR): Arunachal Pradesh
Pakke Tiger Reserve (declared in 1999 - 2000) lies in the foothills of the eastern Himalaya in the East Kameng
district of Arunachal Pradesh.
It is also known as Pakhui Tiger Reserve. It is known for its amazing sightings of four resident hornbill species.
□ Nameri Tiger Reserve: Assam
Nameri Tiger Reserve (declared In 1999-2000) is located In the foothills of the eastern Himalayas in the Sonitpur
district of the state of Assam.
The river Jia-Bharell (a tributary of Brahmaputra River) with its tributaries, the Nameri, Upper Dikorai and
BorDikorai flows through the Reserve.
□ Buxa Tiger Reserve: West Bengal
Buxa Tiger Reserve is located in Alipurduar district of West Bengal, parts of which border Bhutan. It was declared
as a tiger reserve in 1983.
□ Nawegaon-Nagzira Tiger Reserve (NNTR): Maharashtra
It is situated in Gondia and Bhandara districts of Maharashtra and was notified as the 4Glh Tiger Reserve of India
in December 2013,
NNTR comprised of the notified area of Nawegaon National Park, Nawegaon Wildlife Sanctuary, Nagzira Wildlife
Sanctuary, New Nagzira Wildlife Sanctuary and Koka Wildlife Sanctuary.
283.
Ans. (B)
Sol. □ Lion Census
The first Lion Census was conducted by the Nawab of Junagadh in 1936 and since 1965, the Forest Department
has been regularly conducting the Lion Census every five years. The last Census was conducted in
2015. It usually runs for more than two days, including a preliminary census and a final census. Hence,
statement 2 is correct.
The Forest Department invites NGOs, experts and wild life enthusiasts to join the Census for transparency and
augmenting manpower.
The Lion Census is done using the block counting method in which census enumerators
remain stationed at water points in a given block and estimate abundance of lions in that
block, based on direct sighting of lions who need to drink water at least once in 24 hours during the
summer.
□ Poonam Avlokan
The Gujarat Forest Department has estimated its lion count from a population observation
exercise called Poonam Avlokan In place of the 15th Lion Census.
It Is a monthly in-house unique exercise carried out every full moon. It was a mechanism
developed by the Forest Department in 2014 as part of preparations for the 2015 Lion Census. Hence,
statement 1 is not correct.
The exercise is carried out for 24 hours assessing the number of lions and their locations in
their respective jurisdictions and it is conducted by the staff only.
The teams keep moving in their respective territories and make their estimates based on
inputs provided by lion trackers and on chance sightings.
The Gir National Park and Wildlife Sanctuary, Gujarat is the only natural habitat of Asiatic
lions. It was declared as a sanctuary in 1965 and a national park in 1975. Hence, statement 3 is
correct.
284. Ans. (C)
Sol. □ Loktak Lake is the largest freshwater lake in Northeast India. It is internationally
recognised, Ramsar designated, freshwater wetland system in the state of Manipur, India.
□ The pristine lake is famously known for its circular floating swamps, called phumdis i.e.,
a collection of heterogeneous mass of vegetation, soil and organic matter at various stages
of decomposition. Resembling miniature islands, these phumdis are found in various forms,
floating on the lake.
□ The largest area of the Phumdi in the Loktak lake is in the Keibul Lamjao National Park,
which is home to Manipur brow-antlered deer also popularly known as the Sangai. The habitat
exclusively consists of floating meadows and an elevated strip of hard ground that dissects the
park into northern and southern zones.
285. Ans. (A)
Sol. □ Uttar Pradesh for the first time, conducted the Census of Otters in its protected areas. The
exercise began in the Pilibhit Tiger Reserve (PTR), located in the foothills of the Himalayas,
south of Nepal.
□ The mammal form an important part of the forest ecosystem and a rich population of
Otters means a healthy ecosystem.
□ An Otter spends much of its time in or close to water bodies. India is home to 3 of the 13
species of Otters found worldwide. These are - Eurasian Otter (Lutralutra); Smooth-coated O tte r
(Lutraperspicillata) and Small-clawed Otter (Aonyxcinereus).
□ The Smooth-coated Otter is distributed throughout the country from the Himalayas
southward. But the Common Otter and the Small-clawed
Otter are restricted to the Himalayas, to the north of the Ganges and to southern India.
286.
Ans. (A)
Sol. □ Kaziranga National Park is located in the State of Assam to the south of
River Brahmaputra. It is the single largest undisturbed and
representative area in the Brahmaputra Valley floodplain.
□ The Greater One-Horned Rhino is one among the five different species of Rhino, the
others include Black Rhino, White Rhino, Javan Rhino and Sumatran Rhino.
□ Only the Great One-Horned Rhino (Indian Rhino) is found in India and it is the largest of
the rhino species.
□ Kaziranga is the home of the world’s most one-horned rhinos. Pobitora Wildlife Sanctuary
has the highest density of one-horned rhinos in the world and second highest number of
Rhinos in Assam after Kaziranga National Park. Hence, statement 1 is not correct.
□ Much of the focus of conservation efforts in Kaziranga are focused on the ‘big four’
species— Rhino, Elephant, Royal Bengal tiger and Asiatic water buffalo.
□ The key points to remember are that the National Highway 37 passes through the park
area and the Diphlu River runs through it. Hence, statement 2 is correct.
287.
Ans. (C)
Sol. □ Talley is a wildlife sanctuary in Arunachal Pradesh, India. The State as a whole contains
40% of the floral and faunal species in India. Hence, 3 is correct.
□ Talley is a plateau with dense forest of silver fir trees, pine clad plateau of beautiful
grandeur and a vast wasteland. The area has some of the
most important endangered species including the clouded leopard.
□ The major drainage system is through the Subansiri River, which ultimately meets the
Bramhaputra in the plains of Assam.
□ Wildlife Sanctuaries and their Locations
Pobitora Wildlife Sanctuary- Assam. Hence, 1 is not correct.
Gumti Wildlife Sanctuary- Tripura. Hence, 2 is not correct.
Rowa Wildlife Sanctuary- Tripura. Hence, 4 is not correct.
288. Ans. (D)
Sol. □ Nokrek Biosphere Reserve
It is located in the northeast of India on the Tura Range, which forms part of the Meghalaya Plateau.
The entire area is mountainous and Nokrek is the highest peak of the Garo hills. 90% of the Reserve
is covered by evergreen forest.
Hoolock Gibbons are the most endangered apes in India and are given special protection here. It was
designated as UNESCO Biosphere Reserve in 2009.
□ Similipal Biosphere Reserve
The Reserve is located in the Mayurbhanj district in the Indian state of Odisha. It was formally designated a tiger
reserve in 1956 and brought under Project Tiger in the year 1973.
It is part of the Similipal-Kuldiha-Hadgarh Elephant Reserve popularly known as Mayurbhanj Elephant
Reserve. The grey hornbill, Indian pied hornbill, Malabar pied hornbill and Indian trogon are also
found in the reserve.
It has been part of the UNESCO World Network of Biosphere Reserve since 2009.
□ Seshachalam Biosphere Reserve
Seshachalam hills, the first Biosphere Reserve in Andhra Pradesh, are located in Southern Eastern
Ghats of Chittoor and Kadapa districts. In 2010, it was designated as a Biosphere Reserve.
It is the richest floristic hot spot harboring many endemic and rare plants. It has large reserves of Red
Sandalwood which is used in medicines, soaps, and spiritual rituals. Tirupati, a major Hindu pilgrimage
town is located in these hills.
□ Dibru-Saikhowa National Park
It is a National Park as well as a Biosphere Reserve situated in the south bank of the river Brahmaputra
in Assam. It was designated a Biosphere Reserve in July 1997.
It is an identified Important Bird Area (IBA), notified by the Birdlife International. It is most famous
for the rare white-winged wood ducks as well as feral horses. It is the largest swamp forest in north-
eastern India.

289. Ans. (C)


Sol.

Natlmud Park Wildlife Sanctuary
National parks air highly restricted areas, Wildlife sanctuaries have lesser
which are not open to all the restrictions than national
air not open to the people. parks
Hence, statement 1 is correct.
In the case of national park, human Human activities are allowed to a
activities are strictly prohibited. limited extent in the wildlife
sanctuaries
The national parks have clearly marked Boundaries of wildlife sanctuaries are
boundaries (fixed by not sacrosanct.
legislation). Hence, statement
3 Is correct.
The International Union of Conservation Wildlife sanctuaries are grouped in
of Nature (IUCN) has declared Category IV of protected
National Parks In Category II of areas as per IUCN.
the protected areas.
Tourism is allowed. Tourism is allowed
□ Most of the national parks were initially wildlife sanctuaries, which were then upgraded to
national parks. Hence, statement 2 Is not correct.
290. Ans. (B)
Sol. □ The Nilgiri Biosphere Reserve (NBR), the first of the Eighteen biosphere reserves of India,
was established in September 1986. It is located in the Western Ghats and encompasses parts
of Tamil Nadu, Kerala and Karnataka. Hence, statement 1 is correct.
□ The Mudumalai Wildlife Sanctuary, Wyanaad Wildlife Sanctuary Bandipur National Park,
Nagarhole National Park, Mukurthi National Park and Silent Valley are the protected areas
present within this reserve.
□ It includes the largest known population of two endangered animal species, namely the
Nilgiri Tahr and the Lion-tailed macaque. 1 he largest south Indian population of elephant, tiger,
gaur, sambar and chital as well as a good number of endemic and endangered plants are also
found in this reserve. Hence, statement 2 is correct.
□ The habitats of a number of tribal groups remarkable for their traditional modes of
harmonious use of the environment are also found here,
including Todas, Kotas, Irullas, Kurumbas, Paniyas, Adiyans, Edanadan Chettis,
Cholanalckens, Allar, Malayan.
□ Agasthyamalai Hills in Kerala come under Agasthyamalai Biosphere reserve. Hence,
statement 3 is not correct.
291. Ans. (D)
Sol. □ Velavadar Blackbuck National Park
The Park was established in 1976 in the Bhal region of Saurashtra, located around 42 km., from the
district headquarters city of Bhavnagar, Gujarat.
Flat land, dry grasses and herds of antelopes have always attracted visitors to this park which has a
unique grassland ecosystem. A successful conservation program for the Blackbuck, the Wolf and the
Lesser Florican (a bustard) among others is ongoing in the Park.
□ Harike National Park
Harike is one of the largest man-made wetlands of northern India which shares its area with the
Tamtaran, Ferozpur and Kapurthala districts of Punjab. It is situated at the confluence of the Beas and
Sutlej rivers.
The area was declared as a Wild life Sanctuary in 1999 and because of its importance as a wetland of
international stature, It was declared a Ramsar site by the international Body of Wetlands under the
UNDP in 1990.
The sanctuary is a bird watchers paradise and attracts thousands of migratory birds during winter,
some from as far off as Siberia and the Arctic.
□ Bhitarkanika National Park
It is one of Odisha’s finest biodiversity hotspots and is famous for its mangroves, migratory birds,
turtles, estuarine crocodiles, and countless creeks.
It is located in the estuary of Brahmani, Baitarani, Dhamra, and Mahanadi river systems.
The Bhitarkanika is represented by 3 protected Areas; the Bhitarkanika National park, the Bhitarkanika
Wildlife Sanctuary and the Gahirmatha Marine Sanctuary.
It is a world famous place for natural crocodile breeding and it is said to house 70% of the country s
estuarine or saltwater crocodiles, conservation of which was started way back in 1975.
The area has been designated as the second Ramsar Site of the state in August 2002.
292. Ans. (D)
Sol. □ The Kharai (swimming camels) is a unique breed of camel found only in Kutch, Gujarat.
They feed on mangroves on the island off-shore. Hence, statements 1 and 3 are correct.
□ Kharai camels have the special ability to survive on both dry land and in the sea that d i s ti n g u i s h
them from the other camels. Hence, statement 2 is correct.
□ The Jat Community lives in a symbiotic relationship with the kharai camels. The former have b e e n
nomadic herders for generations.
□ They have been declared “highly threatened” by the Indian Government and a massive
conservation programme has been launched to increase Kharai numbers.
293. Ans. (D)
Sol.
Features of Great Explanation
Himalayan
National
Park
Location □ It is located in Banjaar Sub-Division of Kullu District of
Himachal Pradesh, India, in the far Western Himalayas.
Designation □ It was declared a UNESCO World Heritage Site in 2014.
□ GHNP is a part of the Himalaya Hotspot – one of Conservation
International’s 34 biodiversity hotspots
Flora and Fauna □ It provides sanctuary for four globally threatened mammals (Snow
leopard, serow, Himalayan tahr, musk deer), three
globally threatened bids (Western tragopan, koklas, cheer
pheasants) and a large number of medicinal plants.
□ GHNP is home to 34 of the 47 medicinal plants categorised as
threatened in Himachal Pradesh.
294.
Ans. (C)
Sol. □ National Tiger Conservation Authority (NTCA) has been granted the permission to
reintroduce the African Cheetahs (Vulnerable as per IUCN Red list) from Namibia on “an
experimental basis in a careful chosen habitat C and nurtured and watched to see whether it c a n
adapt to the Indian conditions”.
□ Asiatic Cheetah was officially declared extinct from India in 1952. It is classified as a
“critically endangered” species as per IUCN Red list. Hence, statement 1 is not correct 3‘ a n d
statement 3 is correct.
□ The re-introduction of foreign Cheetahs will initiate in the Palpur Kuno Sanctuary in
Madhya Pradesh on an experimental basis. Hence, statement 2 is not correct.
□ Others in the priority list include Nauradehi Wildlife Sanctuary in Madhya Pradesh and
Gandhi Sagar-Chittorgarh-Bhainsrorgarh sanctuaries in Rajasthan.
295.
Ans. (D)
Sol. □ The Great Indian Bustard can easily be distinguished by its black crown on the forehead
contrasting with the pale neck and head.
□ It is listed in Schedule I of the Indian Wildlife (Protection) Act, 1972, in the CMS (Convention
on the Conservation of Migratory Species) Convention and in Appendix I of CITES (Convention on
International Trade in Endangered Species of Wild Flora & Fauna). Hence, statement 1 is correct.
□ It has also been identified as one of the species for the recovery programme under the
Integrated Development of Wildlife Habitats of the Ministry of Environment and forests,
Government of India. Hence, statement 2 is correct.
□ Its population is confined mostly to Rajasthan and Gujarat. Small population occur in
Maharashtra, Karnataka and Andhra Pradesh. Hence, statement 3 is correct.
□ Bustards generally favour flat open landscapes with minimal visual obstruction and
disturbance, therefore adapt well in grasslands.
296.
Ans. (A)
Sol. □ Clouded Leopard is named after cloud shaped pattern on its skin. It is listed as Vulnerable
on the IUCN Red List.
□ It is the State animal of Meghalaya and has been added to India’s Recovery Programme
for Critically Endangered Species to aid more research and strengthen conservation efforts.
Hence, statement 1 is correct.
□ Dampa Tiger Reserve in Mizoram has one of the highest population densities of Clouded
Leopards, from the sites surveyed. In India, it occurs in Sikkim, Northern West Bengal, M e g h a l a y a
subtropical forests, Tripura, Mizoram, Manipur, Assam, Nagaland and Arunachal Pradesh. Hence,
statement 2 is correct and statement 3 is not correct.
□ Clouded Leopard’s presence is positively associated with dense forest cover, high rainfall,
hard terrain and low human presence. This suggests that deforestation and reduction in
rainfall patterns as a result of climate change may negatively influence Clouded Leopard
distributions.
297.
Ans. (B)
Sol. □ Wildlife means the native wild fauna and flora of a region. As per Section 2(37) of the Wild
Life (Protection) Act, 1972, “wildlife” includes any animal, aquatic or land vegetation which forms
part of any habitat.
□ Wildlife crime can be defined as taking possession, trade or movement, processing,
consumption of wild animals and plants or their derivatives in contravention of any international,
regional, or national legislation(s). Hunting and illegal trade are the major wildlife offences.
□ The WildLife (Protection) Act, 1972, is umbrella legislation in the country for wildlife crime
enforcement. State Forest and Police Departments are the primary enforcement agencies with
regards to wildlife crimes. Hence, statement 3 is correct.
□ Wildlife crimes are also investigated by the Central Bureau of Investigation (CBI) with the
permission of State Governments concerned. Hence, statement 1 is not correct.
□ Wildlife Crime Control Bureau is a statutory multidisciplinary body established by the
Government of India under the Ministry of Environment and Forests, to combat organized
wildlife crime in the country. It also takes up selected wildlife offences with trans-border
ramifications for investigation. Hence, statement 2 is correct.
298.
Ans. (B)
Sol. □ The forests of north-east India are well known for the rich biodiversity. These forests are
also known for the highest diversity of primates in India, including the only apes found in t h e
country, the Western Hoolock Gibbon and the Eastern Hoolock Gibbon. Both species’ p o p u l ati o n s
have been declining due to habitat destruction of various forms and hunting for meat. Hence,
statement 1 is not correct.
□ Western Hoolock Gibbon
It has a much wider range, as it is found in all the states of the north-east, restricted
between the south of the Brahmaputra river and east of the Dibang river.
Outside India, it is found in eastern Bangladesh and north-west Myanmar.
It is listed as Endangered in the IUCN Red List. Hence statement 2 is correct.
□ Eastern Hoolock Gibbon
It inhabits specific pockets of Arunachal Pradesh and Assam in India; southern China
and north-cast Myanmar.
It is listed as Vulnerable in the IUCN List. Hence, statement 2 is correct.
□ In order to enhance protection for the species, the Government of Assam upgraded the
status of the Hoollongapar Reserve Forest in the Jorhat District of Assam to a Gibbon W i l d l i f e
Sanctuary in 1997, making this the first Protected Area ever named after a primate s p e c i e s .
Hence, statement 3 is correct.
299.
Ans. (A)
Sol. □ The Convention on the Conservation of Migratory Species of Wild Animals (Bonn
Convention; CMS) is an environmental treaty under the aegis of the United Nations Environment
Programme. Hence, statement 1 is correct.
□ It provides a global platform for the conservation and sustainable use of migratory animals
and their habitats. CMS is the only global and UN-based intergovernmental organisation
established exclusively for the conservation and management of terrestrial, aquatic and avian
migratory species throughout their range. Hence, statement 2 is not correct.
□ CMS has two Appendices:
Appendix I: Endangered migratory species
It comprises migratory species that have been assessed as being in danger of extinction
throughout all or a significant portion of their range. Hence, statement 3 is correct.
CMS Parties strive towards strictly protecting these animals, conserving or restoring the
places where they live, mitigating obstacles to migration and controlling other factors that might endanger them.
Appendix II: Migratory species conserved through Agreements
It covers migratory species that have an unfavourable conservation status and that require international
agreements for their conservation and management, as well as those that have a conservation status
which would significantly benefit from the international cooperation that could be achieved by an
international agreement.
300. Ans. (D)
Sol. □ Nandur Madhmeshwar Wildlife Sanctuary: Located in Maharashtra, it is recently
recognized as Ramsar Wetland and is an important Bird Area declared by Birdlife International.
Critically Endangered species under this sanctuary are White-rumped Vulture, I n d i a n
Vulture, Deolali Minnow.
□ Sandi Bird Sanctuary: It is a freshwater marsh, which is located in Uttar Pradesh. It is
recently recognized as Ramsar Wetland and is an important Bird Area declared by Birdlife
International.
□ Keoladeo National Park: It is a freshwater marsh and swamp located in Rajasthan. It is a
UNESCO World Heritage Site but it is not a World Biosphere Reserve. Notable species include Sarus
Crane, Greater spotted Eagle, Painted Stork, Hog Deer.
□ Sunderban Biosphere Reserve: It is a mangrove forest and Ramsar wetland. It is also
recognized as an Important Bird Area. It is an UNESCO World Heritage Site and also a W o r l d
Biosphere Reserve.
□ Famous for Bengal Tiger Sunderban Biosphere Reserve includes Critically Endangered
species like Ganges Shark, Northern River Terrapin, Spoon-billed Sandpiper and Endangered
species like Arrow-headed hammer-head Fish, Irrawaddy Dolphin, Greater adjutant. Olive Ridley
Turtle and Irrawaddy Dolphin are also the part of this reserve.
301.
Ans. (A)
Sol. □ The concept of Biosphere Reserves (BR) was launched in 1975 as a part of UNESCO’s M a n
and Biosphere Programme, dealing with the conservation of ecosystems and the genetic
material they contain.
□ These are representative parts of natural and cultural landscapes extending over large
areas of terrestrial or coastal/marine ecosystems which are internationally recognized within
UNESCO’s Man and the Biosphere Programme.
□ 18 biodiversity rich representative ecosystems, largely within the forest have been
designated as Biosphere Reserves in India.
□ 12 of the eighteen biosphere reserves are a part of the World Network of Biosphere
Reserves, based on the UNESCO Man and the Biosphere
(MAB) Programme list.
□ In the case of large natural areas which straddle national boundaries, trans-boundary
biosphere reserves can be established jointly by the
countries concerned, testifying to long-term cooperative efforts.
□ BRs are not intended to replace existing protected areas, but it widens the scope of the
conventional approach of protection and strengthens existing protected areas network.
□ Existing Wildlife sanctuaries, National parks can be part of Biosphere reserves without
affecting their own legal status. A Biosphere Reserve consists of core, buffer and transition
zones.
□ Criteria for selection of Biosphere Reserves is mentioned below:
A site must contain a protected and minimally disturbed core area of value of nature
conservation. Hence, statement 1 is correct.
The core area should be characterized with typical fauna and flora. Hence, statement 2 is
correct.
Areas having rare and endangered species. Hence, statement 3 is correct.
Areas having typical geo-ecological and pedological conditions.
Areas characterized with tribal culture and traditions.
□ The management of Biosphere Reserves is the responsibility of the concerned State/Union
Territory with necessary Financial assistance and technical expertise provided by the Central
Government.
302.
Ans. (C)
Sol. □ Conservation reserves and community reserves are terms denoting protected areas of the
country which typically act as buffer zones to or connectors and migration corridors between
established national parks, wildlife sanctuaries and reserved and protected forests.
Hence, statement 1 is correct.
□ Such areas are designated as conservation areas, if they are uninhabited and completely
owned by the Government of India but used for subsistence by communities and community
areas if parts of the lands are privately owned. Hence, statement 2 is correct.
□ There are 97 existing Conservation Reserves in India covering an area of 44483 km2, which
is 0.14% of the geographical area of the country (National Wildlife Database, December
2020).
303.
Ans. (C)
Sol. □ National Parks are declared by State governments under the Wildlife Protection Act, 1972
for the protection and conservation of outstanding natural fauna, flora and geological fo r m a ti o n s .
Following activities are prohibited in it:
Hunting, killing or capturing of fauna,
Deprivation of any wild animal of its habitat,
Destruction and collection of flora,
Use of weapons,
Grazing of any livestocks. Hence, statement 1 is correct.
□ No alteration of the boundaries of a National Park shall be made except on the resolution
passed by the legislature of the State. National parks protect one or more species and
follow an ecosystem approach where whole ecosystem is protected rather than the habitat of a
particular animal. National parks provide more protection than wildlife sanctuaries.
□ IUCN protected area management categories classify protected areas according to their
management objectives. The categories are recognised by international bodies such as the
United Nations and by many national governments as the global standard for defining and
recording protected areas and as such are increasingly being incorporated into government
legislation.
□ Protected Area Categories of IUCN
Category la: Strict Nature Reserve
Category lb: Wilderness Area
Category II: National Park, hence, statement 2 is correct.
Category III: Natural Monument or Feature
Category IV: Habitat/Species Management Area
Category V: Protected Landscape/Seascape
Category VI: Protected area with sustainable use of natural resources.
304.
Ans. (A)
Sol. □ Mahananda Wildlife Sanctuary: It is located on the west bank of River Teesta in the
southern part of Darjeeling district in West Bengal.
□ Lakhari Valley Wildlife Sanctuary: It is situated in the state of Odisha was established in
the year 1985. The place receives a huge amount of rainfall every year due to its proximity to the
Bay of Bengal. The sanctuary is full of rich tropical forest, valleys, hills, water sources etc. It
is one of the elephant populated sanctuaries in India.
□ Pachmarhi Wildlife Sanctuary: It is located in Madhya Pradesh in the biogeographical
region of the Deccan Peninsula and the Biotic Province
of Central India. UNESCO designated it a biosphere reserve in 2009.
□ Chandra Prabha Wildlife Sanctuary: It is situated in eastern region of Uttar Pradesh. The
forest is named after the Chandraprabha River; meaning Luminescence of Moon’. A variety of
wild animals like Black Duck, Cheetal, Sambhar, Nilgai, Wild Boar Porcupine Indian Gazelle, etc.,
are found here.
305.
Ans. (A)
Sol. □ Biosphere Reserves (BRs) are representative parts of natural and cultural landscapes
extending over large areas of terrestrial or coastal/marine ecosystems or a combination
thereof and representative examples of biogeographic zones/provinces.
□ The World Network of Biosphere Reserves of the MAB Programme consists of a dynamic
and interactive network of sites of excellence. Launched in 1971, UNESCO’s Man and the
Biosphere Programme (MAB) is an intergovernmental scientific programme that aims to
establish a scientific basis for the improvement of relationships between people and their
environments-□ The United Nations Educational, Scientific and Cultural Organization (UNESCO) has
included the Panna Biosphere Reserve (PBR) in its World Network of Biosphere Reserves (WNBR) in
2020.
□ India has a total of 18 Biosphere Reserves and with the inclusion of PBR, the number of
internationally designated WNBR has become 12.
□ In 2000, the Nilgiri Biosphere Reserve became the Is Biosphere Reserve from India to be
included in the WNBR-
□ List of biosphere reserves of India which have been recognized internationally under Man
and Biosphere Reserve program:
Nilgiri (First to be included)
Gulf of Mannar
Sunderban
Nanda Devi
Nokrek
Pachmarhi
Similipal
Achanakmar-Amarkantak
Great Nicobar
Agasthyamala
Khangchendzonga
Panna
306.
Ans. (A)
Sol. □ Blackbuck National Park, declared as National Park in 1976, is an area of open grassland.
The sanctuary has been declared primarily for Blackbucks.
A small wetland in the southern part of the Park attracts birds like Pelicans, Flamingoes,
Ducks, Waders, Coots, White storks, Painted storks and Sarus cranes etc.
The Park provides one of the world’s best roosting sites to thousands of Harriers that arrive here from Central
Europe for wintering.
The grassland lies between two rivers namely Parvalia and Alang, which drain into the Gulf of Cambay. The Alang
river forms the southern border of the Park. Hence, pair 1 is correctly matched.
□ Harike, also known as Hari-ke-Pattan, is situated at the i confluence of the Beas and Sutlej
rivers. The wetland ecosystem was created following the construction of the Harike barrage
across the Sutlej River. Hence, pair 2 is correctly matched.
It is an internationally recognised Ramsar site. During winters, thousands of migratory
birds arrive from the Himalayas, Europe and Siberia. The rare Indus River Dolphin can also be seen here.
□ Bhitarkanika lies in the estuarine confluence of the Brahmani, Baitarani, Dhamra and
Patasala rivers. Hence, pair 3 is not correctly matched.
It has three protected areas: Bhitarkanika National Park, Bhitarkanika Wildlife
Sanctuary and Gahirmatha Marine Sanctuary.
307.
Ans. (C)
Sol. □ Bandipur Tiger Reserve was established in 1973 under Project Tiger. In 1985, by including
adjacent areas from Venugopala Wildlife Park, it was enlarged and named as Bandipur
National Park.
□ It is situated in two contiguous districts (Mysore and Chamarajanagar) of Karnataka and is
located at the tri-junction area of the States Karnataka, Tamil Nadu and Kerala. It forms a part
of Nilgiri Biosphere Reserve-Hence, statement 1 is not correct.
□ It is surrounded by:
Mudumalai Tiger Reserve (Tamil Nadu) in the South
Wayanad Wildlife Sanctuary (Kerala) in the Southwest, and
The Kabini Reservoir separates the Bandipur and Nagarahole Tiger Reserve on the North-west.
□ It is endowed with rich floral and faunal diversity and is recognized as one of the Mega
Biodiversity Areas in the country.
The Bandipur along with Nagarahole, Mudumalai, Sathyamangalam and Wayanad constitutes the single largest
Wild population of Tigers in the world.
This landscape is also home to the single largest Asian Elephant population in the world
and is part of the Mysore Elephant Reserve (MER). Hence, statement 3 is correct.
□ The park is located between the Kabini river in the north and the Moyar river in the south.
The Nugu river runs through the park. The
highest point in the park is on a hill called HimavadGopalaswamyBetta.
Hence, statement 2 is not correct
308.
Ans. (B)
Sol. □ Biosphere Reserves (BRs) are representative parts of natural and cultural landscapes
extending over a large area of terrestrial or coastal/marine ecosystems or a combination
thereof and representative examples of bio- geographic zones/provinces.
□ Criteria for designation of Biosphere Reserves:
A site that must contain an effectively protected and minimally-disturbed core area of value of nature
conservation. Hence, statement 1 is correct.
The core area should be typical of a bio-geographical unit and large enough to sustain viable populations
representing all trophic levels in the ecosystern.
The management authority to ensure the involvement/ cooperation of local communities to bring a variety of
knowledge and experiences to link biodiversity conservation and socio-economic development while
managing and containing the conflicts.
Areas potential for preservation of traditional tribal or rural modes of living for harmonious use of the
environment.
□ Biosphere Reserve differs from the protected areas:
Biosphere Reserve is not intended to replace existing protected areas but it widens the scope of conventional
approach of protection.
Biosphere Reserve emphasis on conservation of overall biodiversity and landscape rather than some specific
flagship species. Hence, statement 2 is not correct.
Biosphere Reserve has the focus on the welfare of local inhabitants through provision of supplementary and
alternate livelihood support to the people in the buffer and transition zones in order to reduce biotic
pressure on biodiversity of the natural reserves of the core zone.
309.
Ans. (C)
Sol. □ Ghodazari Wildlife Sanctuary was established in 2018, in the Chandrapur district of
Maharashtra, India. It includes 159 sq. km., area of Bramhapuri forest and the area is home t o
tigers and leopards.
□ The wildlife sanctuary is located North East of Tadoba-Andhari Tiger Reserve and the area
in the sanctuary also important corridor for wild animals moving from Tadoba-Andhari T i ge r
Reserve to Umrer-Karhandla Wildlife Sanctuary and vice-versa.
□ Ghodazari lake is an important destination for the native and migratory birds due to its
undisturbed water body and abundance of food.
310.
Ans. (C)
Sol. □ Nanda Devi National Park is situated around the peak of Nanda Devi (7816 m) in the state
of Uttarakhand in northern India. The Park was established as Sanjay Gandhi National Park by
Notification in 1982 but was later renamed Nanda Devi National Park.
□ The park encompasses the Nanda Devi Sanctuary, a glacial basin surrounded by a ring of
peaks, and drained by the Rishi Ganga through the Rishi Ganga Gorge-Hence, statement 2 is not
correct.
□ It was inscribed a World Heritage Site by the United । Nations Educational, Scientific and
Cultural Organization (UNESCO) in 1988. Hence, statement 1 is correct.
□ Fir, birch, rhododendron, and juniper are the main flora, while fauna includes Himalayan
black bear, Snow leopard, Himalayan Musk Deer etc. Hence, statement 3 is correct.
311.
Ans. (A)
Sol. □ Periyar Tiger Reserve is located in the districts of Idukki and Pathanamthitta in Kerala
(saddled in the southern region of Western Ghats). Hence, statement 1 is correct.
□ Declared a Sanctuary during 1950 and declared as Tiger Reserve during 1978. It gets its
name from the river Periyar which has its origin deep inside the reserve.
□ The major rivers through the reserve are Mullayar and Periyar. The sanctuary comprises
tropical evergreen, semi evergreen, moist deciduous forests and grasslands.
□ The sanctuary is a repository of medicinal plants numbering to about 300. Some are
endemic to the region like Syzygium periyarensis (a tree), Habenaria periyarensis (an orchid) and
Mucuna pruriense thekkadiensis (a climber) etc.
□ There are six tribal communities nestled inside the reserve such as Mannans, Paliyans,
Malayarayans, Mala Pandarams, Uralis and Ulladans. Hence, statement 3 is not correct.
□ Fauna includes; Tiger, Elephant, Lion-tailed macaque, Nilgiri Tahr, Kingfishers, the great
Malabar Hornbill etc. Hence, statement 2 is correct.
312.
Ans. (D)
Sol. □ The State animal of Madhya Pradesh - Hard Ground Barasingha (Swamp deer or Rucervus
duvaucelii) is found exclusively in Kanha Tiger Reserve. Hence, statements 1 and 2 are correct.
□ There are three subspecies of swamp deer found in the Indian Subcontinent.
The western swamp deer (Rucervus duvaucelii) found in Nepal,
Southern swamp deer/Hard Ground Barasingha (Rucervus duvaucelii branderi) found in central and north India,
and
Eastern swamp deer (Rucervus duvaucelii ranjitsinhi) found in the Kaziranga (Assam) and Dudhwa National Parks
(Uttar Pradesh).
□ Protection Status of Swamp Deer:
IUCN Red List: Vulnerable; hence, statement 3 is correct.
CITES: Appendix I
Wildlife Protection Act, 1972: Schedule I
□ Kanha National Park in Madhya Pradesh was created in 1955 and in 1973 was made the
Kanha Tiger Reserve. It is the largest National Park in Central India.
□ Kanha tiger reserve is the first tiger reserve in India to officially introduce a mascot,
“Bhoorsingh the Barasingha”.
313.
Ans. (C)
Sol. □ The Thane Creek Flamingo Sanctuary is the second marine sanctuary of Maharashtra after
Malvan sanctuary and lies along the western bank of the Thane Creek. Thane Creek is an inlet
in the shoreline of the Arabian Sea that isolates the city of Mumbai from the Indian mainland.
□ The Thane Creek Flamingo Sanctuary is recognized as an “Important Bird Area” by the
Bombay Natural History Society, and is home to categories of mangrove species, birds like
flamingos, fish and butterflies species and mammals. Hence, statement 2 is correct.
□ The Ministry of Environment, Forest and Climate Change (MoEFCC) has notified areas
around the Thane Creek Flamingo Sanctuary (TCFS, Maharashtra) as an Eco Sensitive Zone (ESZ).
Eco Sensitive Zone or Ecologically Fragile Areas are areas within 10 kms around Protected
Areas, National Parks and Wildlife Sanctuaries. Hence, statement 1 is correct.
□ ESZs are notified by the Ministry of Environment, Forest and Climate Change (MoEFCC),
under Environment Protection Act, 1986.
□ ESZs minimises the impact of urbanisation and other developmental activities around
areas adjacent to protected areas. They behave as a shock-absorber and also act as a transition
zone from areas of high protection to areas involving lesser protection.
□ Other Protected Areas of Maharashtra:
Sanjay Gandhi National Park,
Tadoba Andhari Tiger Reserve,
Koyna Wildlife Sanctuary,
Bor Wildlife Sanctuary,
Umed Paoni Kardhandla WS,
Sahyadri Tiger Reserve,
Melghat Tiger Reserve,
Navegaon National Park.

4. Environmental Pollution
Solution:
314.
Ans. (D)
Sol. □ Benzene (C6H6) is a colorless, flammable liquid with a sweet odor. It evaporates quickly
when exposed to air. Benzene is formed from natural processes, such as volcanoes and forest
fires, but most of the exposure to benzene results from human activities.
□ The main sources of benzene in the environment includes automobile exhaust, industrial
sources, good buring and fuel evaporation from gasoline filling stations. Hence, 1 and 3 are
correct.
□ Benzene has been detected at high levels in indoor air. Although some of this exposure
might be from building materials (paints, adhesives, etc.), but most is from cigarette smoke.
Hence, 2, 4 and 5 are correct.
315.
Ans. (D)
Sol. □ Coal ash, also referred as coal combustion residuals or CCRs, is produced primarily from
the burning of coal in coal-fired power plants. It contains contaminants like mercury, cadmium and
arsenic. Without proper management, these contaminants can pollute waterways, ground
water, drinking water, and the air. Hence, statement 1 is correct.
□ Coal based thermal power plants are major source of Sulphur dioxide and Nitrogen oxides
emissions. Hence, statement 2 is correct.
□ According to Coal India Limited, the ash content of coal produced in the country is
generally 25 to 45% whereas average ash content of imported coal varies from 10 to 20%.
Hence, statement 3 is correct.
316.
Ans. (C)
Sol. □ Eutrophication is when a water body becomes overly enriched with minerals and nutrients
which induce excessive growth of algae or algal bloom. This process also results in oxygen
depletion of the water body. Hence, statements 1 and 2 are correct.
□ Eutrophication is a leading cause of impairment of many freshwater and coastal marine
ecosystems in the world. Harmful algal blooms, dead zones, and fish kills are also the results of
eutrophication.
317.
Ans. (A)
Sol. □ Microbeads are small, solid, manufactured plastic particles that are less than 5mm and do
not degrade or dissolve in water.
□ Mainly made of polyethylene, microbeads can also be prepared from petrochemical
plastics such as polystyrene and polypropylene. They may be added to a range of products,
including rinse-off cosmetics, personal care and cleaning products.
□ Microbeads, because of their small size pass unfiltered through the sewage treatment
system and reach the water bodies. The untreated microbeads in thewater bodies are taken up by
the marine animals, thus producing toxicity and causing harm to the marine ecosystem.
□ In 2014, Netherland became the first country to ban cosmetics microbeads.
318.
Ans. (D)
Sol. □ Biomass is organic material that comes from plants and animals, and it is a renewable
source of energy. Biomass contains stored energy from the Sun. Plants absorb the Sun’s
energy in a process called photosynthesis. When biomass is burned, the chemical energy in
biomass is released as heat.
□ Crop residue and biomass burning (forest fires) is considered as a major source of Carbon
Dioxide (CO2 ), Carbon Monoxide (CO), Methane (CH4), volatile organic compounds (VOC), a n d
Nitrogen Oxides (NO×). Burning of paddy crop residue releases Suspended Particulate Matter, SO2
and NO2 in the atmosphere. Hence, 1, 2, 3 and 4 are correct.
319.
Ans. (D)
Sol. □ Methanehydrate is a crystalline solid that consists of a methane molecule surrounded by
a cage of interlocking water molecules. It is an “ice” that only occurs naturally in subsurface
deposits where temperature and pressure conditions are favorable for its formation.
□ Regions with suitable temperature and pressure conditions for the formation and stability
of methane hydrate- sediment and sedimentary rock units below the Arctic permafrost;
sedimentary deposits along continental margins; deep-water sediments of inland lakes and s e a s ;
and, under Antarctic ice. Hence, statement 2 is correct.
□ Methane hydrates, the sensitive sediments, can rapidly dissociate with an increase in
temperature or a decrease in pressure. The dissociation produces free methane and water, which
can be triggered by global warming. Hence, statement 1 is correct.
□ Methane is removed from the atmosphere in about 9 to 12-year period by oxidation
reaction where it is converted into Carbon dioxide. Hence, statement 3 is correct.
320.
Ans. (D)
Sol. □ According to FAO, poultry manure releases considerable amounts of Nitrogen mainly in
form of Ammonia (NH3), Dinitrogen (N2), Nitrous Oxide (N2O) and Nitrate(NO3−). Hence,
statement 3 is correct.
□ According to FAO, major forms of pollution associated with intensive livestock production
include eutrophication of surface water, leaching of Nitrates and pathogens and release of
Ammonia, Methane and other gases into the air. Hence, statement 2 is correct.
□ A range of pollutant gases, and especially nitrogen compounds (NO2, NO, NH3, N2O) are
emitted to the atmosphere from agricultural activities, including fertilizer use, farm m a c h i n e r y
and livestock waste. Hence, statement 1 is correct.
321.
Ans. (C)
Sol. □ The National Clean Air Programme (NCAP) was launched by the Ministry of Environment,
Forest and Climate Change (MoEFCC) in 2019 to cut the concentration of coarse ( p a r ti c u l ate
matter of diameter 10 micrometer or less, or PM10) and fine particles (particulate matter of
diameter 2.5 micrometer or less, or PM2.5) by at least 20% -30% by 2024, with 2017 as the base
year for comparison of concentration.
□ The plan includes 102 non-attainment cities, across 23 states and Union territories, which
were identified by the Central Pollution Control Board (CPCB) on the basis of their ambient
air quality data between 2011 and 2015.
□ Non-attainment cities are those that have fallen short of the National Ambient Air
Quality Standards (NAAQS) for over five years.
322.
Ans. (C)
Sol. Catalytic converter is an exhaust emission control device made of platinum and rhodium which helps
in reduction of toxic gases and pollutants from the engines of cars into less toxic pollutants like carbon
dioxide, nitrogen and water by catalyzing a redox reaction.
323.
Ans. (D)
Sol. □ Venturi scrubber is a type of pollution control equipment that is used to remove
contaminated particles (primarily particulate matters) from gaseous exhaust streams. It works
on the principle of removal by atomized water vapour.
□ It is a part of the group of air pollution control technologies collectively referred to as wet
scrubbers and uses the energy from a high-velocity inlet gas stream to atomize the liquid being
used to scrub the gas stream.
324. Ans. (B)
Sol. □ Acid rain results when sulfur dioxide (SO2) and nitrogen oxides (NOX) are emitted into
the atmosphere, react with water, oxygen and other chemicals in the presence of sunlight to f o r m
sulfuric and nitric acids, which then falls to the ground in the form of wet or dry deposition.
□ As the pH level of the precipitation is less than 5.6, it can severely affect flora and fauna in
the region.
325. Ans. (A)
Sol. □ A bioindicator is a living organism that gives us an idea of the health of an ecosystem. It is
also known as the health indicator of the ecosystem.
□ Lichens are a complex life form, living in a symbiotic partnership between fungus and
alga. It can be used as air pollution indicator, especially of the concentration of sulphur
dioxide in the atmosphere.
□ They are sensitive to sulphur dioxide (SO2) and do not grow in polluted areas. Monitoring
population numbers of lichens might indicate damage to the ecosystem in which they live.
□ Lichens live on the surfaces such as trees, rocks, soil etc. Lichens do not have roots;
instead they receive all their nutrients from the atmosphere.
326. Ans. (D)
Sol. □ Smog is a specific type of air pollution. It is a combination of harmful pollutants (often
appearing relatively low to the ground as a yellow-brown haze) that are introduced into the
atmosphere by both natural and human induced processes.
□ The term Smog is used to describe a mix of smoke and fog. It is majorly a composition
of sulphur oxides, nitrogen oxides and hydrocarbon vapours emitted by automobiles. The
smog causes a light brownish coloration of the atmosphere, reduced visibility, plant damage,
irritation of the eyes and respiratory distress.
327.
Ans. (D)
Sol. □ Wetland rice fields (Paddy fields) have been identified as a major source of atmospheric
methane. Although the potential for methane release from rice fields has long been noted, t h e
first comprehensive measurements of methane fluxes in rice fields were reported only i n
the early 1980s. As in a natural wetland, flooding a rice field cuts off the oxygen supply from
the atmosphere to the soil, which results in anaerobic fermentation of soil organic matter.
□ Methane is a major end product of anaerobic fermentation. It is released from submerged
soils to the atmosphere by diffusion and ebullition (the action of bubbling or boiling) and
through roots and stems of rice plants. Global estimates of emission rates of methane from
wetland rice fields range from 20 to 100 Tg/yr (IPCC 1992), which corresponds to 6-29% of the
total annual anthropogenic methane emission.
328. Ans.
Sol. □ A spinning wheel was inaugurated at Gautam Buddh Nagar, Noida by Union Government
and Noida Authority. The ‘charkha’, which symbolises Gandhi’s dream of Swadeshi (self
sufficiency and self reliance), measures 14 ft × 20 ft × 8ft and is made of 1,250 kg of used
plastic. Hence, statements 1 and 2 are correct.
□ It was inaugurated on Mahatma Gandhi’s 150th birth anniversary.
□ The gigantic spinning wheel is not only representative °t construction and beautification b u t
also represents Our commitment towards the plastic-free campaign. It has been recognised as
the largest made of used plastic Waste in the country by India Book of Records.
329.
Ans. (A)
Sol. □ Aral Sea: It lies between Kazakhstan and Uzbekistan. It has been shrinking steadily since
the 1960s after its tributaries were diverted by Soviet irrigation projects. By 2007, the lake h a d
declined to 10% of its original size and had split into four separate lakes. Hence, 1 is correct.
□ Black Sea: It is also known as Euxine Sea. It is one of the major water bodies and a famous
inland sea of the world. The countries sharing a border with the BlackSea include Romania,
Turkey, Bulgaria, Ukraine, Russia, and Georgia. There has been no dramatic shrinkage of the Black
Sea in the recent past. Hence, 2 is not correct.
□ Lake Baikal: Located in Siberian Russia, this UNESCO World Heritage site has not
undergone any dramatic shrinkage in the recent past. One of the most recognizable shifts
affecting Lake Baikal is the rapidly increasing number of Spirogyra, a diverse form of algae. Hence,
3 is not correct.
330. Ans. (D)
Sol. □ Sixth Mass Extinction/Sixth Extinction is the extinction | of a large number of species
within a relatively short j period of geological time, thought to be, due to factors such as a
catastrophic global event or widespread environmental change that occurs too rapidly for most
species toadapt.
□ Natural disasters have triggered at least five mass extinctions in the past 500 million years,
each of which wiped out between 50% and 90% of all species on the planet. The most
recent extinction occurred about 65 million years ago, when an asteroid ended the reign of
dinosaurs and opened new doors for mammals.
□ The recent studies reported that the sixth mass extinction of Earth’s wildlife is already
underway as there is killing of large ocean dwellers (like sharks, whales, giant clams, sea
turtles and tuna) in disproportionately greater numbers than smaller animals.
□ The striking difference between the previous mass extinctions and the anticipated sixth
mass extinction is the cause of its occurrence. It is anticipated that sixth mass extinction is d u e
to mankind’s over-exploitation/ misuse of natural resources, fragmentation/loss of n a t u r a l
habitats, and destruction of ecosystems, pollution and global climate change.
331. Ans. (B)
Sol. □ Sand mining is the process of extracting sand from the river bed or from the coastal region.
□ Excessive sand mining results in pollution of river water by lowering the pH value of
water, mixing of various metal oxides, reduction of oxygen and thus, increasing the Biological
Oxygen Demand (BOD). Polluted river water causes pollution of groundwater. Hence, 2 i s
correct.
□ Due to increased metal oxides and their mixing in river water, the salinity of the water
increases. Hence, 1 is not correct.
□ Volume of water flow in the river is decreased, thus it results in lowering of the water
table. Hence, 3 is correct.
332. Ans. (B)
Sol. □ The product of on-site biological decomposition (organic matter) affects the chemical
and physical properties of the soil and its overall health. Its composition and breakdown rate
affects the soil structure and porosity; the water infiltration rate and moisture holding
capacity of soils; the diversity and biological activity of soil organisms; and plant nutrient
availability. It helps in increasing the water holding capacity of the soil. Hence, statement 1 is not
correct.
□ Sulphur Cycle is a biogeochemical cycle, which contains both atmospheric and terrestrial
processes. Within the terrestrial portion, the cycle begins with the weathering of rocks,
releasing the stored Sulphur. The Sulphur, then comes into contact with air where it is c o n v e r t e d
into Sulphate (SO2−). The Sulphate is taken up by plants and microorganisms and is converted into
organic forms. Animals then consume these organic forms through the foods they eat, thereby
moving the Sulphur through the foodchain. Hence, statement 2 is not correct.
□ Irrigation over a long period of time can lead to salinization because irrigated water
contains dissolved salts which get left behind when water evaporates. Over time, concentrations
of these salts can reach levels that make it more difficult for plants to take up water from t h e
soil. Hence, statement 3 is correct.
333. Ans. (A)
Sol. □ According to the World Health Organization (WHO) a carbon footprint is a measure of the
impact people’s activities have on the amount of carbon dioxide (CO2) produced through t h e
burning of fossil fuels and is expressed as a weight of CO2 emissions produced in tonnes.
□ It may be applied to the actions of an individual, a family an event, an organization or even
an entire nation.
□ Its usual measurement is tonnes of CO2 emitted per year.
334. Ans. (C)
Sol. □ Biochemical Oxygen Demand (BOD) refers to the amount of Oxygen required by
microorganism to stabilize decomposable organic matter in one litre of polluted water.
□ Higher, the organic matter presence in the water body, higher will be the presence of a
large number of microorganisms and the lower the amount of dissolved oxygen available f o r
aquatic organisms. Thus, increased BOD suggests a high level of microbial contamination and is
a reliable gauge to check the organic pollution of the water body. Hence, statements 1 and 2 are
not correct and statement 3 is correct.
335. Ans. (C)
Sol. □ The decrease in ozone concentration mainly in the stratosphere is referred to as the
depletion of the ozone layer. Ozone depleting substances include Chlorofluorocarbons
(CFCs), hydrochlorofluorocarbon (HCFCs), carbon tetrachloride, and methyl chloroform.
These compounds release chlorine or bromine when they are exposed to intense UV rays in t h e
stratosphere and contribute to ozone depletion. One chlorine atom can destroy over 1 0 0 , 0 0 0
ozone molecules before it is removed from the stratosphere. Hence, statement 1 is correct.
□ Sunspots are areas that appear dark on the surface of the Sun (photosphere). They appear
dark because they are cooler than other parts of the Sun’s surface as they are formed at areas
where magnetic fields are particularly strong. The magnetic field lines near sunspots often tangle,
cross, and reorganize. This can cause a sudden explosion of energy called a solar flare that release
a lot of radiation into space, which can disrupt satellite and radio transmission on the Earth, and
more severe ones can cause ‘geomagnetic storms’ that can damage transformers in power grids.
□ Further, unusual solar activity can cause the ozone levels in the upper stratosphere to be
substantially depleted because solar storms carries energetic electrons and protons into the
Earth’s upper atmosphere, where they increase production of nitrogen oxides which depletes
ozone. Hence, statement 2 is correct.
□ Very strong winds inside the polar stratospheric vortex, also increase the rate of
transportation of nitrogen gases further into the atmosphere which then attacks the ozone layer.
□ Ozone depletion increases the amount of ultraviolet (UV) radiation that reaches Earth’s
surface, which causes photo-oxidative degradation that results in breaking of the polymer
chains, produces free radical and reduces the molecular weight, causing deterioration of
mechanical properties and leading to useless materials, after an unpredictable time. H o w e v e r ,
increase in UV radiation is the effect of Ozone depletion. Hence, statement 3 is not correct.
□ Volcanic eruptions can actually impact the ozone layer if they’re strong enough; that is,
if they blast gases (such as sulfur dioxide) and volcanic particles so high that these particles
manage to reach the stratosphere. Produced by major volcanic eruptions, particles or aerosols
accelerate ozone destruction. The particles themselves do not directly destroy ozone but
they do provide a surface upon which chemical reactions can take place. This facilitates
chlorine-driven ozone depletion. Hence, statement 4 is correct.
336. Ans. (B)
Sol. □ Air pollution refers to the release of pollutants ( Material or factors that cause adverse
effects on natural quality of the environment) into the air that are detrimental to human
health and the planet as a whole.
□ In the context of climate change, normally carbon dioxide is not considered as an air
pollutant rather, it is a greenhouse gas which, as its concentration increases in the atmosphere,
contributes to the warming of the planet. Further, carbon dioxide is also not covered under 12
pollutants classified by National Ambient Air Quality Standards (NAAQS). Hence, statement I
is true.
□ The gases in Earth’s atmosphere include:
Nitrogen — 78%
Oxygen — 21%
Argon — 0.93%
Carbon dioxide — 0.04%; hence, statement II is true.
Trace amounts of neon, helium, methane, krypton and hydrogen, as well as water
vapour.
Both Statement (I) and Statement (II) are individually true, but Statement (II) is not the correct
explanation of Statement (I).
337. Ans. (B)
Sol. □ The accumulation of garbage by spreading in layers and covering with ashes or dirt to a
depth sufficient to control rats, flies and odors are commonly referred to as sanitary/municipal
fills and waste heaps.
□ They are hazardous because of the following:
Leachates: Leachate is formed when rain water filters through wastes placed in a landfill. When this
liquid comes in contact with buried wastes, it leaches, or draws out, chemicals or constituents from
those wastes and contaminates groundwater. Hence, 1 is correct.
Release of harmful gases: Poisonous gases such as methane and sulphur dioxide are released which pollute the
air and are harmful to human health. Hence, 2 is correct.
□ Rodents and wandering animals: These do not make the sanitary/municipal fills hazardous.
Hence, 3 is not correct.
□ Automobile workshops that seem to have an affinity for such neighbourhoods: Due to the
least amount of pollution created by them, they have negligible effect on sanitary / m u n i c i p a l
fills. Hence, 4 is not correct.
338. Ans. (A)
Sol. □ The greenhouse effect refers to the rise in the global average temperature of the earth.
□ The greenhouse effect is a natural process that warms the Earth s surface. When the Sun’s
energy reaches the Earth’s atmosphere, some of it is reflected back to space and the rest is
absorbed and re-radiated by greenhouse gases.
□ Greenhouse gases include water vapour, carbon dioxide, methane, nitrous oxide, ozone
and some artificial chemicals such as chlorofluorocarbons (CFCs).
□ Human activities - particularly burning fossil fuels (coal, oil and natural gas), agriculture
and land clearing-are increasing the concentrations of greenhouse gases. This is the e n h a n c e d
greenhouse effect, which is contributing to warming of the Earth.
339.
Ans. (D)
Sol. □ Peroxyacetyl Nitrates (PAN) is an important constituent of photochemical smog. It is also
a ubiquitous chemical present throughout the global troposphere. A unique property of
PAN is that it is very stable at cold temperatures and easily decomposes to release NOx at warm
temperatures.
□ in remote atmospheres, it can act as a carrier and a reservoir of NOx, which is necessary
for ozone (O3) formation. PANs are known to be eye irritants in the smoke (lachrymators),
phytotoxins, and bacterial mutagens. The most serious biological effects of PANs are of a
phytotoxic nature resulting in injury to plants and vegetation.
340. Ans. (A)
Sol. □ Primary pollutants are those which are emitted directly into the atmosphere, like sulphur
dioxide, nitric oxides and carbon monoxide.
□ The pollutants that are not directly emitted from a source but form/result into a new
pollutant, when primary are known as secondary pollutants. Examples include ozone (O3),
sulphuric acid and nitric acid (components of acid rain), particulate matter, nitrogen dioxide ( NO 2 ,
peroxyacyl nitrates (PANs), smog etc.
341. Ans. (C)
Sol. □ Ozone (O3) is a highly reactive gas composed of three oxygen atoms. It is both a natural and
a man-made product that occurs in the Earth’s upper atmosphere ozone molecule (the
stratosphere) and lover atmosphere (the troposphere)
□ Stratospheric ozone is formed naturally through the interaction of solar ultraviolet (UV)
radiation with molecular oxygen (O2).
□ Tropospheric or Ground-level or “bad” ozone is not emitted directly into the air, but is
created by chemical reactions between oxides of nitrogen (NOx) and volatile organic compounds
(VOC) in the presence of sunlight.
342. Ans. (A)
Sol. □ Incineration is a process of waste disposal that involves the combustion of organic
substances contained in waste materials. In this method, waste materials are burnt in huge
incinerators at high temperature. It converts waste material into ash, flue gas and heat.
□ Advantages of Incineration
The volume of waste is reduced by turning it into ashes. Incineration plants can reduce the mass of
waste from 95% to 96%.
The huge amount of unburnt waste is difficult in comparison to ashes disposal.
Incineration plants generate energy from waste that can be used to generate electricity or heat.
□ Disadvantages of Incineration
The installation of an incineration plant is an expensive process.
It produces harmful gases that pollute the atmosphere.
343. Ans. (C)
Sol. □ Wetland rice fields (Paddy fields) have been identified as a major source of atmospheric
methane. As in a natural wetland, flooding a rice field cuts off the oxygen supply from the
atmosphere to the soil, which results in anaerobic fermentation of soil organic matter.
□ Methane is a major end product of anaerobic fermentation. It is released from submerged
soils to the atmosphere by diffusion and ebullition (the action of bubbling or boiling) and
through roots and stems of rice plants. Global estimates of emission rates of methane from wetland
rice fields range from 20 to 100 Tg/yr (IPCC 1992), which corresponds to 6-29% of the total
annual anthropogenic methane emission.
□ Terrestrial arthropod-like termite contributes to the methane emission resulting from their
digestive system. Global emissions of methane due to termites are estimated to be between
20,00,000 and 2,20,00,000 metric tonnes per year.
344.
Ans. (D)
Sol. □ Vulture numbers saw a decline as much as 90% in some species in India since the 1990s in
one of the most drastic declines in bird populations in the world.
□ The cause of the decline was established as diclofenac, a veterinary nonsteroidal anti-
inflammatory drug (NSAID), which is used to treat pain and inflammatory diseases. Widespread
use of diclofenac in cattle was linked to the deaths of millions of vultures that ate carcasses
containing the drug.
□ The Ministry of Environment, Forest and Climate Change released the Action Plan for
Vulture Conservation, 2006 with the Drugs Controller General of India (DCGI) banning the veterinary
use of diclofenac in the same year.
345.
Ans. (B)
Sol. □ Gomti river has become the most polluted river in Uttar Pradesh. Monitoring by the state
pollution control board reveals that water is unfit for consumption.
□ The extent of pollution is such that the river’s biodiversity is being affected. A marine
species of molluscs, Solariella was found in the river. This is alarming, as Solariella is endemic to
coastal waters that ordinarily have high levels of pH (alkaline/polluted conditions).
□ The presence of Solariella was revealed during the first-ever bio-mapping of the river done
by the Geological Survey of India, Lucknow. In this context, Gomati river has been declared a
‘Biological Disaster’, due to environmental and biological consequences.
346.
Ans. (A)
Sol. □ Primary pollutants: The pollutants that directly cause air pollution or the pollutants that
are formed and emitted directly from particular sources. Examples are particulate matter,
carbon monoxide, nitrogen oxide, and sulfur oxide.
□ Secondary pollutants: The pollutants formed by the intermingling and reaction of primary
pollutants are known as secondary pollutants. Examples are ozone and smog.
□ Smog is a specific type of air pollution. It is a combination of harmful pollutants (often
appearing relatively low to the ground as a yellow-brown haze) that are introduced into the
atmosphere by both natural and human induced processes.
The suspended particles like unburnt carbon particles and hydrocarbons mix with smoke and fog in the
atmosphere, especially in the cold weather to form smog which results in a lowered visibility.
Whereas, photochemical smog is produced when sunlight reacts with oxides of nitrogen and at least
one volatile organic compound (VOC) in the atmosphere.
347.
Ans. (D)
Sol. □ Carbon sequestration is the process of trapping CO2 from the atmosphere and then storing
it in leak proof containers. Geological formations like coal seams depleted oil and gas reserves,
subterranean deep saline formations, etc., offer a great capacity for Carbon sequestration.
Hence, 1 and 3 are correct.
□ Injection of CO2 in uneconomic or depleted hydrocarbon reserves can provide additional
benefits in terms of enhanced recovery of the traces left in the reservoirs and formation of coal
bed methane in coal seams. Hence, 2 is correct.
348.
Ans. (C)
Sol. □ Biological Oxygen Demand (BOD) is the amount of Dissolved Oxygen needed by aerobic
organisms to decompose organic material in a given sample of water at a certain temperature
over a particular time period.
□ BOD is one of the most common measures of pollutive organic material in water. BOD
indicates the amount of putrescible organic matter present in water. Therefore, a low BOD is an
indicator of good quality water, while a high BOD indicates polluted water.
□ Sewage and untreated water discharge results in the decrease in the amount of Dissolved
Oxygen as much of the available dissolved oxygen is consumed by aerobic bacteria in the
degradation process, robbing other aquatic organisms of the Oxygen they need to live.
349.
Ans. (C)
Sol. □ Bioremediation is a treatment process that uses naturally occurring microorganisms (yeast,
fungi, or bacteria) to break down, or degrade, hazardous substances into less toxic or non- t o x i c
substances.
□ The microorganisms break down the organic contaminants into harmless products-mainly
Carbon Dioxide and water. It is a cost effective, natural process applicable to many common
organic wastes. Many bioremediation techniques can be conducted on-site. Hence, 1 is
correct.
□ All contaminants cannot be easily treated by bioremediation using microorganisms. For
example, heavy metals such as Cadmium and Lead are not readily absorbed or captured by
microorganisms. Hence, 2 is not correct.
□ Genetic engineering can be used to create microorganisms designed for specific purposes
for bioremediation. For e.g. bacterium Deinococcusradiodurans (the most radioresistant
organism known) has been modified to consume and digest Toluene and ionic Mercury from
highly radioactive nuclear waste. Hence, 3 is correct.
350.
Ans. (C)
Sol. □ Diesel engines convert the chemical energy contained in the fuel into mechanical power.
Diesel fuel is injected under pressure into the engine cylinder where it mixes with air and
where the combustion occurs. The exhaust gases which are discharged from the engine
contain several constituents that are harmful to human health and to the environment.
□ Carbon monoxide (CO), hydrocarbons (HC), and aldehydes are generated in the exhaust as
the result of incomplete combustion of fuel or at lower temperature.
□ Nitrogen oxides (NOx) are generated from nitrogen and oxygen under the high pressure
and temperature conditions in the engine cylinder. NOx consist mostly of nitric oxide (NO) and
a small fraction of nitrogen dioxide (NO2).
□ Nitrogen dioxide is very toxic. NOx emissions are also a serious environmental concern
because of their role in the smog formation.
351.
Ans. (C)
Sol. □ Minamata disease, sometimes referred to as Chisso-Minamata disease, is a neurological
syndrome caused by severe mercury poisoning. Symptoms include ataxia, numbness in the hands
and feet, general muscle weakness, narrowing of the field of vision and damage to hearing and
speech.
□ Blue Baby Syndrome (methaemoglobinaemia) is caused by the decreased ability of blood
to carry vital oxygen around the body. One of the most common causes is nitrate in drinking
water.
352.
Ans. (A)
Sol. □ Bhopal Gas Tragedy (1984) was caused by the release of Methyl Isocyanate (CH3NCO or
C2H3NO) from the pesticide plant of Union Carbide (now Dow Chemicals), an MNC in B h o p a l
(Madhya Pradesh).
□ Methyl isocyanate is extremely toxic gas and if its concentration in the air touches 21ppm
(parts per million), it can cause death within minutes of inhaling the gas.
353.
Ans. (A)
Sol. Low fluoride concentration in drinking water (less than 0.5 ppm) could cause dental caries (tooth
decay or breakdown of teeth), lack of formation of dental enamel and reduced bone mineralization.
However, an overdose of fluoride leads to dental fluorosis also known as Mottling of tooth.
354.
Ans. (A)
Sol. □ Global warming is the unusually rapid increase in Earth’s average surface temperature over
the past century primarily due to the greenhouse gases released as people burn fossil fuels.
Hence, statement I is true.
□ Global warming occurs when carbon dioxide (CO2) and other air pollutants in the
atmosphere and absorb sunlight and solar radiation that have bounced off the earth’s
surface. Normally this radiation would escape into space, but these pollutants, which can l a s t
for years to centuries in the atmosphere, trap the heat and cause the planet to get hotter.
□ These heat-trapping pollutants specifically carbon dioxide, methane, nitrous oxide, water
vapor, and synthetic fluorinated gases are known as greenhouse Eases, and their impact is called
the greenhouse effect.
□ Increasing concentrations of greenhouse gases results in enhanced greenhouse effect,
which is contributing to warming of the Earth. Hence, statement II is true.
Both the statements are individually true and Statement II is the correct explanation of
Statement I.
355.
Ans. (D)
Sol. □ A carbon credit is a generic term for any tradable certificate or permit representing
the right to emit one ton of carbon dioxide or another greenhouse gas with a carbon
dioxide equivalent to one ton of carbon dioxide. Each carbon credit represents 1000 kg of
atmospheric carbon dioxide which has been avoided or
captured.
□ Under Carbon Credits Trading mechanism countries that emit more carbon than the
quota allotted to them buy carbon credits from those that emit less. More carbon emitting
countries, by this way, try to keep the limit of carbon emission specified to them.
□ Carbon credits are traded on various exchanges across the world. Multi-Commodity
Exchange of India (MCX) launched futures trading in carbon credits in 2009. The principles
that support carbon credits are enshrined in the Kyoto Protocol.
356.
Ans. (D)
Sol. □ The Ministry of Environment, Forest and Climate Change launched the National Air
Quality Index (AQI) under the Swachh Bharat Abhiyan. It is published every month by the
Central Pollution Control Board (CPCB) along with a numerical value and colour code which helps
in comparing air pollution level in cities.
□ There are six AQI categories, namely Good, Satisfactory, Moderately polluted, Poor, Very
Poor, and Severe. Each of these categories is decided based on ambient concentration
values of air pollutants and their likely health impacts (known as health breakpoints).
□ The measurement of air quality is based on eight pollutants, namely:
Particulate Matter (size less than 10 µm) or (PM10),
Particulate Matter (size less than 2.5 µm) or (PM2.5),
Nitrogen Dioxide (NO2),
Sulphur Dioxide (SO2),
Carbon Monoxide (CO),
Ozone (O3),
Ammonia (NH3), and
Lead(Pb)
357.
Ans. (A)
Sol. □ The Paris Agreement is a legally binding international treaty on climate change. It was
adopted by 196 Parties at COP 21 in Paris in 2015 and entered into force on 4th N o v e m b e r,
2016.
□ Its goal is to limit global warming to well below 2, preferably to 1.5 degrees Celsius,
compared to preindustrial levels.
□ India ratified the Paris Agreement on 2nd October, 2016.
□ Some of major features of the Paris Agreement are as follows:
It acknowledges the development imperatives of developing countries.
The Agreement recognizes the developing countries’ right to development and their efforts to
harmonize development with environment, while protecting the interests of the most vulnerable.
Hence, statement 1 is correct.
It follows bottom-up approach where countries by themselves submit their plans for climate action
known as nationally determined contributions (NDCs) keeping in mind the national interest.
The objective of the Agreement further ensures that it is not mitigation-centric and includes other
important elements such as adaptation, loss and damage, finance, technology, capacity building and
transparency faction and support. Hence, statement 2 is not correct.
The Paris Agreement recognizes the importance of sustainable lifestyles and sustainable patterns of
consumption with developed countries taking the lead, and notes the importance of ‘ c l i m a t e
justice’ in its Preamble.
358.
Ans. (A)
Sol. □ A green building is one which uses less water, optimizes energy efficiency, conserves
natural resources, generates less waste and provides healthier spaces for occupants.
□ The Green Rating for Integrated Habitat Assessment (GRIHA) is a national rating system
for green buildings that is adopted while designing and evaluating new buildings. It was
conceived by The Energy and Resources Institute (TERI) and developed jointly with the
Ministry of New and Renewable Energy.
□ GRIHA attempts to minimize a building’s resource consumption, waste generation, and
overall ecological impact to within certain nationally acceptable limits/ benchmarks.
□ Leadership in Energy and Environmental Design (LEED) is an internationally recognized
green building certification system devised by the United States Green Building Council
(USGBC) to evaluate the environmental performance of a building and encourage market
transformation towards sustainable design.
□ It provides third-party verification that a building or community was designed and built
using strategies aimed at improving performance across all the metrics that matter most:
Energy savings, water efficiency, emissions reduction, improved indoor environments quality and
stewardship of resources and sensitivity to their impacts.
□ The Indian Green Building Council has adapted LEED system and has launched LEED I n d i a
version for rating of new construction.
□ LEED is for all building types and all building phases including new construction, interior
fit outs, operations and maintenance and core and shell. Hence, 1, 2 are correct and 3, 4 are n o t
correct.
□ Some of the benefits of a green design to a building owner, user, and the society as a
whole are as follows:
Reduced energy consumption without sacrificing the comfort levels,
Reduced destruction of natural areas, habitats, and biodiversity, and reduced soil loss from erosion etc.
Reduced air and water pollution (with direct health benefits),
Reduced water consumption,
Limited waste generation due to recycling and reuse,
Reduced pollution loads,
Increased user productivity,
Enhanced image and marketability.

359.
Ans. (A)
Sol. □ The United Nations Conference on Environment and Development (UNCED), also known
as the ‘Earth Summit’, was held in Rio de Janeiro, Brazil, in 1992. It led to the final development
of the Kyoto Protocol that was adopted in Kyoto, Japan in 1997 and entered into force in 2005.
□ The Kyoto Protocol is an international agreement linked to the UNFCCC, which commits
its parties by netting internationally binding emission reduction targets. It commits industrialized
countries to stabilize greenhouse emission based on principles of the convention.
□ It recognized that developed countries are principally responsible for the current high
levels of GHG emissions in the atmosphere as a result of more than 150 years of Industrial
activity.
□ The detailed rules for the implementation of the Protocol were adopted at COP-7 in
Marrakesh, in 2001 and are referred to as the Marrakesh Accords.
360.
Ans. (A)
Sol. □ The term smog was first coined during the 1950s when it was used to describe a mixture
of smoke and fog experienced in London in 1952. The smog covered the city of London for
five days, popularly known as Great Smog of London. Hence, statement 1 is correct.
□ The principal component of smog are sulphur oxides, smoke and particulate matter; other
components include nitrogen oxides, carbon monoxide and volatile organic compounds.
Hence, statement 2 is correct.
□ The London SMOG occurs generally early in the morning on cold wet winter days, as under
high pressure during winters, wind speeds are low causing smoke and fog to stagnate.
Hence, statement 3 is correct.
361.
Ans. (D)
Sol. Tertiary treatment is the final cleaning process that improves wastewater quality before it is reused,
recycled or discharged to the environment.


Tertiary Treatment Features
methods for
wastewater
Ion-exchange method □ Ion-exchange also known as Demineralization, is a process by which
ions held on a porous, essentially insoluble solid are exchanged for
ions in solution that is brought in contact with it.
□ Both the contaminant and the exchanged substance must be
dissolved and have the same type of electrical charge (positive or
negative),
□ A typical example of ion exchange is a process called “water
softening aiming to reduce calcium and magnesium content.
The process is also efficient in removing toxic metals from water.
Hence, 1 is correct.
Reverse osmosis □ Reverse osmosis is the process that removes contaminants from
water by using pressure to force water molecules through
a semi
permeable membrane.
□ The process can be used to remove ions, mineral chemicals such
as lead, fluoride, chlorine, and other impurities from
drinking water.
Hence, 2 is correct.
Chemical oxidation □ Chemical oxidation involves adding or generating oxidants such
method as hydrogen peroxide (H2O2), bleaching liquor
(NaOCI), chlorine dioxide
(CIO2), chlorine gas (Cl2), peroxy acetic acid (C2H4O3) and
pure oxygen (O2) in the wastewater.
□ In water and wastewater engineering, chemical oxidation serves
the purpose of converting putrescible pollutant
substances to innocuous or
stabilized products. Hence, 3 is correct.
Activated sludge □ Activated sludge process is a secondary treatment but not
process tertiary treatment.
□ The activated sludge process is the biological process that is used
to treat sewage and waste water commonly referred as
effluent.
□ The process uses bacteria in aerobic conditions to degrade the
biodegradable organics into carbon dioxide, water, and
other inorganic compounds. Hence, 4 is not correct.
362.
Ans. (A)
Sol. □ Fly Ash is a byproduct from burning of coal in electric power generating plants. It is called
fly ash because it is transported from the combustion chamber by exhaust gases.
□ It is collected from the exhaust gases by electrostatic precipitators or bag filters.
□ Fly ash includes substantial amounts of silicon dioxide (SiO2), aluminium oxide (Al2O3),
ferric oxide (Fe2O3) and calcium oxide (CaO).
□ Fly ash possesses good pozzolanic properties due to the presence of active and finely
divided silica, alumina and calcium oxide, which provide it with cement like qualities in
combination with lime rich material.
□ Thus, fly ash emitted by thermal power plants can be used for manufacturing bricks,
blocks, aggregates and cement production. It is also used as a landfill material.
363.
Ans. (B)
Sol. □ Rotterdam Convention was adopted in 1998 in Rotterdam, Netherlands and entered into
force on 24th February, 2004.
□ It covers toxic hazardous pesticides and industrial chemicals that have been banned or
severely restricted for health or environmental reasons by Parties and which have been
notified by Parties for inclusion in the Prior Informed Consent (PIC) procedure.
□ The Convention creates legally binding obligations for the implementation of the Prior
Informed Consent (PIC) procedure.
□ The objective of the Convention is to contribute to the environmentally sound use of those
hazardous chemicals and facilitate information exchange about their characteristics.
364.
Ans. (D)
Sol. □ Besides other actions like shock and blast, nuclear explosions also generate high intensity
flux of radioactive radiations such as neutron and gamma radiation which are highly damaging to
living organism. Hence, statement 1 is correct.
□ Most tsunamis are caused by large earthquakes on the seafloor when tectonic plates
move past each other suddenly. It causes sudden vertical displacements which disturbs the
ocean’s surface, displace water, and generate destructive tsunami waves. Hence, statement 2 is
not correct.
□ Acid rain, or acid deposition, is a broad term that includes any form of precipitation with
acidic components, such as sulfuric or nitric acid that fall to the ground from the
atmosphere in wet or dry forms.
Acid rain results when sulfur dioxide (SO2) and nitrogen oxides (NOx) are emitted into the atmosphere and
transported by wind and air currents. The SO2 and NOx react with water, oxygen and other chemicals
to form sulfuric and nitric acids. falling to the ground, disturbing the flora and fauna in the region.
Thus, it is a major environmental concern. Hence, statement 3 is not correct.

□ Meteorological conditions such as high atmospheric pressure, low temperature, low humidity
and large diurnal amplitude have an important effect on the amount of pollution in the
atmosphere. Air movements influence the dispersion of air pollutants. If the air is calm and
pollutants cannot disperse, then the concentration pollutants will build up.
On the other hand, when strong, turbulent winds blow, pollutants disperse quickly, resulting in lower pollutant
concentrations. Another example can be of photochemical smog, which requires Sunlight. Thus, we
can say that Air pollution also has some impact on meteorology. Hence, statement 4 is correct.
365.
Ans. (B)
Sol.
Greenhouse Source
Gas
Carbon dioxide □ There are both natural and anthropogenic
sources of carbon dioxide emissions. Natural
sources include decomposition, ocean release
and respiration.
□ Whereas anthropogenic emission results from
burning of fossil fuels like coal, oil and natural
gas in various industrial processes as well as from
thermal power stations.
Chlorofluorocarbon (CFCs) □ The most common emitters of CFCs are
refrigerants, air
conditioners, aircraft Halon (a chemical
compound
formerly used in firefighting) and aerosol
sprays.
Nitrous Oxide □ Nitrous oxide is emitted during agricultural, land
use, industrial activities, combustion
of fossil fuels, solid
waste, water-logged paddy fields as well
as during treatment
of a wastewater
Sulphur dioxide □ Sulphur dioxide (SO2) is primarily emitted from
fossil fuel
combustion at power plants, brick Kilns,
automobile
emission etc.
366.
Ans. (A)
Sol. □ Hydrogen is a clean fuel that, when consumed in a fuel cell, produces only water,
electricity, and heat. Due to their high efficiency and zero-or near zero-emissions operation, it
is seen as a green alternative to conventional fossil fuels.
□ Whereas conventional fuel such as diesel, kerosene and coal when go through combustion
produces harmful compounds/ pollutants such as carbon dioxide, carbon monoxide, sulphur
dioxide, nitrogen oxide etc.
367.
Ans. (C)
Sol. Acid rain results when sulfur dioxide (SO2) and nitrogen oxides (NOx) are emitted into the atmosphere
react with water, oxygen and other chemicals in the presence of sunlight to form sulfuric and nitric
acids, which then falls to the ground in the form of wet or dry deposition.
368.
Ans. (D)
Sol. □ Energy labeling is one of the most cost-effective policy tools for improving energy
efficiency and lowering energy cost of appliances/equipment for the consumers.
□ Bureau of Energy Efficiency (BEE) under Ministry of Power provides Standards and
Labeling for electrical appliances.
□ The star rating is a measure of energy efficiency of an appliance.
□ Higher rating means lower the energy consumptions and thereby better saving. Usually,
a BEE Star Label can be found on heavy electrical home appliances like air conditioners,
refrigerators and washing machines, etc., with the number of stars it has got and an e s ti m a t e d
power consumption of the appliance in a year.
□ BEE has introduced a new star rating methodology for air conditioners manufactured and
sold in India starting 1st January, 2018. It is called Indian Seasonal Energy Efficient Ratio
(ISEER), which is defined as the energy efficiency of cooling.
□ All the given appliances, namely, ceiling fans, electric geysers and tubular fluorescent
lamp carry Bureau of Energy Efficiency Star Label.
369.
Ans. (B)
Sol. □ National Air Quality Index (AQI) is a tool for effective communication of air quality status
to people in terms which are easy to understand. It transforms complex air quality data of
various pollutants into a single numbs (index value), nomenclature and colour.
□ There are six AQI categories, namely Good, Satisfactory Moderately Polluted, Poor, Very
Poor, and Severe.
□ It considers eight pollutants namely:
Carbon Monoxide (CO); hence, 2 is correct.
Nitrogen Dioxide (NO2); hence, 3 is correct.
Sulphur Dioxide (SO2); hence, 4 is correct.
Ozone (O3);
PM 2.5;
PM 10;
Ammonia.(NH3);
Lead (Pb)
□ Carbon dioxide is not included in National Air Quality Index (AQI). Hence, 1 is not correct.
370.
Ans. (A)
Sol. □ Central Government in exercise of the powers conferred under the Environment
(Protection) Act, 1986 in 2009 had set up ‘National Ganga River Basin Authority’ (NGRBA) as
an empowered planning, financing, monitoring and coordinating authority for the GangaRiver.
□ Prime Minister is the ex-officio Chairperson of the Authority, and it has as its members,
the concerned Union Ministers and the Chief Ministers of states through which Ganga flows,
viz., Uttarakhand, Uttar Pradesh, Bihar, Jharkhand and West Bengal, among others. Hence, 3 is
not correct.
□ Key Features
River basin is the unit of planning and management. This is an internationally accepted strategy for the integrated
management of rivers. Hence, 1 is correct.
NGRBA will spearhead river conservation efforts at the national level. Implementation will be by the State
Agencies and Urban Local Bodies. Hence, 2 is correct.
It will take into account the competing demands of water and will seek to ensure minimum ecological flows.
Solid Waste Treatment Plants will be made functional to minimize the pollution load up to discharge standard of
BOD of 30mg/litre requiring dilution to achieve river water quality of 3mg/litre.
The minimum ecological flows or the entire Ganga will be determined through modelling exercises.
NGRBA will take appropriate measures to regulate water abstraction for maintaining ecological flows in the
river.
371.
Ans. (C)
Sol. □ National Mission for a Green India, also known as Green India Mission (GIM), is one of
the eight missions outlined under India’s National Action Plan on Climate Change. It was
launched in February, 2014.
□ Goals of Mission
To increase forest/tree cover to the extent of 5 million hectares (mha) and improve quality of forest/tree cover
on another 5 mha of forest/non- forest lands. Separate sub-targets exist for different forest types and
ecosystems (e.g., wetland, grassland, dense forest, etc.). Hence, 3 is correct.
Improvement in quality of forest cover and ecosystem services of forests/non-forests, including moderately
dense, open forests, degraded grassland and wetlands (5 mha).
Eco-restoration/afforestation of scrub, shifting cultivation areas, cold deserts, mangroves, ravines and abandoned
mining areas (1.8 mha) with separate sub-targets for each one of those.
Improvement in forest and tree cover in urban/ peri-urban lands (0.20 mha).
Improvement in forest and tree cover on marginal agricultural lands/fallows and other non-forest lands under
agro-forestry/social forestry (3 mha.)
To improve/enhance eco-system services like carbon sequestration and storage (in forests and other ecosystems),
hydrological services and biodiversity, along with provisioning services like fuel ,timber and non-timber
forest produces (minor forest produces or MFPs) etc., which are expected to from the treatment of
10 mha.
To increase forest-based livelihood income for about 3 million households in and around these forest areas; and
enhanced annual CO2 sequestration by 50 to 60 million tons by the year 2020.
Launching 2nd Green Revolution and incorporating green accounting in the Union and State budget are not the
objectives of Green India Mission. Hence, 1 and 2 are not correct.
372.
Ans. (A)
Sol. □ Bioremediation can be defined as any process that uses microorganisms (bacteria and
fungi) or their enzymes to remove and/or neutralize contaminants within the environment to
their original condition.
□ The process is used to treat land or sea which is contaminated by the agents such as
pesticides, oil or solvents. For example, The Energy and Resources Institute (TERI) has developed a
mixture of bacteria called Oilzapper and Oilivorous-S’ which degrades the pollutants of oil-
contaminated sites, leaving behind no harmful residues.
□ However, bioremediation may not provide a feasible strategy at sites with high
concentrations of chemicals that are toxic to most microorganisms. These chemicals include
metals such as cadmium or lead and salts such as sodium chloride.
373.
Ans. (A)
Sol. □ Acid rain, or acid deposition, is a broad term that includes any form of precipitation with
acidic components, such as sulfuric or nitric acid that fall to the ground from the atmosphere
in wet or dry forms. This can include rain, snow, fog, hail or even dust that is acidic.
□ When sulfurous, sulfuric, and nitric acids in polluted air and rain react with the calcite in
marble and limestone, the calcite dissolves. Roughened surfaces, removal of material and l o s s
of carved details are often visible in exposed areas of buildings and statues. Hence, statement 1 is
correct.
□ While burning fossil fuels such as coal to produce electricity, power plants release the
majority of sulfur dioxide and much of the nitrogen oxides. Additionally, nitrogen oxides
and sulfur dioxide are released into the air by the exhaust from cars, trucks and buses. These
pollutants contribute to acid rain. Hence, statement 2 is correct.
□ Eutrophication is the enrichment of a standing water body with nutrients such as
phosphorus and nitrogen. It is characterized by excessive plant and algal growth due to the
increased availability of one or more limiting growth factors needed for photosynthesis such
as sunlight, carbon dioxide and nutrient fertilizers.
□ Eutrophication is the leading cause of impairment of many freshwater and coastal marine
ecosystems in the world. Harmful algal blooms, dead zones, and fish kills are the results of
eutrophication, which begins with the increased load of nutrients to estuaries and coastal
waters. Hence, statement 3 is not correct.
374.
Ans. (A)
Sol. □ Amphibians are one of the most sensitive groups to change, whether that is caused by
habitat loss, invasive species, disease, trade or climate change. Nearly 33% of amphibian
species of the world are categorized as threatened on the Red List.
□ Amphibians are characterized by a rather permeable skin due to which they possess
sensitivity to environmental threats. Based on their complex life cycles and other traits,
amphibians are often recognized as indicators of ecosystem health.
□ Greater than 70% of the world’s amphibian species are in decline. There are two immediate
(proximate) causes of amphibian declines: death and decreased recruitment (reproductive failure).
Pathogens have caused death amongst the amphibians however, the success of the pathogens has
been due to environmental changes, atmospheric changes, habitat modifications and invasive
species. This relates the earth’s sixth mass extinction.
375.
Ans. (D)
Sol. □ The Montreal Protocol on substances that deplete the ozone layer is an international
treaty designed to protect the ozone layer by phasing out the production of ozone depleting
substances like chlorofluorocarbon (CFCs), Methyl Chloroform, CTC (Carbon Tetra Chloride)
halons and Methyl Bromide, etc.
□ The Montreal Protocol has led to the phase-out of 99% of ozone-depleting chemicals in
refrigerators, air conditioners and many other products.
376.
Ans. (A)
Sol. □ The Bah Road Map was adopted at the 13th Conference of the Parties and the 3rd Meeting
of the Parties in December, 2007 in Bali. The Road Map is a set of a forward-looking d e c i s i o n s
that represent the work that needs to be done under various negotiating “tracks’ that essent.ai
to reaching a secure climate future.
□ The Bali Road Map includes the Bali Action Plan, which charts the course for a new
negotiating process designed to tackle climate change.
□ The Bali Action Plan is a comprehensive process to enable the full, effective and sustained
implementation of the Convention through long-term cooperative action beyond 2012, in order
to reach an agreed outcome and adopt a decision. All Parties to the Convention were involved in
crafting the Bali Road Map.
□ The Bali Action Plan is divided into five main categories: shared vision, mitigation,
adaptation, technology and financing. The shared vision refers to a long-term vision for
action on climate change, including a long-term goal for emission reductions.
377.
Ans. (D)
Sol. □ Fly Ash: It is a byproduct from burning of coal in electric power generating plants. It is
called fly ash because it is transported from the combustion chamber by exhaust gases. It is
collected from the exhaust gases by electrostatic precipitators or bag filters.
□ Composition: Fly ash includes substantial amounts of silicon dioxide (SiO2), aluminium
oxide (Al2O3), ferric oxide (Fe2O3) and calcium oxide (CaO).
□ Properties:
Resemble Portland cement but is chemically different.
Exhibit cementitious properties.
□ Uses: It is used in concrete and cement products, road base, metal recovery, and mineral
filler among others.
378.
Ans. (B)
Sol. □ Fertilizer is defined as any organic or inorganic substance, natural or artificial supplying
one or more of the chemical elements/nutrients required for plant growth. They provide six
macronutrients such as Nitrogen (N), Phosphorus (P), and Potassium (K), Calcium (Ca),
Magnesium (Mg), Sulphur (S) and eight micronutrients namely; Boron (B), Chlorine (Cl),
Copper (Cu), Iron (Fe), Zinc (Zn), etc., to the plants for well-balanced growth.
□ Farmers use fertilizers for crops that help them to grow better quality. But at the same
time, it is harmful as the runoff from fields to rivers and water bodies make the quality of t h e
water deteriorate. Precipitation leads to fertilizers making its way to the nearby water bodies.
□ Chemical fertilizers contain phosphates, nitrates that can contribute in enriching the water
bodies with nutrients thus causing eutrophication and water pollution.
379.
Ans. (A)
Sol. □ Major proportion of carbon dioxide emissions are because of combustion of fossil fuels
to produce the heat which is required to power steam turbines. This process is required for
electricity and heat generation.
□ Burning these fuels results in production of carbon dioxide, which is majorly responsible
for global warming.
380.
Ans. (A)
Sol. □ ‘Fly ash’ is a byproduct of burning coal in electricity generating plant. It has some
commercial uses:
‘Fly ash’ can be used in production of bricks for building construction. Fly ash bricks are light in weight, have high
compressive strength and absorbs less heat. Hence, statement 1 is correct.
‘Fly ash’ can be used as a replacement for some of the cement contents of concrete. When mixed with lime and
water, fly ash forms a compound similar to Portland cement. Hence, statement 2 is correct.
□ Composition of fly ash depends upon the composition of coal being burnt. It may contain
Beryllium, Arsenic, unburnt Carbon, Silicon Oxides, Dioxins, etc., which are toxic in nature a n d
are considered as environmental pollutant. Hence, statement 3 is not correct.
381.
Ans. (C)
Sol. □ Nitrogen is an essential element for plant growth. However, due to excessive use of
nitrogenous fertilizers, nitrate concentration accumulates in the edible parts of leafy v e g e t a b l e s .
Consuming these crops can harm human health.
□ Negative impact of excessive use of nitrogenous fertilizers:
Increase in acidity of soil; Hence, 2 is correct.
Leaching of Nitrate into groundwater; Hence, 3 is correct.
□ Commercial Nitrogen fertilizers should be used to supplement and balance the available
Nitrogen supply in soil; however, if applied in excessive amounts, Nitrogen fertilizers may lower
crop quality much more quickly than large applications of the other fertilizerelements.
□ Proliferation of Nitrogen fixing bacteria is the positive outcome of judicious use of
nitrogenous fertilizer m agriculture. Hence, 1 is not correct.
382.
Ans. (C)
Sol. □ The burning of plastic wastes releases toxic gases, the use of chemical fertilizers causes
eutrophication and the burning of dry leaves releases carbon dioxide and other pollutants i n t o
the atmosphere. Thus, they all are harmful for the environment.
□ Hence, the only beneficial practice for a healthy environment is the treatment of domestic
sewage before it is released into the environment, thus, preventing water pollution and
eutrophication.
383.
Ans. (C)
Sol. □ Acid Rain (A-2): It refers to precipitation that is more acidic than the normal. Normally,
rain has a pH value between 5.0 and 5.5 which is slightly acidic. However, when rain c o m b i n e s
with sulphur dioxide or nitrogen oxides, produced from power plants and automobiles, the rain
becomes much more acidic. Typical acid rain has a pH value of 4.0.
□ Depletion in Ozone layer in Atmosphere (B-l): The decrease in ozone concentration mainly
in the stratosphere is referred to as the depletion of the ozone layer. Ozone depleting s u b s t a n c e s
include chlorofluorocarbons (CFCs), hydrochlorofluorocarbons (HCFCs), carbontetrachloride
and methyl chloroform. These compounds release chlorine or bromine when they are
exposed to intense UV light in the stratosphere and contribute to ozone depletion. One
chlorine atom can destroy over 100,000 ozone molecules before it is removed from the
stratosphere.
□ Harmful for human nervous system (C-3): Exposure to high amounts of lead can cause
gastrointestinal symptoms, severe damage to the brain and kidneys and may cause reproductive
effects.
□ Topmost Contribution to greenhouse gas effect (D-4): Gases that trap heat in the
atmosphere are called greenhouse gases. Carbon dioxide, methane, nitrous oxide, sulphur
hexafluoride, etc., are some of the greenhouse gases. Though carbon dioxide has relatively l o w
global warming potential, yet due to longer atmospheric life, it is considered as topmost
contributor to greenhouse effect.
384.
Ans. (C)
Sol. □ Raw material extraction and processing always make an impact on the environment as
they do in soil degradation, water shortages, biodiversity loss, damage to ecosystem functions
and global warming exacerbation. For example, excess use of ground water in leather industry
leads to decrease in water table of ground water and also it pollutes nearby water bodies f r o m
the surface runoff. Hence, 1 is correct.
□ Discharge of toxic chemicals used during industrial processes and activities further
pollutes soil, water bodies and nearby flora. For example, use of chromium and sulphide in
leather industries. Hence, 2 is correct.
□ Although product use and disposal causes a certain amount of pollution, it is not as such
caused by the manufacturing sector as it primarily occurs during and after the consumption of the
product by the final consumer and not the producer. Hence, 3 is not correct.
385.
Ans. (C)
Sol. □ A hydrocarbon is an organic chemical compound composed exclusively of hydrogen and
carbon atoms. Hydrocarbons are naturally-occurring compounds and form the basis of crude
oil, natural gas, coal, and other important energy sources.
□ Hydrocarbons are highly combustible and produce carbon dioxide, water, and heat when
they are burned. Therefore, hydrocarbons are highly effective as a source of fuel.
□ One of the major cause of air pollution is due to automobiles that run on these
hydrocarbon- based fuel5 and due to incomplete combustion
releases harmful gaseous pollutant such as oxides of nitrogen and
sulphur, carbon monoxide etc.
□ Other options such as Oxides of Sulphur, Oxides of nitrogen and smoke are gaseous
pollutant.
386.
Ans. (A)
Sol. □ Mercury (Hg) is a global pollutant that affects human and ecosystem health.
□ Natural sources of mercury include volcanic eruptions and emissions from the ocean.
Anthropogenic (human-caused) emissions include mercury that is released from fuels or r a w
materials, or from uses in products or industrial processes.
□ Globally, artisanal and small-scale gold mining (ASGM) is the largest source of
anthropogenic mercury emissions (37.7%), followed by stationary combustion of coal (21%) or Coal
based power plants-
□ Products like batteries, compact fluorescent lamps, switches and relays, soaps and
cosmetics, thermometers and blood pressure devices, dental fillings, which use mercury
amalgam, if not treated properly can cause severe health hazard.
□ The Minamata Convention on Mercury is an international treaty designed to protect
human health and the environment from anthropogenic emissions and releases of mercury and
mercury compounds.
387.
Ans. (D)
Sol. □ Solid waste treatment
At solid waste disposal sites (SWDS) the degradable organic carbon in waste is decomposed by bacteria under
anaerobic conditions into methane (CH4) and other compounds. The CH4 emissions from SWDS are
important contributors of global anthropogenic CH4 emissions. Hence, 1 is correct.
□ Natural wetlands
Emissions from wetlands and oceans are the major natural sources of methane. These account for 78% of natural
methane emissions.
The water-logged conditions of wetlands are perfect for microbes.
Part of wetland related emissions gets absorbed by methane-consuming microbes, but a large percentage
escapes into the atmosphere. Hence, 2 is correct.
□ Ruminant animals
Raising cattle and other ruminant animals, digestive processes of termites, etc., cause methane emissions.
Ruminant livestock are the primary producers of methane (CH4). They can produce 250 to 500 litres of methane
per day. Hence, 3 is correct.
388.
Ans. (C)
Sol. □ Greenhouse gases are those gases that trap heat in the atmosphere and warm the planet.
□ The main gases responsible for the greenhouse effect include carbon dioxide, methane,
nitrous oxide, and water vapor and fluorinated gases.
□ Carbon monoxide: Carbon monoxide (CO) is a clear, odourless gas and is a common air
pollutant. It is not considered as a primary greenhouse gas, due to its weak absorption of
terrestrial Infrared radiation. However, its presence affects the abundance of greenhouse gases
such as methane and carbon dioxide. Hence, 1 is not correct.
□ Oxygen: Oxygen is transparent to infrared radiation, thus cannot absorb heat and
contribute to the greenhouse effect. Hence, 2 is not correct.
□ Soot: Sometimes called as lampblack or black carbon soot is a fine black or brown powder
that can be slightly sticky and is a product of incomplete combustion. Soot warms the Earth, as
when deposited on snow and ice, it reduces the albedo of the surface i.e., the ability of the
surface to reflect solar radiation. Hence, 3 is correct.
□ Water vapour: The most abundant greenhouse gas overall, water vapor differs from other
greenhouse gases because changes in its atmospheric concentrations are linked not to human
activities directly but rather to the warming that results from the other greenhouse gases we emit.
Warmer air holds more water and since water vapor is a greenhouse gas, more water absorbs more
heat, inducing even greater warming and perpetuating a positive feedback loop. Hence, 4 is
correct.
389.
Ans. (D)
Sol. □ Hydrofluorocarbons (HFCs) are entirely man-made a group of industrial chemicals
primarily used for cooling and refrigeration. HFCs are potent greenhouse gases that can be
hundreds to thousands of times more potent than carbon dioxide (COJ in contributing to
climate change per unit of mass. Hence, statements 1 and 2 are not correct.
□ Ozone in the upper atmosphere (stratosphere) is destroyed by man-made compounds
such as chlorofluorocarbons (CFCs), hydrofluorocarbons (HFCs) and halons. By shielding the
earth from harmful ultraviolet (UV-B) rays generated from the sun, the stratospheric ozone l a y e r
makes life possible. Decreased concentration of stratospheric ozone leads to increased amounts of
UV-B (shorter wavelengths) reaching the earth’s surface.
□ The Kigali Amendment to phase down HFCs under the Montreal Protocol entered into
force in 2019. Under the amendment, countries commit to cut the production and consumption
of HFCs by more than 80% over the next 30 years to avoid more than 70 billion metric tons
of carbon dioxide equivalent emissions by 2050 and up to 0.5° C warming by the end of
the century. Solutions are available to replace high-global warming potential HFCs in many
sectors and reduce emissions.
390.
Ans. (C)
Sol. □ Cigarette smoke is a complex mixture of chemicals.
□ some smoke components, such as carbon monoxide (CO), hydrogen cyanide (HCN),
and nitrogen oxides, are gases. Others, such as formaldehyde, acrolein, benzene, and certain
N- nitrosamines, are volatile chemicals contained in the liquid vapor portion of the smoke
aerosol.
□ The most abundant pollutants in the smoke coming from cigarettes are:
Carbon Monoxide: It is a poisonous gas with no smell or taste. Carbon monoxide (CO) prevents the blood system
from effectively carrying oxygen around the body, specifically to vital organs such as the heart and
brain. High doses of CO, therefore, can cause death from asphyxiation or lack of oxygen to the brain
and can cause death. The gas is toxic for babies in the womb, infants and individuals with heart or
lung disease.
Tar: It consists of several cancer-causing chemicals. When a smoker inhales cigarette smoke, 70% of the tar
remains in the lungs.
Benzene: It is used in gasoline/cigarettes and can cause blood cancer (leukemia).
Nicotine: It is not carcinogenic (having the potential to cause cancer). However, it is highly addictive. Nicotine is
an extremely fast-acting drug. Exposure to large amount can lead to vomiting, seizures, depression of
the CNS (central nervous system), and growth retardation.
391.
Ans. (D)
Sol. □ Arsenic contamination in groundwater is one of the most serious issues in the drinking
water scenario of India.
□ In India, the groundwater arsenic contamination was first surfaced from West Bengal in 1983.
□ Arsenic (As) is introduced into soil and groundwater during weathering of rocks and minerals
followed by subsequent leaching and runoff. It can also be introduced into soil and groundwater
from anthropogenic sources.
□ The states along the Ganga-Brahmaputra-Meghna (GBM) river basin − Uttar Pradesh, B i h a r ,
Jharkhand, West Bengal and Assam are the worst affected by this human-amplified g e o g e n i c
occurrence.
□ According to data published by the National Rural Drinking Water Programme (NRDWP) of the
Ministry of Jal Shakti (MoJS), 9.6 million people in West Bengal, 1.6 million in Assam, 1.2 million in
Bihar, 0.5 million in Uttar Pradesh and 0.013 million in Jharkhand are at immediate risk from
arsenic contamination in groundwater.
392.
Ans. (B)
Sol. □ A non-fiction book ‘Silent Spring’ written by Rachel Carson became one of the most-
influential books in the modern environmental movement.
□ In this book, a woman took on the chemical industry and raised important questions about
humankind’s impact on nature.
□ Rachel Carson was an American biologist well known for her writings on environmental
pollution and the natural history of the sea.
393.
Ans. (B)
Sol. □ Temperature rise above 3°C would lead to rising sea levels and loss of plant species. The
Amazon rainforest, whose plants produce 10% of the world’s terrestrial photosynthesis, may
turn to savannah as drought and mega-fires would destroy the rain forest, turning trees back
into CO2 as they burn or rot and decompose.
□ The carbon released by the forest destruction will be joined by still more carbon from the
world s soils, together, boosting global temperatures by a further 1.5°C. Hence, 1 is correct.
□ Increase in global temperature will result in mass coral bleaching and further, addition
of CO2 into the ocean will reduce calcification rates and increase coral mortality. Hence, 2 is
correct.
□ Wetland habitat responses to climate change and the implications for restoration will be
realized differently on a regional and global level. Thus, it can be restored and will not d i s a p p e a r
permanently. Hence, 3 is not correct.
□ Climate change affects the ecosystem that provides food, and therefore our security of
food is linked to the security of those ecosystems. Rising carbon dioxide concentrations -could
increase production of some crops, such as rice, soybean and wheat due to increase in the
rate of photosynthesis. However, the changing climate would affect the length and quality of the
growing season. Thus, the cultivation of cereals would see difference in production rather than
being extinct. Hence, 4 is not correct.
394.
Ans. (D)
Sol. □ Nanoparticle emissions in the environment can either be directly or indirectly via a
technical system such as Waste Water Treatment Plants (WWTPs) or landfills. Indirect emissions are
likely to occur either via the effluent of WWTPs, application of biosolids to soil, or leachates from
landfills. Hence, 1 is correct.
□ Commercially manufactured polystyrene nanoparticles are transported through the
aquatic food chain from algae. Plastics do not degrade easily. In the marine environment, plastics
are usually broken down into smaller pieces by the Sun, waves, wind and microbial action.
These micro-and nano plastic particles in the water maybe ingested by filter-feeding marine
organism; such as barnacles, tube worms and sea-squirts. Hence 2 is correct.
□ Free radicals are unstable atoms or molecules with free outer electrons. Free radical
formation can be triggered by nanoparticles such as
fullerenes, carbon nanotubes, quantum dots, emission particles, which in turn could lead to
formation of so-called giant cells, resulting in the development of nodular new tissue
formation called granulomas and overtime can develop into mesothelioma (a type of cancer).
Hence, 3 is correct.
395.
Ans. (D)
Sol. □ Steel industry creates pollution as it uses coal and Iron ore whose combustion releases
various Polycyclic Aromatic Hydrocarbons (PAH) compounds and oxides into the air.
□ In steel furnace, coke reacts with iron ore, releasing iron and generating major
environmental pollutants.
□ The pollutants released from steel producing units are:
Carbon Monoxide (CO); hence, 3 is correct.
Carbon Dioxide (CO2); hence, 4 is correct.
Oxides of Sulphur (SOx); hence, 1 is correct.
Oxides of Nitrogen (NOx); hence, 2 is correct.
PM 2.5;
Waste Water;
Hazardous waste;
Solid waste
□ However, technological interventions in the form of air filters, water filters and other water
saving, power saving and closed container can reduce emissions.
396.
Ans. (C)
Sol. □ Brominated Flame Retardants (BFRs) are mixtures of man-made chemicals that are added
to a wide variety of products, to make them less flammable. They are commonly used in
plastics, textiles and electrical/ electronic equipment.
□ The BFRs are highly resistant to degradation in natural environment. Hence, 1 is correct.
□ BFRs can be accumulated in humans and animals and alter liver enzymes, affect thyroid
levels, and are associated with developmental reproductive and neurotoxicity. Hence, 2 is
correct.
397.
Ans. (B)
Sol. □ Diesel: When diesel fuel is burned in engines, the emissions that result contributes to
air pollution that has serious human health and environmental effects. It produces greater
quantities of certain air pollutants such as sulphur and solid carbon particulates and is
considered as one of the largest contributors to environmental pollution. Burning of 1 kg/L o f
diesel can produce 2.66 kg/L of CO2.
□ Petrol: It is a mixture of volatile, flammable liquid hydrocarbons, and is derived from
crude petroleum. It is used as a fuel for internal-combustion engines and produces toxic gases
such as carbon monoxide, nitrogen oxides, particulate matter and unburned hydrocarbons
which contribute to air pollution. Burning of 1L of Petrol/ gasoline, which contains about 0.63 kg
of carbon, can produce about 2.3 kg of CO2.
□ Compressed Natural Gas (CNG): Natural gas is an odorless, gaseous mixture of
hydrocarbons predominantly made up of methane (CH4).
Two forms of natural gas that are currently used in vehicles are compressed natural gas (CNG) and
liquefied natural gas (LNG). CNG is produced by compressing natural gas to less than 1% of
its volume at standard atmospheric pressure CNG fuel is safer than gasoline and diesel
because it is nontoxic and environment friendly.
398.
Ans. (B)
Sol. □ The pesticide treadmill is a term indicating a situation in which it becomes necessary for
a farmer to continue using pesticides regularly because they have become an indispensable
part of an agricultural cycle.
□ This can occur if pesticides are used on crops or animal herds such that other natural
remedies are no longer effective. The farmer then has no option but to use the pesticides year
after year in each agricultural cycle.
□ It can happen that the effective elimination of one target insect pest allows other insect
pests to thrive, resulting in the farmer having to use other insecticides to eliminate the new p e s t
problem. This can lead to more pests and the need for yet more insecticide types to deal with the
additional problems.
399.
Ans. (C)
Sol. Fluoride is a natural compound that protects teeth from cavities. It helps prevent cavities and decay
by coming in direct contact with the tooth enamel (the outside of the tooth). However, too much
fluoride (higher than 1.5-2 mg/L) ingested for longer durations can cause dental fluorosis.
400.
Ans. (A)
Sol. Compact fluorescent lamp (CFL) contains mercury and its disposal is a major environmental concern
because mercury is a neurotoxin. High levels of mercury intake can damage the brain, the reproductive
system, foetuses and cause behavioural problems.
401.
Ans. (B)
Sol. □ Ethylene bromide (C2H4Br2), also called ethylene dibromide or 1,2-dibromoethane, is
a colourless, sweet-smelling, non-inflammable, toxic liquid belonging to the family of
organohalogen compounds.
□ Ethylene bromide in conjunction with tetraethyl lead (an antiknock agent) is added as a
component of petrol or gasoline to avoid lead oxide deposit around the spark plugs and other
components. The resulting byproduct on combustion leads to the formation of volatile l e a d
bromide.
□ Ethylene bromide is toxic to humans and animals, causing severe irritation to the eyes and
skin and damage to the liver, kidneys and lungs.
402.
Ans. (B)
Sol. □ Filtration is a process where solid particles in a liquid or gaseous fluid are removed by
the use of filter medium that permits the fluid to pass through but retains the solid particles.
However, it does not kill or stop the microorganism’s ability to reproduce.
□ Boiling kills or inactivates viruses, bacteria, protozoa and other pathogens by using heat
to damage structural components and disrupt essential life processes for example, denatured
proteins.
□ Chlorination is a process of adding chlorine compound^ such as sodium hypochlorite to
water. Chlorine kills or inactivates all types of microorganisms such as protozoa bacteria and
viruses. The process is also used to prevent the spread of waterborne diseases such as cholera
dysentery and typhoid.
□ UV-irradiation also known as Ultraviolet germicidal irradiation is a disinfection method
that uses short wavelength ultraviolet light to kill or inactivate microorganisms by destroying
nucleic acids and disrupting their DNA, leaving them unable to perform vital cellular functions.
403.
Ans. (D)
Sol. □ Nitrogen Oxides (NOx) pollution occurs when nitrogen oxides are released as a gas into
the atmosphere during the high-temperature combustion of fossil fuels. These nitrogen
oxides consist mainly of two molecules, nitric oxide (NO) and nitrogen dioxide (NO2).
□ Nitric Oxide pollution can lead to:
Leaf spotting in plants as higher concentration of Nitric Oxide hinders the rate of photosynthesis.
Bronchitis-related respiratory problems in humans as inhaling Nitric Oxide causes pulmonary irritant,
causing priming of lung macrophages and oxidative damage to lung epithelial cells. Even short-term
exposure can irritate the lungs of healthy people.
Production of corrosive gases through photochemical reaction as Nitric Oxide plays an important role in the
formation of Ozone and photochemical smog which increases the rate of corrosion in exposed metals,
Particulate matter and ozone in the smog reacts with the metal surface to form corrosion or rust.
□ Silicosis in human which is a chronic disease related to lungs is caused by inhalation of
silica dust over a successive period of time.
404.
Ans. (B)
Sol. □ Landfill gas (LFG) is a natural byproduct of the decomposition of organic material in
landfills. Methane and Carbon dioxide are the major gases that emit from landfills, which made
up to 90-98% of landfill gases.
□ LFG is composed of roughly 50% methane (the primary component of natural gas) and the
other 50% consists of majorly carbon dioxide (CO2) and small amounts of nitrogen, oxygen,
ammonia, sulfides, hydrogen, carbon monoxide, and non-methane organic compounds
(NMOCs) such as trichloroethylene, benzene, and vinyl chloride.

405.
Ans. (D)
Sol. □ Methane is a naturally occurring gas which has global warming impact 25 times greater
than that of carbon dioxide over a 100-year period.
□ Methane is emitted during the production of coal, natural gas and oil. It also results from
livestock and other agricultural practices, wetlands, land use and by the decay of organic waste
in municipal solid waste landfills. Hence, 1, 2 and 3 are correct.
406.
Ans. (C)
Sol. □ Green Mufler is a technique that involves planting of trees, usually in 4-5 rows, to reduce
the noise pollution. The trees planted act like a buffer and absorb the noise coming from
densely populated areas, roadsides, industrial areas, etc., thereby reducing noise pollution.
□ In addition, it also helps in reducing air pollution by acting as a sink for gaseous pollutants
like carbon dioxide and causing settling of particulate matter.
407.
Ans. (B)
Sol. □ Biodegradable pollutants are the pollutants which can be broken down into simpler,
harmless, substances in nature in due course of time by the actions of microorganisms,
naturally occurring agents such as water, oxygen, sunlight etc., further help or accelerate t h e
decomposition of biodegradable pollutants.
Biodegradable pollutants include domestic waste (waste food, animal products, leather, cloth, paper, wood etc.),
sewage which carries faecal matter, urine etc.
□ Non- biodegradable pollutants are those pollutants which cannot be broken down into
simpler, harmless products by the actions of natural agents such as microorganism, heat, water,
oxygen etc., and act as a pollutant.
Non-biodegradable wastes includes plastics, insecticides like DDT, synthetic fibres, metal, toxic chemicals like
mercury, arsenic, lead etc.
408.
Ans. (C)
Sol. □ Primary pollutants are those pollutants which persist in the environment in the same form
in which they are released into the environment. They are emitted directly from the source. F o r
example, Carbon Monoxide, Sulphur dioxide, Nitrogen Oxides, Hydrocarbons, Particulate matter,
plastic, DDT etc.
□ A secondary pollutant is not directly emitted as such, but forms when it reacts with the
primary pollutant. For example, Ozone, PAN, Smog etc.
Ozone: It is formed by the chemical reaction with the primary pollutant such as oxides of nitrogen (NOX ) and
Volatile Organic Compounds (VOC) in the presence of sunlight.
Peroxyacetil nitrates (PAN): It is formed by the photochemical reaction of hydrocarbons with nitrogen oxides in
the atmosphere.
Smog: The suspended particles like unburnt carbon particles and hydrocarbons mix with smoke and fog in
the atmosphere, especially in the cold weather to form smog which results in a lowered visibility.
Whereas, photochemical smog is produced when sunlight reacts with oxides of nitrogen and at least
one volatile organic compound (VOC) in the atmosphere.
409.
Ans. (A)
Sol. □ The United Nations General Assembly (UNGA) in 1994, proclaimed 16th September as
the International Day for the Preservation of the Ozone Layer, commemorating the date of
signing the Montreal Protocol on Substances that Deplete the Ozone Layer in 1987.
□ The Vienna Convention and its Montreal Protocol in 2009 became the first treaties of the
United Nations to achieve universal ratification.
□ The Parties to the Montreal Protocol on Substances that Deplete the Ozone Layer reached
agreement at their 28th Meeting of the Parties in 2016 at Kigali, Rwanda to phase-down
hydrofluorocarbons (HFCs), a family of potent greenhouse gases by the late 2040s.
410.
Ans. (C)
Sol. □ A bioindicators are living organisms such as plants, planktons, animals, and microbes,
which are utilized to screen the health of the natural ecosystem in the environment.
□ Lichens can be used as air pollution indicators, especially of the concentration of sulphur
dioxide in the atmosphere.
□ They are sensitive to sulphur dioxide (SO2) and do not grow in polluted areas. Since they
derive their water and essential nutrients mainly from the atmosphere rather than, from the s o i l ,
their presence indicates the absence of pollution in an area and vice-versa.
411.
Ans. (D)
Sol. As per the NITI Aayog’s report “Study of Assesment of Water Foot Prints of India’s Long term Energy
Scenarios” thermal power plants of India account for 87.8% of total industrial water consumption in
the country while Paper and pulp consumes 2.27%, Engineering 5% and Textiles 2%.
412.
Ans. (B)
Sol. □ Electronic waste (or e-Waste) includes electronics and other items that have reached
the end of their useful life, such as computers, laptops, TVs, DVD players, mobile phones,
mp3 players,etc.
□ Toxic chemicals in a computer system are Lead, Cadmium, Mercury, Beryllium,
Brominated Flame Retardants (BFR), Polyvinyl Chloride and Phosphorus compounds. T h e i r
improper handling and burning releases hydrocarbons and pollute waterbodies.
□ Metals contained in circuit boards are Cadmium, Antimony, Lead and Chromium. Mercury
is present in switches and lamps of many photocopiers, scanners, and fax machines. Lead c a n
also be found in monitors. Hence, 2, 3, 5 and 6 are correct.
□ Copper Beryllium alloy is used to provide “spring memory” that ensures continuous,
fatigue- free electrical connections, meaning higher processing speeds and better performance for
personal computers, routers, and the internet as well as radars, avionics, and defense system.
Hence, 1 is correct.
□ Plutonium is a highly reactive synthetic element of the actinide family which occurs in Uranium ores
and is used as a fuel in nuclear power plants because of its ability to undergo nuclear fission. About
15 isotopes of Plutonium are known to exist and all of these isotopes are radioactive. It is not
used in computers or their parts. Hence, 7 is not correct.
□ Heptachlor is an organochlorine (Cyclodiene) insecticide which was first isolated from
technical chlordane in 1946 and was used primarily by farmers to kill insects in seed grains and on
crops, as well as by exterminators and home owners to kill termites. Hence, 4 is not correct.
413.
Ans. (C)
Sol. □ Drinking water in some parts of India have contaminants like Arsenic and Fluoride.
□ The sources of Arsenic are run off from orchards. Hence, 1 is correct.
□ The sources of Fluoride are erosion of natural deposits, discharge from fertilizers and
Aluminium factories. Hence, 3 is correct.
□ The drinking water in some parts of India also contains traces of Uranium. The main
source of the Uranium contamination is natural, but human factors such as groundwater- table
decline and Nitrate pollution may exacerbate this problem. Hence, 5 is correct.
□ Some naturally occurring elements in drinking water are Lead, Arsenic, Mercury, Radium,
Chloride, Iron and Copper compounds. Most of these are not harmful when consumed in small
quantities. But when the consumption is higher than the prescribed amount, it could be harmful
and sometimes even fatal.
□ Water in India is contaminated with salinity, Arsenic, Fluoride, Iron, Nitrate, and heavy
metals.
□ sorbitol is a sugar alcohol found in fruits and plants with diuretic, laxative and cathartic
property. Unabsorbed Sorbitol retains water in the large intestine through osmotic pressure,
thereby stimulating peristalsis of the intestine and exerting its diuretic, laxative and cathartic
effect. In addition, Sorbitol has one-third fewer calories and 60 % the sweetening activity of
sucrose and is used as a sugar replacement in diabetes. Hence, 2 is not correct.
□ Formaldehyde is a colorless, strong-smelling gas used in making building materials and
many household products. Formaldehyde is quickly broken down in the air -generally within
hours. It dissolves easily in water, but does not last long there, either. Hence, 4 is not correct.
414.
Ans. (D)
Sol. □ Acid rain or acid deposition is a broad term that includes any form of precipitation with
acidic components, such as Sulphuric or Nitric Acid that fall to the ground from the atmosphere
in wetordry forms.
□ This can include rain, snow, fog, hail or even dust that is acidic or has a pH level below
that of normal rain (about pH of 5 to 5.5).
□ Volcanoes are the natural source of the Sulphur Dioxide (SO2) and Nitrous Oxide (NOX),
while a substantial amount of SO2 and NOX is released from fossil-fuel power plants, vehicles
and oil refineries.
□ Electric power generation accounts for two third of SO2 and one fourth of NOx release.
□ A chemical reaction happens when Sulphur Dioxide and Nitrogen Oxides mix with water,
Oxygen and other chemicals in the air. They then become Sulphuric and Nitric Acids that m i x
with precipitation and fall to the ground.
415.
Ans. (D)
Sol. □ Carbon Dioxide (CO2) is an important heat-trappjnp (greenhouse) gas, which is released
through human activities such as deforestation and burning fossil fuels as well as natural
processes such as respiration and volcanic eruptions.
□ It has a long lifetime in Earth’s atmosphere. Carbon Dioxide strongly absorbs energy with a
wavelength of 15 µm (micrometers). This makes carbon dioxide a good absorber of wavelengths
falling in the infrared radiation region of the spectrum.
416.
Ans. (B)
Sol. □ Carbon sequestration refers to the process of removing carbon from the atmosphere and
depositing it in a reservoir. In simple language, carbon sequestration encompasses all forms of
carbon storage such as oceans, plants, soil and underground geologic formations.
□ Contour bunding is a traditional low-cost method of soil conservation suitable for sloping
land; it promotes water retention and helps prevent erosion. It is a proven sustainable land
management practice for marginal, sloping, and hilly land where the soil productivity is very
low. However, it has no role in carbon sequestration. Hence, 1 is not correct.
□ Relay cropping is the growing of two or more crops in the same field with the planting of
the second crop after the first one has completed its development. Row intercropping is the
cultivation of two or more crops simultaneously on the same field with a row arrangement. It has
no role in carbon sequestration. Hence, 2 is not correct.
□ No-till farming is also called zero tillage or direct drilling-It is a way of growing crops or
pasture from year to year without disturbing the soil through tillage. No-till is an a g r i c u l t u ra l
technique which increases the amount of water that infiltrates into the soil, the s o i l ’ s
retention of organic matter and its cycling of nutrients.
□ It has the potential through storage of soil organic matter in the soil of crop fields. Tilling
inverts soil layer, mixes air, and greatly increases microbial activity. Organic matter breaks down
much faster, releasing its carbon into the atmosphere. Hence, 3 is correct.
417.
Ans. (A)
Sol. □ When Carbon Dioxide (CO2) is absorbed by sea water, a chemical reaction occurs and that
reduces the sea water pH, Carbonate ion concentration, and saturation states of biologically
important Calcium Carbonate minerals. These chemical reactions are termed “ocean acidification”.
□ Calcium Carbonate minerals are the building blocks for the skeletons and shells of many
marine organisms. In areas where most life now congregates in the ocean; the seawater is
supersaturated with respect to Calcium Carbonate minerals. This means there are abundant
building blocks for calcifying organisms to build their skeletons and shells. However, c o n ti n u e d
ocean acidification is causing many parts of the ocean to become under saturated with these
minerals, which is likely to affect the ability of some organisms to produce and maintain their
shells. Hence, statement 1 is correct.
□ Many marine organisms that produce Calcium Carbonate shells or skeletons are negatively
impacted by increasing CO2 levels and decreasing pH in seawater. For example, increasing ocean
acidification has been shown to significantly reduce the ability of reef-building corals to
produce their skeletons. Hence, statement 2 is correct.
□ Oysters and planktons cannot calcify resulting in food web interruption. Hence, statement 3
is correct.
□ Cloud seeding and cloud formation has nothing to do with ocean acidification, as it is the
type of weather modification by dispersing substances in the air that serve as cloud condensation
or ice nuclei that alter the microphysical processes within the clouds. Hence, statement 4 is
not correct.

418.
Ans. (C)
Sol. □ Chlorofluorocarbons (CFCs) are nontoxic, non-flammable chemicals containing atoms of
Carbon, Chlorine and Fluorine. They are widely used as refrigerants, propellants (in aerosol
applications), and solvents. The manufacture of such compounds has been phased out (and
replaced with products such as R-410A) by the Montreal Protocol because they contribute to
ozone depletion in the upper atmosphere.
□ Use of CFCs:
Refrigeration;
Making of plastic foams; Hence, statement 1 is correct.
As pressurizing agent in aerosol cans; Hence, statement 4 is correct.
As cleaning agent for electronic products. Hence, statement 3 is correct.
□ However tubeless tyre industry does not use CFCs. Hence, statement 2 is not correct.
419.
Ans. (B)
Sol. □ Lead poisoning is caused by the absorption of Lead in the system and is characterized
especially by fatigue, abdominal pain, nausea, diarrhoea, loss of appetite, anaemia, a dark l i n e
along the gums, and muscle paralysis or weakness of limbs.
□ Common source of lead poisoning are:
Minning
Burning of fossil fuels
Manufacturing
Smelting units; Hence, 1 is correct.
Paints and Gasoline; Hence, 3 is correct.
□ In pen, colour of the ink comes from either a dye, which can dissolve in water, or a
pigment, which is water insoluble. The dye eosin lends
red ink its color and is made by adding the element Bromine to a
fluorescent compound. Pencils contain a form of solid carbon known as graphite. Hence, 2 and 4 are
not correct.
420.
Ans. (D)
Sol. □ Harmful algal blooms (HABs), occur when colonies of algae (photosynthetic organisms)
that live in the sea and freshwater grow out of control while producing toxic or harmful
effects on people, fish, marine mammals, birds, etc.
□ HABs occur naturally, but human activities contribute to the increased occurrence of these
blooms.
HABs are steadily increasing in the Indian waters b of the following factors:
HAB is linked to excessive discharge of nutrients (mainly Phosphorus and Nitrogen) from sources such
as industries and farmlands flow down the river to the sea and build up at a rate that ‘overfeeds’ the
algae that exist normally in the environment. Hence 1 is correct.
There are reports that indicate presence of HABs in the aftermath of natural phenomena like sluggish water
circulation, unusually high-water temperatures and extreme weather events such as hurricanes
floods, and drought.
Run-off from the land during southwest and northeast monsoons cause some algae blooms in Indian coastal
waters. Hence, 2 is correct.
Upwelling in the sea brings up nutrients, providing conducive condition for growth of algae. Hence, 3 is correct.
Formatting of mud banks and river runoff also causes algal bloom.
Incidents of paralytic shell fish poisoning, following an algal bloom, were reported in past from Tamil Nadu,
Karnataka and Maharashtra.
421.
Ans. (B)
Sol. □ The atmosphere is a blanket of gases that surrounds the earth and contains life saving gas
like O2. It is held around by the earth’s gravity. The most abundant gases in terms of volume a r e
Nitrogen (N2), Oxygen (O2), Argon (Ad), Carbon Dioxide (CO2) and small amounts of other
gases.
□ CO2 has a higher density than the other gases found in air, which makes CO2 heavier than
the air and hence remains in the lower atmosphere and does not rise up or escapes in the outer
atmosphere. Hence, statement 1 is not correct.
□ Oceans are the largest available carbon sink present on earth. The two main mechanisms
of consuming carbon are:
Carbon sequestration − when dead marine organisms get deposited beneath the surface of the oceans.
Photosynthesis by phytoplankton, which are microscopic organisms that live in water bodies, both saline and
fresh. They consume carbon dioxide, and release oxygen into the atmosphere. Hence, statement 2 is
correct.
□ The trapping of air in polar ice is not related to the storage of CO2. Hence, statement 3 is
not correct.
422.
Ans. (D)
Sol. □ Coal based power plants are the major contributor of the atmospheric air pollution and
contribute significantly to the global warming and adverse health effects which can u l ti m a t e l y
lead to diseases like lung cancer.
□ Toxic compounds released from the burning of coal include:
Carbon Dioxide and Carbon Monoxide; Hence, 1 is correct.
Oxides of Nitrogen (NOX); Hence, 2 is correct.
Oxides of Sulphur (SOX); Hence, 3 is correct.
Fly Ash
□ Trace elements like Mercury, Cadmium and lead are also emitted which are also health
hazards.
423.
Ans. (B)
Sol. □ Asphalt is a bitumen, which is obtained as a by- product from crude oil processing. It is
a dark viscous liquid found in natural deposits besides crude oil and can also be obtained
during the processing of petroleum products.
□ Asphalt is mainly used in road construction as a glue or binder to create asphalt concrete.
It is also used in waterproofing of buildings, etc.
□ Bioasphalt is made from renewable sources like household organic waste containing
vegetables, edible and non-edible oil, cereals, wastes from other agro based industry, tree
plantation products like rubber, resins, etc. It is an alternative of conventional asphalt.
Hence, statements 1 and 3 are correct and statement 2 is not correct.
□ Benefits
Asphalt is burnt to prepare a binding glue which releases harmful gases in the environment. Substituting
traditional asphalt with bioasphalt can minimize the release of harmful toxic gases in the environment
and prevent global warming.
Bioasphalt is light in color as compared to traditional asphalt which is a dark-colored petroleum product. Roads
made using traditional dark asphalt gives rise to urban heat islands because of their low albedo effect.
Formation of heat island can be prevented by using bioasphalt which is light colored and has high
albedo effect (reflect more energy). Hence, statement 4 is correct.
424.
Ans. (A)
Sol. □ Appropriate technologies are still not sufficiently available to combat climate change.
Hence, statement 1 is correct.
□ India still being in a developing phase, investing a huge amount on research and
development might be a challenging task, since poverty, unemployment and infrastructure
development still being the core issues. Hence, statement 2 is correct.
□ India has strict regulatory laws regarding the establishment of polluting industries by
developed countries in India. Hence, statement 3 is not correct.
425.
Ans. (B)
Sol. □ Inhalation of Carbon Monoxide (CO) leads to formation of Carboxyhemoglobin, a stable
complex of Carbon Monoxide and Haemoglobin.
□ Exposure to small concentrations of CO hinders the ability of Haemoglobin to deliver
Oxygen to the body, because CO has much higher affinity for Haemoglobin as compared t o
Oxygen and thus, Carboxyhemoglobin forms more readily than does Oxyhemoglobin (HbO2).
426.
Ans. (D)
Sol. □ Soot, also known as black Carbon, is a black, carbonaceous substance produced during
incomplete combustion of coal, wood, etc. Black Carbon contributes to climate warming in t w o
ways. First, black soot particles in the air absorb sunlight and directly heat the surrounding
air. Second, soot falling on snow changes the reflecting surfaces into absorbing ones, i.e., soot
decreases the albedo and therefore, increases the melting rate of snow and ice. Hence, 1 is
correct.
□ As per IPCC, Sulphur Hexafluoride (SF6) is the most potent greenhouse gas, with a global
warming potential of 23,900 times that of CO2 over a 100 year of time period. Hence, 2 is
correct.
□ Water vapor is known to be Earth’s most abundant greenhouse gas, which results in
heating of the atmosphere. Hence, 3 is correct.
427.
Ans. (C)
Sol. □ Rice paddies are one of the largest anthropogenic sources of methane.
□ Anaerobic decomposition of organic material in flooded rice fields produces Methane
(CH4), which escapes to the atmosphere primarily by diffusive transport through the rice
plants during the growing season. Methane (CH4) is a greenhouse gas with a Global W a r m i n g
Potential (GWP) of 28-36 over a 100- year period. Hence, statement 1 is correct.
□ Nitrous Oxide gets released into the atmosphere when Nitrogen based fertilizers are used
during cultivation. Among oxides of Nitrogen, Nitrous Oxide (N2O) is a greenhouse gas with
a GWP of 265-298 times that of CO2 in a 100-year time scale. Hence, statement 2 is correct.
428.
Ans. (B)
Sol. □ A fluorescent lamp is a low-pressure Mercury-vapor gas discharge lamp that uses
fluorescence to produce visible light. An electric current in the gas excites Mercury vapor, which
produces short-wave ultraviolet light that then causes a phosphor coating on the inside of the
lamp to glow.
□ A fluorescent lamp converts electrical energy into useful light much more efficiently than
incandescent lamps. But, its indiscriminate disposal causes Mercury pollution in the environment.
429.
Ans. (C)
Sol. □ Acid rain, or acid deposition, is a broad term that includes 8ny form of precipitation with
acidic components, such as Sulfuric or Nitric Acid that fall to the ground from the atmosphere
in wet or dry forms. This can include rain, snow, fog, hail or even dust that is acidic.
□ Acid rain results when Sulphur Dioxide (SO2) and Nitrogen Oxides (NOx) are emitted into t h e
atmosphere and transported by wind and air currents. The SO2 and NOx react with water, Oxygen
and other chemicals to form Sulfuric and Nitric Acids. These then mixes with water and other
materials before falling to the ground. Hence, 2 and 3 are correct.
□ Oxide of Hydrogen, i.e., H2O does not cause acid rain on its own. It is only when it combines w i t h
Oxides of Sulphur or Nitrogen the acid rain occurs. Hence, 1 is not correct.
430.
Ans. (B)
Sol. □ The Climate Change Accountability Bill was submitted in the Canadian Parliament in 2 0 0 6
and eventually passed in 2008.
□ The object of the bill was to meet Canada’s global climate change obligation under
UNFCCC.
431.
Ans. (D)
Sol. □ Rice agriculture is a major source of atmospheric Methane. The warm, waterlogged soil
of rice paddies provides ideal conditions for methanogenesis (i.e., formation of Methane by
microbes known as methanogens). The vast majority of Methane is released into the atmosphere
through rice fields. Hence, 1 is correct.
□ Methane (CH4) is a gas formed as part of the process of coal formation. When coal is
mined, Methane is released from the coal seam and the surrounding disturbed rock strata.
Hence, 2 is correct.
□ Much of the world’s livestock are ruminants - such as sheep, goats, camel, cattle, and
buffalo - who have a unique, four-chambered stomach. In the chamber called the rumen,
bacteria break down food and generate methane as a by-product. Hence, 3 is correct.
□ Wetlands are the largest natural source of atmospheric Methane in the world, and
therefore remain a major area of concern with respect
to climate change. Wetlands are characterized by waterlogged soils and distinctive communities
of plant and animal species that have evolved and adapted to the constant presence of water.
This high level of water saturation creates conditions conducive to Methane production.
Hence 4 is correct.
PRACTICE QUESTION
432.
Ans. (B)
Sol. □ Ozone (composed of three atoms of oxygen) occurs both in the Earth’s upper atmosphere
(stratosphere) and at ground level (troposphere). However, in stratosphere, it occurs naturally
and forms a protective layer that shields us from the Sun’s harmful ultraviolet rays while in t h e
troposphere near ground level, ozone is formed when pollutants emitted by automobiles,
power plants, industrial boilers, refineries, chemical plants and other sources react in the
presence of sunlight. Hence, statement 1 is not correct and statement 2 is correct.
□ Ozone at the ground level is a pollutant with high toxic effects. It makes our eyes itch,
burn and water. It also lowers our resistance to cold and pneumonia. Hence, statement 3 is
correct.
433.
Ans. (B)
Sol. □ Suspended Particulate Matter (SPM) comprises of any solid and liquid particles suspended
in the air like dust, pollen, soot, smoke and liquid droplets for an extended period of time.
Hence, statement 1 is not correct.
□ Major sources of SPM are vehicular emission, power plants, construction activities, oil
refineries, industries etc. An increase in concentration of SPM causes haze and reduces
visibility.
□ The finer particles, when inhaled, can lodge in our lungs and cause lung damage and
respiratory problems. Of these, particles less than 2.5 micrometers in diameter, also known as PM
2.5, pose greater risk to health. Hence, statement 2 is correct.
434.
Ans. (D)
Sol. □ Air Pollution can be defined as the presence of any solid, liquid or gaseous substance
including noise and radioactive radiation in the atmosphere in such concentrations that may
be directly and indirectly injurious to humans or other living organisms, plants or interfering
with the normal environmental processes.
□ The level of air pollution can be affected by :
Increasing traffic: Vehicular emission can lead to increase in concentration of pollutants such as carbon monoxide,
suspended particulate matter (SPM), lead, nitrogen compounds, etc., that have adverse effect on
human well being. Hence, 1 is correct.
Increase in Population: Large concentrations of people and their activities can lead to increased energy
consumption, transportation/motorization, etc., thus, further increasing air pollution. Hence, 2 is
correct.
Economic Development: Economic development in the initial phase leads to increase in consumption of non-
renewable resources and, therefore, higher level of pollution to satisfy increased energy consumption.
However, after certain level of economic and scientific growth, a society begins to improve its
relationship with the environment and levels of pollution as improved technology enables higher
output with less pollution. Hence, 3 is correct.
Industrialization: Industrialization results in a shift of an economy from an underdeveloped agriculture-based to
manufacturing which results in higher level of air pollution as energy consumption is fulfilled by fossil
fuel based resources that emits harmful gases such as carbon dioxide, sulphur
dioxide, ozone, carbon monoxide, etc. Hence, 4 is correct.
Mining: The process of mining increases the dust particles and particulate matter in the air. Hence, 5 is correct.
435.
Ans. (C)
Sol. □ Ozone (composed of three atoms of oxygen) occurs both in the Earth’s upper atmosphere
(stratosphere) and at ground level (troposphere).
□ Though stratospheric ozone is found naturally, ozone in troposphere or ground-level is
formed through a complex reaction involving hydrocarbons, nitrogen oxides and sunlight. It is
formed when pollutants released from gasoline, diesel powered vehicles and oil-based
solvents react with heat and sunlight. Fossil fuel combustion is a primary source of pollutant
gases that lead to tropospheric ozone production. Hence, statement 1 is correct.
E NOx + VOC + Heat & Sunlight = Ozone
□ Smog is a kind of air pollution that reduces visibility. The ozone formed at the ground
level again reacts with volatile organic compounds to produce photochemical Smog. Heavy
smog results in the decrease of natural Vitamin D production, leading to a rise in the cases of
rickets as the level of air pollution is inversely related to the extent of solar ultra violet (UV) radiation
that reaches earth surface. Consequently, it leads to more pollutant areas, less UV passage and as
a result, it lowers vitamin D synthesis. Hence, statement 2 is correct.
436.
Ans. (D)
Sol. A carcinogen is any substance, radionuclide or radiation that can cause or promotes cancer
(carcinogenesis).

Source Health Effects


Name of Pollutant
Asbestos Dust Asbestos Mining, Asbestosis which involves
asbestos sheet severe respiratory
manufacturing problems and may
lead to cancer.
Hence, 1 is correct.
Tobacco Cigarettes, cigars, etc Chronic bronchitis, asthma and
lung cancer, irritation
of eyes, nose and
throat. Hence, 2 is
correct.
Radioactive pollutants Cosmic rays, x-rays, beta Destroy living tissues and
rays, radon and blood cells; affect cell
radium membrane and cell
enzyme functions,
leukemia and
permanent genetic
changes. Hence, 3 is
correct.
Coal dust and particles Coal mines Black lung cancer, pulmonary
fibrosis which leads
to respiratory failure.
Hence, 4 is correct.

437.
Ans. (B)
Sol. □ Fly Ash is the end product of combustion during the process of power generation in the
coal-based thermal power plants. It majorily contains silicon dioxide, aluminum oxide and
calcium oxide. Further, toxic heavy metals like lead, arsenic, cobalt and copper are also found
in fly ash. Hence, statement 1 is not correct.
□ It possesses good pozzolanic properties (having cementations properties) and, therefore,
can be used for making concrete, brick and portland cement. Hence, statement 2 is correct.

438.
Ans. (D)
Sol. □ Cyclone Separator: It is a separation device (dry scrubbers) that uses the principle of
inertia to remove particulate matter from flue gases. The device generally collects heavier
particles more efficiently.
□ Electrostatic Precipitators: An electrostatic precipitator (ESP) removes particles from a
gas stream by using electrical energy to charge particles either positively or negatively. The
emanating dust is charged with ions and the ionized particulate matter is collected and
removed. It is frequently used in boilers, furnace of thermal power plants, cement factories, etc.
□ Arrester: It is used to separate particulate matters from contaminated air.
□ Booms: Booms are floating barriers that are used to clean oil from the surface of water
and to prevent slicks from spreading.
439.
Ans. (A)
Sol. □ HCFC-141b is one of the most powerful ozone-depleting chemicals after
Chlorofluorocarbons (CFCs). It is mainly used as a blowing agent in the foam- m a n u f a c t u r i n g
industries. Hence, statement 1 is correct.
□ HCFC-141b is not produced domestically and all requirements are met through imports.
Hence, statement 2 is not correct.
□ India has successfully phased out Hydrochlorofluorocarbon (HCFC)-141b and is one of the
few countries that are pioneer in the use of non-ozone depleting technologies and have a l o w
Global Warming Potential (GWP).
□ The Ministry of Environment, Forest and Climate Change (MoEFCC) notified that the
issuance of import license for HCFC-141b is prohibited from 1st January, 2020 under O z o n e
Depleting Substances (Regulation and Control) Amendment Rules, 2019.
440.
Ans. (A)
Sol. □ Dissolved Oxygen is the amount of oxygen dissolved in water. It is an important parameter
in assessing water quality because of its influence on the organisms living within a body of
water. Presence of organic and inorganic wastes in water decreases the dissolved oxygen
content of the water.
□ Factors such as photosynthetic activities, oxygen consumption and decomposition of
organic matter by organisms, surface turbulence, etc., determine the amount of DO present i n
water.
□ Eutrophication is a phenomenon where a water body becomes overly enriched with
minerals and nutrients which induces excessive growth of algae or algal bloom. As the algae die and
decompose, the process consumes dissolved oxygen. Thus, it depletes oxygen concentration in
the water body. Hence, statement 1 is correct.
□ Water having DO content below 8.0 mg may be considered as contaminated and water
having DO content below 4.0 mg is considered to be highly polluted. Hence, statement 2 is n o t
correct.
441.
Ans. (C)
Sol. □ Bioremediation is the use of microorganisms (bacteria and fungi) to degrade the
environmental contaminants into less toxic forms.
□ Biomagnification, also known as bioamplification or biological magnification, is a process
by which a compound (such as a pollutant or pesticide) increases its concentration in the
tissues of organisms at successively higher trophic levels or higher levels in a food chain.
□ Bioaccumulation is the gradual accumulation of substances such as pesticides or other
chemicals in an organism. It occurs when an organism absorbs a substance at a rate faster t h a n
that at which the substance is lost by catabolism and excretion.
□ Fortification can be described as a method of adding essential vitamins and minerals to
foods to increase their nutritional value.
442.
Ans. (C)
Sol. □ Water pollution caused by fecal contamination is a serious problem due to the potential
for contracting diseases from pathogens (disease-causing organisms). The coliform bacteria
can be used as an indicator of human fecal pollution in lakes, rivers, coastal areas estuaries a n d
within organisms like fishes. Hence, statement 1 is correct.
□ Biochemical Oxygen Demand (BOD) refers to the amount of oxygen required by
microorganisms to stabilize decomposable organic matter in one litre of polluted water. T h e
more organic matter, the greater the BOD; and the greater the BOD, the lower the amount of
dissolved oxygen available for aquatic organisms. The BOD is, therefore, a reliable gauge of the
organic pollution of a body of water. Hence, statement 2 is correct.
443.
Ans. (C)
Sol. □ Fly ash is the end product of combustion during the process of power generation in the
coal-based thermal power plants. ASHTRACK Mobile App has been launched by the Ministry of
Power for better management of fly ash produced by thermal power plants and b e tt e r
utilization data of thermal power plants. The App will provide an interface between fly ash
producers and potential ash users such as road contractors, cement plants, etc. Hence,
statement 1 is correct.
□ Fly ash can enhance water holding capacity of soil, as a result, crop yield also increases.
Further, it can be used as soil conditioner and amendments. Hence, statement 2 is correct.
444.
Ans. (B)
Sol. □ A consequence of an action that affects someone other than the agent undertaking that
action, and for which the agent is neither compensated nor penalized. An externality can be b o t h
positive or negative.
□ When an individual, a firm or a country takes an action but does not bear all the costs
(negative externality) or all the benefits (positive externality) of the action.
□ A negative externality is a business that causes pollution that diminishes the property
values or health of people in the surrounding area. A positive externality occurs when the
consumption or production of a good causes a benefit to a third party. For example, when o n e
gets education he/she gets a private benefit. But there are also benefits for the rest of
society.
□ Health hazard caused to the person due to smoking by himself/herself is not an example
of externality as the consequence of an action of the agent is suffered by him/ her.
445.
Ans. (B)
Sol. □ Bharat stage (BS) Emission Standards are the emission standards instituted by the
Government of India to regulate the output of air pollutants
from motor vehicles. These regulations are based on the European (EURO) emission
standards. Hence, statement 2 is correct.
□ BS emission norms were first introduced in 2000 under the head “India 2000”. This was
followed by BS2 in 2001, BS in 2005 and BS4 in 2010. However, to drastically reduce v e h i c u l a r
pollution, Government skipped BS5 norms and directly introduced BS6 from 2020.
In BS-VI fuel, the volume of PM 2.5 ranges from 20 to 40 micrograms per cubic metre whereas in BS-IV fuel, it is
up to 120 micrograms per cubic metre.
BS-VI fuel will bring down sulphur content by 5 times from the current BS-IV levels. It has 10 ppm of sulphur as
against 50 ppm in BS-IV. Hence, statement 1 is not correct.
Air pollutants in BS-VI fuel are much less as compared to BS-IV fuel.
BS-VI norms also seek to reduce the level of certain harmful hydrocarbons in the emissions that are produced
due to incomplete combustion of fuel.
On-Board Diagnostics (OBD) are mandatory for all automobiles.
□ The Environment Ministry is responsible for deciding the fuel standard in the country and
the Central Pollution Control Board implements these standards. Hence, statement 3 is correct.
446.
Ans. (C)
Sol. Landfill gas (LFG) is a natural byproduct of the decomposition of organic material in landfills. LFG
is composed of roughly 50% methane (the primary component of natural gas) and the other
50% consists of majorly carbon dioxide (CO2) and small amounts of nitrogen, oxygen, ammonia,
sulphides, hydrogen, carbon monoxide, and non-methane organic compounds (NMOCs) such as
trichloroethylene, benzene and vinyl chloride.
447.
Ans. (A)
Sol. □ pH is a measure of how acidic/basic an aqueous solution is. The range goes from 0 -14 with
7 being neutral. pH of less than 7 indicate acidity whereas a pH of greater than 7 indicates a
base. pH is a measure of the relative amount of free hydrogen and hydroxyl ions In the water.
Water that has more free hydrogen ions is acidic whereas water that has more free hydroxyl i o n s
is basic.
□ This scale, however, is not a linear one but a logarithmic scale in which two adjacent
values increase or decrease by a factor of 10.
□ In a highly polluted city, air is primarily contaminated with the emissions of sulphur
dioxide (SO2) and nitrogen oxides (NOx) resulting from fossil fuel combustion. From these
gases, airborne sulphuric acid (H.SO4) and nitric acid (HNO3) can be formed and dissolved in the
water vapor in the air. So, if you subject the first rain water of a highly polluted city to a pH t e s t ,
it will show a low reading as it would in all likelihood be an acid rain.
448.
Ans. (C)
Sol. □ Mosses are a phylum of non-vascular plants. They produce spores for reproduction instead
of seeds and do not have flowers, wood or true roots and chiefly grow in moist, shaded areas
such as wooded areas and at the edges of streams.
□ Bioindicators are living organisms such as plants, planktons, animals and microbes which
are utilized to screen the health of the natural ecosystem in the environment.
□ Mosses are common plants in all cities and they respond to pollution or drought-stress by
changing shape, density or disappearing, allowing scientists to calculate atmospheric a l t e ra ti o n s .
Thus, they can be used as a ‘bioindicator’ to measure the impact of atmospheric change and
could prove a low-cost way to monitor urban pollution. Hence, statements 1 and 2 are correct.
449.
Ans. (A)
Sol. □ Persistent Organic Pollutants (POPs) are toxic chemicals that persist in the environment
because they resist degradation, thus, they can biomagnify and have adverse affect on
human health and the environment around the world.
□ These have the following characteristics:
Lipophilic (tending to combine with or dissolve in lipids or fats) in nature, thus, can accumulate in the fatty tissue
of animals and human beings. Hence, statement 1 is correct.
Less soluble in water. Hence, statement 2 is not correct.
Long-range environmental transport (LRET) that makes them spread widely in the atmosphere.
□ It is Stockholm Convention that aims to eliminate the production and use of Persistent
Organic Pollutants (POPs). Rotterdam Convention aims to promote shared responsibilities i n
relation to importation of hazardous chemicals. Hence, statement 3 is not correct.
□ The Convention listed twelve distinct chemicals in three categories:
Eight pesticides (aldrin, chlordane, DDT, dieldrin, endrin, heptachlor, mirex and toxaphene)
Two industrial chemicals (poly chlorinated biphenyls and hexachlorobenzene)
Two unintended by-products of many industrial processes such as poly chlorinated dibenzo-p-dioxins and
dibenzofurans commonly referred to as dioxins and furans.
□ Exposure to POPs can lead to cancer, damage to central & peripheral nervous systems,
diseases of the immune system, reproductive disorders and interference with normal infant a n d
child development.
450.
Ans. (D)
Sol. □ Quantitative pollutants are the substances which naturally occur in the
environment and get the status of pollutants when their
concentration increases due to anthropogenic activities, for example, carbon dioxide, when its
concentration increases, it causes harmful effects on the flora and fauna.
□ Insecticides and pesticides come under the category of Qualitative pollutants where
substances do not normally occur in nature but are added by anthropogenic activities.
Hence, statement 1 is not correct.
□ Ozone is an example of secondary pollutant and it is formed by the chemical reaction
with the primary pollutants such as oxides of nitrogen (NOx ) and Volatile Organic Compounds
(VOCs) in the presence of sunlight. Primary pollutants are those pollutants which persist in t h e
environment in the same form in which they are released into the environment like plastics, DDT,
etc. Hence, statement 2 is not correct.
451.
Ans. (D)
Sol. □ National Clean Air Programme (NCAP) was launched by the Ministry of Environment,
Forest and Climate Change in January 2019, is a time-bound national level strategy for pan
India implementation to tackle the increasing air Pollution problem across the country in a
comprehensive manner.
□ It will utilise a collaborative and participatory approach involving relevant Central
Ministries, State Governments, local bodies and other Stakeholders with focus on all sources
of pollution.
□ The tentative national level target of 20%-30% reduction of PM2.5 and PM10
concentration by 2024 is proposed under the NCAP taking 2017
as the base year for the comparison of concentration. Hence,
statement 1 is not correct.
□ Under NCAP, 122 non-attainment cities across 23 states and Union territories have been
identified by the Central Pollution Control Board (CPCB) based on the Air Quality data from (2011-
2015). The city specific action plans have been prepared which, inter-alia, include measures for
strengthening the monitoring network, reducing vehicular/industrial emissions, increasing public
awareness, etc.
□ Non-attainment cities are those that have fallen short of the National Ambient Air Quality
Standards (NAAQS) for over five years. Hence, statement 2 is not correct.
□ Air quality of cities is monitored by State Pollution Control Boards which publish their
results from time to time.
452.
Ans. (A)
Sol. □ United Nations Environment Programme (UNEP) launched the ‘Clean Seas’ programme
in 2017. It aims to engage governments, the general public, civil society and the private
sector in the fight against marine plastic by reducing the use of non-recoverable and single- use
plastics.
□ The campaign contributes to the goals of the Global Partnership on Marine litter, a
voluntary open-ended partnership for international agencies, governments, businesses,
academia, local authorities and nongovernmental organizations hosted by UNEP.
□ India is a member country of the Clean Seas campaign. With the threat of plastic pollution
mounting and its deadly impacts on marine ecosystems, India proposes to phase out of
single-use plastic items by 2022.
453.
Ans. (B)
Sol. □ Dissolved oxygen is the amount of oxygen dissolved in water and it is important for the
survival of aquatic organisms. Presence of organic and inorganic wastes in water decreases t h e
dissolved oxygen (DO) content of the water.
□ Biochemical Oxygen Demand (BOD) is the amount of dissolved oxygen needed by
bacteria in decomposing the organic wastes present in water. It is expressed in milligrams
of oxygen per litre of water. Thus, it is a measure of water pollution by organic wastes. The
higher value of BOD indicates low DO content of water. Water pollution by organic wastes is
measured in terms of Biological Oxygen Demand. Hence, statement 1 is not correct.
□ The run-offs from agricultural field contains high nutrients which enriches the water bodies
with dissolved nutrients (phosphates), stimulating the growth of aquatic plant life usually
resulting in the depletion of dissolved oxygen. The higher amounts of waste increase the ra t e s
of decomposition and oxygen consumption, thereby, decreasing the DO content of water. Hence,
statement 2 is correct.
454.
Ans. (C)
Sol. □ Council of Scientific and Industrial Research (CSIR) has developed less polluting
firecrackers named as safe water releaser (SWAS), safe minimal aluminium (SAFAL) and safe
the rmite cracker (STAR). These are not only environment-friendly but have the unique
property of releasing water vapour.
□ SWAS and STAR crackers eliminate usage of (KNO3) potassium nitrate and sulphur with
consequent reduction in particulate matter SO2 and NOx. It has matching sound intensity with
commercial crackers in the range of 105-110 dBA.
□ SAFAL has minimal usage of aluminium with consequent significant reduction in
particulate matter (35-40%) compared to commercial crackers. It has matching sound intensity
with commercial crackers in the range of 110-115 dBA.
455.
Ans. (C)
Sol. □ Fly ash is a by-product from burning pulverized coal in thermal power plants. During
combustion, mineral impurities in the coal (clay, feldspar, quartz and shale) fuse in s u s p e n s i o n
and float out of the combustion chamber with the exhaust gases. As the fused material rises,
it cools and solidifies into spherical glassy particles called fly ash. Hence, statement 1 is correct.
□ It consists of silica, alumina, oxides of iron, calcium and magnesium and toxic heavy
materials like lead, arsenic, cobalt and copper. If it is not captured and disposed o properly, it
can pollute air and water considerably.
□ Fly ash resembles portland cement but it is chemically different. Fly ash chemically
reacts with the by-product calcium hydroxide released by the chemical reaction between cement
and water to form additional cementitious products that improve many desirable properties
of concrete. All fly ashes exhibit cementitious properties to varying degrees depending on
the chemical and physical properties of both the fly ash and cement. Hence, statement 2 is not
correct.
□ Fly ash enhances water-holding capacity of the land and increases the crop yield. Hence,
statement 3 is correct.
□ Other applications of fly ash are bricks construction, fill materials for road embankments
and abandoned mines reclamation of waste land.

456.
Ans. (C)
Sol. □ Acid rain or acid deposition is a broad term that includes any form of precipitation with
acidic components such as sulphuric or nitric acid that fall to the ground from the atmosphere
in wet or dry forms. This can include rain, snow, fog, hail or even dust that is acidic.
□ Acid rain results when sulphur dioxide (SO2) and nitrogen oxides (NOx) are emitted into
the atmosphere and transported by wind and air currents. The SO2 and NOx react with
water, oxygen and other chemicals to form sulphuric and nitric acids. These then mix with water
and other materials before falling to the ground. Hence, statement 1 is correct.
□ SOx and NOx that create acid rain are often transported to distances far away from their
points of origin by the wind so that the adverse effects of pollution are also experienced at
places remote from the place of genesis. Acid rain can occur outside industrial areas. Hence,
statement 2 is not correct.
□ The pH determines the proliferation of any microbial soecies in a particular environment
and the rate at which it can produce. The optimum pH of most bacteria and protozoa is near
neutrality; most fungi prefer an acidic environment, most blue-green bacteria prefer an alkaline
environment.
□ Therefore, after a long spell of acid rain, microbial content of the soil and water shift from
bacteria bound to fungi bound and causes an imbalance in the microflora. This causes a delay
in the decomposition of soil organic material and an increase in fungal diseases in aquatic life
and forests. Hence, statement 3 is correct.
457.
Ans. (D)
Sol. □ Bioremediation is the use of microorganisms to degrade the environmental contaminants
into less toxic forms. In-situ bioremediation refers to treatment of the contaminated material
at the site whereas ex-situ techniques involve excavating pollutants from polluted sites and
subsequently transporting them to another site : for treatment.
□ In-situ bioremediation techniques are as follows:
Bioventing: This technique involves controlled stimulation of airflow by delivering oxygen to unsaturated zone
(the unsaturated zone lies immediately below the land surface, contains water and air in the open
spaces or pores) in order to increase bioremediation by increasing activities of indigenous microbes.
In bioventing, amendments are made by adding nutrients and moisture to enhance bioremediation
with the ultimate goal being to achieve microbial transformation of pollutants to a harmless state.
Hence, 3 is correct.
Biosparging: This technology uses indigenous microorganisms to biodegrade organic constituents in the
saturated zone (the saturated zone is a zone in which all the pores and rock fractures are filled with
water, it underlies the unsaturated zone). In biosparging, air (or oxygen) and nutrients (if needed) are
injected into the saturated zone to increase the biological activity of the indigenous microorganisms.
Biosparging can be used to reduce concentrations of petroleum constituents that are dissolved in
groundwater, adsorbed to soil below the water table Hence, 4 is correct.
Bioaugmentation: In this, microorganisms are imported to a contaminated site to enhance the
degradation process. Hence, 2 is correct.
□ Ex-situ bioremediation techniques include biopiles which is a bioremediation technology
in which excavated soils are mixed with soil amendments, formed into compost piles and
enclosed for treatment. The basic bio-pile system includes a treatment bed, an aeration
system, an irrigation/nutrient system and a leachate collection system. Moisture, heat,
nutrients, oxygen and pH are controlled to enhance biodegradation. Hence, 1 is not correct.
458.
Ans. (A)
Sol. □ IMD has initiated Color Coded Weather Warning system to alert people ahead of severe
or hazardous weather which has the potential to cause damage, widespread disruption or
danger to life.
□ The four-color codes used by IMD are:
Green (All is well): No advisory is issued visa-vis no action is required.
Yellow (Be Aware): Yellow indicates severely bad weather spanning across several days. It also suggests that the
weather could change for the worse, causing disruption in day-to-day activities. Hence, statement
1 is correct.
Orange/Amber (Be prepared): The orange alert is issued as a warning of extremely bad weather with the
potential of disruption in commute with road and rail closures, and interruption of power supply.
Hence, statement 2 is not correct.
Red (Take Action): When the extremely bad weather conditions are certainly going to disrupt travel and power
and have significant risk to life, the red alert is issued.
□ These alerts are universal in nature and are also issued during floods, depending on the
amount of water rising above land/in a river as a result of torrential rainfall. For instance, when
the water in a river is ‘above normal’ level, or between the ‘warning’ and ‘danger’ levels, a
yellow alert is issued.
459.
Ans. (B)
Sol. □ In 2009, the Ministry of Environment, Forests and Climate Change (MoEF) in association
with Central Pollution Control Board (CPCB), New Delhi and Indian Institute of Te c h n o l o g y
(IIT), New Delhi have carried out an environmental assessment of Industrial clusters across the
country named Comprehensive Environmental Pollution Index (CEPI). Hence, statement 1 is not
correct
□ CEPI provides comprehensive information on the extent and types of pollution caused by a
particular industrial cluster, which then can be used to identify appropriate pollution mitigation
measures. The index captures the various dimensions of environment including air, water and l a n d .
The evaluated CEPI score reflects the environmental quality of the identified industrial
clusters and is a yardstick to assess the progress achieved in the implementation of pollution
mitigation measures. Hence, statement 2 is correct
□ The industrial clusters having CEPI greater than 70, on a scale of 0 to 100, has been
identified as critically polluted and those between 60 to
70 were classified as Severely Polluted.
460.
Ans. (A)
Sol. □ Acid rains affect trees and undergrowth in the forest in several ways, causing reduced
growth or abnormal growth. Dead or dying trees are a common sight in areas effected by acid
rain. Acid rain leaches aluminium from the soil, which is harmful to plants as well as animals. Acid
rain also removes minerals and nutrients from the soil that trees need to grow.
□ At higher elevations, acidic fog and clouds strip nutrients from trees’ foliage, leaving them
with brown or dead leaves and needles. These trees are then less able to absorb sunlight, which
makes them weak and unable to withstand freezing temperatures.
□ The exchange between hydrogen ions and the nutrient cations like potassium and
magnesium in the soil cause leaching of the nutrients, making the soil infertile. Thus, the plant
growth is reduced due to the decrease in nutrients available in the soil.
461.
Ans. (A)
Sol. □ The World Air Quality Report is prepared by Swiss organisation IQAir, which assesses
PM2.5 data from log countries.
□ PM2.5 is an atmospheric particulate matter of diameter of fewer than 2.5 micrometres. It
causes respiratory problems and also reduces visibility. It is an endocrine disruptor that can affect
insulin secretion and insulin sensitivity, thus contributing to diabetes. It is very small and can only
be detected with the help of an electron microscope.
□ Key Points of World Air Quality Report, 2020:
World Capital City Ranking: Delhi has been ranked as the world’s most polluted capital city followed by Dhaka
(Bangladesh), Ulaanbaatar (Mongolia), Kabul (Afghanistan), Doha (Qatar).
World Country Ranking: Bangladesh has been ranked as the most polluted country followed by Pakistan and India.
The least polluted country is Puerto Rico, followed by New Caledonia, US Virgin Islands respectively.
World City Ranking: Hotan in China is the most polluted city, followed by Ghaziabad, Bulandshahar, Bisrakh
Jalalpur, Bhiwadi, Noida, Greater Noida, Kanpur and Lucknow.
Major sources of India’s air pollution include transportation, biomass burning for cooking, electricity
generation, industry, construction, waste burning, and episodic agricultural burning.
462.
Ans. (C)
Sol. □ The Clean Seas campaign works with governments, businesses and citizens to eliminate
the needless use of disposable plastics in order to protect our rivers, seas and the ecosystems,
which sustain life on earth and our very well-being.
□ The Clean Seas campaign was launched at the Economist World Ocean Summit in Bali,
Indonesia in 2017. Indonesia pledged to reduce its plastic footprint by 70 per cent by 2025.
Hence, statement 1 is not correct.
□ The campaign represents the world’s largest global alliance for combatting marine plastic
pollution with commitments covering more than 60 per cent of the worlds coastlines. It seeks
to address the root causes of marine litter and pollution through a three-phased strategy
over five years (2017-2021). Hence, statement 2 is correct.
□ The campaign pledges to cut back on single-use plastics and microbeads, protect national
waters and encourage more recycling. It will be done through working with Governments i n
support action, private sector contribution and with all the citizens who call for an action o n
this issue. Hence, statement 3 is correct.
□ Microbeads are manufactured solid plastic particles of less than five millimetres in their
largest dimension and are used in exfoliating personal care products, toothpastes and in
biomedical and health-science research. Microbeads can cause plastic particle water pollution and
pose an environmental hazard for aquatic animals in freshwater and ocean
water. Greenpeace refers to it as a “toxic time bomb”.
□ India joined the ‘Clean Seas Campaign’ on World Environment Day in 2018. It made a
commitment to address plastic pollution upstream by banning all singleuse plastics by 2022.
463.
Ans. (D)
Sol. The acidic particles corrode metal and cause paint and stone to deteriorate more quickly. Hence, pair
1 is correctly matched.
Acid rain results in embrittlement and discoloration of Paper. Hence, pair 2 is correctly matched.
Acid rain can cause cracking in leather and rubber. Hence, pair 3 is correctly matched.
464.
Ans. (B)
Sol. □ Coal gasification is the process of converting coal into synthesis gas (also called syngas),
which is a mixture of hydrogen (H2), carbon monoxide (CO) and carbon dioxide (CO2). H e n c e
statement 1 is correct.
The syngas can be used in a variety of applications such as in the production of electricity and making chemical
products, such as fertilisers.
The hydrogen obtained from coal gasification can be used for various purposes such as making ammonia,
powering a hydrogen economy. Hence statement 2 is correct.
□ The ammonia is reacted with the carbon dioxide to produce urea melt.
□ In-situ gasification of coal-or Underground Coal Gasification (UCG)-is the technique of
converting coal into gas while it is still in the seam and then extracting it through wells.
□ India has set the target that by 2030, it will gasify 100 million tonne of coal under four
major projects with an overall investment of Rs. 20,000 crore. Hence statement 3 is not
correct.
465.
Ans. (C)
Sol. □ Particulate Matter 2.5 (PM 2.5) refers to fine particulate matter smaller than 2.5
micrometers in diameter. It causes respiratory problems and also reduces visibility. It is an
endocrine disruptor that can affect insulin secretion and insulin sensitivity, thus contributing
to diabetes. Hence, statement 1 is correct.
□ National Air Quality Index (AQI) transforms complex air quality data of eight pollutants
into a single number (index value), nomenclature and colour. The measurement of air quality i s
based on eight pollutants, namely,
Particulate Matter (size less than 10 µm) or (PM10),
Particulate Matter (size less than 2.5 µm) or (PM2.5), hence, statement 2 is correct.
Nitrogen Dioxide (NO2),
Sulphur Dioxide (SO2),
Carbon Monoxide (CO),
Ozone (O3),
Ammonia (NH3), and
Lead (Pb).
□ AQI has six categories of air quality. These are. Good, Satisfactory, Moderately Polluted,
Poor, Very Poor and Severe.
466.
Ans. (D)
Sol. □ The GOBAR (Galvanizing Organic Bio-Agro resources)- DHAN scheme was launched by
the ministry of Jai Shakti. Hence statement 1 is not correct.
□ The scheme is being implemented as part of the Swachh Bharat Mission (Gramin). Hence,
statement 2 is not correct.
□ The Swachh Bharat Mission (Gramin) comprises two main components for creating clean
villages - creating open defecation free (ODF) villages and managing solid and liquid waste in
villages.
□ Aim:
Keeping villages clean, increasing the income of rural households, and generation of energy from cattle
waste.
The scheme also aims at creating new rural livelihood opportunities and enhancing income for farmers and
other rural people.

□ A Unified Portal of Gobardhan is also launched to ensure smooth implementation of
Biogas schemes/initiatives and their real time tracking. It is coordinated by the Department of
Drinking Water and Sanitation.
467.
Ans. (D)
Sol. □ Hydrogen is the lightest and first element on the periodic table. Since the weight of
hydrogen is less than air, it rises in the atmosphere and is therefore rarely found in its pure form.
□ At standard temperature and pressure, hydrogen is a non-toxic, non-metallic, odorless,
tasteless, colorless, and highly combustible diatomic gas.
□ Hydrogen fuel is a zero-emission fuel burned with oxygen. It can be used in fuel cells or
internal combustion engines. It is also used as a fuel for spacecraft propulsion.
□ Types of Hydrogen:
Grey Hydrogen
Constitutes India’s bulk Production.
Extracted from hydrocarbons (fossil fuels, natural gas).
By product: CO2; hence, statement 2 is not correct.
Blue Hydrogen
Sourced from fossil fuels.
By product: CO, CO2
By products are captured and stored, so better than grey hydrogen. Hence, statement 1 is not correct.
Green Hydrogen
Generated from renewable energy (like Solar, Wind).
Electricity splits water into hydrogen and oxygen.
By Products: Water, Water Vapor
□ India has a huge edge in green hydrogen production owing to its favorable geographic
conditions and presence of abundant natural elements.
□ The Government of India has announced, a National Hydrogen Energy Mission (NHM)
that will draw up a road map for using hydrogen as an energy source.
□ National Hydrogen Energy Mission
Focus on generation of hydrogen from green power resources.
To link India’s growing renewable capacity with the hydrogen economy.
India’s ambitious goal of 175 GW by 2022 got an impetus in the 2021-22 budget which allocated Rs.1500 crore
for renewable energy development and NHM.
468.
Ans. (B)
Sol. □ The Stockholm Convention is a global treaty to protect human health and the environment
from persistent organic pollutants (POPs). It became effective in 2004 and ratified by India in 2006.
□ Objectives
Support the transition to safer alternatives.
Target additional POPs for action.
Cleanup old stockpiles and equipment containing POPs.
In implementing the Convention, Governments of countries signatories to the convention will take measures to
eliminate or reduce the release of POPs into the environment.
□ Persistent Organic Pollutants: POPs are identified chemical substances that are
characterised by:
Persistence in the environment.
Bio-accumulation in the fatty acids in living organisms.
Less soluble in water.
Adverse effect on human health/ environment.
Exposure to POPs can lead to cancer, damage to central & peripheral nervous systems, diseases of the immune
system, reproductive disorders and interference with normal infant and child development.
The property of long-range environmental transport (LRET) makes them spread widely in the atmosphere.
469.
Ans. (D)
Sol. □ Carbon sequestration is the process involved in carbon capture and the long-term storage
of atmospheric carbon dioxide (CO2) and other forms of carbon to mitigate global warming
and reduce global warming. Hence, statement 2 is not correct.
□ Carbon farming is the process of changing agricultural practices or adopting agricultural
methods that increase the amount of carbon stored in the soil and reduce greenhouse gases. I t
therefore will enhance the process of carbon sequestration. Hence, statement 1 is not
correct.

You might also like